NCLEX Adult Health

Ace your homework & exams now with Quizwiz!

Which statement by the nurse would be the correct response to a client who is postmenopausal with a uterus when the client asks about temporary hormonal therapy for hot flashes? 1. "Hormonal therapy with a combination of low doses of estrogen and progestin may be prescribed." 2. "Unopposed estrogen hormonal therapy would be most appropriate." 3. "Hormonal therapy is an outdated treatment and can no longer be prescribed so you should try an alternative such as ginseng." 4. "Hormonal therapy is not an option for women with a uterus so you may need to consider a hysterectomy."

1. "Hormonal therapy with a combination of low doses of estrogen and progestin may be prescribed." (1. Correct: Estrogen and progestin are prescribed for women who have not had a hysterectomy.​ 2. Incorrect: Only women who no longer have a uterus can take estrogen without progestin.​ 3. Incorrect: Few data exists about the safety and effectiveness of alternative treatments. 4. Incorrect: Combination therapy for women with a uterus; and estrogen alone for women without a uterus.​)

The school nurse has educated a group of teens concerned about acquiring the Ebola virus. Which statement by the students would indicate to the nurse that further teaching is necessary? 1. "I can get a vaccine to prevent getting the Ebola virus." 2. "Ebola is not spread through casual contact, so my risk of getting the virus is low." 3. "The Ebola virus is passed from person to person through blood and body fluid." 4. "Ebola viruses are mainly found in primates in Africa."

1. "I can get a vaccine to prevent getting the Ebola virus." (1. Correct: This is an incorrect statement. At present, there is no vaccine to prevent Ebola. 2. Incorrect: This is a correct statement about the Ebola virus. Ebola is not spread through casual contact. 3. Incorrect: This is a correct statement about the Ebola virus. Ebola virus is passed from person to person through blood and body fluid. 4. Incorrect: This is a correct statement about the Ebola virus. Ebola viruses are mainly found in primates in Africa.)

A client has recently been diagnosed with systemic scleroderma. Which of the following client complaints would be of most concern to the homecare nurse? 1. "I feel like food gets stuck in my throat when I eat." 2. "I have a hard time brushing my teeth properly." 3. "My fingers burn when I go outside in the winter." 4. "I get short of breath whenever I exercise."

1. "I feel like food gets stuck in my throat when I eat." (1. Correct: Scleroderma is an autoimmune disorder characterized by the excess production of collagen and hardening of tissues. In systemic scleroderma, body organs lose the ability to function as the disease progresses. When parts of the digestive system build up collagen, clients experience frequent acid reflex, constipation, and difficulty swallowing. The nurse would be most concerned about aspiration during or after meals. 2. Incorrect: Because facial skin tightens, clients have difficulty opening the mouth completely. It becomes challenging to properly brush teeth or perform personal mouth care, increasing the likelihood of tooth decay. However, this would not be the biggest concern to the nurse at this time. 3. Incorrect: It is common for clients with one autoimmune disorder to develop other disorders. These symptoms indicate Raynaud's phenomenon, which is often reported by scleroderma clients. Advance of the disease triggers skin on the hands to become tight, stiff, and slightly shiny. The client begins to experience severe pain when fingers are exposed to the cold. Fingertips start out white in color, progressively turning red until re-warmed. Impaired circulation and pain are certainly an area of concern but not the most immediate worry to the nurse at this time. 4. Incorrect: Clients with scleroderma develop scarring ("fibrosis") of lung tissue, decreasing respiratory capabilities and eventually leading to pulmonary hypertension. This client reports shortness of breath just during exercise, indicating that simple daily activities are still achievable. It is obvious the disease has not yet progressed enough to impact ADL's, and therefore this is not the most concerning complaint at this time.)

A client with renal failure has returned to the unit post kidney transplant. Which postoperative interventions should the nurse provide? Select all that apply 1. Administer furosemide. 2. Maintain fluid replacement at 150 ml per hour for 8 hours. 3. Measure abdominal girth every 24 hours. 4. Weigh daily. 5. Measure urine output every 30-60 minutes.

1. Administer furosemide. 4. Weigh daily. 5. Measure urine output every 30-60 minutes. (1., 4., & 5. Correct: Diuretics are administered to promote postoperative diuresis. Daily weights are done to make sure there is not rapid weight gain which is a pertinent of fluid retention. Careful and frequent assessment of UOP helps determine fluid balance and transplant function. Oliguria is an early sign of acute tubular necrosis and should be detected as soon as possible post-op. 2. Incorrect: Fluid replacement is generally calculated to replace urine output over the previous 30-60 minutes, milliliter for milliliter. The stem of the question does not give you enough information about the client to know that this rate of fluid replacement is safe. 3. Incorrect: Signs of hemorrhage after this surgery include swelling at the op site, increased abdominal girth, signs of shock and decreased level of consciousness. Assessment for signs of hemorrhage should be done much more frequently than every 24 hours.)

A case manager is assessing an unresponsive client diagnosed with terminal hepatic encephalopathy for equipment needs upon discharge home for hospice care. Which equipment should the case manager obtain for this client? Select all that apply 1. Alternating pressure mattress 2. Hospital bed 3. Walker 4. Suction equipment 5. Oxygen

1. Alternating pressure mattress 2. Hospital bed 4. Suction equipment 5. Oxygen (1., 2., 4., & 5. Correct: An alternating pressure mattress will help to prevent pressure ulcers. A hospital bed is needed so that the head of the client's bed can be elevated to 30 degrees to ease respirations and decrease the work of breathing. The client with hepatic encephalopathy is unresponsive and may need suctioning if unable to clear secretions from the oropharynx. The client at the end stages of liver disease will be hypoxemic, so oxygen therapy is provided. 3. Incorrect: The unresponsive client will not need a walker.)

Which food items, if chosen by a client diagnosed with diverticulosis, would indicate to the nurse that the client understands the prescribed diet? Select all that apply 1. Avocados 2. Acorn squash 3. Applesauce 4. Lima beans 5. Raspberries 6. Cottage cheese

1. Avocados 2. Acorn squash 4. Lima beans 5. Raspberries (1., 2., 4., & 5. Correct: High fiber foods include raw fruits, legumes, vegetables, whole breads, and cereals. Avocados have 10.5 grams of fiber per cup. Acorn squash has 9 grams of fiber per cup. Lima beans 13.2 grams of fiber per cup. Raspberries have 8 grams of fiber per cup. 3. Incorrect: Raw fruits have more fiber than cooked or processed fruits. A raw apple would provide more fiber than applesauce. 6. Incorrect: Milk and foods made from milk: such as yogurt, pudding, ice cream, cheeses, cottage cheese and sour cream are low fiber.)

A client is being treated for fluid volume deficit with D5W, oral hydration, and management of viral symptoms. Which client data would indicate to the nurse that treatment has been successful? Select all that apply 1. BP 110/70 lying; 100/68 standing 2. Moist mucous membranes 3. Skin turgor recoil below clavicle is 3 seconds 4. Urine specific gravity of 1.033 5. Serum sodium 152 mEq (152 mmol/L)

1. BP 110/70 lying; 100/68 standing 2. Moist mucous membranes (1. & 2. Correct: These BP readings are within normal limits. Moist mucous membranes is a normal, desired finding. 3. Incorrect: Skin recoil delayed for more than 2 seconds indicates severe dehydration. 4. Incorrect: With fluid volume deficit, the specific gravity can be expected to be abnormally high. 5. Incorrect: This indicates hypernatremia, which is the same thing as dehydration.)

The client reports intense headaches with increasing pain for the past month. A magnetic resonance imaging (MRI) is prescribed. In reviewing the client's history, which information is of concern to the nurse? Select all that apply 1. Coronary artery stent 2. Cardiac pacemaker 3. Prescribed glimepiride every morning 4. Extreme obesity 5. History of working with metal fragments

1. Coronary artery stent 2. Cardiac pacemaker 4. Extreme obesity 5. History of working with metal fragments (1., 2., 4. & 5. Correct: With a coronary artery stent the magnet in the MRI may exert too much of a pull on the stent and cause damage. If a client with a cardiac pacemaker has an MRI, the pacemaker is turned off and the client could die. Extreme obesity, usually over 300 pounds is contraindicated. Magnetic substances in the body may become dislodged by the magnet, so history of working with metal fragments must be reviewed. 3. Incorrect: The client does not need to be NPO or have any modifications of their medications, so hypoglycemia is not a concern for MRI.)

The nurse recognizes which manifestations as signs of community-acquired pneumonia? Select all that apply 1. Cough 2. Decreased respiratory rate 3. Fever 4. Myalgia 5. Pleuritic chest pain

1. Cough 3. Fever 4. Myalgia 5. Pleuritic chest pain (1., 3., 4. & 5. Correct: Signs of community-acquired pneumonia include cough, crackles, egophony, tactile fremitus, fever, dyspnea, sputum production, myalgias, and pleuritic chest pain. A client with an infection (particularly pneumonia) will exhibit these symptoms. 2. Incorrect: Decreased respiratory rate is not a sign of community-acquired pneumonia; respiratory rate increases with fever and dyspnea.)

A school nurse is concerned that a teenager may have bulimia. What assessment findings would substantiate this belief? Select all that apply 1. Discolored teeth 2. Calluses on knuckles 3. Underweight 4. Dehydration 5. Hoarseness

1. Discolored teeth 2. Calluses on knuckles 4. Dehydration 5. Hoarseness (1., 2., 4., & 5. Correct: Discolored teeth occur from exposure to stomach acid when throwing up. The teeth may look yellow, ragged, or translucent. Calluses or scars on the knuckles or hands from sticking fingers down the throat to induce vomiting. Dehydration can occur from excessive vomiting and laxative or diuretic abuse. Hoarseness and chronic sore throat can occur due to stomach acid getting in the throat when vomiting. 3. Incorrect: The client is not underweight, but has a changing weight. Most are within normal weight range but may become slightly underweight or slightly overweight.)

Which nursing intervention should the nurse include when planning care for a client with Parkinson's disease? Select all that apply 1. Encourage high fiber diet with increased fluid intake. 2. Schedule activities early in the morning. 3. Help client perform stretching and strengthening exercises. 4. Teach client to sit upright at 90 degree angle when eating. 5. Maintain a high protein diet while taking levodopa.

1. Encourage high fiber diet with increased fluid intake. 3. Help client perform stretching and strengthening exercises. 4. Teach client to sit upright at 90 degree angle when eating. (1., 3., & 4. Correct: These are appropriate interventions for a client with Parkinson's disease. Constipation is a common problem in clients with Parkinson's disease. Increasing dietary fiber and fluid intake is useful in managing this problem. These clients have impaired mobility, so stretching and strengthening exercises are necessary to keep muscles working as long as possible. Loss of control of the mouth and throat muscles causes difficulty swallowing. Sitting upright at 90 degrees makes chewing and swallowing easier by using your posture to assist with the process. 2. Incorrect: Appointments and activities are best scheduled for late morning so that the client will not be rushed in performing self-care. 5. Incorrect: For clients on levodopa, explain that they shouldn't take this drug with high-protein meals as this can reduce its absorption and availability. Caution them to avoid foods and supplements high in pyridoxine (vitamin B6), which can inhibit the drug's action.)

A nurse notes that a client with end-stage chronic renal failure has dry, itchy skin, white crystals on the skin, and uremic halitosis. Which nursing interventions would be appropriate for this client? Select all that apply 1. Encourage use of cotton gloves during sleep 2. Apply emollients to the skin 3. Increase protein rich foods in the diet. 4. Cut fingernails short 5. Provide mouth care prior to meals

1. Encourage use of cotton gloves during sleep 2. Apply emollients to the skin 4. Cut fingernails short 5. Provide mouth care prior to meals (1., 2., 4. & 5. Correct: The build up of uremic frost associated with end stage renal disease causes pruritus. Gloves reduce the risk of dermal injury. Emollients and lotion will aid dry, itchy skin. Apply after bathing. Cutting nails short will decrease risk of skin breakdown when scratching. Uremic halitosis occurs from a build-up of urea in the body. It produces a metallic taste in the mouth. Mouth care prior to meals will help in eliminating this taste. 3. Incorrect: A client in end stage renal disease needs to decrease the amount of protein in the diet. Dietary restrictions include protein, sodium, potassium, and phosphate.)

The family of a bedfast 80 year old is providing care in the home. Which reports by the family indicate adequate understanding of interventions that will reduce the risk for skin breakdown? Select all that apply 1. I make sure that the sheets and the foam pad in the chair stay dry. 2. I will not encourage my parent to turn in the bed at night. 3. The perineal area should be kept dry and clean. 4. My parent eats 2 meals per day and drinks a supplement. 5. I may reposition my parent more than every 2 hours if their perception of pressure is intact.

1. I make sure that the sheets and the foam pad in the chair stay dry. 3. The perineal area should be kept dry and clean. 4. My parent eats 2 meals per day and drinks a supplement. 5. I may reposition my parent more than every 2 hours if their perception of pressure is intact. (1., 3., 4. & 5. Correct: Keeping moisture from the skin is important for reducing the risk of skin breakdown. Keeping the client dry after using a bedpan is important to maintain healthy skin. As long as the intake of food is adequate, no further action is needed with nutrition. The client who is aware of sensations of pressure on the body has less risk of skin breakdown than those that have lost sensation. 2. Incorrect: If the client is not repositioned at night, the resulting pressure on one site may lead to skin breakdown, even when the sensations of pressure are intact. A client in a state of sleep would not be as likely to respond to sensations of pressure appropriately, so the family would need to do this for her.)

A client with a history of peptic ulcer disease arrives at the emergency department reporting weakness, and vomiting "a lot of dark coffee-looking stomach contents." The client's skin is cool and moist to the touch. BP 90/50, HR 110, RR 26, T 98, O₂ sat 88%. Which primary healthcare provider prescription should the nurse perform first? 1. Initiate oxygen at 2 liters/nasal cannula. 2. Start an IV of NS at 150 ml/hr. 3. Insert nasogastric (NG) tube to low suction. 4. Attach client to the electrocardiography (ECG) monitor.

1. Initiate oxygen at 2 liters/nasal cannula. (1. Correct: The client is showing signs of shock and needs all of the above interventions. However, go back to the ABC's. Oxygen needs to be initiated first because the O₂ sat and the increased respiratory rate. 2. Incorrect: Fluids are needed to increase blood pressure and tissue perfusion. If O₂ sats were above 90 then this would be the first priority. 3. Incorrect: The "coffee looking" contents indicate GI bleeding. The NG tube will empty the stomach and monitor the bleeding but is not the top priority to prevent harm to the client. 4. Incorrect: The client has an increased heart rate and if the oxygen and circulation are not improved, problems could occur. Attaching the client to an ECG monitor will allow you to monitor thew heart for arrthymias or impending damage due to decrease oxygen. Necessary but not the first priority.)

A farm worker comes into the clinic reporting headache, dizziness, and muscle twitching after working in the fields. What condition does the nurse suspect? 1. Pesticide exposure 2. Heat stroke 3. Anthrax poisoning 4. Gastroenteritis

1. Pesticide exposure (1. Correct: These are symptoms of pesticide exposure when combined with the details given of coming from the fields. Death can result from severe acute pesticide poisoning. 2. Incorrect: The data provided does not lead the nurse to suspect heat stroke. The stem does not tell the temperature the farmer is working in. Heat stroke signs and symptoms include increased sweating, tachypnea and temperature greater than 105.8°F (41.0°C). 3. Incorrect: The data provided does not lead the nurse to suspect anthrax poisoning. The worker has been outside in a field. This is not a risk factor for anthrax exposure. Inhalation anthrax develops when you breathe in anthrax spores. It's the most deadly way to contract the disease, and even with treatment it is often fatal. Initial signs and symptoms of inhalation anthrax include: Flu-like symptoms, such as sore throat, mild fever, fatigue and muscle aches, which may last a few hours or days. 4. Incorrect: The data provided does not lead the nurse to suspect gastroenteritis. These signs and symptoms do not go with gastroenteritis. Gastroenteritis signs and symptoms include diarrhea, nausea, vomiting, fever and abdominal cramping.)

The nurse is caring for a client who has a history of sleep apnea. The client is scheduled for a colon resection the following morning and asks if the sleep apnea machine should be brought to the hospital. What is the nurse's best response? 1. Yes, bring the sleep apnea machine. 2. No, do not bring the sleep apnea machine. 3. It is your choice. 4. Call your primary healthcare provider.

1. Yes, bring the sleep apnea machine. (1. Correct: A client with sleep apnea is at risk for cardiac and respiratory complications postop due to decreasing oxygenation. So yes, the client needs to use the CPAP machine. Remember this client will also be receiving narcotics for pain and have a decreased activity level as well. All of these things can decrease oxygenation. 2. Incorrect: The client will need to have the machine after surgery. 3. Incorrect: Best response is for nurse to recommend that the client bring machine. 4. Incorrect: The nurse can answer this question.)

A client is preparing to be discharged after a total colectomy with the creation of an ileoanal reservoir for ulcerative colitis. The nurse recognizes that education has been successful if the client makes which statement? 1. "Ulcerative colitis cannot be cured." 2. "I look forward to having the ileostomy closed." 3. "I am going to eat a hamburger and fries for dinner." 4. "Because of this surgery, I am at a higher risk of developing colon cancer."

2. "I look forward to having the ileostomy closed." (2. Correct: Once the reservoir has healed, the ileostomy will be closed. 1. Incorrect: A total colectomy is removal of the entire colon. 3. Incorrect: It may take several days before solid food is tolerated. 4. Incorrect: The entire colon is removed so the client is not at risk for colon cancer.)

Which finding would the nurse expect to see in a client diagnosed with pneumocystis carinii pneumonia (PCP)? Select all that apply 1. Hemoptysis 2. Fever 3. Dyspnea 4. CD4 count of 500 cells/cubic millimeter 5. Wheezing

2. Fever 3. Dyspnea 5. Wheezing (2., 3., & 5. At first, PCP may cause only mild symptoms or none. Common signs/symptoms include fever (usually low-grade if with HIV), dry cough or wheezing, shortness of breath or dyspnea on exertion, fatigue, and pleuritic pain on inspiration. 1. Incorrect: Clients with pneumocystis pneumonia have a nonproductive cough. Hemoptysis is a late sign of lung cancer or tuberculosis. 4. Incorrect: The CD4 count is a test that measures how many CD4 cells are in the blood. These are a type of white blood cells, called T-cells, that move throughout the body to find and destroy bacteria, viruses, and other invading germs. A normal CD4 count is from 500 to 1,400 cells per cubic millimeter of blood. CD4 counts decrease over time in persons who are not receiving antiretroviral therapy. At levels below 200 cells per cubic millimeter, clients become susceptible to a wide variety of opportunistic infections, many of which can be fatal.)

A client arrives to the emergency department with reports of palpitations, chest discomfort, and light-headedness. The nurse connects the client to a cardiac monitor and notes a weak, thready pulse, and a BP of 90/50. What actions should the nurse take? Exhibit Select all that apply 1. Administer Lidocaine 50 mg intravenous push (IVP). 2. Initiate oxygen at 2 liters per nasal cannula. 3. Apply oxygen saturation monitor to client. 4. Prepare for immediate synchronized cardioversion. 5. Perform carotid massage. 6. Begin cardiopulmonary resuscitation.

2. Initiate oxygen at 2 liters per nasal cannula. 3. Apply oxygen saturation monitor to client. 4. Prepare for immediate synchronized cardioversion. (2., 3., & 4. Correct: This client has a rapid heart rate of 188/min. The actual rhythm is atrial tachycardia but can also be identified as supraventricular tachycardia because the heartrate is greater than 150/min. This client is considered unstable so requires oxygen therapy, with O₂ saturation monitoring, and synchronized cardioversion. 1. Incorrect: Lidocaine is not indicated for an atrial or supraventricular dysrhythmia. 5. Incorrect: Carotid massage is not within the scope of practice of the nurse. Asystole could result. 6. Incorrect: This client has a pulse, so CPR is not needed at this time.)

A client received a severe burn to the right hand. When dressing the wound, it is important for the nurse to do what? 1. Apply a wet to dry dressing for debridement. 2. Wrap each digit individually to prevent webbing. 3. Open blisters to allow drainage prior to dressing. 4. Allow the client to do as much of the dressing change as possible.

2. Wrap each digit individually to prevent webbing. (2. Correct: Each finger must be wrapped individually to prevent webbing. If not done appropriately the client could develop contractures and lose functional use of the hand. 1. Incorrect: No debridement is needed if dressing changes are done as ordered. 3. Incorrect: Blisters should be left intact so as not to create an open wound and an environment for infection to easily start. 4. Incorrect: This is not appropriate at this time and is not the most important option for the nurse to do to properly care for the wound and enhance healing.)

Which interventions should the nurse include in the plan of care for a client following chest tube placement for a spontaneous pneumothorax? Select all that apply 1. Keep the water seal chamber at the level of the right atrium. 2. Tape all connections between the chest tube and drainage system. 3. Strip the tubing when visible clots noted. 4. Empty the collection chamber of drainage every 24 hours. 5. Perform pulmonary assessment every two hours.

2., & 5. Correct: Securely tape all connections to prevent the tube from becoming disconnected. Pulmonary assessment should be done at least every 2 hours. Document a comprehensive pulmonary assessment including respiratory rate, work of breathing, breath sounds, pulse oximetry. 1. Incorrect: Must be kept below the client's chest. 3. Incorrect: Avoid aggressive chest tube manipulation, including stripping, or milking, as it can cause extreme negative pressure and damage tissue. 4. Incorrect: Chest tubes are changed, not emptied, when they become full.)

A home care nurse is assessing a client with a forearm cast recently applied for a displaced radial fracture. What client comment should the nurse consider the priority concern? 1. "The cast feels tight on my arm." 2. "There is an odd smell inside my cast." 3. "I can't open up my fingers this morning." 4. "The pain medicine is not relieving my pain."

3. "I can't open up my fingers this morning." (3. Correct: All the reported problems have the potential to be serious and must be investigated; however, one problem has already occurred and could permanently impair the client's mobility. The inability to extend fingers, particularly in a casted extremity, is a contracture resulting from prolonged ischemia of muscle tissues. Swelling inside the cast causes muscles to shorten and scar, leading to deformities or contractures called Volkmann contractures. Mild cases may be treated with splinting and exercise but severe cases need surgical intervention and possible even transplanted tissues with no guarantee of restored dexterity or mobility. 1. Incorrect: Obviously a 'really tight' sensation of the arm is of great concern, since swelling could be an early indication of compartment syndrome. However, the nurse should seek further clarification from the client regarding the "tightness" and its exact location. Another problem is of even greater concern. 2. Incorrect: Many odors could emanate from casting material, from skin breakdown to the odor of drying cast material. The client may even have put something down inside the cast so the smell should definitely be investigated. However, the nurse has greater priority at the moment. 4. Incorrect: There are many reasons pain medication may not relieve discomfort, including too low a dose or patient noncompliance with medication regime. On-going pain should certainly be investigated as a potential sign of greater problems but this is not the nurse's priority at this time.)

A client with a history of intolerance to fatty foods is admitted to the hospital with a sudden onset of severe right upper quadrant pain radiating to the right shoulder. What should be included in the nurse's initial focused assessment of this client? 1. "Do you have pain in the middle of your stomach that is relieved by vomiting?" 2. "Have you noticed any red splotches on your skin?" 3. "Please describe your bowel habits and stool." 4. "Tell me how often you eat high fat meals."

3. "Please describe your bowel habits and stool." (3. Correct: Clay colored stools are a sign of biliary obstruction and are due to lack of bile in the stool. Bile adds a darker color to the stool. Asking the client to describe stool is open ended and will give the nurse more detail. 1. Incorrect: Epigastric pain relieved by vomiting is found with clients who suffer from peptic ulcers. 2. Incorrect: Spider angiomas are seen in clients with liver disease 4. Incorrect: This does not relate to the client's pain and will not obtain needed information about the client's current condition.)

What should the nurse tell a 68 year old client who states that they have started experiencing tremors? 1. "This is nothing to worry about and is common with aging." 2. "You should increase your intake of potassium." 3. "We need to let your primary health care provider know because it may indicate a problem." 4. "Have someone check your blood pressure the next time you experience tremors."

3. "We need to let your primary health care provider know because it may indicate a problem." (3. Correct: Fine tremors are the first symptom reported in 70% of client's diagnosed with Parkinson's Disease. 1. Incorrect: Tremors are not a normal age change. 2. Incorrect: Tremors may indicate a problem. 4. Incorrect: Tremors may indicate early onset Parkinson's Disease.)

A client arrives at the emergency room with active gastrointestinal bleeding. What is the most important nursing action? 1. Treat the cause of the bleeding. 2. Record the amount of blood loss. 3. Initiate an intravenous access line. 4. Prepare client for stat endoscopy.

3. Initiate an intravenous access line. (3. Correct: The client has active gastrointestinal bleeding, which can quickly lead to hypovolemic shock. Active bleeding would be treated with fluids, and in certain cases, blood products. Establishing an IV site allows for immediate initiation of treatment before veins vasoconstrict and become too difficult to access secondary to shock. 1. Incorrect: While it is crucial to find and treat the cause of the bleeding, diagnosing is the responsibility of the primary healthcare provider. Because this client is experiencing internal bleeding, further tests may be needed to determine the source of the hemorrhage. 2. Incorrect: Gastrointestinal bleeding is very difficult to measure since there is no effective way to collect the fluid. The primary healthcare provider could order a hemoglobin and hematocrit but that does not precisely measure the actual amount of blood loss. 4. Incorrect: An endoscopy is an internal examination of a portion of the gastric system. However, proper preparation requires the client to be NPO for hours in order to properly visualize that system. This is not the most important nursing action.)

A construction worker comes into the occupational health nurse's clinic reporting chest heaviness. The nurse should assess for what additional signs and symptoms? Select all that apply 1. Headache 2. Dry, flushed skin 3. Lightheadedness 4. Dyspnea 5. Irregular pulse

3. Lightheadedness 4. Dyspnea 5. Irregular pulse (3., 4. & 5. Correct: Common heart attack symptoms and warning signs may include: Chest discomfort that feels like pressure, fullness, or a squeezing pain in the center of the chest that lasts for more than a few minutes, or goes away and comes back. Pain and discomfort that extend beyond the chest to other parts of the upper body, such as one or both arms, back, neck, shoulder, stomach, teeth, and jaw. Unexplained shortness of breath, with or without chest discomfort, dyspnea and tachypnea. Other symptoms, such as cold sweats, cool and clammy skin, nausea or vomiting, lightheadedness, anxiety, restlessness, indigestion, unexplained fatigue, irregular pulse. 1. Incorrect: Headaches do not commonly occur with MI. 2. Incorrect: Skin would be cool and clammy.)

What independent nursing interventions should the nurse include when planning care for a client who is in a fluid volume excess (FVE)? Select all that apply 1. Monitor Central venous pressure (CVP) 2. Administer diuretic 3. Monitor for orthopnea 4. Raise head of bed (HOB) to 45 degrees 5. Elevate edematous extremities

3. Monitor for orthopnea 4. Raise head of bed (HOB) to 45 degrees 5. Elevate edematous extremities (3., 4. & 5. Correct: These are independent nursing actions that will increase venous return and decrease edema. Also the nurse should assess for crackles, changes in respiratory pattern, shortness of breath (SOB), orthopnea. 1. Incorrect: This is a collaborative intervention. 2. Incorrect: This is a dependent intervention.)

The post-operative craniotomy client's urinary output suddenly increases to 325 mL in 30 minutes. Which nursing action takes priority? 1. Check urine specific gravity 2. Measure ICP level 3. Obtain blood pressure 4. Monitor CVP

3. Obtain blood pressure (3. Correct: This is the best answer because we are worried this client is going into shock. This is a time where checking the BP is appropriate. 1. Incorrect: Not the priority here. We are worried about shock. 2. Incorrect: We worry about increased ICP, however, an increased UOP indicates possible diabetes insipidus, so shock is likely. 4. Incorrect: If my client is going into shock, the highest priority is to assess the blood pressure. CVP will let us know if the client has FVD, but the BP will let us know if the client is tolerating it.)

The emergency department nurse is assuming care of a client with full thickness burns to both legs. Which primary healthcare provider prescription should be implemented first? 1. Administer IV morphine 2. Insert oropharyngeal airway 3. Start two large bore IVs 4. Apply silver sulfadiazine to burn area

3. Start two large bore IVs (3. Correct: Full thickness burns of both legs would result in a severe fluid volume deficit. A priority treatment for burns include fluid replacement; therefore, insertion of 2 large bore IVs is a priority. 1. Incorrect: Pain is important but not priority over fluid volume status. Remember, pain never killed anybody. 2. Incorrect: This client does not have airway involvement. These burns are on the legs; there is no indication in the stem that the airway is involved. 4. Incorrect: Application of silver sulfadiazine does not take priority over fluid replacement.)

Which client diagnosed with chronic peptic ulcer disease is at the highest risk for gastrointestinal bleeding? 1. 50 year old who consumes 4 ounces (120 mL) of wine at bedtime. 2. 55 year old who is positive for Helicobacter pylori (H. pylori). 3. 60 year old who requires corticosteroid inhalers for asthma. 4. 70 year old who takes clopidogrel daily for unstable angina.

4. 70 year old who takes clopidogrel daily for unstable angina. (4. Correct: The older client has a higher risk of bleeding and he is on a platelet aggregation inhibitor. This client is at highest risk for bleeding. 1. Incorrect: Alcohol is a cause of gastritis but not typically in this amount. The role of wine in small amounts as a cause of bleeding is not confirmed. 2. Incorrect: A positive H. pylori is present in 50% of the world's population, so this is not a deciding factor for increased risk of bleeding. 3. Incorrect: This would be my second pick because of the slight risk of the inhaled corticosteroid's effect on the protective lining of the stomach.)

The telemetry nurse sees this ECG strip go across the monitor on a client admitted with unexplained syncope. What initial action should the nurse take? Exhibit 1. Initiate the code for cardiac arrest. 2. Begin cardiopulmonary resuscitation (CPR). 3. Administer Epinephrine 1mg IVP. 4. Check for apical pulse.

4. Check for apical pulse. (4. Correct: Treat the client not the strip. Make sure this is indeed ventricular fibrillation. If it is, the client will be unresponsive and will be pulseless. There may simply be an electrode missing, and the client is actually fine. 1. Incorrect: Make sure the client is indeed unresponsive and does not have a pulse. 2. Incorrect: CPR should be done after verifying that the client is unresponsive and without a pulse. 3. Incorrect: Epinephrine is given after unsuccessful defibrillation.)

A client is admitted to the hospital due to a deep vein thrombosis (DVT). Which intervention should the nurse initiate? 1. Ambulate client around room every 2 hours. 2. Assess Homans' sign every 8 hours. 3. Place sequential compression device on both legs. 4. Apply intermittent warm, moist soaks to affected area.

4. Apply intermittent warm, moist soaks to affected area. (4. Correct. Warm, moist soaks help to decrease edema and ease the discomfort. 1. Incorrect. The client is placed on bedrest with a gradual increase in ambulation over several days to allow time for the clot to adhere to the vessel wall which will prevent embolization. 2. Incorrect. Manipulating the leg to determine Homans' sign can dislodge the clot. 3. Incorrect. Do not use sequential compression devices to treat a DVT. It could cause the clot to break loose or dislodge.)

A nurse is providing care to a post-operative parathyroidectomy client. Which occurrence takes highest priority? 1. Psychoses 2. Renal calculi 3. Positive Trousseau's sign 4. Laryngospasm

4. Laryngospasm (4. Correct: When the parathyroids are removed, calcium is affected because these glands help control calcium levels in the blood. 1. Incorrect: This is disturbing, and important, but AIRWAY is priority. 2. Incorrect: Renal calculi can cause problems and lead to pain and possibly renal failure but are not as important as airway obstruction. 3. Incorrect: A positive Trousseau's sign is seen with hypocalcemia but is not the highest priority. Airway is the most important in this question.)

Which client should the nurse recognize as being at greatest risk for the development of cancer? 1. Smoker for 30 plus years 2. Body builder taking steroids and using tanning salons 3. Newborn with multiple birth defects 4. Older individual with acquired immunodeficiency syndrome

4. Older individual with acquired immunodeficiency syndrome (4. Correct: Cancer has a high incidence in the immune deficiency client and in the older adult with both of these risk factors together, this one is the highest risk for cancer. 1. Incorrect: Although smoking is a known environmental carcinogen, this one risk factor alone is not the highest risk. 2. Incorrect: These are known environmental carcinogens, but do not rank as highly as aging and immune deficiency. 3. Incorrect: Birth defects are not a risk factor for cancer.)

A nurse enters a client's room to find the client on the floor having a seizure. Which nursing action is appropriate for this client? 1. Hold the client's arms and legs. 2. Insert a padded tongue blade in the client's mouth. 3. Assist the client back into the bed. 4. Place a rolled towel under the client's head.

4. Place a rolled towel under the client's head. (4. Correct: Placing a rolled towel under the client's head prevents further injury to the client. 1. Incorrect: Restraining the client may cause further injury to the client. 2. Incorrect: Forcing an object into the client's mouth can result in choking the client or injuring the client's teeth and mouth. 3. Incorrect: Lifting the client may cause injury to the nurse and client.)

The pathology report on a client diagnosed with urolithiasis reveals calcium oxalate stones. Which food selections by the client would indicate to the nurse that the client understands the prescribed low oxalate diet? Select all that apply 1. Spinach 2. Raspberries 3. Almonds 4. 100% bran cereal 5. Bananas 6. Raisins

5. Bananas 6. Raisins (5., & 6. Correct: Fruits provide valuable amounts of water, fiber, and antioxidants, all of which may help lower your risk for kidney stone symptoms. Many fruits are considered low-oxalate, meaning they contain less than 2 milligrams per serving. These include bananas, cherries, grapefruit, grapes, mangoes, melons, green and yellow plums and nectarines. Canned fruits, including peaches, pears, and dried fruits such as raisins, are also low in oxalate. 1. Incorrect: 1 cup of cooked spinach contains 1510 mg of oxalate. 2. Incorrect: Raspberries are the most significant fruit source of oxalate. One cup of raspberries contains 48 mg of oxalate. 3. Incorrect: 1 oz (28 g) of almonds contains 122 mg of oxalate. 4. Incorrect: One cup of 100% bran cereal contains 75 mg of oxalate.)

A client is admitted to an ED after sustaining a head injury in a motor vehicle crash. The client opens eyes and moans as pressure is applied to the nail bed of fingers and then pulls hand away. Based on this information, what Glasgow Coma Scale score should a nurse document for this client? Ans:______

8 (The nurse should document a Glasgow Coma Scale score of 8 for this client indicating that this client has a severe head injury. The nurse should receive a score of 2 for eye opening in response to pain a score of 2 for an incomprehensible verbal response, and a score of 4 for withdrawing from pain. Generally, head injury is classified as: Severe head injury: GCS score of 8 or less Moderate head injury: GCS score of 9 to 12 Mild head injury: GCS score of 13 to 15)

The nurse, caring for a client who has chronic renal failure, suspects that the client is experiencing anxiety. Which statements by the client would validate the nurse's suspicion? Select all that apply 1. "I do not think I can continue working." 2. "My husband has taken over the house cleaning and cooking." 3. "I fear I am dying." 4. "I have an "uneasy" feeling most of the time." 5. "Most of the time I feel very 'down and blue'."

1. "I do not think I can continue working." 2. "My husband has taken over the house cleaning and cooking." 3. "I fear I am dying." 4. "I have an "uneasy" feeling most of the time." (1., 2., 3. & 4. Correct: The inability to maintain employment is of concern to most clients who have been used to working. With a chronic illness, the client is unlikely to be able to return to work. Anxiety related to role strain is common. The client may not be able to perform the duties that she once did, thus causing others to have to assume their roles. Death is a possible outcome if transplant does not occur. Fear may be a later diagnosis as the client's condition deteriorates. Clients with anxiety often report feeling uneasy or on edge. 5. Incorrect: These comments are more indicative of a depressed mood than anxiety. Depression may also occur in the client who has chronic renal failure.)

A client is preparing to be discharged after a total hip replacement. Which client statement would indicate teaching has been successful regarding prevention of hip prosthesis dislocation? Select all that apply 1. "I should not cross my affected leg over my other leg." 2. "I should not bend at the waist more than 90 degrees." 3. "While lying in bed, I should not turn my affected leg inward." 4. "It is necessary to keep my knees together at all times." 5. "When I sleep, I should keep a pillow between my legs."

1. "I should not cross my affected leg over my other leg." 2. "I should not bend at the waist more than 90 degrees." 3. "While lying in bed, I should not turn my affected leg inward." 5. "When I sleep, I should keep a pillow between my legs." (1., 2., 3. & 5. Correct: These are appropriate actions to prevent hip prosthesis dislocation. Until the hip prosthesis stabilizes it is necessary to follow these instructions for proper positioning to avoid dislocation. 4. Incorrect: The knees should be kept apart to prevent dislocation.)

A client diagnosed with gout has received instruction on maintaining a low-purine diet. Which statements, if made by the client, would indicate to the nurse that teaching was successful? Select all that apply 1. "I will eliminate foods from my diet that contain 150 mg or more of purine per serving." 2. "Rather than drinking a glass of wine, I should drink a glass of beer." 3. "Losing weight can help reduce the uric acid levels in my blood." 4. "Potatoes, rice, and barley are high in purine and should be eliminated from my diet." 5. "Vegetables that should be limited to 2 times/week include cauliflower, spinach, and mushrooms." 6. "Increasing fluid intake to 8-10 cups/day will help to eliminate purines through my urine."

1. "I will eliminate foods from my diet that contain 150 mg or more of purine per serving." 3. "Losing weight can help reduce the uric acid levels in my blood." 5. "Vegetables that should be limited to 2 times/week include cauliflower, spinach, and mushrooms." 6. "Increasing fluid intake to 8-10 cups/day will help to eliminate purines through my urine." (1., 3., 5., & 6. Correct: Foods that contain 150 mg or more of purine are considered high purine foods and should be eliminated from the diet. Weight loss has been shown to improve insulin resistance, and therefore reduce uric acid levels in the blood. Vegetables that have high purine content include cauliflower, spinach, peas, asparagus, and mushrooms. These should be limited to no more than 2 times per week. Ensuring a sufficient fluid intake helps to reduce the risk of crystals forming in joints. Keeping hydrated and avoiding dehydration can lessen this risk and help to prevent gout attacks. 2. Incorrect: Alcohol - These cause increased dehydration and interfere with uric acid elimination. The metabolism of alcohol in your body is thought to increase uric acid production, and alcohol contributes to dehydration. Beer is associated with an increased risk of gout and recurring attacks, as are distilled liquors to some extent. The effect of wine is not as well understood. 4. Incorrect: Potatoes, rice, barley, noodles, and pastas are low in purine and can contribute to the 4 or more servings of starches needed per day.)

A nurse is teaching a client who has frequent urinary tract infections how to prevent future infections. What statement by the client would indicate to the nurse that treatment has been successful? Select all that apply 1. "I will go to the bathroom as soon as the urge to void hits me." 2. "It is important for me to drink five to six 8 ounce glasses of water every day." 3. "I should eat foods such as plums and prunes to increase the acidity of my urine." 4. "Nylon underwear should be worn when I am free from infection." 5. "When I clean after voiding, I will discard toilet paper after each swipe."

1. "I will go to the bathroom as soon as the urge to void hits me." 3. "I should eat foods such as plums and prunes to increase the acidity of my urine." 5. "When I clean after voiding, I will discard toilet paper after each swipe." (1., 3., & 5. Correct: Holding urine can lead to stasis of urine and increasing the risk for infection. Foods such as eggs, cheese, meat and poultry, whole grains, cranberries, plums and prunes, and tomatoes tend to increase the acidity of urine. Acidic urine is less likely to allow for bacterial growth. Discarding toilet paper after each swipe will decrease exposure and accidental introduction of bacteria into the urinary meatus. 2. Incorrect: Emphasize the importance of drinking eight to ten 8 ounce glasses per day. Water helps flush bacteria from the urinary tract. 4. Incorrect: Cotton underwear is recommended. The natural fibers work to wick moisture away from the skin which discourages yeast growth.)

Which comment by the client indicates understanding of possible complications of long term hypertension? 1. "I would like to have my serum creatinine checked at this visit." 2. "My blurred vision is part of getting older." 3. "I have leg pain caused by excessive exercise." 4. "Adding salt to my food is permissible."

1. "I would like to have my serum creatinine checked at this visit." (1. Correct: Hypertension is one of the leading causes of end stage renal disease. The client understands that renal function is reflected by serum creatinine levels. This request demonstrates understanding of the disease and possible complications. 2. Incorrect: The appearance of the retina provides important information about the severity and duration of hypertension. Manifestations of severe retinal damage include blurred vision, retinal hemorrhage, and loss of vision. 3. Incorrect: Intermittent claudication is a complication of peripheral vascular disease (PVD). Hypertension speeds up the process of PVD. 4. Incorrect: Lifestyle modifications include dietary sodium reduction, weight reduction, Dietary Approaches to Stop Hypertension (DASH) eating plan, moderation of alcohol consumption, regular physical activity, avoidance of tobacco use, and management of psychosocial risk factors.)

Which statements by an older adult indicate that teaching about adequate nutrition and hydration have been effective? Select all that apply 1. "Taking a multivitamin every day will help me get enough calcium and vitamin C." 2. "Enrolling in Meals on Wheels will provide me with a nutritious meal every day." 3. "I am less likely to become constipated if I increase my fiber intake to 20 grams a day." 4. "Drinking 1 liter of water a day will keep me hydrated." 5. "I will strive to eat at least 5 servings of fruits and vegetables a day."

1. "Taking a multivitamin every day will help me get enough calcium and vitamin C." 2. "Enrolling in Meals on Wheels will provide me with a nutritious meal every day." 5. "I will strive to eat at least 5 servings of fruits and vegetables a day." (1., 2., & 5. Correct: Older adults need an increased dietary intake of calcium and vitamins D, C, and A because aging changes disrupt the ability to store, use, and absorb these substances. If the client cannot eat enough foods containing these nutrients, then a multivitamin can help. There are many programs available which can assist the elder in getting more nutritious foods, such as food stamps, meals on wheels, and food banks. Five servings of fruits and vegetables a day reduces the risk of heart disease and stroke. 3. Incorrect: Older adults should consume 35-50 grams of fiber each day. 4. Incorrect: Older adults sometimes limit their fluid intake, especially in the evening, because of decreased mobility, prescribed diuretics, and urinary incontinence. Teach older adults the importance of drinking 2 liters of water a day plus other fluids as desired.)

Which statement made by a client diagnosed with Addison's disease indicates to the nurse that the client needs further teaching about fludrocortisone therapy? 1. "Taking my medicine at night will help me sleep." 2. "It is important to wear a medical alert bracelet all of the time." 3. "My medication dose may change based on my daily weight." 4. "I may need more medication if I feel weak or dizzy."

1. "Taking my medicine at night will help me sleep." (1. Correct: Steroids can cause insomnia so the client does not need to take the medication prior to going to bed. 2. Incorrect: This is a correct statement of understanding by the client. Wearing a medical alert bracelet is an excellent way of informing healthcare providers of a life threatening condition if the client is unable to verbalize that information. 3. Incorrect: Another correct statement. Steroid therapy is adjusted according to the client's weight and signs of fluid volume status. 4. Incorrect: This statement indicates that the client understands therapy. Signs of being undermedicated include weakness, fatigue, and dizziness. The client will need to report these symptoms so more medication can be given to the client.)

A nurse has completed education on safe sexual practices to a group of college students. Which comments by the students would indicate that education has been successful? Select all that apply 1. "The best way to prevent HIV is to abstain from sex." 2. "Contraceptives should contain spermicide N-9." 3. "Douching is recommended after intercourse." 4. "Drinking too much alcohol can increase the risk exposure to sexually transmitted disease (STDs)." 5. "If my partner will not use a condom, I will."

1. "The best way to prevent HIV is to abstain from sex." 4. "Drinking too much alcohol can increase the risk exposure to sexually transmitted disease (STDs)." 5. "If my partner will not use a condom, I will." (1., 4., & 5. Correct: These are correct statements. The best way to prevent HIV or STIs is to abstain from sex. If one decides to have sex, know your HIV status and your partners. Use condoms, male or female. Practice monogamy. Limit sexual partners. Use protection for all kinds of sexual contact. Get screened for sexually transmitted infections (STIs). Don't abuse drugs or alcohol, which are linked to sexual risk taking. 2. Incorrect: Spermicide N-9 will lower chances of pregnancy, but not HIV and other STIs. Spermicide N-9 actually makes your risk of HIV infection higher, caused by irritation of the vagina, which can make it easier for HIV to go into the body. 3. Incorrect: Douching removes some of the normal bacteria in the vagina that protects you from infection. This can increase your risk of getting HIV.)

When providing instructions, the nurse asks the client to repeat the techniques for crutch walking. The nurse is aware that further teaching is needed when the client makes which statement? 1. "The elbows should be flexed at 10 degrees." 2. "I should not lean on the crutches with my armpit." 3. "When going upstairs, my non-surgical leg goes up first." 4. "Both crutches are held in one hand when sitting down".

1. "The elbows should be flexed at 10 degrees." (1. Correct: The nurse is looking for an incorrect statement from the client. This statement indicates the client will need further instruction prior to discharge. When using crutches, the client's elbows should be flexed at 30 degrees. 2. Incorrect: This is a correct statement by the client. The weight of the body is placed on the hands and handgrips rather than being supported by the armpits, which could cause axillary nerve damage. This is a correct statement by the client; however, the question asks for an incorrect statement by the client. 3. Incorrect: The client is aware that the non-surgical "good" leg should be placed on the steps first when going upstairs, while the surgical "bad" leg is placed on the stairs first when coming down steps. This is a correct statement, indicating that the client did understand teaching; however, this question is looking for an indication that the client needs further instructions. 4. Incorrect: When sitting down in a chair, the client would indeed place both crutches in one hand while safely reaching for the chair with the free hand. This is a correct statement and does not indicate the need for further teaching.)

The nurse in the emergency department is caring for a client admitted in diabetic ketoacidosis (DKA). Which central venous pressure (CVP) reading would the nurse anticipate? 1. 1 mm of Hg 2. 3 mm of Hg 3. 6 mm of Hg 4. 12 mm of Hg

1. 1 mm of Hg (1. Correct: Normal CVP is 2-6 mmHg. This is a CVP reading that would indicate fluid volume deficit. A client in DKA will have polyuria. 2. Incorrect: This is a normal CVP reading. Normal CVP is 2 to 6 mm of Hg. 3. Incorrect: This is a normal CVP reading. 4. Incorrect: This CVP reading indicates fluid volume overload. The client in DKA will not be experiencing fluid volume excess.)

A client has arrived in the emergency department with partial thickness burns to 52 percent of the body. Which central venous pressure (CVP) reading would the nurse anticipate? 1. 1 mm of Hg 2. 2 mm of Hg 3. 6 mm of Hg 4. 10 mm of Hg

1. 1 mm of Hg (1. Correct: Normal CVP is 2-6mmHg. This CVP reading indicates fluid volume deficit. A client with 52 percent of the body burned with partial thickness burns would lose fluid from the vascular space out into the tissues resulting in fluid volume deficit. 2. Incorrect: This is a normal CVP reading. Normal CVP is 2 to 6 mm of Hg. 3. Incorrect: This is a normal CVP reading. Normal CVP is 2 to 6 mm of Hg. 4. Incorrect: An increased CVP reading indicates fluid volume excess. There is no indication in the stem that the client is experiencing a fluid volume excess.)

The nurse is assessing the injection site of a healthy client who received a Mantoux skin test 48 hours ago. Which finding at the injection site indicates a need for further evaluation? 1. 15 mm induration 2. 4 mm erythrokeratodemia 3. 0.1 mL bluish colored hard wheal 4. 0 mm induration

1. 15 mm induration (1. Correct: An induration of 15 mm or greater is usually considered significant in people who have normal or mildly impaired immunity. 2. Incorrect: This is a small, red, hard area that is smaller than 10 mm. Therefore the size is not considered significant. Induration is roughness, not hardness. The induration is what nurses assess to determine significance. 3. Incorrect: When administering a Mantoux skin test, 0.1 mL of solution is injected under the top layer of the skin to produce a wheal. The presence of the 0.1 mL wheal is not expected at this time. 4. Incorrect: This is normal finding in someone who has not been exposed to TB.)

Which client is at the greatest risk for developing pancreatic cancer? 1. 70 year old obese client who smokes one pack of cigarettes a day 2. 64 year old client who had gallbladder surgery less than 5 years ago 3. 58 year old client with Chron's Disease 4. 52 year old client whose mother died from pancreatic cancer

1. 70 year old obese client who smokes one pack of cigarettes a day (1. Correct: The incidence of pancreatic cancer increases with age. Cigarette smoking, exposure to industrial chemicals or toxins in the environment, and a diet high in fat, meat, or both are associated risk factors. 2. Incorrect: Diabetes and pancreatitis are associated with pancreatic cancer. 3. Incorrect: Diabetes and pancreatitis are associated with pancreatic cancer. 4. Incorrect: The inherited risk is small.)

Which clients can the nurse assign to the same room? Select all that apply 1. A 48 year old female one day postoperative appendectomy and a 30 year old female with nephrolithiasis 2. A 41 year old male with nausea, vomiting, and diarrhea and a 62 year old male with neutropenia 3. A 41 year old male with Methicillin-resistant Staphylococcus aureus (MRSA) infection and a 42 year old male with Clostridium difficile 4. A 14 year old two days postoperative splenectomy and an 80 year old female with Parkinson's disease 5. A 57 year old female with chronic obstructive pulmonary disease (COPD) and an 68 year old female with asthma

1. A 48 year old female one day postoperative appendectomy and a 30 year old female with nephrolithiasis 5. A 57 year old female with chronic obstructive pulmonary disease (COPD) and an 68 year old female with asthma (1 & 5. Correct: Both the client with a postoperative appendectomy and the client with nephrolithiasis will need frequent pain assessments. Also neither client has an infection that could be transmitted to the other client. These 2 clients can be assigned to the same room. The clients with asthma and COPD are noninfectious respiratory diseases, so they also can be assigned to the same room. 2. Incorrect: The client with neutropenia has a low number of neutrophils which are a common type of white blood cell important to fighting off infections. The client should be assigned to a single-client room. In addition the other client could be contagious depending on the causative factor of the nausea,vomiting and diarrhea. The client with neutropenia should not be assign with this client since their diagnosis has not been identified. 3. Incorrect: MRSA and C difficile require contact isolation due to different causative organisms. Both of these clients should be assigned to a single-client room. In the healthcare setting it is recommended that clients requiring Contact Precautions should be assigned a single-client room. 4. Incorrect: Think about it an adolescent and an older adult in the same room. The 14 year old client is in the early adolescent stage. Since an admission to the hospital is a stressful situation, the client may exhibit "immature" behaviors and be embarrassed about the healthcare team seeing their bodies. There is a wide gap between a 14 year old and a 80 year old developmental stage. The 80 year old is experiencing developmental changes for older adult client. In addition, this client is exhibiting symptoms of Parkinson's disease.)

A client diagnosed with cancer has been losing weight. What should the nurse teach the client regarding methods for improving nutritional needs to maintain weight? Select all that apply 1. Add butter to foods 2. Spread peanut butter on toast 3. Use biscuits to make sandwiches 4. Put honey on top of hot cereal 5. Eat Caesar salads once per day

1. Add butter to foods 2. Spread peanut butter on toast 3. Use biscuits to make sandwiches 4. Put honey on top of hot cereal (1., 2., 3., & 4. Correct: Butter and oil added to food will add calories. This client needs more calories and more protein. Spread peanut butter or other nut butters, which contain protein and healthy fats, on toast, bread, or crackers. Use croissants or biscuits to make sandwiches which provides more calories. Top hot cereal with brown sugar, honey, dried fruit, cream or nut butter. 5. Incorrect: This will not add calories for weight gain. Choose meat salads, such as chicken, ham, turkey, or tuna.)

A home health nurse is planning home safety education for a client and spouse. Which actions should be included to promote fire safety in the home setting? Select all that apply 1. A fire extinguisher should be kept on each level of the home. 2. Keep matches and lighters away from children by storing them in a locked cabinet. 3. Install carbon monoxide smoke alarms, and test them monthly. 4. You may leave Christmas lights lit all night as long as the tree is artificial. 5. Have a planned route of exit and a place where all family members will meet.

1. A fire extinguisher should be kept on each level of the home. 2. Keep matches and lighters away from children by storing them in a locked cabinet. 3. Install carbon monoxide smoke alarms, and test them monthly. 5. Have a planned route of exit and a place where all family members will meet. (1., 2., 3. & 5. Correct: A fire extinguisher should be placed on each level of the home, near an exit, but out of reach of children. Keeping matches and lighters away from children by storing them in a locked cabinet can prevent fire-related deaths. Carbon monoxide smoke alarms will alarm for smoke as well as carbon monoxide, which is an odorless gas than can kill quickly. Alarms should be tested every month and repaired or replaced immediately if malfunction occurs. A plan facilitates exit from the building, and a place to meet helps identify that all family is out of the building. 4. Incorrect: Lit Christmas lights should be turned off when no one is home and when people go to bed for the night. It does not matter whether the tree is real or artificial.)

A client who has been receiving care for cirrhosis arrives to the clinic for follow-up care. Which new signs and symptoms noted by the nurse would indicate that the client has developed hepatic encephalopathy? Select all that apply 1. A musty breath odor 2. Poor concentration 3. Fatigue 4. Slow movements 5. Asterixis

1. A musty breath odor 2. Poor concentration 4. Slow movements 5. Asterixis (1., 2., 4., & 5. Correct: These are signs and symptoms of hepatic encephalopathy, a severe complication of hepatitis and cirrhosis: A musty or sweet breath odor, poor concentration, fatigue, slow movement, asterixis (an abnormal tremor consisting of involuntary jerking movements, especially in the hands). These occur due to increasing ammonia levels in the blood. However, fatigue would not be a correct symptom to look for since the client will have fatigue already due to the cirrhosis. 3. Incorrect: Fatigue would not be a new symptom because it is a symptom of cirrhosis. The client will already have fatigue due to the cirrhosis, so fatigue would not indicate that the client is going into encephalopathy.)

The nurse is caring for a client being treated for hypertensive crisis and suspects that the client may be developing an abdominal aortic aneurysm (AAA). Which assessment findings by the nurse suggest that the client is developing this complication? Select all that apply 1. Abdominal bruit 2. Upper back pain 3. Hoarseness 4. Pulsations around umbilicus 5. Shortness of breath

1. Abdominal bruit 4. Pulsations around umbilicus (1., & 4. Correct. A bruit heard over the abdomen is an indicator of an abdominal aortic aneurysm and warrants further investigation. An abdominal aortic aneurysm usually causes a balloon-like swelling. The wall of the aorta bulges out which results in a pulsating mass in the abdomen. 2. Incorrect. Upper back pain is not associated with AAA but rather with a thoracic aneurysm. 3. Incorrect. Hoarseness can be caused by any number of disorders, but not AAA. It can be seen with a thoracic aneurysm. 5. Incorrect. Shortness of breath is indicative of a respiratory problem but can be seen with a thoracic aneurysm. It is not a symptom of AAA.)

The nurse suspects a client admitted with myasthenia gravis is going into a cholinergic crisis. Which signs and symptoms would validate the nurse's suspicions? Select all that apply 1. Abdominal cramping 2. Lethargy 3. Salivation 4. Hypertension 5. Lacrimation 6. Miosis

1. Abdominal cramping 2. Lethargy 3. Salivation 5. Lacrimation 6. Miosis (1., 2., 3., 5., & 6. Correct: Remember this: DUMBELLS as a mnemonic to help you recall these signs and symptoms. The signs of cholinergic crisis include Diarrhea and abdominal cramping, Urination increased, Miosis (pinpoint pupils), Bradycardia, Emesis (nausea and vomiting), Lacrimation, Lethargy, Salivation. 4. Incorrect: Hypertension is not a sign of cholinergic crisis. Muscles get weaker so BP would go down.)

A client has been admitted to the telemetry unit with a diagnosis of a cerebral vascular accident. What should the nurse assess to determine the client's risk for aspiration? Select all that apply 1. Ability to swallow 2. Gag reflex 3. Level of consciousness 4. Cough reflex 5. Ability to follow commands

1. Ability to swallow 2. Gag reflex 3. Level of consciousness 4. Cough reflex (1., 2., 3., & 4. Correct: Assessing the ability of a client to swallow is something the nurse can and should do. A small amount of water should be given to the client as the nurse observes for coughing or gurgling. If the nurse suspects a client is having difficulty safely swallowing, further assessment by a speech and language therapist is recommended. To test for a gag reflex use a tongue depressor. Ask the client to open the mouth and look at their throat with a penlight. If the uvula and pharynx rise as the client says "aaahh" then the gag reflex is intact. If it does not rise, touch the back of the throat at the soft palate and watch for the rise in the pharynx in a gag response, If intact,the client should not be at risk for aspiration with eating. A client with a decrease level of consciousness is always at risk for dysphagia and aspiration. A cough reflex is assessed by administering a small sip of water and observing for a cough. if the client coughs, feeding should be withheld until further testing can be performed. 5. Incorrect: Assessing ability to follow commands does not identify a problem with swallowing. It does not provide a great deal of information about cognitive function. The other tests provide more information specific to aspiration.)

What signs/symptoms would the nurse expect to assess in a client diagnosed with tabes dorsalis neurosyphilis due to untreated syphilis? Select all that apply 1. Abnormal gait 2. Blindness 3. Hyperreflexia 4. Stiff neck 5. Hearing loss

1. Abnormal gait 2. Blindness (1., 2. Correct: Symptoms of tabes dorsalis are caused by damage to the nervous system. Problems walking occur such as an abnormal gait or inability to walk at all. Vision changes can occur. Blindness is a complication of tabes dorsalis. 3. Incorrect: Loss of coordination and diminished reflexes occur rather than hyperreflexia. 4. Incorrect: Stiff neck is seen with meningitis, but also with meningovascular neurosyphilis. Meningeal neurosyphilis usually manifests with the clinical features of acute meningitis. 5. Incorrect: Hearing is not affected by neurosyphilis. However, vision changes, including blindness can occur.)

A client diagnosed with cancer has been losing weight. What should the nurse teach the client regarding methods for improving nutritional needs to maintain weight? Select all that apply 1. Add butter to foods. 2. Cup of cubed beef broth. 3. Add powdered creamer to milkshake. 4. Use biscuits to make sandwiches. 5. Fish sauted in olive oil. 6. Put honey on top of hot cereal.

1. Add butter to foods. 3. Add powdered creamer to milkshake. 4. Use biscuits to make sandwiches. 6. Put honey on top of hot cereal. (1., 3., 4., & 6. Correct: Butter added to foods adds calories. This client needs more calories and more protein. Spread peanut butter or other nut butters, which contain protein and healthy fats, on toast, bread, apple or banana slices, crackers or celery. Use croissants or biscuits to make sandwiches which provides more calories. Add powered creamer or dry milk powder to hot cocoa, milkshakes, hot cereal, gravy, sauces, meatloaf, cream soups, or puddings to add more calories. Top hot cereal with brown sugar, honey, dried fruit, cream or nut butter. 2. Incorrect: One cube of beef broth is 11 calories. Supplementing the diet with beef broth would not add significant calories. 5. Incorrect: Although cooked in olive oil, fish is low in calories.)

Twelve hours post coronary artery bypass surgery (CABG), the nurse notes the client's level of consciousness has decreased from alert to somnolent. BP 88/50, HR 130 and thready, resp 32, urinary output (UOP) has dropped from 100 mL one hour earlier to 20 mL this hour. What would be the nurse's first action? 1. Administer 100% oxygen per mask. 2. Lower the head of the bed. 3. Give furosemide STAT. 4. Re-check the BP in the other arm.

1. Administer 100% oxygen per mask. (1. Correct: This client has developed signs of cardiogenic shock, one of the complications post CABG. Cardiac output is decreased, so the client needs more oxygen for the circulating blood volume. 2. Incorrect: Lowering the HOB will not help in cardiogenic shock but will actually make it harder for the heart to pump. 3. Incorrect: Poor kidney perfusion is the reason for the decreased UOP. The kidneys are trying to conserve what little volume the body has to maintain vital organ perfusion as long as possible. 4. Incorrect: Rechecking the BP will not help the problem. With the other symptoms, this BP is most likely accurate. This would only delay treatment and would not fix the problem.)

An alert elderly client has been admitted to the hospital and placed on bedrest following a fall at home. During evening medication rounds, the nurse notes the client has become disoriented to time and place. The nurse is aware a new onset of confusion could be the result of what factors? Select all that apply 1. Admission to the hospital. 2. Amount of physical pain. 3. Current bed confinement. 4. Advanced age. 5. Response to analgesic.

1. Admission to the hospital. 2. Amount of physical pain. 3. Current bed confinement. 5. Response to analgesic. (1, 2, 3, & 5. Correct: The nurse is aware that multiple factors can contribute to acute confusion in clients. The sudden relocation to a new environment, along with pain from injury, could definitely contribute to an acute onset of confusion. The client's ordered bedrest and response to new pain medications are additional factors that could produce an acute change in mental status. 4. Incorrect: Age alone is not a factor for confusion. New onset of confusion may be successfully resolved once any contributing factors are addressed.)

The nurse is discussing frostbite prevention with a group of teenagers who participate in cold weather activities. What risk factors for developing frostbite will the nurse include? Select all that apply 1. Alcohol use 2. Dehydration 3. Diabetes 4. Exhaustion 5. Low level altitude

1. Alcohol use 2. Dehydration 3. Diabetes 4. Exhaustion (1., 2., 3., & 4. Correct: Risk factors for developing frostbite include alcohol and drug abuse, dehydration, medical conditions such as diabetes or any condition that results in poor blood flow to the extremities, fatigue and exhaustion. 5. Incorrect: Being at a high altitude reduces the oxygen supply to extremities and places the person at increased risk for developing frostbite.)

An oncology client with a Hickman catheter is being discharged to receive chemotherapy via cassette pump at home. The nurse is aware that discharge instructions should include what information? Select all that apply 1. Always use two pairs of gloves when preparing chemotherapy medications. 2. Discarded chemotherapy cassettes and tubings can be placed in regular trash. 3. Used needles or syringes must be placed into plastic chemotherapy receptacle. 4. Linens soiled with chemotherapy drugs can be washed with regular laundry. 5. Waste is placed into chemotherapy bags and picked up by medical supplier. 6. Regular home cleaning products are appropriate for spilled chemotherapy medications.

1. Always use two pairs of gloves when preparing chemotherapy medications. 3. Used needles or syringes must be placed into plastic chemotherapy receptacle. 5. Waste is placed into chemotherapy bags and picked up by medical supplier. (1, 3 and 5. Correct: Administering chemotherapy medications at home would require the same diligence and precautions that are used in the hospital setting. In order to prevent contamination, the individual preparing the chemotherapy should wear two pairs of gloves and should not prepare the drugs in an area where food is prepared. Used needles or syringes must be discarded in a hard, yellow plastic receptacle marked "chemotherapy". Any "soft" waste products, such as dressings or towels used to clean up spills, must be double-bagged and then placed into the designated "chemotherapy bag". These wastes are then picked up by the medical supplier for disposal. 2. Incorrect: No equipment used to prepare or administer chemotherapy medications can ever be placed in regular trash. Specially designated chemotherapy disposal receptacles must be used for all types of chemotherapy waste. 4. Incorrect: Several days after receiving chemotherapy, the human body eliminates unused or excess product through body waste such as stool, urine, or even emesis. Linens that become soiled with such waste products must be washed separately from normal linens for the first washing. A second washing is necessary, although the linens may be thrown in with other clothing for the second washing. 6. Incorrect: It is never appropriate to use regular home cleaning products when cleaning up spilled chemotherapy drugs. The medical supplier who delivers the equipment will also deliver the specific "chemotherapy spill kit" needed for the medication in use. In the home setting, it is advised to clean up the area of the spill at least three times.)

A 65 year old client is admitted for management of dehydration with an IV infusion of LR @ 125 mL/hr. What assessment findings would be of concern to the nurse? Select all that apply 1. Anxiety 2. BP 136/80 3. CVP 5 mmHg 4. Crackles noted right posterior lung field 5. S3 heart sound

1. Anxiety 4. Crackles noted right posterior lung field 5. S3 heart sound (1., 4. & 5. Correct: Volume overload is an adverse effect of IV therapy in the elderly. Anxiety is an early sign of hypoxia due to FVE. Crackles to the bases are an early sign of fluid volume excess (FVE). S3 heart sounds are also an indication of FVE. 2. Incorrect: This blood pressure is not considered hypertension in this age group. Blood pressure of >140/90 is cause for concern in this age group. Also, one BP is not cause for concern. In assessing for FVE, it is important to compare to the client's baseline. 3. Incorrect: Normal CVP is 2-6 mmHg. A CVP reading of 5mmHg does not indicate FVE.)

A palliative care client is suffering from persistent diarrhea. What foods should the nurse suggest? Select all that apply 1. Applesauce 2. Rice 3. Bananas 4. Tea 5. Yogurt

1. Applesauce 2. Rice 3. Bananas (1., 2., & 3. Correct: The BRAT diet is recommended for clients with persistent diarrhea. This diet consists of bananas, rice, applesauce, and toast. Rice and potatoes help to reduce diarrhea. Bananas will help replace potassium. Once the diarrhea subsides, the client can add easily digestible foods like eggs. 4. Incorrect: Avoid coffee and tea because caffeine containing beverages may have a laxative effect. Caffeine is a stimulant and will increase the peristalsis even more. 5. Incorrect: Dairy products may make the diarrhea worse. Avoid these until the diarrhea subsides.)

When caring for a client on bedrest, which interventions should the nurse implement to decrease the risk of deep vein thrombosis? Select all that apply 1. Apply compression hose. 2. Place pillow under knees while supine. 3. Assist client to perform active foot and leg exercises. 4. Place client on intermittent pneumatic compression device. 5. Assess extremities for negative Homan's sign.

1. Apply compression hose. 3. Assist client to perform active foot and leg exercises. 4. Place client on intermittent pneumatic compression device. (1., 3., & 4. Correct. The client will need compression or compression hose and/or intermittent pneumatic compression device. The client should perform leg and foot exercises to decrease stagnation of blood. Compression hose, foot and leg exercises as well as pneumatic compression devices increase venous return and prevents stasis of blood. Other interventions to decrease deep vein thrombosis (DVT) include early ambulation, passive and active range of motion, isometric exercises and anticoagulant drugs such as heparin. 2. Incorrect: Do not compromise blood flow by placing pillows under the knees, crossing legs, or sitting for long periods of time. Pillows under the knees help with pressure on the lower back. However, if pillows are left under the knees for an extended time, venous return could be compromised. A pillow under the knees is not a recommended intervention for DVT prevention. 5. Incorrect: Do not assess Homan's sign, as it may dislodge a clot. Homan's sign is not a preventative intervention. Assessing a Homan's sign is considered to be controversial, and this test may contribute to the release or dislodgement of a clot.)

When shopping at the mall, a nurse witnesses an individual collapse in cardiac arrest. A bystander begins CPR while the nurse opens an automatic external defibrillator (AED) brought by security. What critical actions should the nurse perform before delivering a shock? Select all that apply 1. Apply defibrillator pads to bare skin. 2. Verify that synchronizer button is on. 3. Continue CPR until advised to deliver shock. 4. Stop CPR while machine analyzes the rhythm. 5. Shout "clear" prior to activating shock button. 6. Apply cream under de-fib pads to prevent burns.

1. Apply defibrillator pads to bare skin. 3. Continue CPR until advised to deliver shock. 4. Stop CPR while machine analyzes the rhythm. 5. Shout "clear" prior to activating shock button. (1, 3, 4 and 5. Correct: Even in a public setting, the defibrillator pads must be applied directly to bare skin for a solid connection, with one pad in the left axillary area and the other pad just below the right clavicle. CPR should be initiated immediately while the machine is set up and the pads are positioned. CPR should stop momentarily while the AED analyzes the rhythm. Then, if a shock is advised, the nurse shouts "clear" to any individual near the client prior to administering a shock. If no shock is advised, CPR should continue. 2. Incorrect: The synchronized cardioversion mode is used only when converting erratic rhythms back into sinus rhythm, such as atrial fibrillation or atrial flutter. Cardioversion administers a low-voltage shock at a specific point during a heartbeat and can only be used on beating heart. When utilizing the AED for a client in cardiac arrest, the machine must be set to the defibrillate mode only. 6. Incorrect: Defibrillator pads are applied directly to dry, bare skin in order to maintain an optimal connection to deliver a shock. In the hospital setting, clients with excessive chest hair may need to be shaved, but not in the public setting. Substances such as cream or oils would actually increase the severity of a burn while diminishing the effectiveness of the shock.)

A client is admitted to the hospital due to a left-sided cerebrovascular accident. Which interventions should the nurse initiate? Select all that apply 1. Apply splint nightly to affected extremities. 2. Approach client from the right side. 3. Provide full range of motion once a shift. 4. Elevate left extremities on a pillow. 5. Place pillow in the right axilla. 6. Wrap affected hand into a fist.

1. Apply splint nightly to affected extremities. 5. Place pillow in the right axilla. (1., & 5. Correct: With a left-sided stroke, the right side of the body is affected. Applying a splint at night to the affected extremity will prevent flexion of that extremity. Prolonged flexion leads to contractures. Prevent adduction of the affected shoulder with a pillow placed in the axilla. 2. Incorrect: Vision is controlled by the left side of the brain. Vision on the right side of both eyes may have decreased (hemianopia) due to this left-sided stroke, so approach the client from the left side. 3. Provide full range of motion four or five times a day to maintain joint mobility. 4. Incorrect: Remember, left-sided cerebrovascular accident = right sided paralysis. The right extremities, which are affected by the left-sided stroke should be elevate on a pillow to prevent dependent edema. 6. Incorrect: The fingers should be positioned so that they are minimally flexed. This will prevent a contracture of the hand. Flexing the fingers into a fist will cause them to contract.)

The nurse is caring for a client with questionable loss of consciousness in the emergency department following a motor vehicle crash. Which action should the nurse take first? 1. Assess airway patency, breathing, and circulation. 2. Assess level of consciousness and movement. 3. Cover wounds with a sterile dressing. 4. Maintain cervical spine immobilization.

1. Assess airway patency, breathing, and circulation. (1. Correct: Any time a nurse is faced with emergency management, the primary survey should be followed. The beginning order of the primary survey is airway, breathing, circulation. 2. Incorrect: After completion of airway, breathing and circulation assessment, the nurse should assess for neurologic disability. This would include assessment of the client's level of consciousness (LOC). 3. Incorrect: Measurement of a full set of vital signs would occur after assessment of airway, breathing and circulation. 4. Incorrect: The secondary survey includes stabilization of the neck and assessment for signs of neck injury.)

What should the nurse monitor when caring for a client post fasciotomy of the arm? Select all that apply 1. Bleeding 2. Capillary refill 3. Color 4. Distal pulses 5. Infection 6. Sensation

1. Bleeding 2. Capillary refill 3. Color 4. Distal pulses 6. Sensation (1., 2., 3., 4., & 6. Correct: Fasciotomy is a surgical procedure that cuts away the fascia to relieve tension or pressure. So after the procedure, the nurse wants to make certain that pressure has been relieved and circulation distally is good. The nurse will thus need to monitor skin color, capillary refill, distal pulses, and sensation. Since this is a surgical procedure, bleeding will also need to be monitored. 5. Incorrect: Infection can be a complication, however, it will not be an immediate concern.)

A long-term care nurse is planning care for a newly admitted client diagnosed with alzheimer's disease. What should the nurse include in the plan of care? Select all that apply 1. Assess client's ability to perform self care. 2. Educate nursing staff to help client in all activities of daily living. 3. Separate tasks into small manageable steps. 4. Relieve family members of stress by advising them to visit 1 time per week. 5. Have nursing staff spend time talking and listening to client.

1. Assess client's ability to perform self care. 3. Separate tasks into small manageable steps. 5. Have nursing staff spend time talking and listening to client. (1., 3., & 5. Correct: All of these should be included in this client's plan of care. Assess the client's ability to perform self care and allow client to perform any care he/she is capable of doing. Separating tasks into small steps helps the client remember the steps. Plan for staff to spend some time talking and listening to the client. 2. Incorrect: Client independence should be promoted. Teach staff to promote self care and independence. If the client is capable of performing activities of daily living, then the client should be encouraged and allowed to do so. 4. Incorrect: Encourage family to visit to maintain socialization. We do not want to limit their visitation time. This will not be helpful to relieve family stress.)

A client arrives at the emergency department (ED) after sustaining a high-voltage electrical injury. Which interventions should the nurse initiate in the ED? Select all that apply 1. Assess entry and exit wound. 2. Monitor vital signs. 3. Monitor for myoglobinuria. 4. Connect to cardiac monitor. 5. Perform the rule of nines.

1. Assess entry and exit wound. 2. Monitor vital signs. 3. Monitor for myoglobinuria. 4. Connect to cardiac monitor. (1., 2., 3., & 4. Correct: These are correct interventions for the nurse to initiate when caring for a client who has sustained a high-voltage electrical injury. Remember, electricity kills vessels, nerves, and organs. 5. Incorrect: The rule of nines would not be used for an electrical injury. Visual examination is not predictive of burn size and severity with an electrical burn injury.)

The nurse is supervising the care of a client on bedrest with a skull fracture from head trauma. Which action, when performed by an unlicensed assistive personnel (UAP), should the nurse interrupt? 1. Assisting with turn, cough, and deep breathing (TCDB) 2. Elevating the head of the bed to 30 degrees. 3. Measuring urinary output every hour. 4. Turning off room lights.

1. Assisting with turn, cough, and deep breathing (TCDB) (1. Correct: The nurse should interrupt the UAP assisting with TCDB because this may increase intracranial pressure (ICP). TCDB increases intrathoracic pressure which then increases ICP. 2. Incorrect: Maintain client with head trauma in the head up position. This position promotes drainage from the head and decreases vascular congestion. 3. Incorrect: This is an acceptable action and one the UAP can do. 4. Incorrect: You want to decrease stimulation and turning off room lights will provide restful environment in an effort to decrease ICP.)

The nurse is caring for a client with cirrhosis of the liver and suspects that the client may be developing hepatic encephalopathy. Which assessments by the nurse suggest that the client is developing this complication? Select all that apply 1. Asterixis 2. Lethargy 3. Amnesia 4. Behavioral changes 5. Kussmaul respirations

1. Asterixis 2. Lethargy 3. Amnesia 4. Behavioral changes (1., 2., 3. & 4. Correct: Hepatic encephalopathy results in changes in neurologic and mental responsiveness due to the accumulation of ammonia. All of the correct options are either mental or neurologic changes. 5. Incorrect: Kussmaul respirations are not a characteristic of hepatic encephalopathy. They are seen in diabetic ketoacidosis (DKA).)

What interventions can an occupational health nurse discuss with a client in an effort to improve lateral epicondylitis (tennis elbow) pain? Select all that apply 1. Avoid activities that make the pain worse. 2. An oral, nonsteroidal, anti-inflammatory drug may be prescribed. 3. Immediately start stretching and exercising the muscle and tendon. 4. If pain persists, a cortisone injection into the inflamed area may be recommended. 5. Apply ice for 45 minutes, six times a day.

1. Avoid activities that make the pain worse. 2. An oral, nonsteroidal, anti-inflammatory drug may be prescribed. 4. If pain persists, a cortisone injection into the inflamed area may be recommended. (1., 2., & 4. Correct: General activities that make the pain worse should be avoided or at least cut back. While continued activity in the presence of mild discomfort is not harmful, activities that cause severe pain will only prolong the necessary recovery time and should be avoided. Oral, nonsteroidal, anti-inflammatory drugs are very helpful in controlling the pain and inflammation of tennis elbow. The medicine is taken daily for at least four to six weeks when treating severe cases. For less severe cases, these medicines may be taken only when needed. Cortisone injections are considered when the other measures have not worked and the pain is severe. The cortisone is injected into the area of the inflamed tendons in order to decrease the inflammation. 3. Incorrect: Stretching and exercising of the involved muscle and tendon unit is one of the mainstays of treatment for this condition once pain and inflammation have subsided, but not during the acute phase. A gentle stretching program is started through a range of motion at the elbow and wrist. This is combined with a program of muscle strengthening. 5. Incorrect: It is recommended to apply ice to the area two to three times a day, for 20 to 30 minutes each time.)

A nurse is planning to educate diabetic clients on how to decrease their risk for developing renal failure. What educational points should the nurse include? Select all that apply 1. Avoid daily use of non-steroidal antiinflammatory medications. 2. Aggressive blood pressure management is necessary. 3. Aim to keep Glycosylated Hemoglobin (HgbA1c) less than 7%. 4. Have estimated glomerular filtration rate measured every five years. 5. Increase protein intake to 30% of total calories eaten per day.

1. Avoid daily use of non-steroidal antiinflammatory medications. 2. Aggressive blood pressure management is necessary. 3. Aim to keep Glycosylated Hemoglobin (HgbA1c) less than 7%. (1., 2. & 3. Correct: NSAIDs can damage the kidneys with chronic use. Risk factors for diabetic related renal complications include hypertension and hyperglycemia; therefore, management of blood pressure and blood glucose is necessary. The ADA treatment goal for HgbA1c is < 7%. 4. Incorrect: The estimated glomerular filtration rate (eGFR) should be assessed at least yearly if not more frequently. 5. Incorrect: A diabetic client's diet should consist of no more than 15-20% caloric intake of protein because protein makes the kidneys work harder.)

After a cholecystectomy, a client experiences palpitations, weakness and diarrhea following meals. Which teachings would be appropriate for the nurse to provide the client? Select all that apply 1. Avoid drinking liquids with meals. 2. Increase high sugar foods as they are well tolerated. 3. Take adequate vitamins, iron and calcium. 4. Lie down on right side after meals. 5. Eat at least six small meals per day.

1. Avoid drinking liquids with meals. 3. Take adequate vitamins, iron and calcium. 5. Eat at least six small meals per day. (1., 3., & 5 Correct: Dumping syndrome is associated with meals having a hyperosmolar composition. To decrease hyperosmolar components, you decrease the carbs and electrolytes. You should avoid fluids with meals because they increase the size of the food bolus. Vitamins, iron, and calcium may become depleted after stomach surgery and due to dumping syndrome so taking these will help to maintain good health. Small frequent meals decrease the extremes of the hyperosmolar content and keep a steady blood sugar level. 2. Incorrect: High sugar foods and carbs speed through the GI tract. Fats and proteins digest slower and stay in the stomach longer. 4. Incorrect: Lying down on the left side slows emptying of the stomach. Lying on the right side will speed up emptying and make the symptoms worse. Sit upright for 30-60 minutes after eating.)

What should the nurse include in a discharge plan for a client diagnosed with lymphoma who will be receiving outpatient treatment? Select all that apply 1. Avoid uncooked meats, seafood or eggs and unwashed fruits and vegetables. 2. Take bleeding precautions. 3. Do not take influenza or pneumonia vaccine during treatment. 4. Avoid individuals with infections. 5. Emphasize importance of frequent oral hygiene with an alcohol based mouthwash.

1. Avoid uncooked meats, seafood or eggs and unwashed fruits and vegetables. 2. Take bleeding precautions. 4. Avoid individuals with infections. (1., 2., & 4. Correct: The client with lymphoma is susceptible to infection and should eat foods low in bacteria. The client should avoid uncooked meats, seafood or eggs and unwashed fruits and vegetables as the bacteria count will be higher than desired. Instruct client and family about bleeding precautions and management of active bleeding due to thrombocytopenia. They should be advised to avoid activities that place them at risk for injury or bleeding (including excessive straining). This client is at risk for infection due to low white count, so the client should avoid individuals who are ill. 3. Incorrect: Encourage clients to maintain current immunizations for influenza and pneumonia. They are more susceptible to infection. Cancer and cancer treatment can weaken the immune system, which puts them at higher risk of serious problems if they get the flu or pneumonia. Only live vaccines (MMR, varicella, oral polio) are contraindicated in clients receiving chemotherapy. 5. Incorrect: This client is at risk for bleeding and infection due to low platelet and white cell counts. The client needs frequent oral care with a soft toothbrush and alcohol free mouthwash. Alcohol-based mouthwashes can dry out the gum and increase bleeding.)

The nurse manager of a long-term care facility notes an increase in pressure ulcers over the last six months. What new protocol developed by the nurse manager is most likely to decrease the occurrence of decubiti? 1. Bedfast clients must be repositioned every two hours. 2. All clients should have egg crate mattress on the bed. 3. Clients bathed in bed need lotion applied to all joints. 4. Provide back massage daily to all clients on bed rest.

1. Bedfast clients must be repositioned every two hours. (1. Correct: Repositioning clients every two hours prevents excessive, prolonged pressure on skin and bony prominences. Such an action also provides an opportunity for visible inspection of the client by staff. This repositioning applies not only to bedfast clients, but also to those who sit in a chair for prolonged periods of time. 2. Incorrect: An egg crate, foam mattress topper can be useful, both on a bed or a chair seat, to decrease shearing forces and cushion skin. But padding a surface does not guarantee the client will not develop a pressure sore. 3. Incorrect: Applying lotion to body will help lubricate dry skin. However, massaging the skin directly over joints is not advised, since that skin is generally thinner and more fragile. Additionally, this action alone would not ensure a reduction in the occurrence of pressure ulcers. 4. Incorrect: A daily back massage does stimulate circulation and allow for inspection of the spine, particularly for clients on bed rest. But this action alone would not decrease the occurrence of decubiti.)

The homecare nurse is visiting a client to assess the response to new medications ordered for benign prostatic hyperplasia (BPH). What symptoms reported by the client would indicate to the nurse the medications are not working? Select all that apply 1. Bladder pain 2. Fever with chills 3. Urinary frequency 4. Terminal dribbling 5. Nighttime sweats

1. Bladder pain 3. Urinary frequency 4. Terminal dribbling (1, 3 and 4. Correct: Symptoms of benign prostatic hyperplasia are very similar to those of a urinary tract infection. As the prostate enlarges and presses against the bladder wall, it becomes more difficult for a client to start and maintain a stream of urine, or even to completely empty the bladder. Medications prescribed for this disorder are meant to shrink the prostate, allowing urine to flow easily when voiding. When the medications are ineffective, the client again experiences the original symptoms such as bladder pain, urinary frequency and a tendency to continue 'dribbling' urine after the bladder is emptied. The client may then need a different medication or a change in the dose currently prescribed. 2. Incorrect: The symptoms of fever with chills are related to infection rather than benign prostatic hyperplasia. Although untreated BPH may lead to a urinary infection because of retained urine, these two symptoms do not relate directly to this prostate disorder. 5. Incorrect: Nighttime sweats are not associated with benign prostatic hyperplasia. Nighttime sweats can be associated with tuberculosis.)

Which signs and symptoms does the nurse expect to see in a client admitted to the medical unit with Parkinson's disease? Select all that apply 1. Blank affect. 2. Decreased ability to swing arms. 3. Waddling gait. 4. Walking on toes. 5. Pill-rolling tremor. 6. Stiff muscles.

1. Blank affect. 2. Decreased ability to swing arms. 5. Pill-rolling tremor. 6. Stiff muscles. (1., 2., 5., & 6. Correct: Classic characteristics of Parkinson's disease include a blank facial expression, forward tilt in the posture, slow/slurred speech, tremor, and a short shuffling gait. These symptoms also are manifested by a decreased ability to swing the arms and stiff muscles. 3. Incorrect: This is a sign of Duchenne Muscular Dystrophy. The client with Parkinson's disease has a shuffling gait. 4. Incorrect: This is a sign of Duchenne Muscular Dystrophy. The client with Parkinson's disease has a shuffling gait.)

A client was admitted 48 hours ago in septic shock. Treatment included oxygen at 40% per ventimask, IV therapy of Lactated Ringer's (LR) at 150 mL/hr, vancomycin 1 gram IV every 8 hours, and methylprednisolone 40 mg IVP twice a day. Which clinical data indicates that treatment has been successful? Select all that apply 1. Blood pressure 96/68; HR 98; RR 20 2. WBC 12,000/mm (12 x 10⁹)/L 3. CVP- 6 mmHg 4. pH- 7.30; pCO₂- 44; pO₂ -92; HCO₃⁻ 20 5. Urinary output of 20 mL/hr

1. Blood pressure 96/68; HR 98; RR 20 3. CVP- 6 mmHg (1., & 3. Correct: The systolic BP should be greater than 90. Normal CVP is 2-6 mmHg. 2. Incorrect: Incorrect: WBC is elevated. 4. Incorrect: The client is still in metabolic acidosis, so no improvement. 5. Incorrect: Urinary output should be adequate if treatment is successful. The urinary output should be 30 mL/hr for an adult.)

A nurse is planning a health fair in a Hispanic community composed of primarily young adults. What would be essential for the nurse to provide to this community at the health fair? Select all that apply 1. Blood pressure screening 2. Glucose monitoring 3. Influenza vaccination 4. BMI calculation 5. Test urine for protein. 6. Pneumococcal vaccination

1. Blood pressure screening 2. Glucose monitoring 3. Influenza vaccination 4. BMI calculation 5. Test urine for protein. (1., 2., 3., 4., & 5. Correct: In order to answer this question correctly, the test taker needs to understand what is considered young adulthood (ages 18 to 40 yrs). Middle adulthood is from ages 40 to 65 yrs and late adulthood is greater than 65+ yrs. You also need to know what Hispanics are at risk for. Hispanics have a higher incidence of death from heart disease and stroke. Blood pressure monitoring is essential to detect and control hypertension. Diabetes is prevalent in the Hispanic community. Early diagnosis is critical to manage and control for the risk of complications. Flu vaccination is recommended for all ethnic groups. Obesity is very high among Hispanic Americans at >70%. Chronic renal failure is a high risk for Hispanic Americans particularly since diabetes is prevalent. Early testing for protein in urine is recommended. 6. Incorrect: Pneumococcal vaccination is recommended for older adults (greater than age 65).)

What assessment data would a nurse expect to find in a client diagnosed with acute inflammatory bowel disease? Select all that apply 1. Bloody stools that contain mucus 2. Pallor 3. Anorectal excoriation 4. Urine output below 30 mL/hr 5. Increased serum prealbumin

1. Bloody stools that contain mucus 2. Pallor 3. Anorectal excoriation 4. Urine output below 30 mL/hr (1., 2., 3., & 4. Correct: Stools are bloody and contain mucus. The client will be malnourished, thus will be pale due to anemia. Anemia is related to folate deficiency. Anorectal excoriation and pain can occur. Hypotension and low urine output indicate possible fluid volume deficit. 5. Incorrect: Serum prealbumin, albumin, and protein levels are decreased in malnourished individuals.)

A client with a history of alcoholism arrives at the clinic reporting severe abdominal pain with nausea and vomiting. What additional findings would make the nurse suspect the client may have pancreatitis? Select all that apply 1. Bruising at the umbilicus. 2. Fever with tachycardia. 3. Positive Trousseau sign. 4. Pain radiating to back. 5. Vague pain at night.

1. Bruising at the umbilicus. 2. Fever with tachycardia. 4. Pain radiating to back. (1, 2 and 4. Correct: Whether the client is experiencing acute or chronic pancreatitis, symptoms are severe and distinct. Bruising around the umbilicus is referred to as "Cullen's Sign", indicating internal bleeding. Because of inflammation in the pancreas, the client generally becomes febrile and pain can cause tachycardia. Considering the location of the pancreas, the client frequently experiences pain that radiates from the mid-gastric area to the back. 3. Incorrect: Trousseau's sign is an indication of low levels of calcium in hypocalcemia, occurring as a hand spasm when the nurse takes the client's blood pressure. This sign does not relate to pancreatitis. 5. Incorrect: Pain caused by pancreatitis is quite severe and continuous, not just at night. Vague night-time pain could be secondary to many disorders, but not pancreatitis.)

Which symptoms should the nurse anticipate when caring for a client with acute cholecystitis? Select all that apply 1. Chills 2. Fever 3. Nausea and vomiting 4. Increased appetite 5. Rigidity of upper right abdomen

1. Chills 2. Fever 3. Nausea and vomiting 5. Rigidity of upper right abdomen (1., 2., 3. & 5. Correct: Many clients with acute cholecystitis present with acute onset of right upper quadrant pain associated with nausea and vomiting. Epigastric pain may also be present as well as fever, chills, and anorexia. A physical examination often reveals rigidity of the upper right abdomen that may radiate to midsternal area or right shoulder. Rebound and guarding are present in some cases. 4. Incorrect: The client with cholecystitis will have nausea and vomiting which usually results in a decreased appetite.)

A 35 year old client asks a clinic nurse how to find out if the client is overweight or obese. The client weighs 135 pounds and is 5 feet 2 inches tall. What should the nurse educate the client about? 1. Calculating body mass index 2. Measuring abdominal circumference 3. Determining lean body mass 4. Finding the nearest hydrostatic testing location

1. Calculating body mass index (1. Correct: Calculating body mass index (BMI) would determine if the client is considered overweight or obese. 2. Incorrect: BMI is the most efficient way to determine if a client is overweight or obese. Measuring the abdominal circumference is one assessment for determining if a client is at risk for metabolic syndrome. 3. Incorrect: BMI calculates whether the client is overweight or obese. Once you have the BMI, you can calculate the lean body mass. 4. Incorrect: Although this is the "gold standard" for measuring body fat percentage by weighing the body in water, it is often performed in hospitals and university labs. It is not the most practical means of monitoring weekly progress.)

What food should the nurse include when teaching an older adult about increasing vitamin B12 intake? Select all that apply 1. Calf liver 2. Feta cheese 3. Fresh spinach 4. Shrimp 5. Tuna 6. Tofu

1. Calf liver 2. Feta cheese 4. Shrimp 5. Tuna (1., 2., 4., & 5. Correct: A serving of just one ounce of beef liver contains well over the amount of B12 that the average person needs for the day. Feta cheese contains a good amount of vitamin B12 as well as several other important nutrients such as calcium and vitamin B2. Shrimp contain about 80 percent of the daily value needed of vitamin B12. Tuna either canned or grilled has a lot of vitamin B12. In fact, just one three ounce serving offers an entire day's worth of vitamin B12. 3. Incorrect: Fresh spinach does not have any vitamin B12. 6. Incorrect: Tofu is low-carb, dairy-free, gluten-free, cholesterol-free, and vegan, so it is popular with people who have specialized diets. However, vitamin B12 is not in tofu.)

A 3 day post-operative client with a left knee replacement is reporting chills and nausea. Temperature: 100.8ºF/38.2ºC, pulse: 94, respiration: 28 and blood pressure is 146/90. What is the nurse's best action? 1. Call the surgeon immediately. 2. Administer extra strength acetaminophen per prescription. 3. Assess the surgical site. 4. Offer extra blankets and increase fluids.

1. Call the surgeon immediately. (1. Correct: The client's symptoms are indicative of infection, and the primary healthcare provider needs to be notified and may want diagnostic tests performed. The other actions are appropriate to treat the symptoms and provide comfort, but they are not the best action to fix the problem. 2. Incorrect: While this may be appropriate, it may also delay treatment of the problem, which is infection. Remember, you can only pick one answer to fix the problem and this action will only treat the symptoms. 3. Incorrect: The primary healthcare provider may want the site assessed, but this also delays treatment. Since you can only pick one option, this is not the best. 4. Incorrect: Comfort measures are always appropriate, but this is not the best action available.)

The nurse is caring for a ventilator-dependent client assisted with positive expiratory end pressure (PEEP). The high-pressure alarm begins sounding. What actions should the nurse initiate? Select all that apply 1. Check to see if client is biting ET tube. 2. Examine tubing for presence of water. 3. Inspect for any loose connections. 4. Reduce the amount of PEEP used. 5. Assess client's need for suctioning.

1. Check to see if client is biting ET tube. 2. Examine tubing for presence of water. 5. Assess client's need for suctioning. (1, 2 and 5. Correct: The high-pressure alarm on a ventilator indicates the machine is pushing against excessive resistance while trying to deliver oxygen to the client. There are multiple potential causes for a high pressure alarm. The client could be fighting against the ventilator and thus biting down on the endotracheal tube. Another possible issue may be the water that accumulates in the vent tubing from condensation. Or the client may have excessive mucus in the airways that requires the nurse to suction lungs frequently. Any of these problems could initiate the high pressure alarm. 3. Incorrect: Loose or disconnected tubing results in a low pressure alarm, indicating the ventilator is unable to exert the expected amount of pressure needed to oxygenate client. 4. Incorrect: The amount of PEEP provided to a ventilator-dependent client is determined by respiratory diagnosis or ABGs and is ordered by the primary healthcare provider. The high pressure alarm does not generally indicate a problem with the PEEP setting.)

A client consumes a lacto-ovo vegetarian diet at home. During hospitalization, the primary healthcare provider prescribes an increased calorie diet. Which foods are appropriate for the nurse to serve as between meal snacks to boost caloric intake? 1. Cheese sandwich and milk 2. Boiled eggs but no dairy products 3. Fish sticks and cocktail sausages 4. Fresh vegetables but no milk or eggs

1. Cheese sandwich and milk (1. Correct: A lacto-ovo vegetarian diet is a vegetarian diet that does not include meat, but does contain eggs and dairy. The client can eat milk and dairy products along with grain products on this diet. 2. Incorrect: Dairy products and eggs are allowed on this diet. Milk, cheese and yogurt can be consumed on a lacto-ovo vegetarian diet. 3. Incorrect: The client does not consume meats. Meats should not be provided as a snack. 4. Incorrect: The client can consume milk and eggs as well as fresh fruits and vegetables. Milk and eggs can be consumed on a lacto-ovo vegetarian diet.)

A client arrives at the emergency department after being removed from a burning building. The nurse suspects carbon monoxide poisoning when the client exhibits which signs and symptoms? Select all that apply 1. Chest pain 2. Hypertension 3. Abdominal cramps 4. Confusion 5. Palpitations

1. Chest pain 4. Confusion 5. Palpitations (1., 4. & 5. Correct: Not enough oxygen is getting to the vital organs, such as the brain and heart, so confusion and palpitations can occur. 2. & 3. Incorrect: These are not signs of carbon monoxide poisoning.)

A client with a severe cough is suspected of having lung cancer. When preparing the client for testing to confirm a diagnosis of cancer, which tests should a nurse anticipate? Select all that apply 1. Chest x-ray 2. Arterial blood gas 3. Bronchoscopy 4. Computed tomography (CT) 5. Pulmonary function test

1. Chest x-ray 3. Bronchoscopy 4. Computed tomography (CT) (1., 3., & 4. Correct: Chest x-ray, bronchoscopy, and CT scan are evidenced-based tests used in the diagnosis of lung cancer due to the efficacy of the tests. 2. Incorrect: Arterial blood gas measures the quantity of oxygen in the blood and acid-base status. 5. Incorrect: Pulmonary function test is used to diagnose obstructive lung diseases, such as emphysema.)

What signs/symptoms would the nurse expect to find in a client diagnosed with acute pyelonephritis? Select all that apply 1. Chills 2. Fishy smelling urine 3. Polyuria 4. Dysuria 5. Headache

1. Chills 2. Fishy smelling urine 4. Dysuria (1., 2., & 4. Correct: Chills, fever, fishy smelling urine, painful and frequent urination are common signs/symptoms of acute pyelonephritis. 3. Incorrect: Polyuria is a symptom of chronic pyelonephritis. 5. Incorrect: Chronic pyelonephritis leads to the buildup of toxins in the body. This can cause the client to have a headache.)

A client presents to the after-hours clinic with reports of pain that occurs with walking but generally subsides with rest. The nurse's assessment reveals coolness and decreased pulses in lower extremities bilaterally. What condition would the nurse recognize these symptoms being most indicative of? 1. Chronic Arterial Insufficiency 2. Chronic Venous Insufficiency 3. Chronic Unstable Angina 4. Chronic Coronary Artery Disease

1. Chronic Arterial Insufficiency (1. Correct: These symptoms are indicative of arterial insufficiency as there is pain with walking that is relieved by rest. This pain is known as intermittent claudication. In addition, the pulses are decreased or may be absent with arterial insufficiency and the extremities are cool to touch. Other s/s include: paleness of extremity when elevated or possible redness when lowered, loss of hair on affected extremity, and thick nails. 2. Incorrect: Venous insufficiency is not characterized by pain with walking. Pulses are generally normal and color is generally normal with the exception of the brown pigmentation that may be noted (especially around the ankles). 3. Incorrect: The description in the stem is evident of peripheral arterial insufficiency and is not descriptive of decreased coronary artery perfusion. No reports of chest pain were noted. 4. Incorrect: The description is evident of peripheral arterial insufficiency and is not descriptive of decreased coronary artery perfusion. The symptoms listed in the stem are indicative of a peripheral artery problem.)

A nurse is in the mall when a shopper who suddenly becomes non-responsive. Obtaining an available AED, the nurse would initiate what emergency interventions? Select all that apply 1. Clear everyone before shock. 2. Turn on the machine. 3. Initiate shock immediately. 4. Place pads on client's chest. 5. Await arrival of paramedics. 6. Shave client's chest.

1. Clear everyone before shock. 2. Turn on the machine. 4. Place pads on client's chest. (1., 2.,& 4. Correct: It is always critical to be certain rescuers are "clear" before providing a shock; that is, not physically in contact with the client or any equipment that is touching the client. It is also important to verify the machine is turned on prior to use. Additionally, the electrode pads need to be properly applied in the correct location on the client's chest, as depicted on the AED and the pads, in order for the machine to analyze the client's rhythm and determine action. 3. Incorrect: A shock is only delivered when the machine determines it is necessary to do so. 5. Incorrect: The purpose of the AED is to provide the public a means of providing emergency care to those experiencing a cardiovascular emergency while awaiting the arrival of qualified medical personnel. 6. Incorrect: Shaving would waste time and risk injury to the client.)

Which assessment finding by the nurse is most indicative of fluid volume overload? 1. Client has pitting edema in lower extremities. 2. Client's blood pressure is 120/80. 3. Client's CVP measurement is 6 mmHg. 4. Weight gain of 1.5 pounds (0.68 kg) in one day.

1. Client has pitting edema in lower extremities. (1. Correct. A client in fluid volume overload may experience pitting edema in lower extremities, a bounding pulse, increased blood pressure, and shortness of breath. 2. Incorrect. This blood pressure reading is considered normal and is not a characteristic of fluid volume overload. 3. Incorrect. This CVP is within the normal range therefore not indicative of a fluid volume excess. In a fluid volume excess, the CVP would be elevated. 4. Incorrect. A weight gain in excess of 2 pounds (0.9 kg) is of concern for fluid volume excess. Any weight gain overnight is reason for concern; however, the stem asked which finding was most indicative.)

Which assessment finding on a client four hours post right femoral percutaneous transluminal coronary angioplasty (PTCA) would require immediate intervention by the nurse? Select all that apply 1. Client reports chest discomfort. 2. Legs elevated 15 degrees. 3. Pressure dressing over puncture site intact/dry. 4. Client reports slight tingling to right foot. 5. Left pedal pulse 2+/4+, Right pedal pulse 1+/4+.

1. Client reports chest discomfort. 2. Legs elevated 15 degrees. 4. Client reports slight tingling to right foot. 5. Left pedal pulse 2+/4+, Right pedal pulse 1+/4+. (1., 2., 4., & 5. Correct: All episodes of chest pain post PTCA should be reported to the primary healthcare provider as the artery may be occluding. After PTCA, the client will need to keep the affected extremity straight for several hours. In some cases, the primary healthcare provider may use a device that seals the small hole in the artery; that may allow the client to move around more quickly. Any neurovascular changes that are abnormal will require intervention. If the client starts reporting tingling or numbness to the right foot and a decrease in the grade force of the right pedal pulse is noted by the nurse, then the primary healthcare provider will need to be notified. 3. Incorrect: Having an intact and dry pressure dressing is a good thing. A pressure dressing will help prevent arterial bleeding. The dressing is intact and there is no bleeding. This is a good thing.)

Which client should the nurse assess first? 1. Client who reports increasing size, firmness, and discomfort in the abdomen. 2. Client who develops a headache and dizziness after being started on losartan. 3. Client with a chest tube whose pulse oximeter reading is 92 mmHg while on 2 liters of oxygen per nasal cannula. 4. Client who is receiving total parenteral nutrition (TPN) and has a blood glucose level of 140 mg/dL (7.8 mmol/L).

1. Client who reports increasing size, firmness, and discomfort in the abdomen. (1. Correct: Increasing size and firmness of the abdomen can be an indicator of a serious issue such as peritonitis, bowel obstruction, or other abdominal issue that would require immediate notification of the healthcare provider.This is an unexpected change in the client's status that would require the nurses immediate respond. 2. Incorrect: Headache and dizziness are some of the common side effects of angiotensin receptor blockers (ARBs), such as losartan. The BP should be carefully monitored on all clients taking ARBs, but this client would not be a priority over the client with increasing abdominal girth, firmness, and tenderness. 3. Incorrect: The client with a pulse oximetry reading of 92 mmHg on 2 liters of oxygen does not take priority over the client with a possible life threatening abdominal problem. This reading is acceptable. 4. Incorrect: The increased amounts of dextrose in TPN can cause hyperglycemia. The client's insulin may need adjusting. But this level is not so high that it would take priority over the client with a possible abdominal complication.)

Which victim would the nurse decontaminate first in a biological terrorist event? 1. Client who was exposed but is exhibiting no symptoms 2. Client who has an open leg fracture and head injury 3. Client who is not breathing and has no palpable pulse 4. Client with minor cuts and abrasions

1. Client who was exposed but is exhibiting no symptoms (1. Correct: Exposed victims with no symptoms are first priority. 2. Incorrect: Victims needing maximum medical care are third priority. 3. Incorrect: Deceased victims are the last priority 4. Incorrect: Those with minor injuries are second priority.)

What signs and symptoms would a nurse assess for in a client who is receiving hospice care and is close to death? Select all that apply 1. Cool extremities 2. Mottling 3. Cheyne-Stokes respirations 4. Loss of appetite 5. Increased blood pressure

1. Cool extremities 2. Mottling 3. Cheyne-Stokes respirations 4. Loss of appetite (1., 2., 3., & 4. Correct: In the hours before death, blood will be shunted to the vital organs and not the periphery. This will make the extremities cool to the touch and mottled in appearance. Both cool extremities and mottling are due to reduced blood flow. Cheyne-Stokes respirations is a respiratory pattern that consists of loud deep inhalations followed by a pause of apnea. Loss of appetite will occur as energy needs decline. The use of moistened clothes around the mouth and lip balm may help with keeping lips moist and comfortable. 5. Incorrect: Blood pressure will not increase as death nears. The pumping action of the heart declines when death is occurring which leads to a decrease in cardiac output and blood pressure.)

A nurse from the maternity unit is pulled to the medical-surgical unit for the first four hours of the shift. Which clients would be appropriate for the charge nurse to assign to the nurse from the maternity unit? Select all that apply 1. Client with rheumatic fever 2. Client scheduled for an appendectomy 3. Client one day post cardiac catheterization 4. Client diagnosed with Methicillin-Resistant Staphylococcus Aureus 5. Client newly admitted with Guillian-Barre Syndrome

1. Client with rheumatic fever 2. Client scheduled for an appendectomy 3. Client one day post cardiac catheterization (1., 2., & 3. Correct: Rheumatic fever is an inflammatory disease that can develop later as a complication of untreated or inadequately treated Group A beta hemolytic strep infections such as strep throat and scarlet fever. It is not contagious at this point, so the maternity nurse could be assigned to this client. The maternity nurse should have the knowledge and skill needed to provide preop care to a client scheduled for a routine appendectomy.This nurse routinely cares for preop surgical clients on the maternity unit. Since the client who had a cardiac catheterization is one day post procedure, the client is stable and could be appropriately assigned to the maternity nurse if needed. 4. Incorrect: This client is contagious and should not be delegated to the maternity nurse. The nurse will be going back to the maternity unit after four hours and will be a potential agent for spreading the infection. Client safety could be compromised. 5. Incorrect: Although the client with Guillian-Barre Syndrome is not contagious, this is a newly admitted client with a complex, possibly life threatening condition. The maternity nurse may not have the knowledge and assessment skills needed to care for this client. This would not be an appropriate assignment for this nurse.)

When teaching a client about lactose intolerance, what should the nurse include? Select all that apply 1. Common symptoms of lactose intolerance include abdominal bloating, diarrhea, and gas. 2. Symptoms of lactose intolerance generally occur three hours after consuming foods high in lactose. 3. Calcium rich foods should be consumed. 4. The client can drink lactose-free milk. 5. Vitamin D foods should be increased in the diet.

1. Common symptoms of lactose intolerance include abdominal bloating, diarrhea, and gas. 3. Calcium rich foods should be consumed. 4. The client can drink lactose-free milk. 5. Vitamin D foods should be increased in the diet. (1., 3., 4., & 5. Correct: These statements are correct. Symptoms include abdominal bloating, pain, diarrhea, and gas. Because milk and milk products cause symptoms, the client may not get enough calcium and vitamin D. Supplementing with calcium or foods high in calcium and vitamin D is important to maintain these levels. The client may have lactose-free milk. 2. Incorrect: Symptoms occur 30 minutes to 2 hours after drinking milk or milk products.)

What should the nurse include in the teaching plan for a client who has iron deficiency anemia? Select all that apply 1. Consume iron rich foods such as dried lentils, peas, and beans. 2. Notify primary healthcare provider of glossitis, anorexia, and paresthesia. 3. Iron is needed for white blood cell development. 4. Educate about ferrous sulfate supplement. 5. After drinking liquid iron, follow immediately by water.

1. Consume iron rich foods such as dried lentils, peas, and beans. 2. Notify primary healthcare provider of glossitis, anorexia, and paresthesia. 4. Educate about ferrous sulfate supplement. (1., 2., & 4. Correct: These are examples of iron rich foods. Foods high in iron will help with correcting iron deficiency anemia. Glossitis, anorexia, and paresthesias can result from iron deficiency anemia. Foods high in vitamin C such as citrus fruits, dark green leafy vegetables and strawberries help with absorption. 3. Incorrect: Iron is needed for red blood cell development and oxygen transport to the cells. Iron is not needed for white blood cell development. White blood cell development. White blood cells are produced in the bone marrow. 5. Incorrect: Clients should dilute liquid iron with water or juice, drink with a straw, and rinse mouth after swallowing. Iron will stain the teeth.)

What is the nursing priority for the client experiencing hyperparathyroidism? 1. Continuous cardiac monitoring. 2. Initiate fall precautions. 3. Administer IV normal saline. 4. Begin preparations for emergency parathyroidectomy.

1. Continuous cardiac monitoring. (1. Correct: To much calcium equals sedation. Life threatening complications such as airway obstruction and cardiact arrest may occur from severely high levels of calcium. 2. Incorrect: Preventing injury is important because of bone density loss and risk of fractures. But it is not the priority. Done after maintaining airway and circulation. 3. Incorrect: IV normal saline in large volumes is done to promote calcium excretion, but it is not the priority over airway and circulation. 4. Incorrect: Surgery is indicated "when medically feasible". Cardiac monitoring is the priority at this time.)

The nurse is teaching a group of clients how to decrease the risk of developing osteoarthritis (OA). What should the nurse include? Select all that apply 1. Control blood sugar. 2. Use largest, strongest joints for lifting. 3. Do intense aerobic exercise, daily. 4. Maintain a healthy weight. 5. Wear joint padding with playing sports.

1. Control blood sugar. 2. Use largest, strongest joints for lifting. 4. Maintain a healthy weight. 5. Wear joint padding with playing sports. (1., 2., 4., & 5. Correct: High glucose levels speed the formation of certain molecules that make cartilage stiffer and more sensitive to mechanical stress. The client should use the largest and strongest joints and muscles. Use both arms when lifting or carrying an object. By using the largest and strongest joints, less stress occurs on single joints or weaker areas of the body. Excess weight is one of the biggest risk factors for osteoarthritis. Although injuries aren't always avoidable, it pays to protect joints. If playing sports, wear protective gear, such as joint padding for soccer or hockey. And make sure any baseball field uses break-away bases. 3. Incorrect: Physical activity is one of the best ways to keep joints healthy. As little as 30 minutes of moderately intense exercise five times a week helps joints stay limber and strengthens the muscles that support and stabilize the hips and knees. However, it does not have to be daily or intense.)

Which food items, if chosen by a new unlicensed assistive personnel (UAP), would indicate to the nurse that the UAP understands a clear liquid diet? Select all that apply 1. White grape juice 2. Gelatin 3. Vanilla pudding 4. Lemon Popsicle 5. Fat free Broth 6. Tea with honey

1. White grape juice 2. Gelatin 4. Lemon Popsicle 5. Fat free Broth 6. Tea with honey (1., 2., 4., 5., & 6. Correct: A clear liquid diet is made up of only clear fluids and foods that are clear fluids when they are at room temperature. These choices are considered to be clear liquids. 3. Incorrect: This is considered appropriate for a full liquid diet.)

What is the first nursing action that should be taken in caring for a client with suspected tuberculosis? 1. Identify the client's symptoms promptly. 2. Instruct the client to cover the mouth and nose with tissues when sneezing. 3. Isolate the client in a negative pressure room. 4. Place a surgical mask on the client.

1. Identify the client's symptoms promptly. (1. Correct: First, identify the client's symptoms. 2. Incorrect: Not before proper identification of client's symptoms. 3. Incorrect: Not before proper identification of client's symptoms. 4. Incorrect: Not before proper identification of client's symptoms.)

What signs/symptoms would the nurse expect to assess in a client diagnosed with exocrine pancreatic cancer? Select all that apply 1. Dark tea colored urine 2. Clay colored stools 3. Jaundice 4. Coffee ground emesis 5. Lower abdominal pain

1. Dark tea colored urine 2. Clay colored stools 3. Jaundice (1., 2., & 3. Correct: Diseases of the head of the pancreas such as pancreatic cancer can lead to darkening of the urine, clay colored stools, and jaundice. All are the result of bile duct blockage. 4. Incorrect: Coffee ground emesis is a symptom of an ulcer that is bleeding. 5. Incorrect: Pain in the upper abdomen that radiates to your back is seen with pancreatic cancer. Lower abdominal pain can be associated with diseases such as diverticulitis.)

A client has been receiving 5-fluorouracil treatments for colon cancer and is admitted with weakness, fatigue, thrombocytopenia and low grade fever. Which actions would be appropriate for this client? Select all that apply 1. Dedicated supplies in room 2. Semi-private room 3. Liquid diet 4. Limit visitors 5. Nasogastric tube placement

1. Dedicated supplies in room 4. Limit visitors (1., & 4. Correct. This client is at risk for infection and bleeding. Keeping dedicated supplies in room will decrease the risk of infection. Limiting visitors will decrease risk of infection. 2. Incorrect. This client needs a private room because the client is immunocompromised. 3. Incorrect. Unless contraindicated, the client would be prescribed a high-calorie, high-protein diet. 5. Incorrect: To minimize the risk of bleeding, restrict the placement of nasogastric tubes, rectal tubes, and suctioning equipment.)

The nurse is planning to teach a group of senior citizens about modifiable risk factors for developing a stroke. Which factors should the nurse include? Select all that apply 1. Diabetes mellitus 2. Hypertension 3. Hispanic ethnicity 4. Atrial fibrillation 5. Sleep apnea 6. Smoking

1. Diabetes mellitus 2. Hypertension 4. Atrial fibrillation 5. Sleep apnea 6. Smoking (1., 2., 4., 5., & 6. Correct: Modifiable risk factors are those that can potentially be altered through lifestyle changes and medical treatment, thus reducing the risk of stroke. Diabetes and hypertension damages vessels by causing plague buildup which can lead to decreased perfusion to the brain. Atrial fibrillation can cause a clot to be thrown from the heart and travel to the brain. Sleep apnea causes hypoxia that can affect brain perfusion. Smoking causes vasoconstriction which can decrease oxygenation to the brain. All of these problems however, can be altered through lifestyle changes or through medical treatment. 3. Incorrect: Hispanics, African Americans, Native Americans, and Asian Americans have a higher incidence of strokes than whites. You cannot change your race or ethnicity so this is a non-modifiable risk factor for stroke.)

Which signs/symptoms should the nurse assess for in the client admitted with a diagnosis of myasthenia gravis? Select all that apply 1. Difficulty holding head erect 2. Limited facial expressions 3. Ptosis 4. Hemiparesis 5. Writhing, twisting movements of the body 6. Pill rolling

1. Difficulty holding head erect 2. Limited facial expressions 3. Ptosis (1., 2., & 3. Correct: These are three of the primary symptoms of myasthenia gravis. The muscles of the head and neck are weak and have difficulty holding the head up. Facial paralysis occurs and drooping of the eyelids develops as the client's muscles get tired. 4. Incorrect: Hemiparesis is a type of physical condition with weakness on one side of the body. 5. Incorrect: Writhing, twisting movements of the face, limbs, and body is known as chorea and is seen in Huntington's disease. 6. Incorrect: The hand tremor described as "pill rolling" is a sign of Parkinson's disease. The thumb and forefinger appear to move in a rotary fashion as if rolling a pill, coin, or other small object.)

A community health nurse is planning to discuss how to prevent pesticide ingestion at a local health fair. What should the nurse include in this teaching session? Select all that apply 1. Discard the outer leaves of lettuce. 2. Wash fruits and vegetables with dish soap. 3. Buy organic produce. 4. Peel fruits prior to eating. 5. Dry produce thoroughly with disposable paper towels after washing. 6. Use a scrub brush when washing fresh fruits and vegetables.

1. Discard the outer leaves of lettuce. 3. Buy organic produce. 4. Peel fruits prior to eating. 5. Dry produce thoroughly with disposable paper towels after washing. 6. Use a scrub brush when washing fresh fruits and vegetables. (1., 3., 4., 5., & 6. Correct: The outer leaves of green, leafy vegetables, such as lettuce and cabbage, should be discarded as pesticide residue likely remains there. Another great idea to reduce overall exposure to pesticides is to buy organic or unsprayed produce. If you can't buy organic, peel fruits and vegetables prior to eating. Washing your fruits and veggies is not enough if you want to reduce the pesticide load you expose yourself to, as it is very important to thoroughly dry them with disposable paper towels as well. This will remove all the remaining pesticide residue and make the produce safer to eat. A scrub brush is very effective in cleaning the crevices and areas around the stem. 2. Incorrect: One of the most common mistakes people make in their attempt to remove all pesticide residue from their produce is that they wash their fruits and vegetables with soap or, even worse, dish soap. Never use detergents, special rinses or soaps of any kind, as this will only do more harm than good. Unless the soap is entirely made of natural and organic materials, it tends to contain harmful compounds that easily penetrate the skin of the fruits, thus doing more harm than the actual pesticides after you ingest them. Simply wash with tap water.)

Which information should the nurse plan to teach family members of a client diagnosed with hepatitis B? 1. Do not share personal items with the client, such as razors or toothbrushes. 2. Wash dishes separately from the rest of the family's. 3. Wear a surgical mask when in close proximity to the client. 4. Use a separate bathroom from the client.

1. Do not share personal items with the client, such as razors or toothbrushes. (1. Correct: Hepatitis B is a bloodborne pathogen that can spread via sharing personal items, such as razors or toothbrushes where infected blood can get into a person's cut, mucous membranes, etc. 2. Incorrect: Unlike some forms of hepatitis, Hepatitis B is not spread through sharing eating utensils, contaminated food or water. Hepatitis B is spread by infected blood or body fluids. 3. Incorrect: Hepatitis B is not airborne, therefore, there is no need to wear a mask. 4. Incorrect: Hepatitis B is not spread by sharing a bathroom. It is blood borne, not spread by the fecal route.)

Two hours post chest tube insertion, the nurse notes 100 mL of dark bloody drainage in the collection chamber of the closed drainage unit (CDU). What action should the nurse take? 1. Document the findings. 2. Notify the primary healthcare provider. 3. Decrease the amount of suction. 4. Use a padded hemostat to clamp the chest tube.

1. Document the findings. (1. Correct: A chest tube is inserted to remove air, blood, or exudate from the pleural space. So 100 mL of dark bloody drainage would not be unusual over the first two hours after insertion. Drainage may range from 100 to 300 mL/hr within the first 2 hours. Documentation is required and continued hourly follow-up. 2. Incorrect: It is not necessary to notify the primary healthcare provider at this time. There is nothing in the stem to indicate that the client is in distress. 3. Incorrect: Suction does not need to be decreased or increased. You want just enough suction to have gentle continuous bubbling in the suction control chamber. 4. Incorrect: You only clamp the chest tube as a last resort and only long enough to find a leak in the system. Clamping a chest tube is dangerous and can lead to a tension pneumothorax.)

A nurse is caring for a nonambulatory client who must be decontaminated after a chemical exposure event. What nursing action will prevent further chemical exposure? 1. Don appropriate personal protective equipment (PPE). 2. Remove only contaminated clothes. 3. Avoid decontaminating the eyes. 4. Use hot water during decontamination.

1. Don appropriate personal protective equipment (PPE). (1. Correct: PPE should be donned prior to contact with the client to prevent contamination of the healthcare worker. The nurse must protect themselves from exposure of the chemical. 2. Incorrect: All clothes, jewelry, and personal belongings should be removed and placed into appropriate containers. There is no way to be certain which articles of clothing are contaminated and which are not. 3. Incorrect: Decontaminate the eyes with a saline solution via nasal cannula or Morgan lens. Preventing cornea damage is very important. 4. Incorrect: Hot water is unnecessary unless the client is hypothermic during decontamination procedures. Hot water causes vasodilation.)

In caring for a client exposed to radiation, the nurse knows that the type of damage due to radiation exposure depends on which factors? Select all that apply 1. Dose rate. 2. Organs exposed. 3. Type of tumor being treated. 4. Presence of metastatic disease. 5. Type of radiation.

1. Dose rate. 2. Organs exposed. 5. Type of radiation. (1., 2. & 5. Correct: The extent of damage due to radiation exposure depends on the quantity of radiation delivered to the body, the dose rate, the organs exposed, the type of radiation, the duration of exposure, and the energy transfer from the radioactive wave or particle to the exposed tissue. 3. Incorrect: The type of tumor being treated is important to know, but this will not affect the type of damage the client receives from the radiation. 4. Incorrect: The client may be receiving radiation therapy for palliative treatment. Damage to the client due to the radiation exposure will not increase or decrease due to the metastatic disease.)

The nurse recognizes which manifestation as a sign/symptom of Hodgkin's lymphoma? Select all that apply 1. Drenching night sweats 2. Small, red, itchy bumps 3. Painful lymph nodes in the neck 4. Weight loss of 2 kg in 1 week 5. Flushed skin 6. Enlarged spleen

1. Drenching night sweats 2. Small, red, itchy bumps 6. Enlarged spleen (1., 2., & 6. Correct: With this disease, the body switches back and forth from fever and chills to excessive sweating. The sweating is the worst at nighttime; and many clients report waking up in the middle of the night to clothing and bedding that is completely drenched. As the lymphoma cells grow, they secrete a chemical that causes a generalized itchiness and irritation of the skin throughout the body. The spleen is part of the lymph system and works as a drainage network that defends the body against infection. Since Hodgkin's lymphoma affects the lymphatic system, the spleen is also affected. 3. Incorrect: The most common symptom of Hodgkin lymphoma is one or more enlarged lymph nodes. The enlarged lymph node may be in the neck, upper chest, armpit, abdomen or groin. The swollen lymph node is usually painless. 4. Incorrect: This is not a significant weight loss. A large decrease in weight is common in many types of cancers and is particularly noticeable in lymphoma cases. 5. Incorrect: Lymphoma lowers the body's red blood cell count, leading to anemia and even greater fatigue. Skin will be pale.)

Which signs/symptoms does the nurse expect to see in a client diagnosed with Bell's Palsy? Select all that apply 1. Drooping of one side of the face. 2. Inability to wrinkle forehead. 3. Excessive tearing. 4. Decreased sensitivity to sound. 5. Inability to taste. 6. Numbness of affected side of face.

1. Drooping of one side of the face. 2. Inability to wrinkle forehead. 3. Excessive tearing. 5. Inability to taste. 6. Numbness of affected side of face. (1., 2., 3., 5., & 6. Correct: Symptoms of Bell's Palsy include sudden weakness or paralysis on one side of the face that causes it to droop (main symptom), drooling, eye problems (such as excessive tearing or a dry eye), loss of ability to taste, pain in or behind ear, numbness in the affected side of face, increased sensitivity to sound. 4. Incorrect: There would be increased sensitivity to sound with Bell's Palsy.)

A nurse suspects that a client admitted to the emergency department is in diabetic ketoacidosis. What data would lead the nurse to this conclusion? Select all that apply 1. Dry mucous membranes 2. Fruity-smelling breath 3. Biot's respirations 4. Glycosuria 5. Client report of abdominal pain

1. Dry mucous membranes 2. Fruity-smelling breath 4. Glycosuria 5. Client report of abdominal pain (1., 2., 4., & 5. Correct: The client with diabetic ketoacidosis will have signs of dehydration due to polyuria and includes dry mucous membranes. Fruity breath odor is from the acetone that occurs with breakdown of fats and formation of ketones, which are acids.. With DKA, the client would be spilling glucose into the urine. Vomiting and abdominal pain are frequently the presenting symptoms of DKA. 3. Incorrect: The client will have Kussmaul respirations. Biot's respiration is a respiratory pattern characterized by periods of rapid respirations, then apnea periods. These are not the type of respirations that occur with diabetic ketoacidosis (metabolic acidosis).)

During the admission examination of a client diagnosed with acute pyelonephritis, what signs or symptoms would the nurse expect to find? Select all that apply 1. Dysuria 2. Costovertebral angle tenderness 3. Weight loss 4. Chills 5. Urinary frequency

1. Dysuria 2. Costovertebral angle tenderness 4. Chills 5. Urinary frequency (1., 2., 4., & 5. Correct: The client with acute pyelonephritis, will often exhibit these signs/symptoms due to the kidney infection. 3. Incorrect: Weight loss is not a symptom of acute pyelonephritis. The client will more likely experience weight gain due to the decreased functioning of the kidneys.)

A nurse is caring for a client with a suspected myocardial infarction (MI). What lab work or diagnostics should the nurse anticipate the primary healthcare provider prescribing to specifically confirm the diagnosis? Select all that apply 1. ECG 2. Troponin Level 3. PTT 4. Metabolic Panel 5. CPK-MB 6. CPK-BB

1. ECG 2. Troponin Level 5. CPK-MB (1., 2., & 5. Correct: Yes, when a client is suspected of having an MI, the client needs an ECG, Troponin, and CPK-MB levels. Remember Troponin is our favorite, because Troponin will confirm an MI even when the client delays seeking care. CPK-MB is right because CPK-MB is cardiac specific. 3.Incorrect: No, PTT looks at clotting factors and does not tell you if the client is having an MI. 4 Incorrect: A metabolic panel will tell you about metabolism and that is not the concern. 6.Incorrect: No, CPK-BB is used to assess for brain damage, not cardiac damage.)

Which assessment findings would the nurse expect to see in a client diagnosed with idiopathic thrombocytopenic purpura (ITP)? Select all that apply 1. Ecchymosis 2. Bleeding gums 3. Palpable spleen 4. Pain 5. Petechiae

1. Ecchymosis 2. Bleeding gums 3. Palpable spleen 5. Petechiae (1., 2., 3., & 5. Correct: The word thrombocytopenia means low platelets. Any client with low platelets is at risk for bleeding, which is indicated by ecchymosis (bruising), bleeding gums, and petechiae (red to purple dots on the skin, 1-3 mm in size). Spleen and liver are often slightly palpable. 4. Incorrect: Pain is not associated with ITP unless there are other associated problems. However, the stem of the question gave no indication that other problems exist.)

What signs/symptoms would the nurse expect to find in a client diagnosed with late stage rheumatoid arthritis? Select all that apply 1. Effusion to knees. 2. Weight loss of 1 kg in 2 weeks. 3. Swan neck deformity. 4. Peripheral neuropathy. 5. Subcutaneous nodules on elbows.

1. Effusion to knees. 3. Swan neck deformity. 4. Peripheral neuropathy. 5. Subcutaneous nodules on elbows. (1., 3., 4., & 5. Correct: As rheumatoid arthritis worsens, the joints become progressively inflamed and very painful. On palpation, these joints feel soft and look puffy because of synovitis and effusions, especially in the knees. Swan-neck deformity is a bending in (flexion) of the base of the finger, a straightening out (extension) of the middle joint, and a bending in (flexion) of the outermost joint. Peripheral neuropathy occurs in later stages of the disease due to vasculitis. Subcutaneous nodules or rheumatoid nodules are firm bumps of tissue most commonly form around pressure points, such as the elbows. 2. Incorrect: A 1 kg weight loss over 2 weeks would more likely occur in the early stages of the disease. As the disease progresses, there is moderate to severe weight loss and accompanying anemia.)

The son of an elderly diabetic client reports that his mother is frequently having low blood sugar. What should the nurse teach this family member about symptoms of hypoglycemia in the elderly? Select all that apply 1. Elders may not be aware that blood sugar is dropping due to decreased release of epinephrine in response to the lowered blood sugar. 2. Suggest that the client and family check with primary healthcare provider to ensure that the medication prescribed has low incidence of hypoglycemic episodes. 3. Symptoms of hypoglycemia may be averted if the client maintains routines and regular meal schedules. 4. Stress the importance of proper foot care and regular eye exams. 5. Check blood glucose levels if client becomes unsteady, has difficulty concentrating, or is tremulous.

1. Elders may not be aware that blood sugar is dropping due to decreased release of epinephrine in response to the lowered blood sugar. 2. Suggest that the client and family check with primary healthcare provider to ensure that the medication prescribed has low incidence of hypoglycemic episodes. 3. Symptoms of hypoglycemia may be averted if the client maintains routines and regular meal schedules. 5. Check blood glucose levels if client becomes unsteady, has difficulty concentrating, or is tremulous. (1., 2., 3. & 5. Correct: Older clients are at risk for hypoglycemia unawareness. Blood sugar levels should be checked frequently. Some oral medications are more likely to cause hypoglycemia episodes. If the client has frequent episodes, perhaps a medication change is warranted. The elderly must maintain regular meal schedules and adequate food intake. This may present challenges for the elder who lives alone. If an elder develops unsteady gait, loss of concentration, and/or lightheadedness, the blood glucose levels should be checked. These symptoms are typical in a hypoglycemic episode. 4. Incorrect: Proper foot care and regular eye exams should be done to avoid complications caused by hyperglycemia, not hypoglycemia.)

A client presents in the emergency department with acute onset of fever, headache, stiff neck, nausea/vomiting, and mental status changes. What interventions should the nurse initiate? Select all that apply 1. Elevate HOB 30 degrees 2. Pad side rails 3. Provide sponge bath if temperature greater than 101°F (38.3°C) 4. Initiate airborne isolation precautions 5. Darken room

1. Elevate HOB 30 degrees 2. Pad side rails 3. Provide sponge bath if temperature greater than 101°F (38.3°C) 5. Darken room (1., 2., 3. & 5. Correct: An acute onset of fever, headache, stiff neck, n/v, and mental status changes are consistent with bacterial meningitis. Elevate the head of the bed to promote comfort and decrease intracranial pressure. The client is at an increased risk for seizures, and the nurse should implement seizure precautions which include padding the side rails. A sponge bath is an independent nursing intervention appropriate for a fever greater than 101°F (38.3°C). Darkening the room is also a comfort measure as this client will have photophobia. 4. Incorrect: Droplet precautions should be initiated for the first 24 hours of antimicrobial therapy.)

Which nursing interventions should the nurse initiate for a client post-thoracotomy with two chest tubes in place connected to a chest drainage unit (CDU)? Select all that apply 1. Elevate head of bed 45 degrees. 2. Educate on use of incentive spirometry. 3. Support chest incision with pillow when client coughs. 4. Document amount and color of drainage. 5. Notify the primary healthcare provider if tidaling is noted in the water-seal chamber.

1. Elevate head of bed 45 degrees. 2. Educate on use of incentive spirometry. 3. Support chest incision with pillow when client coughs. 4. Document amount and color of drainage. (1., 2., 3., & 4. Correct: Elevate HOB to promote breathing. Educate on incentive spirometry to promote lung expansion. Supporting chest incision decreases pain and can help client produce a productive cough and can improve deep breathing efforts. Assessing for excessive bleeding is important. Keep sterile water at bedside in case the tubing becomes disconnected. 5. Incorrect: Tidaling—fluctuations in the water-seal chamber with respiratory effort—is normal. The water level increases during spontaneous inspiration and decreases with expiration.)

A client has been admitted with advanced Cirrhosis. The nurse's assessment of the abdominal girth verifies an increase in 5 inches (12.7 cm) and an increase in 6 lbs. (2.72 kg) since yesterday's measurements. Which interventions would the nurse expect to see in this client's plan of care? Select all that apply 1. Elevate head of bed to a semi-fowlers position. 2. Monitor the color of urine and stools. 3. Turn every 2 hours. 4. Instruct about a 1200 calorie diet. 5. Monitor creatinine levels daily.

1. Elevate head of bed to a semi-fowlers position. 2. Monitor the color of urine and stools. 3. Turn every 2 hours. 5. Monitor creatinine levels daily. (1., 2., 3., & 5. Correct: The client needs to have the head of the bed elevated in order to relieve the pressure of ascites off of the diaphragm. The client with ascites is in a fluid volume deficit (FVD) and has the risk for postural hypotension and falls. It is important to monitor for jaundice. When jaundice is present the urine may be dark brown and the stool light gray to tan color. The distended tissue with ascites is fragile and can breakdown. Remember that the problem is the loss of protein into peritoneal cavity. Protein is necessary for tissue repair. This lab level would indicate renal function that can occur due to shock. 4. Incorrect: Diet instructions should focus on low salt, not low calories.)

The nurse is monitoring the infection risk in a client that is to begin chemotherapy. Which activity should alert the nurse that the client is at a higher risk for infection? Select all that apply 1. Enjoys getting manicures and pedicures every two weeks. 2. Loves to go with the children to the local water park. 3. Relaxes in hot tubs when traveling. 4. Selects steamed vegetables as part of routine dietary intake. 5. Prefers to go barefooted when at home. 6. Keeps cats in the home and cleans the litter boxes once a week.

1. Enjoys getting manicures and pedicures every two weeks. 2. Loves to go with the children to the local water park. 3. Relaxes in hot tubs when traveling. 5. Prefers to go barefooted when at home. 6. Keeps cats in the home and cleans the litter boxes once a week. (1., 2., 3., 5., & 6. Correct: Infection is one of the most common life-threatening complications associated with cancer and chemotherapy. You know that both the cancer and chemotherapy weakens the immune system. Therefore, clients on chemotherapy should be familiar with activities that should be avoided due to the risk of infection with the immunosuppressed state. There are several things that are known to increase the risk of infection in these clients. Did you pick up on these? Well, let's look at a few of these. Clients on chemo should not get manicures or pedicures at salons or spas and should avoid having false nails or nail tips applied. There is too great a risk of contamination at the public salons, so clients are encourage to use their own personal and well-cleaned tools for nail care at home. Another source of bacterial contamination is public water parks. Although these parks take measures to reduce the risk of infection to the general public, the risk is too great for a client on chemo. Swimming can result in accidental ingestion of water which increases the risk of cryptosporidium or other waterborne pathogens. Same thing applies to hot tubs. ​So why is going barefoot at home such a big deal? Well, this increases the risk of cuts, scrapes, or other injury that would increase the portal for infectious agents to enter. In addition, the exposure to potential infectious agents is greater. The oncologist may direct the client in the best way to deal with this client having cats in the home and cleaning the litter box due to the risk of exposure to bacteria and parasites. If allowed to clean the litter box, latex or rubber gloves, along with a mask over the nose and mouth is generally recommended to reduce the risk of infection. In addition, the client should be instructed to thoroughly wash the hands with soap and water after cleaning the litter box or after touching the cats. 4. Incorrect: Although the intake of fresh fruits and vegetables has been controversial, most agree that if washed properly, even fresh fruits and vegetables can be consumed. However, the oncologist should be the one to approve the dietary intake of these. Here, we have vegetables that have not only been washed, but steamed as well. These should be safe for consumption for clients on chemo.)

A male client diagnosed with primary hyperaldosteronism is receiving spironolactone. Which potential side effect should the nurse educate the client regarding? Select all that apply 1. Erectile dysfunction 2. Gastrointestinal upset 3. Gynecomastia 4. Hypernatremia 5. Hypokalemia

1. Erectile dysfunction 2. Gastrointestinal upset 3. Gynecomastia (1., 2., & 3. Correct: Spironolactone blocks androgen and progesterone receptors and may inhibit the action of these hormones. Side effects can include gynecomastia, decreased sexual desire, impotence, menstrual irregularities, and gastrointestinal distress. 4. Incorrect: Hyponatremia, rather than hypernatremia, may be seen. 5. Incorrect: Hyperkalemia, rather than hypokalemia, may be seen.)

A nurse is developing a proposal to implement a pet therapy program at a nursing home. What information should the nurse include in the proposal to support this program? Select all that apply 1. Evidence has shown that animals can directly influence a person's mental and physical well-being. 2. Bringing a pet into a nursing home for the elderly has been shown to enhance social interaction. 3. Petting an animal can be helpful in lowering a client's blood pressure. 4. Some researchers believe that animals actually may retard the aging process among those who live alone. 5. Nursing home clients are more submissive after petting an animal.

1. Evidence has shown that animals can directly influence a person's mental and physical well-being. 2. Bringing a pet into a nursing home for the elderly has been shown to enhance social interaction. 3. Petting an animal can be helpful in lowering a client's blood pressure. 4. Some researchers believe that animals actually may retard the aging process among those who live alone. (1., 2., 3. & 4. Correct: All of these statements are correct in reference to pet therapy programs. Petting a dog or cat has been shown to lower blood pressure. Studies indicate a 7 mm Hg drop in systolic and an 8 mm Hg decrease in diastolic BP when volunteers talked to or would pet their dogs as opposed to reading aloud or resting quietly. 5. Incorrect: The clients are not more submissive or passive after participating in pet therapy. Evidence does show increased mental and physical well being with pet therapy.)

A client who needs to have a stool specimen for an occult blood test is instructed by the nurse to avoid which substances two hours prior to testing? Select all that apply 1. Liver 2. Tomato 3. Ibuprofen 4. Sardines 5. Ascorbic acid

1. Liver 3. Ibuprofen 4. Sardines 5. Ascorbic acid (1., 3., 4., 5. Correct: The following foods can cause a false-positive reading: red meats, liver, turnips, broccoli, cauliflower, melons, salmon, sardines, and horseradish. Medications altering the test include aspirin, ibuprofen, ascorbic acid, indomethacin, colchicines, corticosteroids, cancer chemotherapeutic agents, and anticoagulants. Ingestion of vitamin rich foods can cause a false negative result. 2. Incorrect: A tomato is not on the food list for false-positive reading and do not have to be avoided.)

A client with distended and tortuous veins along the inner aspects of both legs asks the nurse how to decrease the development of these veins. What should the nurse advise? Select all that apply 1. Exercise 2. Follow a low protein diet 3. Wear low heeled shoes 4. Elevate legs above heart several times per day 5. Do not cross legs

1. Exercise 3. Wear low heeled shoes 4. Elevate legs above heart several times per day 5. Do not cross legs (1., 3., 4., & 5. Correct: These are varicose veins. Get moving. Walking is a great way to encourage blood circulation to the legs. Low-heeled shoes work calf muscles more, which is better for veins. To improve circulation in legs, take several short breaks daily to elevate legs above the level of the heart. Do not cross legs as it decreases circulation distally. 2. Incorrect: Low sodium diet will prevent swelling caused from water retention. A diet low in protein will not decrease the development of these veins. The key is to keep swelling down so that pressure on the veins is reduced.)

A client with Hepatitis C has returned from surgery with a total laryngectomy. The nurse knows that what personal protective equipment is necessary when providing trach care? Select all that apply 1. Face mask 2. Shoe covers 3. N-95 mask 4. Goggles 5. Gloves 6. Gown

1. Face mask 4. Goggles 5. Gloves 6. Gown (1, 4, 5 and 6. Correct: The client has had a total laryngectomy which will initially produce large amounts of thick, bloody mucus. Hepatitis C is transmitted through blood and body fluids. During trach care, the nurse needs to be protected by specific personal protective equipment (PPE's). For this procedure, the nurse should utilize gown, gloves, goggles and face mask. 2. Incorrect: Tracheostomy care is completed in close proximity to the client. Splattering of blood and body fluids on the floor is unlikely, so shoe covers are unnecessary. 3. Incorrect: The N-95 face mask is a specially fitted mask used by nurses when providing care for clients with active tuberculosis. It is not necessary for a client with Hepatitis C.)

A nurse is completing a nursing history on a client admitted with peripheral vascular disease (PVD). Which data from the nursing history should the nurse identify as contributing to this diagnosis? Select all that apply 1. Family history of hyperlipidemia 2. Postmenopausal 3. BMI of 24 4. Swims three times a week 5. Leg pain when walking

1. Family history of hyperlipidemia 2. Postmenopausal 5. Leg pain when walking (1., 2 & 5. Correct: A family history of hyperlipidemia, hypertension, or PVD increases the risk of a client developing PVD as well. Men over age 50 and postmenopausal women are at increased risk. A decline in the natural hormone estrogen may be a factor in heart disease increase among post-menopausal women. Estrogen is believed to have a positive effect on the inner layer of artery wall, helping to keep blood vessels flexible. Developing PVD risk also increases if the client has hyperlipidemia, cerebrovascular disease, heart disease, diabetes, hypertension, and/or renal failure. Leg pain with activity such as walking is a sign of PVD increases risk. 3. Incorrect: Overweight clients are at increased risk for PVD. A BMI of 24 means the client is of normal weight for height. Body mass index (BMI) is a measure of body fat based on height and weight that applies to adult men and women. Underweight = <18.5; Normal weight = 18.5-24.9; Overweight = 25-29.9; Obesity = BMI of 30 or greater. 4. Incorrect: Swimming three times per week is good exercise for the client. Sedentary life style increases the risk for development of PVD.)

Which signs and symptoms experienced by the client correlate with chronic renal failure diagnosis? Select all that apply 1. Fatigue 2. Anorexia 3. Dark skin pigmentation 4. Swollen extremities 5. Hyperkalemia

1. Fatigue 2. Anorexia 4. Swollen extremities 5. Hyperkalemia (1., 2., 4. & 5. Correct: The client will have fatigue from anemia and anorexia from toxins. Fluid volume excess leads to swollen extremities. Hyperkalemia can be caused by reduced renal excretion or excessive intake. 3. Incorrect: The client may have an uremic frost not dark skin pigmentation.)

What signs/symptoms would the nurse expect to assess in a client diagnosed with multiple sclerosis (MS)? Select all that apply 1. Fatigue 2. Ptosis 3. Blurry vision 4. Leg weakness 5. Limited facial expression 6. Electric shock sensation when bending neck forward

1. Fatigue 3. Blurry vision 4. Leg weakness 6. Electric shock sensation when bending neck forward (1., 3., 4., & 6. Correct: Multiple sclerosis causes fatigue which often comes on in the afternoon and causes weak muscles, slowed thinking, or sleepiness. Vision problems are common with this diagnosis and include blurry vision, double vision, and pain on eye movement. Partial or complete vision loss can occur in one eye. Because this disease affects nerves, symptoms often affect movement such as extremity weakness, numbness, tingling, and coordination. Electric-shock sensations that occur with certain neck movements, especially bending the neck forward (Lhermitte sign) develop because of the nerve damage that is occurring. 2. Incorrect: Drooping of one or both eyelids (ptosis) would be seen in myasthenia gravis rather than multiple sclerosis. 5. Incorrect: Limited facial expressions occur in myasthenia gravis rather than multiple sclerosis. The muscles (not nerves) that control facial expressions have been affected.)

A client has been admitted with a diagnosis of pneumocystis carinii pneumonia (PCP). What initial assessment findings would the nurse expect? Select all that apply 1. Fever 2. Night sweats 3. Hemoptysis 4. Dry cough 5. Dyspnea

1. Fever 4. Dry cough 5. Dyspnea (1, 4 and 5. Correct: Pneumocystis carinii pneumonia, now known as pneumocystis jirovecii, is caused by a fungus and occurs in clients with weakened immune systems. Expected assessment findings include fever, dry non-productive cough and dyspnea. Any additional symptoms are related to other co-morbidities and not the pneumonia itself. 2. Incorrect: Night sweats are an early symptom of active tuberculosis and are often the definitive symptom, along with a productive cough, that indicates the need for immediate testing and isolation. 3. Incorrect: Hemoptysis is among the late signs of lung cancer, in addition to weight loss. Lung cancer is asymptomatic in its early stages.)

A client admitted with somnolence has a history of chronic bronchitis and heart failure. Vital signs on admit are T 101.8ºF (38.8ºC), HR 106, R 26/shallow, BP 90/58. ABGs are pH 7.2, PCO₂ 75, HCO₃⁻ 26. The nurse determines that this client has which acid/base imbalance? 1. Respiratory acidosis 2. Respiratory alkalosis 3. Metabolic acidosis 4. Metabolic alkalosis

1. Respiratory acidosis (1. Correct: This client has a respiratory problem. Respiratory failure, COPD, and muscular weakness can lead to respiratory acidosis. Signs & symptoms: hypoventilation, sensorium changes, somnolence, semicomatose to comatose state. pH < 7.35, pCO₂ > 45, HCO₃⁻ normal. 2. Incorrect: This is not alkalosis since the pH is 7.2 showing acidosis. 3. Incorrect: Not a metabolic related acid/base imbalance, because the HCO​₃⁻ is 26 and within the normal range. 4. Incorrect: Not a metabolic related acid/base imbalance, because the HCO​₃⁻​ is 26 and within the normal range.)

The nurse has been trained to work in a decontamination station for hazardous exposure victims. What should the nurse tell the victim about the process? Select all that apply 1. First you will remove clothing and dispose of it in hazardous material containment area. 2. You will be placed in a warm shower for decontamination. 3. You will spend a minute or so using soap over the entire body before rinsing. 4. You will spend approximately 15 minutes in the shower. 5. You will apply soap from head to toe and then rinse for a few minutes.

1. First you will remove clothing and dispose of it in hazardous material containment area. 2. You will be placed in a warm shower for decontamination. 3. You will spend a minute or so using soap over the entire body before rinsing. 5. You will apply soap from head to toe and then rinse for a few minutes. (1., 2., 3. & 5. Correct: If the victim can remove his/her own clothing, then instructions should be given to do so and dispose of in hazardous material container. The person will wash for several minutes, beginning with a minute or so of full body rinsing with water to remove any visible contaminants, followed by soap and finally the rinse. The length of time for washing and rinsing will vary with institution and known contaminants. Using soap with good surfactant qualities is important. Generally, the victim is instructed to rinse with tepid water, apply soap from head to toe, and then rinse again with copious amounts of water. 4. Incorrect: Most procedures require about 5 to 6 minutes for the decontamination process. Times may vary depending on policy, contaminants, and the level of ability of the victim.)

The nurse is preparing to educate a client diagnosed with essential hypertension on how to decrease the risk of developing complications. What topics should the nurse include? Select all that apply 1. Following the DASH dietary plan. 2. Use of blood pressure monitoring device. 3. Diaphragmatic breathing exercises. 4. Brisk walking for 30 minutes 3-4 times/week. 5. Reduce sodium intake to less than 2700 mg/day.

1. Following the DASH dietary plan. 2. Use of blood pressure monitoring device. 3. Diaphragmatic breathing exercises. 4. Brisk walking for 30 minutes 3-4 times/week. (1., 2., 3., & 4. Correct: The DASH Eating Plan is recommended for clients who have hypertension. It is high in vegetables, fruits, low-fat dairy products, whole grains, poultry, fish, beans, and nuts and is low in sweets, sugar-sweetened beverages, and red meats. Home blood pressure monitoring can help the client keep closer tabs on their blood pressure, show if medication is working, and even alert the client and primary healthcare provider to potential complications. However, home blood pressure monitoring isn't a substitute for follow-up visits. Reducing stress as much as possible helps to decrease blood pressure. Healthy coping techniques, such as muscle relaxation, deep breathing or meditation are good options. Getting regular physical activity and plenty of sleep can help, too. Regular physical activity can lower blood pressure, manage stress, reduce the risk of several health problems and keep weight under control. 5. Incorrect: A limit of 1500 mg of sodium per day is preferred on the low sodium DASH diet. On the standard DASH diet 2,300 mg of sodium are allowed each day.)

What signs and symptoms of ovarian cancer should a nurse include when educating women? Select all that apply 1. GI disturbances 2. Menstrual changes 3. Malnutrition 4. Increasing abdominal girth 5. Pain radiating down the legs

1. GI disturbances 2. Menstrual changes 4. Increasing abdominal girth (1., 2., & 4. Correct: Signs and symptoms of ovarian cancer include irregular menses, increasing premenstrual tension, menorrhagia with breast tenderness, early menopause, abdominal discomfort, dyspepsia, pelvic pressure and urinary frequency. Indigestion, flatulence, and fullness after a light meal, and increasing abdominal girth are significant symptoms. 3. Incorrect: Malnutrition is a complication of advanced metastatic cancer. 5. Incorrect: Pain will be in the abdomen and pelvis, and does not radiate down the legs.)

Which of the following should the nurse teach regarding nutrition for a client with celiac disease? Select all that apply 1. Gluten is a protein found in wheat and oats. 2. A gluten intolerant person can eat foods that are made with barley or rye. 3. Fruits can be eaten on a gluten free diet. 4. Gluten causes inflammation of the large intestines of people with celiac disease. 5. Accidentally eating a product containing gluten may result in abdominal pain and diarrhea.

1. Gluten is a protein found in wheat and oats. 3. Fruits can be eaten on a gluten free diet. 5. Accidentally eating a product containing gluten may result in abdominal pain and diarrhea. (1., 3., & 5. Correct: A gluten-free diet is used in the treatment of celiac disease. Gluten is a protein found in barley, oats, rye, and wheat. All products containing these grains are to be avoided. Rice and corn may be used. Fruits, vegetables, nuts, diary products and meats not prepared with gluten containing ingredients can be eaten. Accidentally ingesting food with gluten may result in abdominal pain and diarrhea. 2. Incorrect: The main starchy foods that a person can eat are made with rice, corn, potatoes, quinoa, and Tapioca. The gluten intolerant clients can not eat barley and rye. Gastrointestinal pain and diarrhea may occur. 4. Incorrect: Gluten causes inflammation in the small intestines of people with celiac disease. Eating a gluten-free diet helps people control their signs and symptoms and prevent complications.)

Which teaching points would the nurse include in a client's nutritional teaching plan to accomplish the goal of a gluten free diet? Select all that apply 1. Gluten is a protein found in wheat. 2. The main starchy foods that a person can eat are made with barley. 3. Fruits can be eaten on a gluten free diet. 4. Pure, non-contaminated oats are an acceptable grain food that the client can consume. 5. Accidentally eating food with gluten may result in abdominal pain and diarrhea.

1. Gluten is a protein found in wheat. 3. Fruits can be eaten on a gluten free diet. 4. Pure, non-contaminated oats are an acceptable grain food that the client can consume. 5. Accidentally eating food with gluten may result in abdominal pain and diarrhea. (1., 3., 4., & 5. Correct: A gluten-free diet is used in the treatment of celiac disease. Gluten is a protein found in barley, rye, and wheat. All products containing these grains are to be avoided. Rice and corn may be used. In addition, pure oats that have not been cross-contaminated with wheat, barley and/or rye can be safely consumed by most individuals with celiac disease or other gluten sensitivity. A reduction in the fiber content of the diet is also frequently recommended. Fruits, vegetables, nuts, diary products and meats not prepared with gluten containing ingredients can be eaten. Accidentally ingesting food with gluten may result in abdominal pain and diarrhea. Some people do not experience signs and symptoms, but this does not mean it is not damaging their small intestines. Even trace amounts of gluten may be damaging. 2. Incorrect: The main starchy foods that a person can eat are made with rice, corn, potatoes, quinoa, and Tapioca.)

A client with an automated internal cardiac defibrillator (AICD) was successfully defibrillated. The telemetry technician shouts out that the client was in ventricular fibrillation (VF). What should the nurse do first? 1. Go to the client to assess for signs and symptoms of decreased cardiac output. 2. Call the primary healthcare provider to report that the client had an episode of VF so medication adjustments can be made. 3. Notify the "on call" person in the cath lab to re-charge the ICD in the event that the client has a recurrence. 4. Document the incident on the code report form and follow up regularly.

1. Go to the client to assess for signs and symptoms of decreased cardiac output. (1. Correct: The client comes first. Check to see how they are doing by completing a head to toe cardiac output assessment. make sure to include LOC, vital signs, skin and urinary output assessment. 2. Incorrect: Do not call before you assess the client who may be unconscious if the arrhythmia has decreased their cardiac output. 3. Incorrect: This is not needed because there is a battery that keeps it charged, so that they don't have to re-charge after each shock. 4. Incorrect: Documentation is not appropriate until the client has been assessed first.)

A client who is at high risk for developing a stroke has been advised to follow a Mediterranean type diet by the primary healthcare provider. Which food choices, if selected by the client, would indicate to the nurse that the client understands this diet. Select all that apply 1. Grilled eggplant 2. Purple grape juice 3. Bacon 4. Cashews 5. Skim milk 6. Salmon

1. Grilled eggplant 2. Purple grape juice 4. Cashews 5. Skim milk 6. Salmon (1., 2., 4., 5., & 6. Correct: It is reasonable to counsel clients to follow a Mediterranean-type diet over a low-fat diet. The Mediterranean type diet emphasizes vegetables, fruits, and whole grains and includes low fat dairy products, poultry, fish, legumes, and nuts. It limits intake of sweets and red meats. 3. Incorrect: Substitute fish and poultry for red meat. When eaten, make sure it's lean and keep portions small (about the size of a deck of cards). Also avoid sausage, bacon and other high-fat meats.)

A client returns to the clinic two days after receiving treatment for diarrhea caused by a Campylobacter jejuni infection. The client reports a tingling sensation that began in the toes yesterday and has spread to the feet and legs today. The nurse notes muscle weakness in the legs and that the client is having difficulty walking steadily. Based on this data, what does the nurse suspect is wrong with the client? 1. Guillain-Barré Syndrome 2. Multiple Sclerosis 3. Myasthenia Gravis 4. Systemic Lupus Erythematosus

1. Guillain-Barré Syndrome (1. Correct: The clues in this stem are diarrhea from Campylobacter jejuni, tingling sensation that began in the toes, spread to the feet and legs today, muscle weakness in the legs, and difficulty walking steadily. These s/s point to Guillain-Barré Syndrome. 2. Incorrect: Multiple Sclerosis damages nerves but not in an ascending progression from toes to head. 3. Incorrect: Myasthenia gravis is caused by a breakdown in the normal communication between nerves and muscles. Myasthenia gravis is characterized by weakness and rapid fatigue of any of the muscles under voluntary control. 4. Incorrect: Systemic lupus erythematosus, the most common form of lupus, is a chronic autoimmune disease that can cause severe fatigue and joint pain.)

A home health nurse is interpreting Mantoux skin test results of clients who received the test 48 hours ago. Which clients have a positive tuberculin skin test reaction? Select all that apply 1. HIV+ client with an induration of 6 millimeters. 2. Client who immigrated from Haiti 6 months ago who has an induration of 10 millimeters. 3. Client working at a nursing home with an induration of 8 millimeters. 4. 3 year old client with an induration of 12 millimeters. 5. Healthy client with no known TB exposure who has an induration of 5 millimeters.

1. HIV+ client with an induration of 6 millimeters. 2. Client who immigrated from Haiti 6 months ago who has an induration of 10 millimeters. 4. 3 year old client with an induration of 12 millimeters. (1., 2. & 4. Correct: HIV infected clients are considered to have a (+) TB skin test with an induration of 5 millimeters or more. An induration of 10 millimeters or more is considered positive in recent immigrants (less than five years) from high-prevalence countries such as Haiti, and in children less than 4 years of age. 3. Incorrect: An induration of 10 millimeters or more is considered positive for residents and employees of high-risk congregate settings.5. Incorrect: An induration of 15 millimeters or more is considered positive in any person with no known risk factors for TB.)

The nurse plans to teach a client how to manage the use of a behind the ear hearing aid. What teaching strategies should the nurse include? Select all that apply 1. Hairspray should not be used while wearing the hearing aid. 2. A whistling sound when the hearing aid is inserted indicates proper placement. 3. Submerse hearing aid in cool water daily to clean. 4. Illustrate where damage commonly occurs on a hearing aid. 5. Batteries last 6 months with daily wearing of 10-12 hours.

1. Hairspray should not be used while wearing the hearing aid. 4. Illustrate where damage commonly occurs on a hearing aid. (1. & 4. Correct: The residual from the hair spray causes the hearing aid to become oily and greasy. The client should routinely inspect the hearing aid for damage, especially where damage is more likely: ear mold, earphone, dials, cord, and connection plugs. 2. Incorrect: A whistling sound indicates incorrect ear mold insertion, improper fit of aid, and buildup of earwax or fluid. 3. Incorrect: Do not submerse hearing aid in water, as it will damage the device. 5. Incorrect: Batteries last 1 week with daily wearing of 10-12 hours.)

What strategies for smoking prevention could the school nurse recommend to the community task force? Select all that apply 1. Have a "Pledge Campaign" asking students not to use tobacco. 2. Include effects of smoking in health classes. 3. Enlist help from celebrities who are against smoking. 4. Conduct a "Don't Smoke" poster contest aimed at seventh graders. 5. Start a smoking cessation class for students who currently smoke.

1. Have a "Pledge Campaign" asking students not to use tobacco. 2. Include effects of smoking in health classes. 3. Enlist help from celebrities who are against smoking. 4. Conduct a "Don't Smoke" poster contest aimed at seventh graders. (1., 2., 3., & 4. Correct: These are all activities that the nurse could recommend. All are primary prevention strategies that may educate and influence students to abstain from smoking. 5. Incorrect: This is a worthy activity; however, it is for students who are already smoking and wish to stop. This is not primary but tertiary prevention.)

Which client should the nurse see first after receiving report on assigned clients? 1. Having dyspnea after surgery. 2. Needing an IV started for the administration of blood. 3. Crying with pain after back surgery. 4. Vomiting dark brown, granular material.

1. Having dyspnea after surgery. (1. Correct: The client may be having a pulmonary embolism after surgery. This client with oxygenation needs takes priority over the other three clients. 2. Incorrect: Needing an IV started for blood administration does not take priority over oxygenation. If blood is needed, tissue perfusion could be altered, so this would need to be addressed in a timely manner after airway issues and other potentially deteriorating situations have been addressed. 3. Incorrect: Pain is expected after back surgery and is not a priority over oxygenation. When possible, the pain should be assessed and medications administered. Remember, pain never killed anyone. 4. Incorrect: This client with dark brown emesis may have an upper GI bleed that has slowed or stopped. This is the second client to see but is not a priority over oxygenation. This could potentially return to active GI bleeding and the client's condition could deteriorate rapidly, so the client would need to be seen following the client with dypnea.)

In what position should the nurse place a client post intracranial surgery? 1. Head of bed elevated 30 degrees 2. Supine 3. Dorsal recumbent 4. Recovery position

1. Head of bed elevated 30 degrees (1. Correct: The goal after intracranial surgery is to keep the intracranial pressure (ICP) from rising while optimizing the cerebral perfusion pressure (CPP). The ideal position for this client is HOB elevated and the head in neutral position. 2. Incorrect: Placing the client in supine position may increase ICP. Supine position is achieved when the client is lying flat. 3. Incorrect: Dorsal recumbent position will increase ICP as this position will increase peripheral return. The client in dorsal recumbent position is lying flat with the knees flexed and separated. 4. Incorrect: The recovery position is side lying position with one knee flexed. This position can also increase ICP.)

Which signs/symptoms should the nurse monitor for in a client admitted with a diagnosis of pheochromocytoma? Select all that apply 1. Headache 2. Hypotension 3. Hyperglycemia 4. Bradycardia 5. Polycythemia 6. Leukopenia

1. Headache 3. Hyperglycemia (1., & 3. Correct: Pheochromocytoma is a rare, catecholamine-secreting tumor that may precipitate life-threatening hypertension. The tumor is malignant in 10% of cases but may be cured completely by surgical removal. This disease is characterized by hypertension, hypermetabolism, hyperglycemia, and headache due to increased release of epinephrine and norepinephrine. 2. Incorrect: Hypertension, rather than hypotension, would be seen in this client. 4. Incorrect: The heart rate will increase rather than decrease. 5. Incorrect: Polycythemia is elevated red blood cell count, which is not seen with this disease. 6. Incorrect: Leukopenia is a low white blood cell count, which is not seen with this disease.)

A client's central venous pressure (CVP) reading has changed significantly from the last hourly reading. Which data would the nurse assess that reflect changes in the CVP reading? Select all that apply 1. Heart sounds 2. Skin turgor 3. Temperature 4. Nail bed color 5. EKG rhythm 6. Urinary output

1. Heart sounds 2. Skin turgor 6. Urinary output (1., 2. & 6. Correct: The CVP reading reflects the client's fluid volume status. If the CVP is elevated, indicating FVE, then the nurse is likely to hear S3 sounds when auscultating the heart sounds. The client's skin turgor and urine output would reflect the client's fluid volume status. 3. Incorrect: The CVP reading reflects the client's fluid volume status. The client's temperature would not reflect the client's fluid volume status. 4. Incorrect: The CVP reading reflects the client's fluid volume status. The nail bed color would not reflect the client's fluid volume status. 5. Incorrect: The CVP reading reflects the client's fluid volume status. The EKG rhythm would not reflect the client's fluid volume status.)

What lab values should the nurse monitor when caring for a client diagnosed with acute leukemia? Select all that apply 1. Hemoglobin 2. Hematocrit 3. Lactate dehydrogenase (LDH) 4. Platelets 5. White blood cells 6. Metanephrine

1. Hemoglobin 2. Hematocrit 4. Platelets 5. White blood cells (1., 2., 4., & 5 Correct: The client with acute leukemia usually has a decreased hemoglobin and hematocrit level, a low platelet count, and an abnormal white blood cell count. 3. Incorrect: If lymphoma has been diagnosed, the lactate dehydrogenase (LDH) level may be checked. LDH levels are often increased in patients with lymphomas. LDH is not a lab test monitored in the client with acute leukemia. 6. Incorrect: Plasma free metanephrine is up to 99% sensitive for diagnosing pheochromocytoma. It measures circulating epinephrine and norepinephrine levels. Grossly elevated plasma norepinephrine renders the diagnosis of pheochromocytoma highly probable.)

When assessing for the development of an infection following the application of a plaster cast to the leg, the nurse should teach the client to observe for the presence of which sign of infection? 1. Hot spots 2. Cold toes 3. Warm toes 4. Paresthesia

1. Hot spots (1. Correct: Hot spots is the best answer. Redness and increased warmth are indicators of localized infection. If the cast covers the extremity, redness cannot be visualized, but the client can feel more warmth (a "hot spot") in an area becoming infected. 2. Incorrect: "Cold toes" is a neurovascular check, not an indication of infection. 3. Incorrect: "Warm toes" is a neurovascular check, not an indication of infection. 4. Incorrect: Paresthesia is a neurovascular check, not an indication of infection.)

A client has been admitted with advanced Cirrhosis. The nurse's assessment reveals an abdominal girth increase of 5 inches (12.7 cm) and a weight increase of 6 lbs.(2.71 kg) since yesterday's measurements. What further assessment findings would the nurse expect? Select all that apply 1. Hypotension 2. Cool extremities 3. Bradycardia 4. CVP readng of 8 mm/Hg 5. Radial pulses 4+/4+

1. Hypotension 2. Cool extremities (1. & 2. Correct: These are signs and symptoms of FVD due to 3rd spacing and shock is what you are afraid of. 3. Incorrect: We would expect the heart rate to increase in FVD in an effort to move what little volume you have left through the system. 4. Incorrect: This is a high CVP, and with FVD you would expect it to be low. 5. Incorrect: Pulses are evaluated on a 4 point scale, so 4 would be a bounding pulse which would indicate fluid volume excess.)

A nurse is planning a teaching session for a group of clients diagnosed with irritable bowel syndrome. What points should the nurse include to help the clients control symptom flare-ups? Select all that apply 1. If you are constipated, try to make sure you have breakfast. 2. Avoid low fat foods. 3. If you think a certain food is a problem, try cutting it out of your diet for about 12 weeks. 4. Drinks containing caffeine are likely to contribute to symptoms. 5. Foods such as broccoli and cabbage are good sources of fiber.

1. If you are constipated, try to make sure you have breakfast. 3. If you think a certain food is a problem, try cutting it out of your diet for about 12 weeks. 4. Drinks containing caffeine are likely to contribute to symptoms. (1., 3. & 4. Correct: If you are constipated, try to make sure you eat breakfast, as this is the meal that is most likely to stimulate the colon and give you a bowel movement. If you think a certain food is a problem, try cutting it out of your diet for about 12 weeks. (If you suspect more than one, cut out one at a time so you know which one causes you problems.) If there's no change, go back to eating it. The foods most likely to cause problems are: Insoluble (cereal) fiber; Coffee/caffeine; Chocolate; Nuts. 2. Incorrect: Avoid meals that over-stimulate the gut, like large meals or high fat foods. 5. Incorrect: Broccoli and cabbage are common gas-producing foods that can cause abdominal distention and flatulence.)

Which information should the community health nurse include when explaining to a group of college students living in a dormitory about receiving an immune globulin (IG) injection for hepatitis A virus (HAV)? Select all that apply 1. Immune globulin contains antibodies that destroy the HAV, preventing infection. 2. Immune globulin protection is permanent, so no other injection is required. 3. Common side effects of the injection include soreness and swelling around the injection site. 4. The sooner you get a shot of IG after being exposed to HAV, the greater the likelihood of protection from the virus. 5. Crowded living environments such as dormitories place people at risk for HAV.

1. Immune globulin contains antibodies that destroy the HAV, preventing infection. 3. Common side effects of the injection include soreness and swelling around the injection site. 4. The sooner you get a shot of IG after being exposed to HAV, the greater the likelihood of protection from the virus. 5. Crowded living environments such as dormitories place people at risk for HAV. (1., 3., 4. & 5. Correct: These are all correct statements about immune globulin for Hepatitis A. The client gets the antibodies immediately. Most injections cause some discomfort and swelling at the site. Antibodies are received immediately with passive immunity, so the client has what it needs to fight off this infection at once. The most common method of transmission for Hepatitis A is through the fecal/oral route. People living in crowded, confined places are at a higher risk of contracting Hepatitis A. 2. Incorrect: IG protection is only temporary, lasting about 3 months.)

A client with acute pancreatitis is prescribed total parenteral nutrition (TPN), methylprednisolone, and sliding scale insulin. What is the rationale for the insulin prescribed? Select all that apply 1. Impaired endocrine function of the pancreas 2. Inability of the liver to convert glucose 3. Steroid therapy side effects 4. Dextrose concentration of TPN 5. Re-establish serum potassium level

1. Impaired endocrine function of the pancreas 3. Steroid therapy side effects 4. Dextrose concentration of TPN (1., 3., & 4. Correct: Really what we are saying here is why would the client be on insulin? Well, the pancreas is damaged and so the endocrine function of the pancreas is impaired. We know that hyperglycemia or pseudo diabetes is a side effect of steroid therapy and TPN is high in glucose and may require additional insulin. These are the three rationales for why they might need insulin. 2. Incorrect: If the liver can't convert glucose that will decrease the insulin need so that one is false. 5. Incorrect: Is not related at all. The NCLEX people want you to say,"I remember something about potassium and glucose, but I am not sure what. Don't fall for that, this is false. The rationale for the Insulin order is not to re-establish potassium in this question.)

What is the most important action for the nurse to take in order to decrease an adverse drug reaction/interaction in an elderly client who takes multiple medications? 1. Implementing a thorough client assessment. 2. Instructing the client about adverse drugs reactions. 3. Explaining to the client that hospital admissions of older adults are often due to a drug reaction. 4. Teaching the client that adverse reactions are directly proportional to the number of medications taken.

1. Implementing a thorough client assessment. (1. Correct: To prevent complications of medication administration, such as adverse drug reactions and interactions, careful planning is priority. A thorough assessment of the client is vital when planning care. 2. Incorrect: Instructing the client about adverse drug reactions is a true statement that supports client education, but not more important than thorough client assessment. 3. Incorrect: Explaining the prevalence of drug reactions in the elderly is a true statement that supports client education, but not more important than thorough client assessment. 4. Incorrect: Teaching the client that risk increases with the number of medications taken is a true statement that supports client education, but not more important than thorough client assessment.)

Which signs/symptoms does the nurse expect to note when caring for a client with a suspected cystitis? Select all that apply 1. Incontinence 2. Urgency 3. Frequency 4. Hematuria 5. Nocturia 6. Flank pain

1. Incontinence 2. Urgency 3. Frequency 4. Hematuria 5. Nocturia (1., 2., 3., 4. & 5. Correct: Signs and symptoms of cystitis include burning on urination, nocturia, incontinence, suprapubic or pelvic pain, hematuria, and back pain. 6. Incorrect: Flank pain is seen when the urinary tract infection progresses to the kidneys.)

A community health nurse is planning to teach a group of caregivers about early warning signs of Alzheimer's Disease (AD). What signs should the nurse include? Select all that apply 1. Mild disorientation 2. Difficulty with words and numbers 3. Poor personal hygiene 4. Agitation 5. Visual agnosia 6. Dysgraphia

1. Mild disorientation 2. Difficulty with words and numbers (1. & 2. Correct: Early warning signs of Alzheimer's Disease include mild disorientation and difficulty with words and numbers. This client may have difficulty recognizing numbers or doing basic calculations. The person may begin to have trouble with words. 3. Incorrect: Poor personal hygiene occurs as Alzheimer's Disease progresses due to ongoing loss of neurons. 4. Incorrect: Behavioral manifestations occur later in the disease process as a result of changes that take place within the brain. They are not intentional or controllable by the person with this disease. 5. Incorrect: With progression of this disease, additional cognitive impairments are noted, including visual agnosia, which is the inability to recognize objects by sight. 6. Incorrect: Dysgraphia is defined as difficulty communicating via writing and occurs during disease progression.)

Which vaccines would a nurse participating at a health fair encourage a 65 year-old adult to receive? Select all that apply 1. Influenza 2. Herpes Zoster 3. Diphtheria 4. Pertussis 5. Pneumococcal vaccine 6. Measles, mumps, and rubella (MMR)

1. Influenza 2. Herpes Zoster 3. Diphtheria 4. Pertussis 5. Pneumococcal vaccine (1., 2., 3., 4., & 5. Correct: Influenza is often quite serious for people 65 and older due to weaker immune defenses. CDC recommends a single dose of herpes zoster vaccine for people 60 years of age or older to prevent shingles.Tetanus, diptheria and pertussis (Tdap) vaccine is given to older adults to protect against whooping cough (pertussis), tetanus and diptheria. Adults should get one dose of the tetanus and diptheria (Td) vaccine every 10 years. For adults who did not get Tdap as a preteen or teen, they should get one dose of Tdap in place of a Td dose to boost protection against whooping cough. However, adults who need protection against whooping cough can get Tdap at anytime, regardless of when they last got Td. Pneumococcal vaccines protect against infections in the lungs and blood stream and are recommended for all adults over 65 years old and for adults younger than 65 who have certain chronic health conditions. 6. Incorrect: A booster for measles, mumps, and rubella is not indicated for this age group.)

A client reports crushing chest pain 3 hours prior to arrival in the emergency department. Initial assessment by the nurse reveals a BP of 90/50, a weak, thready pulse at 108/min, cool, clammy skin, and confusion. Which interventions should the nurse perform? Select all that apply 1. Initiate cardiac monitoring. 2. Monitor intake and output hourly. 3. Position client in recumbant position. 4. Limit physical activity. 5. Administer dopamine at 5 micrograms/kg/min.

1. Initiate cardiac monitoring. 2. Monitor intake and output hourly. 4. Limit physical activity. 5. Administer dopamine at 5 micrograms/kg/min. (1., 2., 4. & 5. Correct: This client exhibits signs of cardiogenic shock, a complication of myocardial infarction. Hypotension accompanied by clinical signs of increased peripheral resistance (weak, thready pulse and cool, clammy skin) and inadequate organ perfusion (altered mental status and decreased urinary output) are found in this client. Initiate cardiac monitoring, watching for dysrhythmias, monitor I&O hourly to make sure kidneys are perfused. Limit activity to decrease oxygen demand. Dopamine is administered to increase BP and cardiac output. 3. Incorrect: Position upright to promote optimal ventilation by reducing venous return and lessen pulmonary edema.)

An elderly, bed-bound client receiving G-tube feeding at home is transported to the emergency department after onset of behavioral changes and hallucinations. Which nursing action is priority while diagnostic testing is underway? 1. Initiate seizure precautions 2. Discontinue G-tube feeding 3. Administer oxygen 4. Obtain blood work for troponin level

1. Initiate seizure precautions (1. Correct: Feeding tube clients tend to get dehydrated, especially clients on bed rest, because bed rest induces diuresis. If the client is already having neurological signs, a grand mal seizure may be next. Better take seizure precautions while awaiting the serum sodium results. 2. Incorrect: You may do this; however, seizure precautions will take priority. 3. Incorrect: The priority here is seizure precautions. 4. Incorrect: This data should lead to the suspicion of dehydration and hypernatremia, not suspected MI, which would be the reason a troponin level would be obtained.)

A nurse assessing a client who is one day post thyroidectomy and identifies an arrhythmia on auscultation. While taking the blood pressure, the nurse notices the client's hand starts to tremble. What interventions are priority? Select all that apply 1. Initiate seizure precautions 2. Monitor potassium level 3. Monitor BUN and creatinine 4. Restrict calicum rich foods 5. Check for airway patency

1. Initiate seizure precautions 5. Check for airway patency (1. & 5. Correct: The parathyroid glands can accidentally be removed with a thyroidectomy. Low calcium causes rigid and tight muscles. 2. Incorrect: Calcium and phosphorous should be monitored. Thyroid and parathyroid do not affect potassium levels. 3. Incorrect: The kidneys are not involved. 4. Incorrect: The client is hypocalcemic. You would not restrict calicum rich foods. Instead you would provide a diet high in calicum rich foods.)

The nurse is providing foot care to the client who has diabetes. What should the nurse include in the teaching? Select all that apply 1. Inspect the feet daily for abrasions or pressure areas. 2. Check water temperature with the hands before getting into tub. 3. Do not use heating pads on the feet or lower legs. 4. Thoroughly dry the feet, especially between the toes. 5. Cut toenails rather than file them. 6. Cut nails in a rounded fashion.

1. Inspect the feet daily for abrasions or pressure areas. 2. Check water temperature with the hands before getting into tub. 3. Do not use heating pads on the feet or lower legs. 4. Thoroughly dry the feet, especially between the toes. (1., 2., 3. & 4. Correct: The feet should be inspected daily. Small tears or abrasions can occur without the client's awareness due to decreased sensation in the feet. The client may be burned by getting into water that is too hot due to decreased sensation in the feet. There is less chance of decreased sensation in the hands. Heating pads may burn the client's feet. It is better to apply blankets for warmth. Drying the feet and between the toes will prevent skin breakdown. 5. Incorrect: Filing is safer, as it is not likely to result in cutting or irritating the skin around the nail. A cut on the lower extremity can result in an infection. 6. Incorrect: The nails should be filed straight across. Filing into a round shape may result in an ingrown toenail, which may lead to infection. Skin breaks on the lower extremity can lead to infection.)

The homecare nurse is providing family teaching on safety issues for a client diagnosed with Parkinson's disease. What adaptations should the nurse instruct the family to initiate? Select all that apply 1. Install grab bars on tub walls. 2. Place nightlights in hallways. 3. Add bran and fiber to daily diet. 4. Remove scatter rugs or loose cords. 5. Keep bedroom dark, cool and quiet. 6. Put tennis balls on legs of walker.

1. Install grab bars on tub walls. 2. Place nightlights in hallways. 3. Add bran and fiber to daily diet. 4. Remove scatter rugs or loose cords. 5. Keep bedroom dark, cool and quiet. (1., 2., 3., 4., & 5. Correct: Parkinson's disease causes deterioration of the basal ganglia, ultimately impacting motor control and function. As muscles become stiff and rigid, mobility slows, resulting in poor coordination and loss of balance. Safety is a chief concern in all ADLs, requiring modifications in activity, nutrition, and the client's environment. Because Parkinson's disease affects mobility, modification such as grab bars and night lights are essential. Clients develop constipation because of decreased peristalsis, so adding bran and fiber can address impending bowel issues. Scatter, or throw, rugs along with loose extension cords on the floor create a fall risk because the client is unable to pick up feet during ambulation. The shuffling gait that develops increases the risk for falls. These clients also have problems with insomnia along with poor REM sleep, leading to daytime drowsiness. Making the bedroom conducive to sleep may help alleviate symptoms for a period of time. A dark, cool room with no distractions is the most appropriate sleep environment. 6. Incorrect: The proper method of utilizing a walker is to step into the walker, pause and then move it forward before stepping again. Even though clients with Parkinson's disease have a shuffling gait and stooped posture, sliding a walker with tennis balls on the walker legs presents a serious safety issue. The client would not have the ability to control the speed or hold the walker steady while stepping into it.)

Which assessment finding would the nurse expect in a client diagnosed with Paget's disease? Select all that apply 1. Severe back pain 2. Walking with a limp 3. Upper extremity grip weakness 4. A shuffled gait 5. Bow legged

1. Severe back pain 2. Walking with a limp 5. Bow legged (1., 2. & 5. Correct: Paget's disease is a chronic skeletal bone disorder in which there is excessive bone resorption followed by the marrow being replaced by fibrous connective tissue. The new bone is larger, disorganized, and weak. These clients have severe pain, may walk with a limp, and may become bow legged. 3. Incorrect: Muscle weakness is not a symptom. 4. Incorrect: This is a manifestation of Parkinson's disease which is related to brain and nerve signals. It is not related to Paget's disease.)

A client has been admitted to the medical unit after sustaining a stroke. The admitting nurse initiates a nursing diagnosis of unilateral neglect related to a decrease in visual field and hemianopia from cerebrovascular problems as evidenced by consistent inattention to stimuli on the affected side. What nursing interventions should the nurse initiate for this client? Select all that apply 1. Instruct client to scan from left to right to visualize the entire environment. 2. Encourage client to practice exercises independently. 3. Position bed in room so that individuals approach the client on the unaffected side. 4. Apply splints to achieve stability of affected joints. 5. Touch unaffected shoulder when initiating conversation with client. 6. Position personal items within view on the unaffected side.

1. Instruct client to scan from left to right to visualize the entire environment. 3. Position bed in room so that individuals approach the client on the unaffected side. 5. Touch unaffected shoulder when initiating conversation with client. 6. Position personal items within view on the unaffected side. (1., 3., 5., & 6. Correct: Instructing the client to scan from left to right will help the client to visualize the entire environment. The client has to be reminded to do this since only one side of the client's visual field is working. By positioning the bed so that individuals approach the client from the unaffected side and by touching the client on the unaffected shoulder, the client is not surprised or frightened when realizing someone is in the room. Placing personal items where the client can see them will allow the client to use the material. Then gradually move personal items and activity to the affected side as the client demonstrates an ability to compensate for neglect. 2. Incorrect: Practicing exercises independently focuses on impaired physical mobility rather than unilateral neglect. 4. Incorrect: Applying splints to affected joints focuses on impaired physical mobility rather than unilateral neglect.)

A client is being evaluated for possible Rheumatoid Arthritis (RA). Which lab data and assessment findings by the nurse would be indicative of RA? Select all that apply 1. Joint pain, swelling, and warmth. 2. Decreased movement in joints. 3. Presence of Rheumatoid factor on lab analysis. 4. Presence of Dupuytren's contractures. 5. Elevated erythrocyte sedimentation rate (ESR). 6. Presence of Cyclic Citrullinated Peptide Antibody.

1. Joint pain, swelling, and warmth. 2. Decreased movement in joints. 3. Presence of Rheumatoid factor on lab analysis. 5. Elevated erythrocyte sedimentation rate (ESR). 6. Presence of Cyclic Citrullinated Peptide Antibody. (1., 2., 3., 5., & 6. Correct: Classic features of RA include joint pain, swelling, and tenderness worsened by movement and stress placed on joint. Morning stiffness that often lasts for one hour or more and limited movement in joints are common manifestations as well. The Rheumatoid Factor is present in 80% of adults who have rheumatoid arthritis. The ESR blood test is elevated with RA and is used to determine if an abnormal level of inflammation exists in the body. The cyclic citrullinated peptide antibody, if present, helps to confirm the diagnosis of RA and may indicate the risk of having severe symptoms. Levels that are at a moderate to high level may indicate that the client is at increased risk for damage to the joints. 4. Incorrect: Dupuytren's contractures are a type of hand deformity where a layer of tissue under the skin in the palms of the hands is affected. Hard knots form in the palm areas and eventually create a thick cord that can pull one or more of the fingers into a bent position. However, this is not associated with RA.)

What potential contributing factors for stress urinary incontinence should a nurse assess for in an elderly female client? Select all that apply 1. Lack of estrogen 2. Rising abdominal pressure 3. Multiparous vaginal births 4. Spinal cord injury 5. Dementiaevery 2 hours if their perception of pressure is intact.

1. Lack of estrogen 2. Rising abdominal pressure 3. Multiparous vaginal births (1., 2., & 3. Correct: During pregnancy and childbirth, the sphincter and pelvic muscles stretch out and are weakened. Increased age, decreased estrogen, and a history of multiple vaginal births/pregnancies are contributing factors for stress incontinence. 4. Incorrect: Spinal cord injury results in urge incontinence because of damage to the nerves of the bladder. Urge incontinence means there is a sudden, involuntary contraction of the muscle wall. 5. Incorrect: With functional incontinence the person knows there is a need to urinate but cannot make it to the restroom. The dementia client cannot make the conscious decision or carry out the task of ambulating to the restroom.)

A nurse is planning discharge education for a client diagnosed with syndrome of inappropriate antidiuretic hormone (SIADH). Which instructions should the nurse include when teaching this client? Select all that apply 1. Limit fluid intake. 2. Report muscle cramping. 3. Measure intake and output. 4. Perform mouth care once a day. 5. Report weight gain of 2 pounds (0.9 kg) over 24 hours.

1. Limit fluid intake. 2. Report muscle cramping. 3. Measure intake and output. 5. Report weight gain of 2 pounds (0.9 kg) over 24 hours. (1., 2., 3. & 5. Correct: The nurse should advise the client diagnosed with syndrome of inappropriate antidiuretic hormone (SIADH) to limit fluid intake. In SIADH, excessive amounts of water are reabsorbed by the kidneys, creating potentially disastrous dilutional hyponatremia. Water must be restricted to prevent water intoxication. The nurse should advise the client diagnosed SIADH to report muscle cramping. Muscle cramping should be reported immediately to the primary healthcare provider because this sign could indicate hyponatremia, which can lead to seizures or coma. The nurse should advise the client diagnosed with SIADH to measure intake and output. Intake and output should be monitored carefully to assess the amount of fluid restriction needed. Weight gain of 2 pounds (0.9 kg) or more should be reported to the primary healthcare provider because this is an indication of fluid retention and increases the client's risk for fluid volume excess. 4. Incorrect: The nurse should teach the client to rinse the mouth frequently to keep mucous membranes moist during fluid restriction, not just once a day.)

What nursing interventions should the nurse initiate in a client who experiences sundowning? Select all that apply 1. Limit naps. 2. Encourage TV watching in the evening. 3. Create a calm, quiet environment. 4. Open window blinds during the day. 5. Leave the lights on at night. 6. Maintain a routine.

1. Limit naps. 3. Create a calm, quiet environment. 4. Open window blinds during the day. 6. Maintain a routine. (1., 3., 4., & 6. Correct: Sundowning occurs when the client becomes more confused and agitated in the late afternoon or evening. Behaviors commonly seen include agitation, aggressiveness, wandering, resistance to redirection, and increased verbal activity such as yelling. Limit naps because too much daytime napping may interfere with sleeping at night. Light therapy may reduce agitation and confusion so open the blinds. Caregivers should remain calm and avoid confrontation. Routine helps the client feel secure. 2. Incorrect: Watching television for this client may lead to restlessness, agitation, and confusion. Calming and more restful activities are better for the evening. 5. Incorrect: Lights should be on during the day but turned off at night (except for low lighting or nightlights so the client can see).)

The nurse enters the client's room and finds the client having a seizure on the floor. Which nursing interventions should the nurse implement? Select all that apply 1. Loosen tight shirt or jacket. 2. Move the client to the couch. 3. Place a pillow under the head. 4. Position the head tilted forward. 5. Insert a wash cloth between the teeth.

1. Loosen tight shirt or jacket. 4. Position the head tilted forward. (1., & 4. Correct: Client safety should be the priority action. The tight clothing should be loosened to reduce the potential of the clothing obstructing the airway. During a seizure, the head is tilted forward to allow the tongue to advance forward. This will assist in the drainage of saliva and mucus. 2. Incorrect: If the nurse tries to move the client during a seizure to the couch, there is the possibility that both the nurse and client could be injured. The client is safer on the floor than trying to move the client. 3. Incorrect: During a seizure, the head should not be placed on a pillow. The pillow may cause the client's airway to become occluded, and an increase of saliva and mucus in the mouth may not drain properly. 5. Incorrect: The nurse should not open the mouth of the client during a seizure. This action may result in injury to the client's teeth and/or jaws. The muscles in the jaws may spasm which will seal the mouth tight.)

What physical changes should the nurse discuss with a client who is entering menopause? Select all that apply 1. Loss of bone density 2. Loss of muscle mass 3. Improved skin elasticity 4. A reduction in waist size 5. Increased fat tissue

1. Loss of bone density 2. Loss of muscle mass 5. Increased fat tissue (1., 2. & 5. Correct: Changes associated with menopause, with its dramatic decline in estrogen, include loss of muscle mass, increased fat tissue leading to thicker waist, dryness of the skin and vagina, hot flashes, sleep abnormalities, and mood changes. 3. Incorrect: A decrease in turgor and elasticity may occur as we grow older. Skin becomes dry and thin and collagen levels decrease after menopause. 4. Incorrect: There is increased abdominal fat. The waist size swells relative to hips after menopause.)

The nurse cares for a client after a gastroscopy examination. Which nursing interventions are appropriate post-procedure? Select all that apply 1. Maintain NPO status until the gag reflex returns. 2. Observe for hematemesis. 3. Monitor intake and output. 4. Assess bowel elimination. 5. Monitor respirations. 6. Connect to oxygen saturation monitor.

1. Maintain NPO status until the gag reflex returns. 2. Observe for hematemesis. 5. Monitor respirations. 6. Connect to oxygen saturation monitor. (1., 2., 5, & 6. Correct: A gastroscopy examination uses a flexible fiber-optic tube to visualize the esophagus and/or stomach. The throat is numbed before the procedure to inhibit the gag reflex and to allow the scope to pass freely. Therefore, it is important to maintain NPO status until the gag reflex returns. The nurse should monitor for hematemesis, respirations, and oxygen saturation to ensure the scope did not damage any structures like the esophagus or lungs. 3. Incorrect: Because the kidneys are not affected during this procedure, monitoring intake and output is not an important intervention. 4. Incorrect: Because the bowels are not affected during this procedure, this is not an important intervention.)

What should a community health nurse include when planning a presentation on prevention and early detection of colon cancer? Select all that apply 1. Maintain a diet high in fruits, vegetables, and whole grains. 2. Exercise regularly. 3. Regular screening should begin at age 30. 4. Yearly fecal occult blood test beginning at age 50. 5. Flexible esophagogastroduodenoscopy every 5 years.

1. Maintain a diet high in fruits, vegetables, and whole grains. 2. Exercise regularly. 4. Yearly fecal occult blood test beginning at age 50. (1., 2., & 4. Correct: A diet high in vegetables, fruits, and whole grains have been linked with a decreased risk of colorectal cancer, whereas, a diet high in red meats, processed meats, and frying can increase risk of colorectal cancer. There is a greater risk of developing colorectal cancer in individuals who live a sedentary life style. The fecal occult blood test detects blood in the stool through a chemical reaction. This test is done yearly. 3. Incorrect: If there is no identified risk factors (other than age), regular screening should begin at age 50. 5. Incorrect: A flexible esophagogastroduodenoscopy is not a recommended procedure for the early detection of cancer of the colon.)

When providing care for a client with a chest drainage unit (CDU) set at 20 cm. of suction, which nursing actions are correct? Select all that apply 1. Maintain chest drainage system below the client's chest during transport. 2. Apply tape to the tubing connection sites. 3. Add sterile water to suction control chamber to achieve 20 cm. 4. Assess respiratory effort every shift. 5. Ensure that tubing is not kinked or looped.

1. Maintain chest drainage system below the client's chest during transport. 2. Apply tape to the tubing connection sites. 3. Add sterile water to suction control chamber to achieve 20 cm. 5. Ensure that tubing is not kinked or looped. (1., 2., 3. & 5. Correct: Never raise the drainage system above the level of the client's chest. All connection sites should be tightly secured. If the water level drops below the prescribed suction, more saline must be added. Tubing must not be kinked or looped. 4. Incorrect: Respiratory effort for a client with a CDU must be assessed more often than every shift. This assessment should be done at least every 2 hours.)

The nurse providing palliative care to a client would include which outcomes in the teaching plan? Select all that apply 1. Maintaining the client's quality of life 2. Minimizing family caregiver stress 3. Managing the client's pain 4. Managing the client's and family's emotional needs 5. Attending to the client's spiritual needs 6. Ensuring the client understands that disease focused treatments will cease

1. Maintaining the client's quality of life 2. Minimizing family caregiver stress 3. Managing the client's pain 4. Managing the client's and family's emotional needs 5. Attending to the client's spiritual needs (1., 2., 3., 4. & 5. Correct: Palliative care includes supporting the client's and family's quality of life. Palliative care includes managing pain and symptoms. Palliative care includes managing the client's and family's emotional needs and attending to their spiritual needs. 6. Incorrect: Palliative care includes treatment of discomfort, symptoms, and stress of serious illness. Disease focused treatments will not cease.)

The nurse has determined that a bedridden client diagnosed with a stroke is at risk for venous thromboembolism (VTE). What interventions should the nurse initiate? Select all that apply 1. Measure the calf and thigh daily. 2. Apply sequential compression device to legs. 3. Position paralyzed leg with each distal joint higher than the proximal joint. 4. Place a trochanter roll at the hip. 5. Perform passive range of motion exercises once daily. 6. Monitor for pain by assessing Homan's sign.

1. Measure the calf and thigh daily. 2. Apply sequential compression device to legs. 3. Position paralyzed leg with each distal joint higher than the proximal joint. (1., 2., & 3. Correct: Assessment for VTEs is accomplished by measuring the calf and thigh daily, observing swelling, noting unusual warmth of the leg, and asking the client about pain in the calf. Prevention of VTEs include the use of sequential compression devices for bedridden clients. This device helps promote venous return. Positioning the paralyzed leg with each distal joint higher than the proximal joint will prevent dependent edema. 4. Incorrect: A trochanter roll does not prevent VTEs. They are used for the prevention of external hip rotation. 5. Incorrect: Passive range of motion exercises should be done several times a day to promote venous return and muscle tone. Once a day is not adequate. 6. Incorrect: Routinely checking the Homan's sign can actually cause a clot to dislodge. The nurse should not perform this procedure.)

A nurse is planning to discuss steps that senior citizens can take to keep the brain healthy. What should the nurse include? Select all that apply 1. Memorize poetry. 2. Eat foods low in Omega 3, fatty acids. 3. Brush teeth with nondominant hand. 4. Do crossword puzzles. 5. Learn a new language. 6. Volunteer.

1. Memorize poetry. 3. Brush teeth with nondominant hand. 4. Do crossword puzzles. 5. Learn a new language. 6. Volunteer. (1., 3., 4., 5., & 6. Correct: Although there is no known definitive way to prevent Alzheimer's disease, steps can be taken to keep the brain healthy. Growing evidence indicates that people can reduce their risk of cognitive decline by adopting key lifestyle habits. When possible, combine these habits to achieve maximum benefit for the brain and body. To keep the brain active, learn something new. Take up a language, an instrument, memorize poetry. A good, healthy diet will improve all areas of our health, but there are many studies and an increasing amount of evidence suggesting that certain foods slow mental decline. Brain-boosting food is any food high in Omega 3 fatty acids, which has been linked to a lower risk of dementia and improved focus and memory. Brain games, puzzles and brainteasers help create new associations between different parts of the brain, which keeps it sharp. Challenge the brain by doing normal activities with the non-dominant hand like brushing your teeth or combing your hair. Research shows that volunteering can lower stress levels and increase mental functioning. It also adds to a person's wellbeing and overall health. Social activity keeps our minds sharp. This is especially true later in life, when aging takes its toll on memory and other complex neurological processes. 2. Incorrect: Brain-boosting food is any food high in Omega 3 fatty acids, which has been linked to a lower risk of dementia and improved focus and memory.)

A 19 year old client preparing to enter college asks the clinic nurse about immunizations. What immunizations should the nurse suggest the client discuss with the primary health care provider? Select all that apply 1. Meningococcal conjugate vaccine 2. Tdap vaccine 3. HPV vaccine 4. Seasonal flu vaccine 5. Hepatitis B 6. Polio

1. Meningococcal conjugate vaccine 2. Tdap vaccine 3. HPV vaccine 4. Seasonal flu vaccine 5. Hepatitis B (1., 2., 3., 4, & 5. Correct: These vaccine are specifically recommended for young adults ages 19-24. Meningococcal conjugate vaccine is recommended as it protects against bacterial meningitis. It is required for students living in a dorm. Tdap vaccine protects against tetanus, diphtheria, and pertussis. HPV vaccine protects against the human papillomavirus, which causes most cases of cervical and anal cancers, as well as genital warts. Seasonal flu vaccine is recommended. Hepatitis B is a blood-born infection, which can also be transmitted through sexual activity. 6. Incorrect: The inactivated polio (IPV) vaccine is a 4-dose series given during early childhood. IPV is not routinely recommended for children aged 18 years or older.)

A client diagnosed with renal failure has been admitted to the medical unit. An arterial blood gas (ABG) analysis has been prescribed by the primary healthcare provider. Which ABG interpretation by the nurse is appropriate? pH - 7.33 PaCO₂ - 36 HCO₃⁻ - 20 1. Metabolic acidosis 2. Metabolic alkalosis 3. Respiratory acidosis 4. Respiratory alkalosis

1. Metabolic acidosis (1. Correct: pH - 7.33 (normal value 7.35 - 7.45) less than 7.35. PaCO₂ 36 mm Hg (normal value 35 - 45 mm Hg) within normal range. HCO₃⁻ 20 mEq/L (normal value 22 - 26 mEq/L) less than 22 mEq/L Metabolic acidosis is reflected in a reduction of the HCO₃⁻ and pH levels. 2. Incorrect: Metabolic alkalosis is indicated by an elevated pH and HCO₃⁻ levels. The client's HCO₃⁻ and the pH are below the normal range. 3. Incorrect: The primary issue with respiratory acidosis is an elevated CO₂ level. The CO₂ level for this client is within the normal range. 4. Incorrect: With respiratory alkalosis the pH is greater than 7.45. The pH for this client is less than 7.35 and the PaCO₂ is within the normal range. The HCO₃⁻ value will decrease or elevate depending if the pulmonary process is acute or chronic.)

How should the nurse interpret this arterial blood gas (ABG) report? pH - 7.33 PaO₂ - 95% PaCO₂ - 28 HCO₃⁻ - 18 Select all that apply 1. Metabolic acidosis 2. Metabolic alkalosis 3. Respiratory acidosis 4. Respiratory alkalosis 5. Uncompensated 6. Partially compensated 7. Fully compensated

1. Metabolic acidosis 6. Partially compensated (1., & 6. Correct: This set of ABGs reflects partially compensated metabolic acidosis. The pH, bicarb, and carbon dioxide are all abnormal, so compensation is beginning. Since the pH is not normal yet, total compensation has not occurred. There is only partial compensation. 2. Incorrect: The pH remains low (acidic) so acidosis rather than alkalosis is occurring. 3. Incorrect: This set of ABGs reflects partially compensated metabolic acidosis. The pH, bicarb, and carbon dioxide are all abnormal, so compensation is beginning. Since the pH is not normal yet, total compensation has occurred. The pH and bicarb match indicating a metabolic problem initially. The lungs are attempting to compensate by blowing off CO₂. 4. Incorrect: This set of ABGs reflects partially compensated metabolic acidosis. The original problem is not a lung problem, but a metabolic problem. 5. Incorrect: The pH, bicarb, and carbon dioxide are all abnormal, so compensation is beginning. 7. Incorrect: Since the pH is not normal yet, total compensation has not occurred. There is only partial compensation.)

A client with type II diabetes reports normal blood glucose levels at bedtime and high blood glucose levels in the morning for the past week. What instruction would the nurse give the client? 1. Monitor blood sugar around 2am. 2. Decrease bedtime snacking. 3. Decrease intermediate acting insulin. 4. Increase intermediate acting insulin.

1. Monitor blood sugar around 2am. (1. Correct: Morning hyperglycemia may be the result of dawn's phenomenon or the Somogyi effect. The client must take their blood sugar between two and three o'clock in the morning for several days to determine the cause of morning hyperglycemia. If the client has decreased blood sugar between two and three o'clock in the morning, suspect Somogyi effect. 2. Incorrect: This is an intervention; assessment should come first. The nurse must determine the cause of morning hyperglycemia in order to treat the condition appropriately. 3. Incorrect: This is an intervention; assessment should come first. The nurse must determine the cause of hyperglycemia in order to treat the condition appropriately. An appropriate intervention for a client with Somogyi effect would be to decrease the evening dose of intermediate acting insulin, however, the nurse must first determine that the client is in fact experiencing the Somogyi effect. 4. Incorrect: This is an intervention; assessment should come first. Increasing the intermediate acting insulin would not be appropriate action for a client experiencing Somogyi effect.)

What nursing interventions should the nurse include when planning care for a client admitted with Guillain-Barre' Syndrome? Select all that apply 1. Monitor for contractures. 2. Place prone for 30 minutes, 4 times per day. 3. Provide therapeutic massage for pain relief. 4. Teach range of motion exercises. 5. Provide high protein meals 3 times a day. 6. Refer to physical therapist.

1. Monitor for contractures. 3. Provide therapeutic massage for pain relief. 4. Teach range of motion exercises. 6. Refer to physical therapist. (1., 3., 4., & 6. Correct: This client will have progressive weakness and paralysis. Contractures and pressure ulcers need to be prevented through ROM exercises and frequent turning. Muscle spasms and pain can be relieved by therapeutic massage, imagery, diversion, and pain medication. 2. Incorrect: The client will need to be repositioned every 2 hours to prevent pressure sores and pneumonia and atelectasis. Elevate the head of the bed to help with lung expansion. Prone will interfere with lung expansion ability. 5. Incorrect: Encourage small, but frequent meals that are both well-balanced and nourishing.)

A client with an acute exacerbation of systemic lupus erythematosus (SLE) is hospitalized with incapacitating fatigue and fever. A urinalysis reveals proteinuria and hematuria. The primary healthcare provider prescribes corticosteroids. During the acute phase of the client's illness, what is most important for the nurse to do? 1. Monitor intake and output and daily weight. 2. Allow for frequent, uninterrupted rest periods. 3. Institute seizure precautions. 4. Protect client from injury that may cause bleeding.

1. Monitor intake and output and daily weight. (1. Correct: Look at the clues in the stem. Proteinuria and hematuria. When you see proteinuria what do you need to worry about? The kidneys! Protein is a great big molecule. The only way for protein to be seen in the urine is if there are holes in the glomerulus. So the kidneys are being damaged. Thus, the nurse knows that the biggest problem to "worry" about here is renal failure. The best methods for monitoring fluid status and renal status for a client are to monitor I and O and daily weights. (Also, remember that one weight doesn't mean anything. The hematuria indicates that there has already been glomerular damage.) 2. Incorrect: Systemic lupus erythematosus (SLE) is an autoimmune disease. In this disease, the body's immune system mistakenly attacks healthy tissue. It can affect the skin, joints, kidneys, brain, and other organs. Fatigue is a major symptom so allowing for frequent, uninterrupted rest periods is important for this client but monitoring for renal failure is more acute. 3. Incorrect: Seizures are a potential problem with SLE, but the ACTUAL problem depicted in the stem of the question, renal failure, takes priority. Look for the option that relates to the renal system. 4. Incorrect: Hemolytic problems can occur with SLE, but this is not the ACTUAL problem depicted in the stem of the question. The stem is indicating a renal problem, so look for a renal answer.)

What nursing interventions should a nurse initiate for a client diagnosed with pyelonephritis? Select all that apply 1. Monitor urine for dark, cloudy, foul smelling urine. 2. Place client on intake and output monitoring. 3. Decrease fluid intake to 1 liter/day. 4. Advise client wear protective clothes outside while taking levofloxacin. 5. Monitor for hypotension, tachycardia, fever.

1. Monitor urine for dark, cloudy, foul smelling urine. 2. Place client on intake and output monitoring. 4. Advise client wear protective clothes outside while taking levofloxacin. 5. Monitor for hypotension, tachycardia, fever. (1., 2., 4., & 5. Correct: With pyelonephritis urine will be dark, cloudy and foul smelling due to the bacteria. Anytime a client has a renal problem, that client should be placed on I&O. Levofloxacin could make the client sunburn more easily. Avoid sunlight or tanning beds. Wear protective clothing and use sunscreen (SPF 30 or higher) when outdoors. Monitor for septic shock, a complication of pyelonephritis. S/S include hypotension, tachycardia, and fever. 3. Incorrect: Fluid intake should be increased to 2-3 liters/day unless contraindicated.)

The charge nurse of a large medical-surgical unit is admitting several clients requiring specific infection control precautions. The nurse is aware that droplet precautions are necessary for which client diagnosis? Select all that apply 1. Mumps 2. Methicillin resistant Staphylococcus aureus (MRSA) 3. Shingles (Herpes Zoster) 4. Human immunodeficiency virus (HIV) 5. Pertussis

1. Mumps 5. Pertussis (1. & 5. Correct: Droplet precautions are utilized whenever a client has specific microorganisms that are spread by coughing, sneezing or talking. Individuals within three feet of the client can be contaminated by breathing in those respiratory droplets. Mumps require approximately 9 days of isolation with droplet precautions after the swelling becomes visible. Clients with pertussis also require droplet precautions. Pertussis is a very contagious disease only found in humans. It is spread from person to person. People with pertussis usually spread the disease to another person by coughing or sneezing or when spending a lot of time near one another where you share breathing space. 2. Incorrect: MRSA is a bacterial infection that can be spread by either direct contact with the client, or indirect contact with environmental surfaces that have been contaminated by the client. This client will require contact precautions. Depending on the type of care being provided to this client, staff may need gloves, gown, mask or eye protection. 3. Incorrect: Shingles (Herpes Zoster) is transmitted through air and direct contact with respiratory secretions and the lesions. Therefore, airborne and contact precautions should be initiated. This client should be placed in a private room with negative pressure to prevent contaminating others. Staff entering this room must wear a special filter mask (N95), gown and gloves. Additionally, pregnant staff or family should not enter the room at all during this time. 4. Incorrect: HIV is a viral disease transmitted through the blood or body fluids of contaminated individuals, blood transfusions, or needles from IV drug usage. Standard precautions are adequate for most HIV clients unless the client has an open wound.)

A nurse is planning to provide information to a group of adults considering smoking cessation. What information should the nurse include? Select all that apply 1. Nicotine is the drug in tobacco products that produces dependence. 2. Withdrawal symptoms may include irritability, difficulty concentrating, and increased appetite. 3. Stopping smoking reduces the risk of coronary heart disease. 4. All smokers need to have a prescription for bupropion SR in order to quit. 5. Refer to smoking quit-lines that offer free support, advice, and counseling from experienced coaches.

1. Nicotine is the drug in tobacco products that produces dependence. 2. Withdrawal symptoms may include irritability, difficulty concentrating, and increased appetite. 3. Stopping smoking reduces the risk of coronary heart disease. 5. Refer to smoking quit-lines that offer free support, advice, and counseling from experienced coaches. (1., 2., 3., 5. Correct: These are correct statements. Nicotine is the drug in tobacco products that produces dependence. Other withdrawal symptoms include anxiety and cravings for a cigarette. There are many health benefits to smoking cessation including reducing the risk of coronary heart disease, stroke, peripheral vascular disease, COPD and reduced risk for infertility in women. Clients should be referred to educational programs and support groups. 4. Incorrect: The majority of cigarette smokers quit without using this prescription; however, treatments can help the smoker quit, so they should discuss possible medications with their primary healthcare provider. Other medications such as the nicotine patch or varenicline (chantix) may also be used to assist with smoking cessation.)

What sign and symptom should the nurse expect to find during the physical assessment of a client who has a history of rheumatoid arthritis? Select all that apply 1. Nodules over bony prominences 2. Reports of morning stiffness lasting over 1 hour 3. Reports of weight loss 4. Cool, swollen joints 5. Joint deformity

1. Nodules over bony prominences 2. Reports of morning stiffness lasting over 1 hour 3. Reports of weight loss 5. Joint deformity (1., 2., 3., & 5. Correct: Firm bumps of tissue (nodules) over bony prominences, such as on elbows, are called rheumatoid nodules, and are a sign of rheumatoid arthritis. Morning stiffness that can last for hours is a symptom of rheumatoid arthritis. Over time, rheumatoid arthritis can cause joints to deform and shift out of place. Weight loss is a common symptom of rheumatoid arthritis. 4. Incorrect: Affected joints will be tender, red, warm, and swollen.)

A 40 year old client reports a diminished ability to visually focus on close objects and has also noticed a need for a well lit environment to enhance vision. To what would the nurse attribute these changes? 1. Normal changes associated with aging. 2. A cataract is forming. 3. Symptoms of a brain tumor. 4. Precipitated by diabetic retinopathy.

1. Normal changes associated with aging. (1. Correct: Aging results in stiffening of the lens, thus lessening the ability to focus. The retina is less sensitive to light, making accurate vision in low-light situations more difficult. Pupillary response diminishes, affecting the ability to adjust to changing light levels. 2. Incorrect: Cataracts present with blurred vision and a glare from lights. 3. Incorrect: Brain tumors increase intracranial pressure, resulting in blurring of vision. 4. Incorrect: Diabetic retinopathy is caused by changes in retinal blood vessels and results in blurred vision and outright impairment in some fields.)

After completing several rounds of chemotherapy, a client's laboratory results indicate severe neutropenia. Following admission assessment, what is the nurse's priority action for this client? 1. Notify dietary no fresh, unpeeled fruits or vegetables. 2. Avoid all venipunctures or IM injections. 3. Have client wear mask when leaving room. 4. Instruct client to use a soft toothbrush.

1. Notify dietary no fresh, unpeeled fruits or vegetables. (1. Correct: Neutropenia is an abnormally low white blood cell count caused, in this case, by the recent chemotherapy. The greatest concern is the client's inability to fight off infection. Fresh fruits and vegetables have a high bacterial count and present an increased risk for infection. Asking dietary to remove fresh fruits and vegetables from meal trays is an important priority action by the nurse. 2. Incorrect: Avoiding venipunctures of any type, including IM injections, is an important precaution for neutropenia, in which infection is the main concern. However, the word "all" makes this statement to definite. The client may need an IV. Remember, nothing is that definite in the world. 3. Incorrect: This immunocompromised client is at risk for infection, as indicated by a low neutrophil count. While airborne bacteria may be a concern at some point, there is another action by the nurse which takes priority. 4. Incorrect: A soft toothbrush is used as part of the precautions for clients at risk for bleeding, which would not apply to this client. The nurse here is concerned about infection control secondary to a low neutrophil count.)

What is the priority nursing action for a client that was admitted with tingling of the toes and feet after having the flu for several days when the client begins to have numbness in the legs and hips? 1. Notify the primary healthcare provider 2. Monitor for paresthesia in the fingers and hands 3. Insert an indwelling urinary catheter 4. Assist the client with performing passive range of motion

1. Notify the primary healthcare provider (1. Correct: Symptoms are classic for Guillain-Barre. The possibility of rapid progression and respiratory failure make this a medical emergency. The nurse's priority action is to notify the healthcare provider. 2. Incorrect: The nurse should continue to monitor for paresthesia in the upper body and arms. The first priority in this situation is to notify the primary healthcare provider of the potential life threatening situations. 3. Incorrect: Urinary retention is a possible complication with Guillain-Barre, and the client may require an indwelling urinary catheter, but the immediate priority is to notify the primary healthcare provider. 4. Incorrect: Passive range of motion is performed to prevent complications of immobility, but this is not the priority at this time. The client is presently able to move their extremities. Passive range of motion is not the priority at this time.)

What risk factors should the nurse include when teaching a group of clients about osteoarthritis? Select all that apply 1. Sports injury to joint 2. Genetic predisposition 3. Obesity 4. Male sex 5. Repetitive joint stress

1. Sports injury to joint 2. Genetic predisposition 3. Obesity 5. Repetitive joint stress (1., 2., 3., & 5. Correct: Injuries, such as those that occur when playing sports or from an accident, can increase the risk of osteoarthritis. Some people inherit a tendency to develop osteoarthritis. Carrying extra body weight contributes to osteoarthritis. If a person's job or a sport places repetitive stress on a joint, that joint might eventually develop osteoarthritis. 4. Incorrect: Women are more likely to develop osteoarthritis, though it isn't clear why.)

The nurse is caring for a client who is scheduled to receive furosemide 40 mg IVP twice daily, as well as 20 meq (20 mmol/l) of potassium chloride twice daily. The client's lab work reveals that the potassium level is 2.4 mEq/L (2.4 mmol/L) this morning. How should the nurse proceed? 1. Notify the primary healthcare provider of the potassium level immediately. 2. Administer the medications as scheduled and notify the primary healthcare provider on rounds. 3. Give the potassium, but hold the furosemide until primary healthcare provider rounds. 4. Assess the client for muscle cramps.

1. Notify the primary healthcare provider of the potassium level immediately. (1. Correct: Yes, this is a very low level. Normal values are 3.5-5.0 mEq/L (3.5-5.0 mmol/L). This client will need more potassium and less furosemide (a potassium wasting diuretic). 2. Incorrect: No, potassium is dangerously low. Giving the furosemide will drop the potassium level further since it is potassium wasting. Do not wait for the primary healthcare providers to make rounds as they often do not make predictable rounds. 3. Incorrect: This is delaying care and confuses the issue of how much potassium needs to be administered now. 4. Incorrect: Delays care. What if there are no symptoms? Will you wait for symptoms to treat?)

Which clinical manifestations would validate to the nurse that a client has developed an electrolyte imbalance due to malabsorption from celiac disease? Select all that apply 1. Numbness 2. Muscle cramps 3. Negative Trousseau 4. Irritable 5. Muscle spasticity 6. Hyperreactivity to sensory stimuli

1. Numbness 2. Muscle cramps 4. Irritable 5. Muscle spasticity 6. Hyperreactivity to sensory stimuli (1., 2., 4., 5., & 6. Correct: Low magnesium is typically due to decreased absorption of magnesium in the gut or increased excretion of magnesium in the urine. Conditions that increase the risk of magnesium deficiency include gastrointestinal (GI) diseases, such as Celiac disease, advanced age, type 2 diabetes, use of loop diuretics, and alcohol dependence. Early signs of low magnesium include nausea, vomiting, weakness, and decreased appetite. As magnesium deficiency worsens, symptoms may include numbness, tingling, muscle cramps, seizures, muscle spasticity, personality changes, dysrhythmias, tremors, hyperactive deep-tendon reflexes, hyperreactivity to sensory stimuli, positive Chvostek and Trousseau signs, tetany, and nystagmus. 3. Incorrect: The client with a low magnesium level will have a positive Trousseau sign.)

A nurse is caring for a poorly controlled type 2 diabetic client. The client does not adhere to the diet and the latest HbA1c is 8%. The serum glucose at this visit is 218mg/dL (12.09 mmol/L). The client is currently taking metformin and exenatide. Based on this history, what should the nurse anticipate will be the first strategy implemented to improve glucose control for this client? 1. Nutritional counseling 2. Increased daily exercise regimen 3. Education regarding Insulin by basal/bolus dosing method 4. More frequent self-monitoring of blood glucose

1. Nutritional counseling (1. Correct: True, nutritional teaching to promote diet compliance should be tried first because the client's HbA1c is 8% and his blood sugar is 218mg/dL (12.09 mmol/L) - not terribly bad for a type 2 diabetic who is non-compliant. Noncompliant is your big hint with this question. 2. Incorrect: No, this is not priority over nutritional counseling. Appropriate, moderate exercise, like walking, can overtime lower blood glucose. Increasing the daily exercise regimen is not appropriate based on the information provided in the question stem. 3. Incorrect: No, insulin is not indicated for a HbA1c of 8%, unless diet and oral hypoglycemics have failed long-term. Insulin is generally prescribed for a sustained HbA1c of > 9%. 4. Incorrect: No,This is not the priority over nutritional counseling. More frequent self monitoring will be important once insulin therapy has begun.)

A client with type 2 diabetes, who is noncompliant, has a HbA1c of 8%. The finger stick blood sugar is 218 mg/dL (12.1 mmol/L) at 0900. The current medications prescribed are metformin and exenatide. Based on this data, what teaching should the nurse reinforce? Select all that apply 1. Nutritional counseling to help improve diet compliance 2. HbA1c measures glycemia control over a period of 1 month 3. Blood glucose testing 4. Vigorous exercise plan to improve glucose control 5. Without glycemic control, eye complications can occur

1. Nutritional counseling to help improve diet compliance 3. Blood glucose testing 5. Without glycemic control, eye complications can occur (1., 3., & 5. Correct: The goal of therapy is to have a HbA1c <7.0% Nutritional teaching to promote diet compliance should be tried first because this clients HbA1c is 8% and blood sugar is 218 mg/dL (12.1 mmol/L). Reinforce need to monitor glucose levels several times a day, before meals and at bedtime. Have client keep results in a log. Eye complications include glaucoma, cataracts, retinopathy, blindness. 2. Incorrect: HbA1c measures glycemic control over a period of 3 months. 4. Incorrect: Physical activity under the supervision of the primary healthcare provider is appropriate teaching. Vigorous exercise is not key to improving blood sugar. Exercise does not have to be intense. A normal exercise plan is recommended based on the client's ability.)

The nurse is preparing to administer a dose of ondansetron 0.15 mg/kg. The nurse has not administered this medication before and is using a drug reference to review information about the medication. Which client and drug reference information supports the nurse's decision to withhold the ondansetron? Client Information: Medical diagnosis: Right mastectomy Current vital signs: BP 112/82, HR 104, R 24. Weight - 54 kg Medical history: Right breast cancer Physical examination: Alert/oriented. Reports a dry mouth and nausea. Vomited x 1. PERRLA. Nystagmus noted. Skin warm/dry. PMI 6 intercostal space 1 cm left of midclavicular line. Clear lung sounds. Right breast incision approximated, clean, dry, without drainage. Pill rolling movement of right thumb and forefinger. Bowel sounds present x 4 quadrants. Lab test results: Hgb - 12.3 g/dL (123 g/L) Hct - 37% (0.370) WBC - 10,000/microliter (10.0 × 10⁹/L). ALT - 16 U/L AST - 12 U/L Current medications: Clarithromycin 500mg IVPB every 8 hours x 10 doses. Hydromorphone 1 mg IM every 3 hours as needed for pain. Ondansetron 0.15 mg/kg IV every 6 hours as needed for nausea/vomiting. Drug Reference: Medication: Ondansetron Classification: Antiemetic Indications: Prevention of nausea and vomiting associated with chemotherapy. Prevention and treatment of postoperative nausea and vomiting. Contraindications/Precautions: Hypersensitivity; Congenital long-term QT syndrome: Concurrent use of apomorphine. Use cautiously in hepatic impairment; abdominal surgery; pregnancy, lactation. Adverse reactions/Side effects: Side effects: Headache, dizziness, drowsiness, fatigue, weakness, constipation, diarrhea, abdominal pain, dry mouth, increased liver enzymes. Adverse effects: Torsade de pointes, extrapyramidal reactions. Interactions: Use with apomorphine increases risk of severe hypotension and loss of consciousness. Carbamazepine may decrease levels. Route/Dosage: 0.15 mg/kg (max dose = 16 mg) IV Select all that apply 1. Nystagmus 2. Concurrent use of apomorphine 3. Pill rolling movement 4. Tachycardia 5. Maximum dose 16 mg 6. Elevated liver enzymes

1. Nystagmus 3. Pill rolling movement (1., & 3. Correct: Nystagmus and the pill rolling movement are signs of extrapyramidal effects of ondansetron, which would require the nurse to hold the medication and notify the primary healthcare provider. 2. Incorrect: This client has been prescribed hydromorphone for pain, which is not the same as apomorphine. Apomorphine is used to treat "wearing-off" episodes in people with advanced Parkinson's disease. 4. Incorrect: The client has mild tachycardia, which does not warrant withholding ondansetron. Bradyarrhythmias due to prolonged QT intervals are of more concern. 5. Incorrect: The prescribed dose is not greater than 16 mg. The dose would be 8.1 mg for this client weighing 54 kg (0.15 mg/kg). 6. Incorrect: The liver enzymes are not elevated. In fact, all lab values are within normal range. The normal number of WBCs in the blood is 4,500 to 11,000 WBC per microliter (4.5 to 11.0 × 10⁹/L). ALT - 3 to 36 U/L AST - 0 to 35 U/L)

A client diagnosed with heart failure has been prescribed a 2 gm sodium diet. Which food choices selected by the client would indicate to the nurse that the client understands this diet? Select all that apply 1. Pork loin 2. Frozen cheese ravioli dinner 3. Instant vanilla pudding 4. Thin crust pepperoni and ham pizza 5. Fresh salad with fresh citrus juice dressing 6. Bottled tomato juice

1. Pork loin 5. Fresh salad with fresh citrus juice dressing (1., & 5. Correct: A 3 ounce serving of pork loin contains approximately 54 mg of sodium. Slices of lemon, lime, or even oranges squeezed over a salad is low sodium (0-85 mg). 2. Incorrect: Canned entrees, and frozen dinners are high in sodium (Up to 1000 mg). 3. Incorrect: Instant puddings and cakes are high in sodium (1400 mg). 4. Incorrect: Pizza is high in sodium, particularly with meats such as pepperoni and ham (690 mg). 6. Incorrect: Bottled or canned tomato juice (980 mg), vegetable juice, mineral water, and softened water is high in sodium. Select coffee, tea, fruit juices, soft drinks, and low sodium tomato and vegetable juices.)

A client arrives by ambulance after being thrown from a horse. The client is pale, clammy and tachycardic with bruising over left upper abdominal quadrant. The nurse is aware what prescription by the primary healthcare provider takes priority? 1. Obtain blood for type and cross match. 2. Administer hydromorphone IV for pain. 3. Increase Lactated Ringers to 150 mL/hour. 4. Send client to radiology for stat CAT scan.

1. Obtain blood for type and cross match. (1. Correct: The signs and symptoms displayed by the client suggest a ruptured spleen and shock. The greatest concern in this situation is internal bleeding and possible emergency surgery. The client will need blood; therefore, the nurse should immediately obtain blood for type and cross match. 2. Incorrect: There is no indication in the scenario the client has pain. Pain medication should never be administered while the client is still being assessed or is in shock. 3. Incorrect: Fluids are crucial for clients in shock and increasing the Lactated Ringers to 150 mL/hr. is important to help maintain blood pressure. However, this is not the nurse's priority action. 4. Incorrect: A CAT scan is often prescribed prior to surgery to verify the extent of splenic injury and the amount of blood in the abdominal cavity. Though the order is written as 'stat', this is not the nurse's priority. Transporting an unstable client to another department requires preparation.)

The palliative care nurse is instructing the family of a client who is experiencing nausea and vomiting on methods of controlling these symptoms. What methods should the nurse include? Select all that apply 1. Offer electrolyte replacement drinks or broths. 2. Avoid cooking close to the client 3. Provide light, bland food. 4. Drink liquids less often 5. Chew 5-30 paw paw seeds

1. Offer electrolyte replacement drinks or broths. 2. Avoid cooking close to the client 3. Provide light, bland food. (1., 2., & 3. Correct: These are all methods that can help control n/v symptoms. Sports drinks and broths can help with hydration. Juices and soft drinks should be avoided. Smells from foods cooking can lead to nausea and vomiting. Bland foods in small portions may be tolerated vs. fried or heavy foods. 4. Incorrect: The client should drink small amounts of liquid more often. If tolerated, fluids will help prevent dehydration. Avoid milk products and sugary drinks as they will increase nausea and loss of fluids. 5. Incorrect: Paw paw seed is an herb that can be used for constipation. The question is not related to relieving constipation. It is related to nausea and vomiting prevention/control.)

Which signs/symptoms should the nurse assess for the presence of in a client diagnosed with valvular heart disease? Select all that apply 1. Orthopnea. 2. Paroxysmal nocturnal dyspnea. 3. Petechiae on the trunk. 4. Increasing CVP with decreasing BP. 5. Pericardial friction rub. 6. Widening pulse pressure.

1. Orthopnea. 2. Paroxysmal nocturnal dyspnea. (1., & 2. Correct: These are signs seen with valvular heart disease. Orthopnea is a condition where the client must sit or stand to breathe comfortably. Paroxysmal nocturnal dyspnea occurs when the client is reclining. It is sudden respiratory distress. 3. Incorrect: This is a sign of endocarditis. 4. Incorrect: This is the hallmark sign for cardiac tamponade. 5. Incorrect: This is a sign of pericarditis. 6. Incorrect: This is a sign of increased intracranial pressure.)

Which interventions should the nurse include in the plan of care for a client who has been admitted with a head injury? Select all that apply 1. Pad side rails. 2. Place hips in flexed position for 15 minutes every 4 hours. 3. Elevate head of bed 35 degrees. 4. Maintain neck in neutral position. 5. Cluster nursing activities. 6. Maintain a quiet environment.

1. Pad side rails. 3. Elevate head of bed 35 degrees. 4. Maintain neck in neutral position. 6. Maintain a quiet environment. (1., 3., 4., and 6. Correct: The client with a head injury is at risk for seizures. Padding the side rails is a safety precaution. Elevate the HOB 30-45 degrees to facilitate venous drainage and reduce ICP. Maintain the client's head midline to facilitate blood flow. A quiet environment is necessary to keep the client calm. An increase in environmental stimuli can increase ICP. 2. Incorrect: Do not allow pronounced neck or hip flexion as ICP will increase. Maintain HOB at 30-45 degrees and body in neutral position to avoid an increase in ICP. 5. Incorrect: Clustering nursing activities will increase ICP. Activities should be spaced out. Remember, the client needs a quiet environment.)

A client requires external radiation therapy. The nurse knows external radiation may cause which problems? Select all that apply 1. Pancytopenia 2. Leukocytosis 3. Erythema 4. Fever 5. Fatigue

1. Pancytopenia 3. Erythema 5. Fatigue (1., 3. & 5. Correct: Effects of radiation therapy include, but are not limited to pancytopenia (marked decrease in the number of RBCs, WBCs and platelets), erythema (redness of the skin), and fatigue. 2. Incorrect: Leukocytosis is an increase in WBCs. External radiation causes pancytopenia which is a decrease in the number of blood cells including WBCs. 4. Incorrect: Fever is not typically seen with external radiation.)

A nurse is providing dietary instructions for a client diagnosed with liver disease. Which food should the nurse instruct the client to increase in their diet? 1. Pasta 2. Olive oil 3. Spinach 4. Cantaloupe

1. Pasta (1. Correct: The client is encouraged to eat an increased amount of carbohydrates in their diet. Carbohydrates should be the major source of calories in their diet. The increased carbohydrates help to reduce the protein break down in the liver. 2. Incorrect: Olive oil has no carbohydrate content. Increasing the amount of olive oil will not affect the need for increase carbohydrates in the diet of clients diagnosed with liver disease. 3. Incorrect: A cup of spinach has about 1 gram of carbohydrates. Spinach is not a vegetable that is high in carbohydrates. 4. Incorrect: One half cup of diced cantaloupe which is a serving size contains 6.5 grams of carbohydrates. This food would not have enough carbohydrates to help reduce the protein break down in the liver.)

The nurse is caring for a burn client in the emergent phase. The client becomes extremely restless while on a ventilator. What is the priority nursing assessment? 1. Patency of endotracheal tube. 2. Adventitious breath sounds. 3. Fluid in the ventilator tubing. 4. Ventilator settings.

1. Patency of endotracheal tube. (1. Correct: With restlessness, think hypoxia so the nurse should start assessment with airway first. Check for patency of the ET tube. If this is patent, then the other options would be next. 2. Incorrect: This is the next best answer, but hypoxia and airway comes first. 3. Incorrect: This is the third step. Rule out the other two before checking tubing for kinks or obstructions. 4. Incorrect: Start with the client first. Then move toward the ventilator. Always assess the client first.)

The community health nurse is planning to teach nutritional education to a group of adults attending a health fair. What tips about health eating should the nurse include? Select all that apply 1. Pay attention to fullness cues during meals. 2. Make one fourth of the plate fruits and vegetables. 3. Drink sweet tea rather than soft drinks with meals. 4. Eat foods low in dietary fiber. 5. Consume less than 30% of calories from saturated fatty acids. 6. Use a smaller plate for meals.

1. Pay attention to fullness cues during meals. 6. Use a smaller plate for meals. (1., & 6. Correct: Pay attention to hunger and fullness cues before, during, and after meals. Use them to recognize when to eat and when you have had enough. Portion out foods before eating. A smaller plate will make the amount of food look larger. 2. Incorrect: Make half the plate fruits and vegetables. 3. Incorrect: Cut calories by drinking water or unsweetened beverages rather than drinks with sugar, such as soft drinks and sweet tea. 4. Incorrect: Diets should be high in fiber coming from fruits, vegetables, and whole grains. 5. Incorrect: Individuals should consume less than 10% of calories from saturated fatty acids (approximately 20 grams of saturated fat per day in a 2000 calorie diet).)

A client has been admitted to the orthopedic floor following application of a long leg cast for a fractured femur. What nursing action takes priority? 1. Perform neurovascular checks of the extremities. 2. Cover the edge of the cast near the groin area. 3. Instruct client not to insert anything into cast. 4. Use palms of hands to lift and position the cast.

1. Perform neurovascular checks of the extremities. (1. Correct: The most vital aspect of care for clients with a fracture and/or cast is frequent neurovascular checks. Circulation can quickly become compromised secondary to edema from the injury or application of the cast, leading to permanent nerve and tissue damage. Neurovascular checks are performed every two hours for the first 24 hours, or more often per hospital protocols, and both extremities must be compared when looking for problems. 2. Incorrect: While this is a vital action by the nurse, it is not the initial priority. Because this client has a long leg cast for a fractured femur, there is the potential for urine to contaminate the cast close to the groin. That would impair the integrity of the cast, or potentially cause an infection. The nurse definitely needs to cover the upper edges of the cast near the groin with water proof material, but there is another action to complete first. 3. Incorrect: Clients must always be instructed on self care or equipment function as part of the recovery process. Proper cast care following discharge is essential and, in particular, the importance of not placing anything down inside the cast. Clients tend to complain of itching skin beneath a cast and may put baby powder, corn starch or other objects inside the cast to scratch. All these can cause serious complications, and the nurse must provide specific teaching to prevent such problems. However, those instructions are not the most immediate priority for the nurse at this time. 4. Incorrect: Casting material can take up to 24 hours to dry hard enough to protect the client's injury. In the meantime, careful handling of the cast when positioning the client is crucial. The nurse is aware that the cast must be lifted using only the palms of the hands to prevent indentations which could injure the client's skin beneath the casting material. These instructions must also be relayed to any personnel providing care to the client; however, this is not the first priority.)

The client is experiencing autonomic dysreflexia. What is the first action by the nurse? 1. Place in high Fowler's position 2. Find and remove the trigger source 3. Notify the primary healthcare provider 4. Check for fecal impaction

1. Place in high Fowler's position (1. Correct: This first action provides some immediate relief to decrease the blood pressure while you are preparing for other interventions. This is one thing the nurse can do immediately to help fix the problem. 2. Incorrect: Later you will look for bladder or bowel distention which is a common precipitating cause of autonomic dysreflexia. 3. Incorrect: The primary healthcare provider will be notified after the nurse intervenes quickly with appropriate nursing measures. 4. Incorrect: Sit client up is the priority and then look for causes.)

A case manager is evaluating a client diagnosed with hemiplegia due to a cerebral vascular accident for assistive devices that will be needed upon discharge. Which resources should the case manager include for this client? Select all that apply 1. Plate guards 2. Transfer belt 3. Raised toilet seat 4. Long handled shoe horn 5. Wide grip utensils 6. Large button closures on clothes

1. Plate guards 2. Transfer belt 3. Raised toilet seat 4. Long handled shoe horn 5. Wide grip utensils (1., 2., 3., 4., & 5. Correct: The goal is to promote self-care by the client as much as possible. The case manager should evaluate the need for assistive devices to help with eating, bathing, dressing, and ambulating. The plate guard will prevent food from being pushed off of the plate. The transfer belt will provide safety for the client and family member who is assisting the client to get up into a chair or back in bed. A raised toilet seat makes it easier for the client to sit on the toilet without falling. The long handled shoe horn allows the client to put on shoes without assistance. Wide grip utensils accommodate a weak grip. 6. Incorrect: It is hard for someone with hemiplegia to use buttons. Velcro fasteners are best.)

The nurse, caring for a client who has terminal cancer, finds that the client is extremely restless. In response to this data, what would be the appropriate nursing action? Select all that apply 1. Play soothing music. 2. Use chamomile aromatherapy. 3. Place soft restraints on arms. 4. Dim room lights. 5. Keep conversations quiet. 6. Massage forehead.

1. Play soothing music. 2. Use chamomile aromatherapy. 4. Dim room lights. 5. Keep conversations quiet. 6. Massage forehead. (1., 2., 4., 5., & 6. Correct: Music therapy may produce relaxation by quieting the mind and promoting a restful state. Aromatherapy with chamomile may also help overcome anxiety, anger, tension, stress, and insomnia in dying clients. When the lights go down and the room darkens, this signals to the brain that it's time for rest. Keeping conversations quiet will help to decrease stimuli. Simple techniques such as repositioning pillows or bed clothes and gentle massage (if tolerated) can also provide relief from pain. 3. Incorrect: Restraints will only agitate the client more. Remember, use restraints as a last resort.)

A nurse, planning an educational seminar on chronic kidney disease, would invite clients with which medical conditions? Select all that apply 1. Polycystic kidney disease 2. Diabetes 3. Hypertension 4. Glomerulonephritis 5. Acute urinary tract infections (UTI)

1. Polycystic kidney disease 2. Diabetes 3. Hypertension 4. Glomerulonephritis (1., 2., 3., & 4. Correct: Polycystic kidney disease is a genetic condition that causes damage to the kidneys. Clients with diabetes and hypertension make up more than 67% of clients diagnosed with chronic kidney disease. Glomerulonephritis damages the kidneys and can lead to permanent damage. 5. Incorrect: Acute UTIs do not generally lead to chronic kidney disease.)

What symptoms of meningeal irritation would the nurse anticipate when performing an assessment on a newly admitted client with a diagnosis of bacterial meningitis? Select all that apply 1. Positive Kernig's sign 2. Positive Brudzinski's sign 3. Presence of Babinski's reflex 4. Photophobia 5. Severe headache 6. Nuchal rigidity

1. Positive Kernig's sign 2. Positive Brudzinski's sign 4. Photophobia 5. Severe headache 6. Nuchal rigidity (1., 2., 4., 5., & 6 Correct: Brudzinski's sign is the involuntary lifting of the legs when the neck is passively flexed (head is lifted off the examining surface). Kernig's sign is positive when the thigh is bent at the hip and knee at 90 degree angles and attempts to extend the knee are painful, resulting in resistance. Both of these signs are thought to indicate meningeal irritation. These seem to be caused when the motor roots become irritated as they pass through inflamed meninges, and the roots are brought under tension. Photophobia (sensitivity to bright light), severe, unrelenting headache, and nuchal rigidity (stiff neck) are all believed to be due to irritation of the meninges. 3. Incorrect: Babinski reflex is a normal reflex in infants up to age 2, but is a pathological reaction in adults. It is often indicative of severe damage to the central nervous system but is not indicative of meningeal irritation.)

What room assignment by the charge nurse is most appropriate for a client who is being admitted with poor appetite, malaise, and temperature of 101.5ºF (38.6ºC)? 1. Private room. 2. Room with a client who has biliary colic. 3. Room with a client who is 3 days post operative hip replacement. 4. Room with a client who is in skeletal traction due to broken femur.

1. Private room. (1. Correct: In this particular situation, a private room is best due to the elevated temperature. This could mean the client has an infection and is contagious. All of the often clients do not need to be exposed to this client with fever of unknown cause. 2. Incorrect: Does not need to be exposed to infection. Biliary colic is pain due to a gallstone blocking the bile duct. The client may need surgery and definitely should not be exposed to infection. 3. Incorrect: Post op client already at risk for infection. This is not the most appropriate client to room with the new admit. 4. Incorrect: Does not need to be exposed to infection. The client is already at risk for infection due to the skeletal traction. Complications of skeletal traction include risk for bone infection due to a screw being placed in a bone.)

A client is admitted to the pediatric unit with a diagnosis to rule out tuberculosis (TB). What room assignment should the charge nurse make? 1. Private room. 2. Private room and place on protective isolation. 3. Room with a client diagnosed with a respiratory infection. 4. Room with a client who is 24 hours post appendectomy.

1. Private room. (1. Correct: This client should be in a private room to prevent the spread by airborne contamination. In addition, standard precautions should be implemented. Remember, you are trying to protect staff and others without the disease from contracting TB. 2. Incorrect: Airborne isolation is needed to protect staff and others. There are no indications for protective isolation. The term 'protective isolation' describes a range of practices used to protect highly susceptible hospital clients from infection. 3. Incorrect: A respiratory infection client needs a private room. Also, it is best for the client with suspected TB to be in a private room. 4. Incorrect: 24 hours postoperative client does not need exposure to infection. Cross contamination is always a concern with a surgical client.)

The nurse is caring for a postoperative client. The client asks the nurse the purpose of anti-embolic stockings. What is the nurse's best response? 1. Promotes the return of venous blood to the heart and assists in preventing blood clots. 2. Stabilizes any clots to prevent embolization. 3. To increase the blood pressure in the venous system in the legs to promote perfusion. 4. Promotes lymphatic drainage to prevent swelling and arterial congestion.

1. Promotes the return of venous blood to the heart and assists in preventing blood clots. (1. Correct: The anti-embolic stockings promote return of venous blood to the heart and assist in preventing the stasis of blood that can lead to blood clots. 2. Incorrect: The purpose of the anti-embolism stockings is to promote venous return and prevent blood stasis which can result in blood clot formation. Anti-embolitic stockings will not stabilize existing blood clots. 3. Incorrect: Anti-embolism stockings are used to increase venous return. They are not used to increase blood pressure or perfusion to the legs. 4. Incorrect: Compression garments, not anti-embolitic stockings, are used by persons with lymphedema to reduce edema by promoting the flow of lymph fluid out of the affected limb. Anti-embolitic stockings are to help with venous return and preventing stasis of blood and blood clots.)

Which interventions would the nurse initiate to lessen acid reflux in a client diagnosed with gastric esophageal reflux disease (GERD)? Select all that apply 1. Provide small, frequent meals. 2. Avoid carbonated beverages. 3. Administer omeprazole as prescribed. 4. Assist with smoking cessation. 5. Place in right lateral position after eating.

1. Provide small, frequent meals. 2. Avoid carbonated beverages. 3. Administer omeprazole as prescribed. 4. Assist with smoking cessation. (1., 2., 3., & 4. Correct: Gastroesophageal reflux disease is a disorder that results from stomach acid moving backward from the stomach into the esophagus. GERD usually happens because the lower esophageal sphincter (LES) — the muscular valve where the esophagus joins the stomach — opens at the wrong time or does not close properly. All of these actions are correct to help alleviate GERD. 5. Incorrect: It is best for the client to sit upright for 3 hours after a meal and to not eat 2 to 3 hours before going to bed.)

A child is brought into the emergency department (ED) after accidently ingesting 3 grams of acetylsalicylic acid. Initial assessment reveals lethargy, excessive sweating, hyperventilation, and hyperthermia. What interventions should the nurse initiate? Select all that apply 1. Provide tepid water sponge bath. 2. Start an IV for fluid resuscitation. 3. Insert a nasogastric tube. 4. Pad side rails. 5. Obtain blood gases. 6. Administer ipecac syrup orally.

1. Provide tepid water sponge bath. 2. Start an IV for fluid resuscitation. 3. Insert a nasogastric tube. 4. Pad side rails. 5. Obtain blood gases. (1., 2., 3., 4., & 5. Correct: This client has hyperthermia. Methods to decrease temperature include external cool down, such as with a tepid water sponge bath. Dehydration occurs early in aspirin poisoning due to vomiting and hyperventilation. IV fluid is needed to offset the dehydration. Gastric lavage and activated charcoal are needed to deactivate the aspirin. The child is at risk for seizures so pad the side rails for safety. Care is based on blood gas results. Metabolic acidosis is the imbalance of the most concern. 6. Incorrect: Although ipecac syrup was used commonly in the past to make a client vomit, it is rarely recommended today. It would not be suggested in aspirin poisoning due to the chance that the client might develop altered mental status or convulsions.)

A nurse is attempting to help a client who has self-care difficulty due to left-sided hemiparesis. Which interventions should the nurse plan to include? Select all that apply 1. Provide the client with a button hook for dressing. 2. Discourage use of electric razors and toothbrushes. 3. Have client comb own hair. 4. Offer to take the client to the toilet every four hours. 5. Avoid relying on furniture for support when walking.

1. Provide the client with a button hook for dressing. 3. Have client comb own hair. 5. Avoid relying on furniture for support when walking. (1., 3. & 5. Correct: The use of a button hook or loop and pile closure on clothes may make it possible for a client to continue independence in this self-care activity. This is a one handed task that will enable the client to maintain autonomy for as long as possible. Having client comb own hair helps maintain autonomy. The client should use prescribed assistive devices for ambulation. Furniture may move or not be in the correct place for support while walking. 2. Incorrect: The client can be helped by using an electric razor and toothbrush. These will improve client safety during self care. 4. Incorrect: Offer bedpan or place client on toilet every 1 to 2 hours during the day and three times during the night.)

A client returns to the unit after a liver biopsy. Which nursing interventions would the nurse implement? Select all that apply 1. Put a pillow under the costal margin. 2. Place in the right side lying position. 3. Perform passive range of motion exercises to right shoulder. 4. Take vital signs every 10 - 15 minutes for first hour. 5. Instruct the client to avoid strenuous exercise for 1 month.

1. Put a pillow under the costal margin. 2. Place in the right side lying position. 4. Take vital signs every 10 - 15 minutes for first hour. (1., 2., & 4 Correct: The client is placed on the right side and a pillow placed under the costal margin. The pillow will place additional pressure on the rib cage which will assist with applying pressure to the liver capsule. By positioning the client on the right side, the liver capsule at the site of the biopsy is compressed against the chest wall. If the puncture site is not compressed, there is the possibility that blood or bile will leak from the puncture site. The vital signs are measured at 10 - 15 minute intervals for the first hour. Variations of the vital signs will indicate complications such as bleeding, severe hemorrhage, and bile leakage. 3. Incorrect: Passive range of motion exercises is not correct. The shoulder is not placed in a position during and after the biopsy to warrant passive exercises to the shoulder. 5. Incorrect: The client should be instructed to avoid strenuous exercise for 1 week not 1 month. The strenuous exercise is restricted to 1 week to prevent the possibility of liver bleeding.)

The nurse is monitoring the client's heart rhythm. The monitor shows sinus tachycardia. What is expected with this assessment finding? Select all that apply 1. Regular rhythm 2. Rate of 101-200 3. P wave normal 4. P-R interval not measurable 5. QRS complex normal

1. Regular rhythm 2. Rate of 101-200 3. P wave normal 5. QRS complex normal (1., 2., 3. & 5. Correct: Sinus tachycardia indicates a regular rhythm, although the rate is elevated. The term tachycardia is defined as a heart rate above 100. The P-wave is normal in a sinus rhythm. Sinus rhythms have a normal QRS complex. 4. Incorrect: P-R interval is not measurable in atrial flutter, atrial fib, PVCs, V tach or V fib.)

A client returns to the nursing unit post-thoracotomy with two chest tubes in place connected to a drainage device. The client's spouse asks the nurse about the reason for having two chest tubes. The nurse's response is based on the knowledge that the upper chest tube is placed to do what? 1. Remove air from the pleural space 2. Create access for irrigating the chest cavity 3. Evacuate secretions from the bronchioles and alveoli 4. Drain blood and fluid from the pleural space

1. Remove air from the pleural space (1. Correct: A chest tube placed in the upper chest is to remove air from the pleural space. Remember air rises and fluid settles down low. 2. Incorrect: Chest tubes are placed in the pleural space to get rid of air, blood, fluid, or exudate so that the lung can re-expand. The purpose is not to create an access for irrigating the chest cavity. 3. Incorrect: The chest tube is inserted into the pleural space because the lung has collapsed due to air, blood, fluid, or exudate. The chest tube does not go into the lung so secretions can not be removed from the bronchioles and alveoli by way of the chest tube. 4. Incorrect: You have to know the purpose of the upper chest tube. Fluid drains down, so the lower one is for fluid.)

A hiker that was lost in the mountains for 3 days experienced exposure to below freezing temperatures. Upon arrival to the emergency department (ED), the nursing assessment reveals hard, mottled, bluish-white toes bilaterally, and the client reports being unable to feel the toes. Which actions should the nurse initially take? Select all that apply 1. Remove any wet or constricting clothing. 2. Initiate a controlled and rapid rewarming process with warm water. 3. Wrap each toe individually with sterile gauze. 4. Encourage the client to walk. 5. Apply a heating pad to the feet. 6. Massage the frozen digits.

1. Remove any wet or constricting clothing. 2. Initiate a controlled and rapid rewarming process with warm water. 3. Wrap each toe individually with sterile gauze. (1., 2., & 3. Correct: Wet clothing is removed to eliminate continued exposure to the cold and allow the warming process to begin. Swelling is common so anything, such as clothing or jewelry that could cause constriction to blood flow should be removed. A controlled and rapid re-warming process is accomplished using a continuous flow of warm water until flushing is noted in the affected areas. Antiseptics or antibiotics are often used, and each digit is wrapped individually with sterile gauze (not constricting) to minimize the risk of infection and assist in the warming process. The core should be re-warmed first to prevent "afterdrop" which is a further drop in core temperature caused by cold peripheral blood returning to the central circulation. 4. Incorrect: Movement of frostbitten areas can cause ice crystals to form in the tissue and cause further damage. In addition, lack of sensation places the client at risk for falls or other injury. 5. Incorrect: External heat such as heating pads, fireplaces, etc. should not be used because burns are more likely to result due to the presence of decreased sensation in the affected areas. 6. Incorrect: Initial rubbing or massage of the frostbitten digits is an absolute contraindication as it can cause further tissue damage. Gentle handling is required to prevent stimulation of the cold myocardium.)

A client who has recurrent episodes of allergic rhinitis asks the nurse what could be done to decrease symptoms. What instruction should the nurse provide to this client? Select all that apply 1. Remove pets from interior of home. 2. Treat a stuffy nose with warm salt water. 3. Remove carpeting. 4. Stay inside when pollen count is at its lowest. 5. Wash bed linens in hot water.

1. Remove pets from interior of home. 2. Treat a stuffy nose with warm salt water. 3. Remove carpeting. 5. Wash bed linens in hot water. (1., 2., 3., & 5. Correct: Controlling pet dander and other pet allergens is one way to reduce allergic rhinitis. Keep pets outside. Cleaning nasal passages with saline solution (warm salt water) will relieve stuffy nose. Control dust and dust mites by dusting regularly, removing carpeting and washing bed linens in hot water. 4. Incorrect: Stay inside with closed doors and windows during high-pollen season.)

Following a thyroidectomy, a client reports shortness of breath and neck pressure. Which nursing action is the best response? 1. Remove the dressing and elevate the head of bed. 2. Call a code, open the trach set, and position the client supine. 3. Obtain vital signs. 4. Immediately go to the nurse's station and call the primary healthcare provider.

1. Remove the dressing and elevate the head of bed. (1. Correct: The nurse should identify that the client is in respiratory distress. So get the dressing off the neck, elevate the HOB and see if they can breathe any better. Stay with the client. 2. Incorrect: Calling a code and opening a trach set is premature. What is likely the problem? Swelling around the airway. Do something that will decrease swelling. Placing the client flat will make the swelling and breathing worse. 3. Incorrect: Don't just look and check. The nurse must do something. This is delaying treatment. Checking the vital signs will not correct the problem. 4. Incorrect: Never leave an unstable client. If the client is having trouble breathing, then that client is unstable. The nurse can call the primary healthcare provider from the room.)

The nurse is reviewing sequential lab results on a newly admitted client with multiple health issues. Critical changes in which body system require the nurse to immediately notify the primary healthcare provider? Lab value on 1/1 → 1/2 → 1/3 Sodium: 143 → 138 → 137 mEq/L Potassium: 4.2 → 4.6 → 5.0 mEq/L Chloride: 100 → 102 → 104 mEq Glucose: 99 → 102 → 104 mg/dL BUN: 16 → 19 → 22 mg/dL Creat.: 1.3 → 1.6 → 2.0 mg/dL Venous CO₂: 24 → 26 → 27 mEq/L 1. Renal 2. Endocrine 3. Pulmonary 4. Cardiovascular

1. Renal (1. Correct: All lab values are fluctuating, but those most significantly outside of normal range are the BUN and Creatinine levels, reflecting possible renal failure. The nurse would need to immediately notify the primary healthcare provider of possible complications in the client's renal system. The sodium, potassium, and glucose are within normal limits. 2. Incorrect: Several lab readings could relate to the endocrine system, but most specifically are glucose and chloride. Both these electrolytes have fluctuating levels but remain well within normal limits. Therefore, the endocrine system is not the nurse's concern at this time. 3. Incorrect: The carbon dioxide levels listed reflect venous, NOT arterial, blood. Norms for venous carbon dioxide are 23 to 29 mEq/L (milliequivalent units per liter of blood), indicating these results are all within normal levels. Although chloride could also reflect the pulmonary system, there are no irregular results in chloride levels. 4. Incorrect: Many of these elements could affect the cardiovascular system, but most specifically sodium and potassium. At present, these levels are all within normal limits, although the potassium has risen to the upper most levels of normal. If those levels continue to climb, this could become a concern; however, this would not require a call to the primary healthcare provider at this time.)

The client is being admitted for a myocardial infarction (MI). Which assessment finding is expected? 1. Reports of nausea and vomiting 2. Elevated temperature higher than 102 degrees F (38.89 degrees C) in the first 24 hours. 3. Pain relieved by two aspirin tablets. 4. Myoglobin will be negative.

1. Reports of nausea and vomiting (1. Correct: Pain with MI may have associated symptoms that include nausea, vomiting, diaphoresis, palpitations, and shortness of breath. 2. Incorrect: Temperature as high as 102 degrees F (38.89 degrees C) may occur for several days after infarction. Temperature higher than this would not be expected. 3. Incorrect: Aspirin is given to prevent platelet aggregation and does not relieve chest pain from an MI. Nitroglycerin and morphine will help with pain associated with an MI. 4. Incorrect: A negative myoglobin will rule out an MI.)

A client is admitted to the critical care unit after suffering from a massive cerebral vascular accident. The client's vital signs include BP 160/110, HR 42, Cheyne-Stokes respirations. Based on this assessment the nurse anticipates the client to be in which acid/base imbalance? 1. Respiratory acidosis 2. Respiratory alkalosis 3. Metabolic acidosis 4. Metabolic alkalosis

1. Respiratory acidosis (1. Correct: Causes of respiratory acidosis include any causes of decreased respiratory drive, such as drugs (narcotics) or central nervous system disorders. With a massive cerebral vascular accident (CVA or stroke), the respiratory center in the brain is impaired and affects oxygenation. Cheyne-Stokes respirations are characterized by progressively deeper and sometimes faster respirations followed by periods of apnea. This leads to acidosis and often times respiratory arrest. 2. Incorrect: Respiratory alkalosis includes hyperventilation and tachypnea which does not describe the characteristics of Cheyne-Stokes respirations. 3. Incorrect: Metabolic acidosis includes tachypnea with deep respirations called Kussmaul's respirations caused by disorders like DKA. 4. Incorrect: The most common cause of metabolic alkalosis is vomiting, and this is a respiratory problem, not metabolic.)

A client is admitted to the hospital with acute exacerbation of COPD following an upper respiratory infection. His daughter found him at home, confused and in respiratory distress, a day after he developed a cold. He was placed on 4 L/min of oxygen via nasal cannula, but oxygen saturation remains at 89%. Based on this assessment, the nurse suspects that the client has developed which acid base imbalance? 1. Respiratory acidosis 2. Respiratory alkalosis 3. Metabolic acidosis 4. Metabolic alkalosis

1. Respiratory acidosis (1. Correct: Look at all the hints in this stem: COPD, upper respiratory infection, respiratory distress, confused, oxygen saturation of 89%. This client is having lung problems. So you should be able to identify the acid base imbalance as a respiratory problem, right? Yes. Why is it acidosis? Poor gas exchange! Respiratory failure, COPD, and muscular weakness can lead to respiratory acidosis. So you would expect the pH to be < 7.35, and the pCO₂ to be > 45. The HCO₃⁻ would be normal. 2. Incorrect: Not alkalosis. You would expect respiratory alkalosis with a client who is hyperventilating, such as the hysterical client. The client in this question would be hypoventilating and having poor gas exchange. 3. Incorrect: Not a metabolic related acid/base imbalance. Metabolic problems do not start with a respiratory problem. Metabolic acidosis is seen with diabetic ketoacidosis or starvation. 4. Incorrect: This is not a metabolic problem but a respiratory problem. Metabolic alkalosis may be seen with prolonged vomiting and hypokalemia.)

The nurse is assessing a client admitted with acute gastritis. Which client information is most significant? 1. Takes ibuprofen for arthritis pain. 2. Had an upper respiratory infection two weeks ago. 3. Has a stressful job. 4. Enjoys spicy food.

1. Takes ibuprofen for arthritis pain. (1. Correct: Ibuprofen is a non-steroidal anti-inflammatory drug (NSAID). NSAIDs are highly associated with GI irritation. 2. Incorrect: Upper respiratory infections have nothing to do with gastritis. 3. Incorrect: Research does not support an association between gastritis and stressful jobs. 4. Incorrect: Spicy foods may not be tolerated by clients with gastritis, but spicy foods have not been linked to causing gastritis.)

Based on the results of the arterial blood gases (ABGs), what imbalance does the nurse understand the client to be exhibiting? pH - 7.35 PaO₂ - 95% PaCO₂ - 49 HCO₃⁻ - 30 1. Respiratory acidosis compensated 2. Respiratory acidosis partially compensated 3. Metabolic acidosis compensated 4. Metabolic acidosis partially compensated

1. Respiratory acidosis compensated (1. Correct: The pH is normal but is on the acidic side of normal. The PaCO₂ is elevated, causing acid formation. The HCO₃⁻ is alkalotic and is increased to buffer the acid. The pH and PaCO₂ match, so the original problem was respiratory acidosis, but compensation has occurred since the pH is now normal. 2. Incorrect: The pH is normal but is on the acidic side of normal. The PaCO₂ is acid. The HCO₃⁻ is alkalotic. The pH and PaCO₂ match, so the original problem was respiratory acidosis, but compensation has occurred since the pH is now low. 3. Incorrect: The pH is normal but is on the acidic side of normal. The PaCO₂ is acid. The HCO₃⁻ is alkalotic. The pH and PaCO₂ match, so the original problem was respiratory acidosis, but compensation has occurred since the pH is now low. 4. Incorrect: The pH is normal but is on the acidic side of normal. The PaCO₂ is acid. The HCO₃⁻ is alkalotic. The pH and PaCO₂ match, so the original problem was respiratory acidosis, but compensation has occurred since the pH is now low.

Which food selections would need to be removed from the tray by the nurse for a client recovering from thyroidectomy? 1. Roasted almonds 2. Mashed vegetables 3. Scrambled eggs 4. Ice cream

1. Roasted almonds (1. Correct: Too hard and crunchy. Need soft diet because esophagus is right behind the thyroid and trachea. This would be difficult to swallow after surgery due to pain. 2. Incorrect: Mashed vegetables will be soft and easy to swallow. 3. Incorrect: This would be good for the client. The food is soft and easy to swallow. 4. Incorrect: Ice cream with neck surgery. Cold and soft.)

A nurse is caring for a client who is on bed rest following admission to the hospital two days ago with a diagnosis of new onset heart failure. While evaluating the client's progress, what assessment findings would indicate to the nurse that further treatment is required? Select all that apply 1. Sacral edema 2. Orthopnea 3. Shiny skin 4. S3 heart sound 5. Heart rate 88/min 6. CVP 8mmHg

1. Sacral edema 2. Orthopnea 3. Shiny skin 4. S3 heart sound 6. CVP 8mmHg (1., 2., 3., 4., & 6. Correct: These are all signs of fluid volume excess seen with heart failure. 5. Incorrect: This is a normal heart rate which would indicate the client is improving.)

Which discussion points should a nurse plan to include when teaching a group of college students on prevention of sexually transmitted infections (STI)? Select all that apply 1. Safe sex practices 2. Routine human immunodeficiency virus (HIV) testing 3. Proper use of birth control pills 4. Sexual abstinence 5. Vaccinations for STIs

1. Safe sex practices 4. Sexual abstinence 5. Vaccinations for STIs (1., 4. & 5. Correct: All of these topics should be included when discussing prevention of STIs. Safe sex practices include proper use of condoms. Abstinence is the best way to prevent STIs. Vaccines are available for some STIs such as human papillomavirus vaccine (HPV). 2. Incorrect: Routine HIV testing is not a way to prevent HIV or other STIs. It will provide early diagnosis. The best course of action is to prevent occurrence.3. Incorrect: Birth control pills help prevent unplanned pregnancy. STIs can still be contracted if proper safe sex techniques are not implemented.)

The homecare nurse is instructing a client with chronic obstructive pulmonary disease (COPD) about the importance of a nutritious diet to avoid weight loss. The nurse knows that teaching has been effective when the client selects which foods for a breakfast menu? Select all that apply 1. Scrambled eggs 2. Cheese omelet 3. Sliced banana 4. Orange juice 5. Whole milk 6. Dry toast

1. Scrambled eggs 2. Cheese omelet 3. Sliced banana 4. Orange juice (1., 2., 3. & 4. Correct: Maintaining weight and nutrition is vital to the health of clients with (COPD). Extreme fatigue along with excessive mucus production decreases the client's ability to eat complete, well-balanced meals, leading to weight loss or malnourishment. Therefore, the nurse would instruct the client to eat small, frequent meals high in protein and fiber. Good sources of protein include eggs, cheese, fish and poultry, beans and even nuts. Fresh fruit such as bananas along with non-carbonated beverages such as orange juice are excellent breakfast food choices. 5. Incorrect: Although milk and dairy products like yogurt could be considered part of a healthy breakfast, it is recommended that COPD clients use 1% or 2 % milk products to avoid increasing mucus production. This client should select the orange juice from the choices provided. 6. Incorrect: Dry toast provides little nutrient value, and may actually increase coughing because of its brittle nature. Coughing quickly leads to exhaustion rather than eating. This client would benefit more from a more palatable choice such as muffin or French toast.)

Which male client condition in the after-hours clinic should the nurse assess first? 1. Scrotal pain and edema. 2. Erection lasting for 2 hours. 3. Inability to void with a history of benign prostatic hyperplasia (BPH). 4. Purulent drainage from the penis.

1. Scrotal pain and edema. (1. Correct: This client is likely to have testicular torsion, which requires immediate intervention. Infarction of the testes can occur if not treated promptly. 2. Incorrect: This is not the most life threatening problem. Priapism, a persistent, often painful erection that lasts for more than 4 hours should be treated. 3. Incorrect: With BPH the prostate gland increases in size, leading to disruption of the outflow of urine. This can cause inability to void and needs to be assessed but is not the first priority. 4. Incorrect: This client does not have the most serious condition and would not take priority.)

A nurse is educating the family of a client in the middle stages of Alzheimer's disease how to encourage independence during meals. What points should the nurse include? Select all that apply 1. Serve meal in a quiet environment 2. Give 30 minutes to eat 3. Serve finger foods 4. Serve one dish at a time 5. Do not worry about neatness

1. Serve meal in a quiet environment 3. Serve finger foods 4. Serve one dish at a time 5. Do not worry about neatness (1., 3., 4., & 5. Correct: Limit distractions by serving meals in quiet surroundings, away from the television and other distractions. Too many foods at once may be overwhelming. Simplify by serving one dish at a time. For example, mashed potatoes followed by meat. Serve finger foods, which are foods easy to pick up to eat. Do not worry about neatness. Let the person feed self as much as possible. Consider plates with suction, built-up rims and no spill glasses to allow users to more easily place food on their utensils. 2. Incorrect: Give the person plenty of time to eat. Remind client to chew and swallow carefully. Keep in mind that it may take an hour or longer to finish eating.)

A nurse is caring for a client diagnosed with the ebola virus who is experiencing vomiting and diarrhea. What personal protective equipment should be worn by the nurse while providing care to this client? Select all that apply 1. Single use impermeable gown 2. Powered Air Purifying Respirator (PAPR) or N95 respirator 3. One pair of sterile gloves 4. Single use boot covers 5. Single use apron

1. Single use impermeable gown 2. Powered Air Purifying Respirator (PAPR) or N95 respirator 4. Single use boot covers 5. Single use apron (1., 2., 4., & 5. Correct: The nurse should wear a single use (disposable) impermeable gown OR a single use impermeable coverall. Either a PAPR or a disposable, NIOSH-certified N95 respirator should be worn to reduce the risk of contamination in the case of an emergency situation where a potentially aerosol-generating procedure would be performed. The PAPR reduces the risk of self-contamination while providing client care, but the N95 respirator is less bulky. If the N95 respirator is selected for use, nurses should be extremely careful to make sure that they do not accidentally touch their faces under the face shield during client care. Disposable boot covers should be worn and should extend to at least mid-calf. Some agencies may add the single use shoe covers over the boot covers to reduce the risk of contaminating the underlying shoes. If clients with Ebola are vomiting or have diarrhea, a single use (disposable) apron should be worn over the gown to cover the torso to mid-calf. This will provide additional protection to reduce the risk of contaminating the gown (or coveralls) by the infectious body fluids and also provides a way to rapidly remove a soiled outer layer if contamination occurs on the apron. 3. Incorrect: Sterile gloves are not required, but two pairs, instead of one pair, of gloves should be worn so that a contaminated outer glove can be safely removed when providing client care or safely removed without self-contamination when removing the PPE. These gloves should at the very least have extended cuffs.)

A community health nurse, participating in a health fair, is educating a community group about risk factors for developing varicose veins. What risk factors should the nurse include? Select all that apply 1. Sitting for prolonged periods 2. Obesity 3. Female 4. Leg exercises 5. Wearing high-heeled shoes

1. Sitting for prolonged periods 2. Obesity 3. Female 5. Wearing high-heeled shoes (1., 2., 3, & 5. Correct: These are all risk factors for developing varicose veins: Sitting or standing for prolonged periods of time, obesity, female gender, wearing high-heeled shoes. 4. Incorrect: Exercise is good for preventing varicose veins. Get moving. Walking is a great way to encourage blood circulation to the legs.)

What discharge instructions should the nurse include for a client following a transsphenoidal hypophysectomy? Select all that apply 1. Sleep with head of bed at 35 degrees. 2. Notify the primary healthcare provider for an increased urinary output. 3. Brush the teeth three times a day followed by rinsing with a commercial mouthwash. 4. Nasal packing will need to be removed in 48 hours. 5. Use a humidifier in the room.

1. Sleep with head of bed at 35 degrees. 2. Notify the primary healthcare provider for an increased urinary output. 5. Use a humidifier in the room. (1.,2., 5. Correct: Sleeping with the head of the bed elevated will promote drainage of cerebrospinal fluid. An increased UOP could indicate diabetes insipidus, an adverse reaction to this surgical procedure. Humidified air prevents drying of nasal passages. 3. Incorrect: Because the incision for this surgery is just above the gumline, the client should not brush the front teeth. Oral care should be performed with a sponge until the incision heals. 4. Incorrect: There is no nasal packing. The incision is located just above the gumline of the upper teeth.)

What should the nurse include in the teaching plan for a client receiving external beam radiation? Select all that apply 1. Small marks will be placed on the skin to mark the treatment area. 2. Lotion may be used around the treatment area to decrease dryness. 3. The radiation therapist can see, hear, and talk with you at all times during treatment. 4. Stay away from babies for 24 hours. 5. You will have to hold your breath during radiation treatment.

1. Small marks will be placed on the skin to mark the treatment area. 3. The radiation therapist can see, hear, and talk with you at all times during treatment. (1., & 3. Correct: Small ink marks or small tattoos will be placed on the skin to mark the treatment area. Do not remove the marks. The radiation therapist can see, hear, and talk to the client at all times during treatment. Relieve anxiety by letting client know he/she is not alone. 2. Incorrect: Do not put lotion, powder or deodorant near or on treatment area. 4. Incorrect: Client is not radioactive and will not radiate others. The client can safely be around other people, babies, and children. 5. Incorrect: The client will need to stay very still so radiation goes to the exact same place each time, but can breathe as always and does not have to hold breath.)

The nurse is working with a group of elderly clients to promote better nutrition. Prior to developing the health promotion plan, the nurse assesses individual members of the group. Which assessment findings are expected as the nurse works with this group? Select all that apply 1. Some clients may have dental issues, making chewing difficult. 2. There may be a decreased appetite in clients. 3. Caloric and nutritional needs may vary somewhat depending on activity levels. 4. Access to fresh foods is adequate. 5. The desire and interest in cooking is increased.

1. Some clients may have dental issues, making chewing difficult. 2. There may be a decreased appetite in clients. 3. Caloric and nutritional needs may vary somewhat depending on activity levels. (1., 2. & 3. Correct: Many elderly people have dental issues that affect chewing and intake of nutritionally dense foods. Appetite may decrease due to changes in taste, medications, depression or isolation. Many elderly people are active; therefore, it is important to assess each one individually in regard to activity levels. 4. Incorrect: Many elderly clients may not have access to fresh foods due to infrequent grocery shopping, limited budgets, and a desire to not waste good food. 5. Incorrect: Many elderly do not have a desire to cook for one or two. Pain and physical impairment may also decrease desire or interest in cooking.)

A petite female client presents to the clinic with symptoms of back pain and states, "I think I am getting shorter." Which information would be appropriate for the nurse to provide? Select all that apply 1. Spend time in the sunlight. 2. Wear low heeled, nonslip sole shoes. 3. Walk at least 30 minutes most days. 4. Include yogurt and cheese in diet. 5. Take regularly scheduled prescribed corticosteroids.

1. Spend time in the sunlight. 2. Wear low heeled, nonslip sole shoes. 3. Walk at least 30 minutes most days. 4. Include yogurt and cheese in diet. (1., 2.,3. & 4. Correct: The client with osteoporosis is usually female, and small framed. Back pain from collapsed vertebrae and shortening are symptoms. Exposure to light converts vitamin D stores in the skin. Low-heeled, nonslip soled shoes are a safety issue to prevent loss of balance and falls. Weight bearing exercises like walking will promote bone density. Yogurt and cheese are high in calcium. 5 Incorrect: This individual should not be taking corticosteroids as these drugs will promote the loss of calcium from the bones.)

A client with Cushing's disease is in a semi-private room. When considering room assignments, which client would be the safest choice to assign to this room? 1. Status asthmaticus receiving respiratory therapy. 2. Newly admitted client with shingles. 3. Client with 50% burn of both arms. 4. Client who is post emergency appendectomy.

1. Status asthmaticus receiving respiratory therapy. (1. Correct: Status asthmaticus is not infectious and would be the best roommate for the client with Cushing's disease who is immunosuppressed due to excessive secretion of glucocorticoids. 2. Incorrect: Shingles is a contagious infectious disease and not the best choice to room with the client who has Cushing's disease. Remember, the client with cushing's is immunosuppressed. 3. Incorrect: Burns are always contaminated wounds, and the client has a decreased immune system, so a high probability for an infected burn would make this client a poor choice to occupy a room with the client who has Cushing's disease. This client is also immunosuppressed. 4. Incorrect: A client post emergency appendectomy is prone to peritonitis or wound infection and not the best client choice.)

A client who has been trying to lose weight reports to the nurse that it is just easier to stop by the fast food restaurant on the way home from work than to go home and prepare a meal. Which interventions could help the client stay on track? Select all that apply 1. Suggest that the client eat yogurt and a piece of fruit upon returning home. 2. Suggest that the client order low fat options at the restaurants. 3. Encourage the client to pack a healthy snack to eat on the way home from work. 4. Inform the client that fast food restaurants do not have healthy food options. 5. Suggest that the client alter her route home from work in the evenings to avoid the fast food restaurants.

1. Suggest that the client eat yogurt and a piece of fruit upon returning home. 2. Suggest that the client order low fat options at the restaurants. 3. Encourage the client to pack a healthy snack to eat on the way home from work. (1., 2. & 3. Correct: The client is describing lack of convenience, a barrier to making better choices. The client can consume yogurt and fruit on the way home and still be making a good choice for dinner. Accessibility of healthier items will help the client stay on track. Availability of healthy foods will help the client stay on her food plan. 4. Incorrect: There are healthier choices currently at most fast food restaurants. Clients should be encouraged to choose from those. 5. Incorrect: While this may help the client stay on track, it may make a healthy choice inaccessible. The client is more likely to make healthy choices when they are accessible, available, and affordable.)

An occupational health nurse works in a factory where loud equipment is used in production of the factory's product. What should the nurse emphasize to factory management persons to reduce the risk of hearing impairment? Select all that apply 1. Supply workers with earplugs when exposed to noise. 2. Replace high noise machinery with low noise machinery. 3. Limit amount of time a person spends at a noise source. 4. Operate noisy machines during shifts when fewer people are exposed. 5. Supply personal noise monitoring to identify employees at risk from hazardous level of noise. 6. Have all employees make an appointment for a hearing test.

1. Supply workers with earplugs when exposed to noise. 2. Replace high noise machinery with low noise machinery. 3. Limit amount of time a person spends at a noise source. 4. Operate noisy machines during shifts when fewer people are exposed. 5. Supply personal noise monitoring to identify employees at risk from hazardous level of noise. (1., 2., 3., 4., & 5. Correct: All of these are primary prevention methods of controlling hearing loss in employees exposed to hazardous noise levels. Earplugs, or earmuffs, are considered an acceptable but less desirable option to control exposures to noise and are generally used during the time necessary to implement engineering and administrative controls to protect worker's hearing. Engineering controls include choosing low noise tools and machinery, maintaining and lubricating machinery and equipment, placing a barrier between the noise source and employed, and enclosing or isolating the noise source. Administrative controls are changes in the workplace that reduce or eliminate the worker's exposure to noise. This includes operating noisy machines during shifts when fewer people are exposed, limiting the amount of time a person spends at a noise source, and using monitoring equipment to monitor hazardous noise level. 6. Incorrect: Some employees may need a hearing test but this will not reduce the risk of hearing impairment. This is secondary prevention, which focuses on screening and early diagnosis of disease.)

What is priority for the client experiencing hyperparathyroid crisis? 1. Support for airway and breathing. 2. Continuous cardiac monitoring for arrhythmias. 3. Provide safety precautions. 4. Prepare for emergency tracheostomy.

1. Support for airway and breathing. (1. Correct: Always remember ABC, if it is relevant, and it is with hyperparathyroid crisis. 2. Incorrect: Circulation is important. This priority comes after attention has been directed toward airway and breathing. What good would come of circulating deoxygenated blood, and how long can the heart muscle last without oxygen? Always remember ABC when prioritizing in emergency situations. 3. Incorrect: Muscle weakness, thus risk for falls is a concern, but airway takes priority! 4. Incorrect: Trach would be more likely with hypoparathyroidism. Remember, in hypoparathyroidism, the client would have rigid and tight muscles which would cause laryngospasms.)

What signs of cannula displacement should the nurse monitor for at an arterial line insertion site? Select all that apply 1. Swelling 2. Fluid leakage 3. Blanching 4. Poor arterial waveform 5. Pyrexia 6. Purulent drainage

1. Swelling 2. Fluid leakage 3. Blanching 4. Poor arterial waveform (1., 2., 3., & 4. Correct: These are signs of cannula displacement. Observe for signs of cannula displacement into the tissues which will be swelling, bleeding, lack of a normal arterial waveform, fluid leakage, blanching, and pain or discomfort. 5. Incorrect: This is a sign of infection rather than cannula displacement. Signs of infection include pain, redness, purulent drainage, and fever. 6. Incorrect: This is a sign of infection rather than cannula displacement.Signs of infection include pain, redness, purulent drainage, and fever.)

Which intervention would the nurse recommend to a client with rheumatoid arthritis to best help relieve joint stiffness? 1. Take a warm shower prior to performing activities of daily living. 2. Take an aspirin after activity to help decrease inflammation. 3. Lose 10 pounds of weight. 4. Apply cold compresses to joints for 30-45 minutes.

1. Take a warm shower prior to performing activities of daily living. (1. Correct: Warm water may provide muscle relaxation, increase blood flow, and reduce stiffness. 2. Incorrect: A mild analgesic may be taken before activity or exercise to decrease pain and inflammation. 3. Incorrect: Weight reduction may be recommended to relieve stress on joints but does not address joint stiffness. 4. Incorrect: Apply cold compresses for 15-20 minutes at a time. Longer than 20 minutes may cause tissue damage.)

The nurse is providing teaching for a client who is being scheduled for outpatient 24 hour electrocardiogram monitoring using a Holter monitor. What should the nurse tell the client to avoid while monitoring is in progress? Select all that apply 1. Taking a shower or bath 2. Performing daily exercises 3. Working around high voltage equipment 4. Being screened at airport security 5. Eating foods that are sources of potassium

1. Taking a shower or bath 3. Working around high voltage equipment 4. Being screened at airport security (1., 3., and 4. Correct: The nurse should teach this client to continue the usual activities while wearing the monitor with a few exceptions. The monitor should be kept dry to ensure that it functions properly. The client should avoid taking a shower or bath or swimming while wearing the monitor. The electrodes could also become detached from the skin if they get wet, which would also interfere with the accuracy of the reading. The client should be advised to not work around high voltage equipment because areas of high voltage can interfere with the function of the electrocardiogram monitoring. In addition, magnetic fields, such as those used for airport screenings, can interfere with the function of the Holter monitor and should be avoided. 2. Incorrect: This client should be encouraged to continue regular routine unless otherwise directed by the primary healthcare provider. The client can perform the usual daily exercise, but should be advised to avoid activities that may cause excessive perspiration that could lead to the electrodes becoming loosened from the skin. 3. Incorrect: There are generally no dietary restrictions while wearing the Holter monitor unless otherwise prescribed by the primary healthcare provider.)

The nurse is caring for a client who has an active herpes simplex 1 lesion on the lip. What measures should be implemented by the nurse? Select all that apply 1. Tell the client to avoid touching the lesion. 2. Scrub the lesion gently with soap and water prior to meals. 3. Apply a thin layer of acyclovir to the lesion 5 times a day. 4. Wear sterile gloves when applying medication to lesion. 5. Ask client to discard lip balm until lesion is resolved.

1. Tell the client to avoid touching the lesion. 3. Apply a thin layer of acyclovir to the lesion 5 times a day. 5. Ask client to discard lip balm until lesion is resolved. (1., 3., & 5. Correct. The number one way of spreading herpes simplex if through oral secretions and sores on the skin. So, the nurse needs to tell the client to avoid touching the lesion. Acyclovir is an antiviral medication that slows the growth and spread of the herpes simplex virus so that the body can fight off the infection. Only enough cream should be applied to cover the sore. Since touching the lesion or secretions from the lesion can spread the virus, lip balm, lip stick, and make up should be discarded until the lesion has healed. 2. Incorrect: Scrubbing the lesion even gently would transfer the virus along the nerves and lips. 4. Incorrect: Sterile gloves are not needed. Clean gloves should be used.)

What would the nurse include in the teaching plan for a client with right sided heart failure? Select all that apply 1. There is a backup of blood in the right upper chamber of the heart. 2. There is swelling of lower extremities. 3. The heart rate decreases. 4. You may experience fatigue and depression. 5. You may have nausea and anorexia.

1. There is a backup of blood in the right upper chamber of the heart. 2. There is swelling of lower extremities. 4. You may experience fatigue and depression. 5. You may have nausea and anorexia. (1., 2. 4. & 5. Correct: The blood backs up into the right atrium and venous circulation. Vascular congestion is evident by swelling of the lower extremities. Clients usually experience fatigue and depression. Ascites may increase pressure on the stomach and intestines causing GI upset with nausea and anorexia. 3. Incorrect: The heart rate increases in an attempt to increase cardiac output.)

Which client could the telemetry charge nurse safely transfer in order to admit a new client? 1. Twenty-four hour post operative carotid endarterectomy. 2. Unstable angina with onset of atrial fibrillation. 3. Status post coronary artery bypass grafting (CABG) with atrial flutter. 4. Myocardial infarction with a history of heart failure.

1. Twenty-four hour post operative carotid endarterectomy. (1. Correct: Yes, this client is the least critical. The carotid endarterectomy is to open or clean the carotid artery and hopefully prevent a stroke. Since no evidence of complications or being unstable is presented, this client should be able to be cared for on a general nursing unit. 2. Incorrect: Needs a telemetry bed with new onset of atrial fibrillation. The description provided also tells you that the client is experiencing unstable angina. Unstable angina can occur at rest and may develop suddenly, progressively worsening in a short time period. Unstable angina should be treated as an emergency because the client is at increased risk for a myocardial infarction (MI). 3. Incorrect: The client who had a CABG and is experiencing an should be considered unstable and requires further cardiac monitoring. Arrhythmias are common complications after CABG and are major causes of morbidity and longer hospital stays. The client should be carefully assessed for how well the rhythm is being tolerated. Ventricular response to the rapid rate from the atria may be slower and the cardiac output can be reduced. 4. Incorrect: Myocardial infarction and history of heart failure needs telemetry. This client is considered unstable. The client needs to be monitored for arrhythmias, signs of decreased cardiac output and for any signs of recurring infarction.)

The homecare nurse is instructing the family of a client recently diagnosed with Parkinson's disease about potential neurologic changes. During the discussion, what signs should the nurse include? Select all that apply 1. Unsteady gait 2. Muscle rigidity 3. Hyperactive reflexes 4. Bradykinesia (slowed movements) 5. Expressive aphasia

1. Unsteady gait 2. Muscle rigidity 4. Bradykinesia (slowed movements) (1., 2 & 4. Correct: Parkinson's disease is a debilitating, progressive neurological disorder of unknown cause. The most classic symptoms include unsteady gait secondary to increasing muscle rigidity and bradykinesia, plus difficulty with purposeful movement. These symptoms worsen over time and are often accompanied by tremors in the extremities at rest. 3. Incorrect: Reflexes in clients with Parkinson's disease become progressively slowed, not hyperactive. Because this disorder affects the midbrain, and ultimately the connection of the basal ganglia, deep tendon reflexes decrease over the course of the disease. Hyperactive reflexes are associated with other neurologic disorders such as multiple sclerosis. 5. Incorrect: Expressive aphasia is associated with brain trauma or cerebral vascular accident (CVA) and prevents the client from verbalizing appropriate or desired terminology. In Parkinson's disease, the client's speech volume becomes too low and very monotone. Also, because of facial muscle rigidity, there is great difficulty articulating words enough to be clearly understood.)

What signs and symptoms of ovarian cancer should a nurse include when educating women? Select all that apply 1. Urinary frequency. 2. Menorrhagia with breast tenderness. 3. Watery vaginal discharge. 4. Increasing abdominal girth. 5. Fullness after a heavy meal.

1. Urinary frequency. 2. Menorrhagia with breast tenderness. 4. Increasing abdominal girth. (1., 2., & 4. Correct: Signs and symptoms of ovarian cancer include irregular menses, increasing premenstrual tension, menorrhagia with breast tenderness, early menopause, abdominal discomfort, dyspepsia, pelvic pressure and urinary frequency. Flatulence, fullness after a light meal, and increasing abdominal girth are significant symptoms. 3. Incorrect: Watery, vaginal discharge is a sign of advanced cervical cancer. 5. Incorrect: A sense of fullness occurs after ingesting a light meal.)

For a client with a major burn, which evaluation criterion identified by the nurse best indicates that fluid resuscitation has been effective during the first 24 hours of care? 1. Urine output of 860 mL / 24 hours. 2. Increase in weight from preburn weight. 3. Heart rate of 122 beats per minute 4. Central venous pressure of 18 mm

1. Urine output of 860 mL / 24 hours. (1. Correct: Urine output is the best indicator of adequate fluid replacement during the first 24 hours. 2. Incorrect: The weight is not a good indicator now because of the large volume of fluids being infused. These extra fluids would increase the weight. Edema is a problem because of third spacing. 3. Incorrect: The heart rate should come down with adequate fluid replacement. 4. Incorrect: The CVP reading is too high. This indicates that too many fluids have been given.)

Which interventions should be included in the nutritional teaching plan to accomplish the goal of a diet lower in fat? Select all that apply 1. Use 2% milk instead of whole milk. 2. Eat air-popped popcorn instead of potato chips. 3. Eat more red meat instead of fish. 4. Incorporate plant sources of protein. 5. Use olive oil instead of vegetable oil when frying.

1. Use 2% milk instead of whole milk. 2. Eat air-popped popcorn instead of potato chips. 4. Incorporate plant sources of protein. (1., 2. & 4. Correct: Two percent milk can reduce the amount of fat consumed daily, not only in milk that the client drinks, but also in foods that contain milk as an ingredient. Air-popped corn contains no fat unless butter is added after popping. The client still is able to have a crunchy snack without the fat. Plant proteins such as kidney, black, or lima beans are good sources of protein without the fat from a meat source. 3. Incorrect: Red meats are high in fat. Chicken, fish, and seafood are better meat choices. 5. Incorrect: Olive oil is low in saturated fat but still a source of fat. While olive oil may be a healthier choice, all fats have essentially the same number of calories per serving. The goal is to reduce the amount of fat in the diet.)

Which health promotion instructions should the nurse provide to a client diagnosed with cirrhosis? Select all that apply 1. Use a shower chair when performing hygiene. 2. Limit alcohol intake. 3. Stop any activity that causes dizziness. 4. Calculate daily sodium intake. 5. Proper hand hygiene.

1. Use a shower chair when performing hygiene. 3. Stop any activity that causes dizziness. 4. Calculate daily sodium intake. 5. Proper hand hygiene. (1., 3., 4., & 5. Correct: Using a shower chair while showering and performing hygiene will help to save energy. Stop any activity that causes chest pain, a marked increase in shortness of breath, dizziness, or extreme fatigue or weakness. High sodium promotes fluid volume excess. The client should maintain a low sodium intake. Proper hand hygiene prevents infection. 2. Incorrect: The client must stop drinking alcohol to halt the progression of cirrhosis.)

What should be included in the discharge teaching plan for a client who has lymphedema post right mastectomy? Select all that apply 1. Use a thimble when sewing. 2. Wear a heavy duty oven mitt for removing hot objects from the oven. 3. Long sleeves should be worn to prevent insect bites. 4. Shave underarms with an electric razor. 5. Avoid wearing jewelry.

1. Use a thimble when sewing. 2. Wear a heavy duty oven mitt for removing hot objects from the oven. 3. Long sleeves should be worn to prevent insect bites. 4. Shave underarms with an electric razor. (1., 2., 3., & 4. Correct: Because lymphedema is a lifelong threat, teach the client hand and arm precautions to minimize the risk of injury, infection, and impaired circulation. All of these options are correct options to minimize these risks. Even a minor injury such as a pin prick or sunburn can cause painful swelling after lymph node removal. 5. Incorrect: The client may wear jewelry that does not inhibit lymph drainage. They should avoid jewelry that constricts the affected arm.)

The nurse is sharing best practice for preventing pressure injuries in clients. What should the nurse include? Select all that apply 1. Use moisturizer daily on dry skin. 2. Massage reddened skin areas. 3. Prevent shearing by maintaining the head of bed at 45 degrees or higher. 4. Place rubber ring (donut) under client's sacral area. 5. Position client at 30 degree tilt when placed on side.

1. Use moisturizer daily on dry skin. 5. Position client at 30 degree tilt when placed on side. (1., & 5. Correct: Moist skin is more pliable than dry skin, so keep dry skin moist by applying moisturizer daily. A good plan for positioning is the 30 degree rule. This plan ensures that the client is positioned and propped so that whatever part of the body is elevated is tilted back to no more than a 30 degree angle to the mattress rather than resting directly on a dependent body prominence. This rule applies to side lying and head of bed elevation positions. 2. Incorrect: Do not massage reddened skin areas. This can damage capillary beds and increase tissue necrosis. 3. Incorrect: Do not keep the head of bed elevated above 30 degrees to prevent shearing. If the client requires greater head elevation because of respiratory problems, they should be tilted up above 30 degrees with pillows behind the back to keep pressure off the sacral/coccyx area. 4. Incorrect: Do not place rubber ring (donut) under client's sacral area. This can cause pressure to the area and can damage capillary beds and increase tissue necrosis. There are products that redistribute tissue load, such as specialty bed mattresses and seat cushions.)

Which interventions should a nurse discuss with a client for primary prevention of skin cancer from exposure to ultraviolet light? Select all that apply 1. Use sunscreen when outdoors. 2. Stay in the shade when outdoors. 3. Wear wide brimmed hats when outdoors. 4. Examine skin every 3 months for changes. 5. Have an annual skin assessment by a dermatologist.

1. Use sunscreen when outdoors. 2. Stay in the shade when outdoors. 3. Wear wide brimmed hats when outdoors. (1., 2. & 3. Correct: Using sunscreen, staying in shaded areas, and wearing wide brimmed hats are effective interventions to prevent skin cancer. 4. Incorrect: Examine your whole body monthly for possible changes that may be precancerous or cancerous lesions. Early detection is considered secondary prevention. 5. Incorrect: Assessment by a dermatologist is not a primary prevention strategy. Early diagnosis is considered secondary prevention.)

A client was admitted to CCU with a diagnosis of acute coronary syndrome. Continuous cardiac monitoring has been implemented. Which assessment finding by the nurse is most significant? 1. Ventricular fibrillation 2. Ventricular tachycardia 3. 2nd degree AV block 4. Atrial fibrillation

1. Ventricular fibrillation (1. Correct: V-fib is the most common lethal dysrhythmia in the initial period following a myocardial infarction. 2. Incorrect: V-tach is significant as it may occur prior to V-Fib. However, V-fib is most significant. 3. Incorrect: The client will still have a cardiac output in second degree heart block. There is no cardiac output with V-fib. The most lethal is V-fib. 4. Incorrect: Atrial fibrillation involves chaotic contractions of the atria, but there is a cardiac output. It is not life-threatening.)

A client who was admitted to coronary care unit with a diagnosis of myocardial infarction is on continuous cardiac monitoring. Which cardiac change noted on the monitor would be of greatest concern? 1. Ventricular tachycardia > 100 bpm 2. Atrial fibrillation with atrial rate > 300 per minute 3. Four premature ventricular contractions within one minute 4. ST segment depression of 0.5 mm

1. Ventricular tachycardia > 100 bpm (1. Correct: Ventricular tachycardia with a ventricular rate greater than 100 per minute can be a precursor to ventricular fibrillation. This rhythm is the most life threatening and would be of greatest concern. 2. Incorrect: Clients diagnosed with atrial fibrillation are at high risk for formation of thrombus. This is a serious concern, but not as great a concern as ventricular tachycardia. 3. Incorrect: Premature ventricular contractions (PVCs) that are less than 6 are worrisome but not considered a precursor to ventricular tachycardia or ventricular fibrillation. 4. Incorrect: ST segment depression of 1 mm or more signifies myocardial ischemia.)

The nurse is caring for a client in the emergency department who presents with hematemesis. What information is most important for the nurse to obtain during the initial assessment? Select all that apply 1. Vital signs 2. History of prior bleeding episodes 3. Medications the client is taking 4. Urinary output 5. Level of consciousness

1. Vital signs 4. Urinary output 5. Level of consciousness (1., 4., & 5. Correct: A set of vital signs and assessment for hypovolemic shock take priority for this client. S/S of shock include thready, rapid pulse, decreased LOC, shortness of breath, cold and clammy skin, and decreased urinary output. 2. Incorrect: History of prior bleeding episodes is important but does not address the immediate problem. 3. Incorrect: Medication history is important, but the nurse must first determine whether or not the client is in shock.)

While completing the nutritional history of a client admitted with pernicious anemia, the nurse determines that the client follows a strict vegan diet. What education should the nurse provide to the client? Select all that apply 1. Vitamin B12, a nutrient needed to prevent pernicious anemia, is found in some foods like meat, fish, eggs, and milk. 2. In order to increase intake of vitamin B12, your diet must contain beef or chicken liver at least once per week. 3. In addition to eating plants, you should eat dairy products and eggs in order to prevent pernicious anemia. 4. Vegetables high in protein include cabbage, carrots and squash. 5. Pernicious anemia occurs when the body produces red blood cells that are larger than normal and result in a lower than normal red blood cell count.

1. Vitamin B12, a nutrient needed to prevent pernicious anemia, is found in some foods like meat, fish, eggs, and milk. 5. Pernicious anemia occurs when the body produces red blood cells that are larger than normal and result in a lower than normal red blood cell count. (1. & 5. Correct: Pernicious anemia is a type of vitamin B12 anemia. The body needs vitamin B12 to make red blood cells. You get this vitamin from eating foods such as meat, poultry, shellfish, eggs, and dairy products. 2. Incorrect: The client does not have to eat meat or dairy products in order to obtain vitamin B-12. Supplements can be taken and the client can eat vegetables that are considered to be high in protein. 3. Incorrect: A strict vegan will not eat dairy products or eggs. 4. Incorrect: For a vegetable to qualify as a low-protein source, it must contain 4g or less of protein. Green vegetables, such as lettuce, cabbage, bell pepper and asparagus provide only 1 to 2g of protein per serving. Orange vegetables, including carrots, sweet potatoes and squash also contain only 1 to 2 g.)

The nurse is caring for a burn victim with a skin graft to the hand. The area is pale and mottled but has good capillary refill. What is the nurse's best action at this time? 1. Warm the room. 2. Submerge the hand in warm water. 3. Order a K pad and apply to hand. 4. Have the client exercise the fingers to increase blood flow.

1. Warm the room. (1. Correct: When caring for clients with skin grafts, we want good circulation, so warm that room up. 2. Incorrect: This will not improve circulation and can lead to infection. 3. Incorrect: This will not improve circulation. Someone who has a skin graft doesn't have good sensation so there is risk of another burn to the graft with this. 4. Incorrect: Working those stiff, cold fingers will further imbalance the oxygen supply. This will not help, particularly if the environment remains cool.)

The emergency room nurse is assessing a client with an eye injury that occurred while chopping wood. The client states the chain saw caused a log to splinter, sending slivers of wood into the right eye. While waiting for the eye specialist, the nurse discusses future safety precautions for such an activity. What safety precautions are most important for the nurse to include in client teaching? Select all that apply 1. Wear heavy gloves. 2. Stand with feet together. 3. Use steel-toed boots. 4. Wear unbreakable googles. 5. Use ear covers and plugs. 6. Wear loose-fitting clothing.

1. Wear heavy gloves. 4. Wear unbreakable googles. 5. Use ear covers and plugs. (1, 4 and 5. Correct: When engaging in a potentially risky activity, precautions should be taken even if the activity has been completed multiple times before. Functional body parts, such as hands, fingers and toes, are particularly vulnerable to injury. Heavy duty work gloves made of leather or suede along with protective eye googles should be worn even before turning on any machines. Ears should also be protected with regulation ear phones or ear plugs because of equipment noise levels. 2. Incorrect: Any activity involving equipment poses a safety risk, no matter how often an individual completes that action. A client should have both feet firmly planted on a flat surface, approximately shoulder-width apart, with weight distributed evenly over the hips. Standing with feet together distributes body weight unevenly, increasing the risk for injury. 3. Incorrect: While sturdy leather boots provide protection for the feet, it is not necessary to have steel-toed boots. However, the client should never wear sneakers, sandals or other light-weight, non-protective foot wear when using any type of machinery or equipment. 6. Incorrect. Loose fitting clothing could easily become caught in equipment, yanking the body in towards sharp blades and other moving parts. A client needs snug fitting clothing to cover exposed extremities to prevent even minor injuries.)

What discharge education should a nurse provide to a client post hip replacement with a metal joint? Select all that apply 1. Weight bearing limits. 2. Use of a high seated chair. 3. Sexual intercourse in dependent position for up to six months. 4. Avoid taking showers. 5. Use of long handled tongs to assist with dressing.

1. Weight bearing limits. 2. Use of a high seated chair. 3. Sexual intercourse in dependent position for up to six months. 5. Use of long handled tongs to assist with dressing. (1., 2., 3., & 5. Correct: Weight bearing limits on the involved extremity varies according to the healthcare providers preference but are commonly prescribed. The client needs to avoid flexion. This includes sitting in low chairs and getting into a bath tub; elevated toilet seats and raised seats are necessary. Sexual intercourse should be carried out with the client in a dependent position (flat on the back) for 3-6 months to avoid excessive adduction and flexion of the new hip. To avoid flexion when dressing, adaptive devices and utensils may be used to help with bathing, dressing and personal hygiene. 4. Incorrect: Showers are preferable as getting into a tub would cause flexion of the new hip. This could cause the hip to dislocate.)

What assessment data is the priority nursing concern in a client receiving prednisolone for the treatment of nephrotic syndrome? 1. Weight gain of 2 lbs (0.907 kg) in 24 hours 2. Temperature 99.6°F (37.5° C) 3. Blood glucose 116 mg/dL 4. Blood pressure 138/88

1. Weight gain of 2 lbs (0.907 kg) in 24 hours (1. Correct: If weight is going up rapidly, it is from fluid, not fat. Worsening edema is a sign of decreased kidney perfusion. Steroids inhibit insulin, thus elevated glucose is expected while taking prednisolone. 2. Incorrect: The temperature is within normal range. 3. Incorrect: The glucose is slightly elevated (normal: 70-110 mg/dL), but edema is more significant. 4. Incorrect: Hypertension can be a sign of nephrotic syndrome. However, a blood pressure of 138/88 is not concerning.)

A client who has diabetes calls the nurse hot-line reporting shakiness, nervousness, and palpitations. Which questions would yield information that would help the nurse decide that this is a hypoglycemic episode? Select all that apply 1. What have you eaten today and at what times? 2. Are you using insulin as a treatment of diabetes, and if so, what kind? 3. Do you feel hungry? 4. Do you have access to a glucose monitor to check your current glucose level? 5. Does your skin feel hot and dry?

1. What have you eaten today and at what times? 2. Are you using insulin as a treatment of diabetes, and if so, what kind? 3. Do you feel hungry? 4. Do you have access to a glucose monitor to check your current glucose level? (1., 2., 3. & 4. Correct. This question will give the nurse information about how much time has elapsed since the last meal and will indicate the amount of protein and carbohydrates consumed at the last meal. Even a minor delay in meal times may result in hypoglycemia. Insulin type will give the nurse information about duration of action and peak time. Hunger is a symptom of hypoglycemia.If the client has a glucose monitor, an accurate reading would give the nurse valuable information about how much food the client should consume now. 5. Incorrect. Hot and dry skin is not an indicator of hypoglycemia and would not help the nurse determine if the client is experiencing a hypoglycemic episode. Cool, clammy skin is a symptom of hypoglycemia.)

What should the nurse consider when caring for a client who is receiving total parenteral nutrition (TPN)? Select all that apply 1. Will need a central line. 2. TPN requires a dedicated line. 3. Weigh the client daily. 4. Check the urine for protein. 5. TPN can only be hung for 12 hours

1. Will need a central line. 2. TPN requires a dedicated line. 3. Weigh the client daily. (1., 2., & 3. Correct: Yes, you will need a central line, remember TPN is very irritating to a vein because of the high glucose content and can only be safely administered through a central line. TPN is incompatible with most other solutions so administer through a dedicated line. Daily weights will help you evaluate effectiveness of the TPN. 4. Incorrect: I bet you thought, "Well, I know that we check the urine for something?" But if you chose that option, then you will miss the question. You may want to check the urine for ketones and glucose. 5. Incorrect: TPN can be hung for 24 hours.)

The nurse is monitoring a client in diabetic ketoacidosis (DKA). Which arterial blood gas value would be expected? 1. pH 7.32 2. PaCO₂ 47 3. HCO₃⁻ 25 4. PaO₂ 78

1. pH 7.32 (1. Correct: In DKA, the client is acidotic. Normal pH is 7.35-7.45. A pH of 7.32 indicates acidosis and will be expected for a client in DKA. 2. Incorrect: Normal PaCO₂ is 35-45. Remember CO₂ is considered an acid. The client in DKA will have an increased respiratory rate, so the PaCO₂ will either be normal or low. This value of 47 is high and not an expected finding. 3. Incorrect: Normal HCO₃⁻ is 22-26. For a client in DKA, the expected HCO₃⁻ would be less than 22. HCO₃⁻ is a base. In acidosis, the expected finding is low HCO₃⁻​. 4. Incorrect: Normal PaO₂ is 80-100. An expected finding in DKA will be normal or increased PaO₂, not decreased.)

The nurse is teaching a client, recovering from a myocardial infarction (MI), about the prescribed diet of low sodium, low saturated fat, and low cholesterol. Which statements, if made by the client, would indicate to the nurse that teaching has been successful? Select all that apply 1. "I should drink fruit juices rather than soft drinks." 2. "A good snack to eat would be unsalted popcorn." 3. "When making homemade tomato sauce, I should not add salt." 4. "I should use 2% milk when cooking." 5. "There is no restriction on egg white consumption."

2. "A good snack to eat would be unsalted popcorn." 3. "When making homemade tomato sauce, I should not add salt." 5. "There is no restriction on egg white consumption." (2., 3., & 5. Correct: Popcorn without salt is a healthy snack choice for clients on a low sodium, low saturated fat, and low cholesterol diet. Homemade tomato sauce can be made without adding salt. The American Heart Association no longer makes recommendations on how many egg yolks to eat or not to eat. A good, general guideline is to eat no more than 1 egg yolk a day, up to 5 total a week. There is no restriction on egg whites (including those used in baking and cooking). 1. Incorrect: Consume whole vegetables and fruits rather than fruit juices. Fruit juices have added sugars and lack fiber. 4. Incorrect: Use skim or 1% milk rather than 2% or whole milk.)

What dietary information should the nurse provide to a client diagnosed with Celiac disease? Select all that apply 1. "The most cost effective way to follow the lactose free diet is to eat more fruits and vegetables." 2. "Creamed based canned soups are a source of hidden wheat." 3. "You can eat foods containing fax, corn, or rice." 4. "Avoid foods and beverages that contain malt." 5. "Do not eat traditional wheat products such as pasta."

2. "Creamed based canned soups are a source of hidden wheat." 3. "You can eat foods containing fax, corn, or rice." 4. "Avoid foods and beverages that contain malt." 5. "Do not eat traditional wheat products such as pasta." (2., 3., 4., & 5. Correct: Soups and sauces are one of the biggest sources of hidden gluten, as many companies use wheat as a thickener. It is always a good idea to read the label of any pre-prepared or canned soups and sauces, paying special attention to those that are cream based. Grains that are naturally gluten free include rice, corn, potato, quinoa, kasha, flax, and nut flours. Malt flavoring or extract, which contains gluten may be found in cornflakes and puffed rice cereal. It is also found in beers, ales, and malt vinegars. As a rule, traditional wheat products such as pastas, breads, crackers, and other baked goods are not gluten-free. However, there are many gluten-free options available that use alternative flours and grains. 1. Incorrect: The client who has Celiac disease is prescribed a gluten free diet rather than a lactose free diet.)

A nurse has taught a group of teenage girls about breast self-awareness. Which statements by the teens would indicate to the nurse that teaching was effective? Select all that apply 1. "I should have a clinical breast exam every 5 years starting at the age of 18." 2. "Doing a monthly breast self-exam will help me learn what is normal for me." 3. "It is only important to know my maternal health history." 4. "Signs I should not ignore include dimpling of the skin, and nipple discharge." 5. "Self-breast exam should be done a few days before my menstrual cycle begins."

2. "Doing a monthly breast self-exam will help me learn what is normal for me." 4. "Signs I should not ignore include dimpling of the skin, and nipple discharge." (2., & 4. Correct: The purpose of breast self-exam is to determine what is normal. This will allow the client to recognize when there is a change in breast tissue. S/S of breast cancer includes dimpling of the skin, nipple discharge, tenderness, change in appearance, retracted nipple, hard lump and itchy or scaly skin. 1. Incorrect: Clinical breast exams are recommended every 3 years starting at age 20, and every year starting at age 40. 3. Incorrect: Talk to both sides of the family to learn about your family health history. 5. Incorrect: The breast self-exam should be done after the menstrual cycle (day 7-12) for a better exam. The breasts will be too tender just prior to the period.)

A client arrives to the after hours clinic with reports of palpitations and skipping heart beats. The nurse notes the client to be alert and oriented with a BP of 124/76, HR irregular at 95 beats per minute, respirations at 18 breaths per minute, and is afebrile. Cardiac monitoring is initiated. Based on this data, what questions should the nurse ask the client? EXHIBIT Select all that apply 1. "Have you been prescribed a tricyclic antidepressant?" 2. "Have you been experiencing more stress than usual in your life?" 3. "Does this generally begin when you are having a bowel movement?" 4. "How many cups of coffee do you drink each day?" 5. "What over the counter medications do you take?" 6. "Have you been running a fever?"

2. "Have you been experiencing more stress than usual in your life?" 4. "How many cups of coffee do you drink each day?" 5. "What over the counter medications do you take?" (2., 4., & 5. Correct: This rhythm strip is showing that the client has normal sinus rhythm with premature atrial contractions (PACs). Common causes of PACs include stress and excess caffeine intake from coffee, tea, colas, and over the counter medications that contain caffeine. 1. Incorrect: Tricyclic antidepressants can result in a client developing premature ventricular contractions (PVCs) due to sedation and decreased oxygenation. 3. Incorrect: When the vagus nerve is stimulated, it can cause bradycardia and sinus arrest rather than premature atrial contractions. 6. Incorrect: Fever tends to increase the heart rate so that sinus tachycardia would be seen.)

Which statement made by a 67 year old client who recently retired indicates to the nurse that client has developed ego integrity? 1. "I want to make my mark on the world." 2. "I am satisfied with my life so far." 3. "I wish I could go back and fix the mistakes I have made." 4. "Life is too short. I have more living to do."

2. "I am satisfied with my life so far." (2. Correct: Those who feel proud of their accomplishments will feel a sense of integrity. Successfully completing this phase means looking back with few regrets and a general feeling of satisfaction. 1. Incorrect: Generativity versus stagnation is the seventh of eight stages of Erik Erikson's theory of psychosocial development. This stage takes place during middle adulthood (40 to 65). Generativity refers to "making your mark" on the world through creating or nurturing things that will outlast an individual. 3. Incorrect: Feeling regret over past decisions or mistakes can lead to despair. To reach ego integrity, the client must accept mistakes made and look at accomplishments achieved in life. 4. Incorrect: Success in this stage will lead to the virtue of wisdom. Wisdom enables a person to look back on their life with a sense of closure and completeness and accept death without fear.)

A client has received discharge education post extracapsular cataract surgery. Which statement made by the client indicates to the nurse that further teaching is needed? 1. "A protective eye patch will be needed for 24 hours." 2. "I will notify my primary heathcare provider for any amount of discharge, redness or scratchy feeling because these symptoms are abnormal." 3. "I will clean the surgical eye with a clean tissue, wiping once from the inner aspect of the closed eye to the outer eye." 4. "When sleeping, I will avoid lying on the same side of my affected eye."

2. "I will notify my primary heathcare provider for any amount of discharge, redness or scratchy feeling because these symptoms are abnormal." (2. Correct: This is an incorrect statement by the client. Slight morning discharge, some redness, and a scratchy feeling may be expected for a few days." Clients are instructed to report any pain that is unrelieved, redness around the eye, nausea or vomiting to the primary healthcare provider. 1. Incorrect: This is a true statement and does not require intervention. Following surgery, the eye is covered with a patch and a metal or plastic shield for protection from light and trauma. 3. Incorrect: This is the correct way to clean the surgical eye. Cleaning from the inner to outer canthus avoids entrance of microorganisms into the lacrimal duct. 4. Incorrect: This is a correct action. Increased intraocular pressure needs to be avoided. Clients are instructed to avoid sleeping on the operative side.)

A home heath nurse is educating a client about home care considerations for clean intermittent catheterization. Which statement by the client would best reflect understanding of the nurse's teaching? 1. "After insertion, I will tape the tubing to my upper thigh." 2. "I will wash the re-usable catheter thoroughly with soap and water after use." 3. "It is important that I keep the drainage bag below the level of my bladder." 4. "Catheterization should be done hourly."

2. "I will wash the re-usable catheter thoroughly with soap and water after use." (2. Correct: For intermittent catheterization in the home, the client should follow clean technique. Wash catheters thoroughly with soap and water after use, dry, and store in a clean place. 1. Incorrect: With intermittent catheterization there is no tubing to secure. 3. Incorrect: With intermittent catheterization there is typically no drainage bag, unless it is a temorary collection bag. Therefore, the bag would not be maintained at a certain level. 4. Incorrect: Catheterization is done when the client feels the need to void, but cannot void. Catheterizing this often can increase the risk of infection. Some clients, such as paralegics, may need to self catheterize every 2-4 hours initially.)

A client with tuberculosis (TB) has been coming to the health department for directly observed therapy (DOT) for the past month. Today, the client states, "I don't think I need to come back anymore. I am feeling much better now." What should the nurse tell the client? 1. "You have taken your medication long enough so, the primary healthcare provider should discontinue it today." 2. "If you stop taking your medication now, your disease could become resistant to this medication, making it harder for you to be cured." 3. "I will be required to have you arrested if you do not come back for further treatment." 4. "Just let us decide when you should stop taking the medication."

2. "If you stop taking your medication now, your disease could become resistant to this medication, making it harder for you to be cured." (2. Correct: This is true regarding TB treatment. The Medication has to be taken for the entire course. The minimal length of time for therapy is 3 months.1. Incorrect: Treatment usually lasts 4-7 months. If the medication regimen is not strictly and continuously followed, the disease may become drug-resistant. It is not the nurse's place to determine when enough medication has been taken. 3. Incorrect: The nurse needs to discuss the reason for continuing to take the medication. This step is premature and intimidating. Also, threatening to have the client arrested will not likely maintain a good patient-nurse relationship. 4. Incorrect: This statement is non-therapeutic and dismissive of the client. This does not address the client's statement of thinking they have had enough medicine and should stop.)

A client hospitalized with a deep vein thrombosis (DVT) is on a heparin infusion. The client asks the nurse why it is necessary to have blood drawn every six hours. What is the best explanation for the nurse to provide to the client? 1. "The medicine might make your blood much too thin." 2. "It helps us monitor and adjust the dose to work better." 3. "It is required for anyone getting heparin intravenously." 4. "The test results tell us whether the treatment is working."

2. "It helps us monitor and adjust the dose to work better." (2. Correct: The nurse has clearly stated the purpose of the frequent venipunctures in a simple and non-technical manner that answers the client's question. 1. Incorrect: This comment by the nurse does not address the inquiry about every 6 hour bloodwork, plus the phrasing of the statement could easily frighten the client. 3. Incorrect: This standard response does not answer the client's question about blood work nor does it provide further information about the treatment. 4. Incorrect: The nurse's statement is vague and does not address the client's question about frequent blood work.)

Which statements made by a client diagnosed with Addison's disease indicates to the nurse that the client understands fludrocortisone therapy? Select all that apply 1. "Taking my medicine at night will help me sleep." 2. "It is important to wear a medical alert bracelet all of the time." 3. "I will limit my sodium intake to 200 mg per day." 4. "My medication dose will change based on my daily weight." 5. "I may need more medication if I feel weak or dizzy."

2. "It is important to wear a medical alert bracelet all of the time." 4. "My medication dose will change based on my daily weight." 5. "I may need more medication if I feel weak or dizzy." (2., 4., & 5. Correct: Medical alert bracelet is an excellent way of informing healthcare providers of a life threatening condition if the client is unable to verbalize that information. Steroid therapy is adjusted according to the client's weight and signs of fluid volume status. Signs of being undermedicated include weakness, fatigue, and dizziness. The client will need to report these symptoms, so more medication can be given to the client. 1. Incorrect: Steroids can cause insomnia so the client does not need to take the medication prior to going to bed. 3. Incorrect: This client needs a high sodium diet as they are losing sodium and retaining potassium.)

The following clients arrive to the emergency department (ED) at the same time. The triage nurse gives priority to which client? 1. A client with a possible fracture of the tibia 45 minutes ago. 2. A client with left hemiparesis and aphasia beginning 1 hour ago. 3. A client smelling of alcohol and reporting of severe abdominal pain. 4. A client involved in a motor vehicle accident (MVA) with a possible fractured pelvis.

2. A client with left hemiparesis and aphasia beginning 1 hour ago. (2. Correct: The client who is started experiencing hemiparesis and aphasia 1 hour ago is likely having a stroke. The window for treatment with fibrolytics is 3 hours, thus taking priority over the other clients. Time is brain! 1. Incorrect: This client has a possible fracture of the tibia. This is not a large bone, which would be at risk for hemorrhage. Splinting and ice packs could be used until after seeing the client having a stroke. 3. Incorrect: With this client, you would worry about pancreatitis. This client needs to be seen soon but not prior to the client having a stroke. 4. Incorrect: The MVA client could have bleeding from a fractured pelvis. This client is high on the admit list, but after the client having a stroke.)

The nurse is teaching a group of teenagers about decreasing the risk of developing skin cancer. What information should the nurse include? Select all that apply 1. Use sunscreen with a sun protection factor (SPF) of at least 30. 2. A self-tanning product containing dihydroxyacetone (DHA) is safe to use. 3. Put on sunscreen every day, even on days when it is cloudy. 4. Stay in the shade between 9 AM and 4 PM. 5. Tanning beds are safer than outdoor tanning.

2. A self-tanning product containing dihydroxyacetone (DHA) is safe to use. 3. Put on sunscreen every day, even on days when it is cloudy. 4. Stay in the shade between 9 AM and 4 PM. (2., 3., & 4. Correct: People of all ages should avoid tanning. If you like the look of tanned skin, choose a self-tanning product that contains the active ingredient DHA. This active ingredient has been approved by the U.S. Food and Drug Administration and is safe to use. Most are fast-acting and will give skin a darker appearance in a matter of a few hours. This "tan" will last about a week. Put on sunscreen every day, even on days when it is cloudy. UV rays from the sun can reach you on cloudy and hazy days, as well as bright and sunny days. UV rays also reflect off surfaces like water, cement, sand, and snow. The hours between 10 a.m. and 4 p.m. Daylight Saving Time (9 a.m. to 3 p.m. standard time) are the most hazardous for UV exposure outdoors in the continental United States. UV rays from sunlight are the greatest during the late spring and early summer in North America. 1. Incorrect: Use sunscreen with a SPF of 15 or higher, and both UVA and UVB (broad spectrum) protection. 5. Incorrect: Tanning beds are dangerous. Avoiding the sun but replacing it with a tanning bed does not reduce the risks that are associated with UV damage to the skin.)

The client presents to the emergency department with nausea, vomiting and anorexia for the last few days. Based on the EKG obtained, what action should the nurse take first? Exhibit 1. Defibrillate at 200 joules x 2 2. Administer amiodarone IV 150 mg over 10 minutes 3. Infuse 500 mL NS with 40 mEq KCl (40 mmol/L) at 100 mL/hour 4. Begin 2 person cardiopulmonary resuscitation

2. Administer amiodarone IV 150 mg over 10 minutes (2. Correct: The one electrolyte we worry about with arrhythmias is potassium. The first line medication is amiodarone. 1. Incorrect: Pulseless v-tach and v-fib require defibrillation. 3. Incorrect: KCL is needed but we need to treat the short run of v-tach first. 4. Incorrect: Not indicated. Treat short run of v-tach and increase potassium.)

Which nursing actions should the nurse initiate for a client with signs of increased intracranial pressure (ICP)? Select all that apply 1. Encourage coughing and deep-breathing. 2. Administer corticosteroids. 3. Position client in the prone position. 4. Determine ability to swallow prior to administering po fluids. 5. Maintain head in neutral alignment.

2. Administer corticosteroids. 4. Determine ability to swallow prior to administering po fluids. 5. Maintain head in neutral alignment. (2., 4., & 5. Correct: Administer corticosteroids to reduce inflammatory response seen in acute brain injury. If I have increased ICP, my reflexes could be suppressed, so check my swallowing. Maintain head in neutral alignment to prevent decrease in venous flow which would increase the ICP. 1. Incorrect: Coughing and deep breathing make the ICP go up. 3. Incorrect: This position makes the ICP go up.)

A client has been admitted for observation after having a minor automobile accident. During the admission history, the client admits to being an alcoholic. Two hours after admission the nurse notes the client's cardiac rhythm displayed on the telemetry monitor. The client reports shortness of breath, chest discomfort, and nausea. What initial action should the nurse take? Exhibit 1. Cardiovert at 200 joules. 2. Administer magnesium 1 gm IVP over 30 seconds. 3. Begin cardiopulmonary resuscitation (CPR). 4. Obtain a 12 lead ECG.

2. Administer magnesium 1 gm IVP over 30 seconds. (2. Correct: Magnesium is the drug of choice for suppressing Torsades and terminating the arrhythmia. Magnesium can be given at 1-2 g IV initially in 30-60 seconds, which then can be repeated in 5-15 minutes. 1. Incorrect: In an otherwise stable client, cardioversion is kept as a last resort because Torsades is paroxysmal in nature and is characterized by its frequent recurrences following cardioversion. Although torsade frequently is self-terminating, it may degenerate into ventricular fibrillation, which requires direct defibrillation. 3. Incorrect: This client is awake and has a pulse. CPR is not indicated at this time. 4. Incorrect: Getting a 12 lead ECG will not fix the problem and it is delaying treatment. Do something to fix the problem.)

The client arrives in the emergency department with crushing substernal chest pain radiating down the left arm. Which measure should the nurse initiate first? 1. Attach to a cardiac monitor 2. Administer oxygen at 2 L/nasal cannula 3. Start an intravenous (IV) line of D5W to keep open 4. Draw blood for troponin level

2. Administer oxygen at 2 L/nasal cannula (2. Correct: A crushing substernal chest pain radiating down the left arm is classic for an MI. So what option can help the client? Oxygen administration to get more oxygen to the heart muscle. 1. Incorrect: Looking and watching is what you are doing if you select this option. Will attaching the client to the cardiac monitor first help the client? No. Give the client oxygen first. 3. Incorrect: Getting an IV line is good so that cardiac medications can be given, but help the client first by starting the O₂. If you wait to provide oxygen until after starting the IV, the client may be waiting a while and heart muscle will be dying. 4. Incorrect: Yes, the lab will be there shortly, but get started with O₂ while you wait.)

Which assigned client should the nurse see first? 1. Diagnosed with urinary tract infection 2 days ago who is to be discharged. 2. Admitted last night with a diagnosis of severe pneumonia. 3. 45 year old who had a hernia repair 24 hours ago. 4. Scheduled for an endoscopy in two hours.

2. Admitted last night with a diagnosis of severe pneumonia. (2. Correct: The client with severe pneumonia is at greatest risk for respiratory difficulty and should be seen first. Clients with severe pneumonia may develop the following complications: bacteremia, septic shock, lung abscesses, pleural effusion, empyema, pleurisy, renal failure, and respiratory failure. 1. Incorrect: The client who is being discharged is considered to be stable. A client who was diagnosed with a urinary tract infection is considered to be stable. This client is not exhibiting signs of potential airway complications. 3. Incorrect: This postoperative client of 24 hours is considered stable. The client's age of 45 also does not suggest that the client was a surgical risk. 4. Incorrect: The client admitted for an endoscopy is considered to be stable at this point. There is no data listed to support the client needing to be assessed first.)

What action should the nurse take first for the 5 year old client brought to the urgent care clinic with a blistering sunburn? 1. Administer analgesics. 2. Apply cool water soaks. 3. Check immunization status for tetanus. 4. Educate family to avoid greasy lotions or butter on the burn.

2. Apply cool water soaks. (2. Correct: The priority is to stop the burning process. This can be done by applying cool water to the burned area. 1. Incorrect: Determining pain level and administering pain medication would be the second priority. 3. Incorrect: Before the client is sent home, tetanus immunization status would need to be determined. 4. Incorrect: This would be included in discharge teaching.)

An elderly client is admitted to the outpatient unit with anemia and is receiving a blood transfusion. What is the nurse's priority assessment? 1. Monitor for peripheral edema. 2. Assess breath sounds. 3. Keep bedrails up at all times. 4. Monitor hemoglobin every 6 hours.

2. Assess breath sounds. (2. Correct: The elderly client receiving a blood transfusion is at greater risk for fluid volume overload. The nurse should recognize that the very old and the very young are at increased risk for fluid volume overload which could manifest as wet breath sounds. 1. Incorrect: Monitoring for edema is valuable in assessing for fluid volume overload, but the priority is the lung assessment. 3. Incorrect: Safety is important, especially with an elderly anemic client, but monitoring for fluid volume excess is the priority. Physiologic needs prioritize higher than safety needs. 4. Incorrect: Monitoring hemoglobin is important but not necessary every 6 hours.)

An elderly client with a history of coronary artery disease (CAD) has just been admitted to the telemetry unit following a syncopal episode at home. The admitting nurse places EKG leads on the client and notes the following rhythm on the monitor. When the client indicates the need to void, the nurse knows that what would be the safest action? Exhibit 1. Request order for a foley catheter. 2. Assist client with the use of a bedpan. 3. Provide a bedside commode chair. 4. Perform in and out straight catherization.

2. Assist client with the use of a bedpan. (2. Correct: The exhibit shows bradycardia with premature ventricular contractions (PVCs), and more specifically, bigeminy. The safest approach for a syncopal client with this rhythm is the use of a bedpan for bathroom needs. Even with assistance, this client would be at risk for falls when ambulating. 1. Incorrect: Because the client has experienced syncope and is bradycardic, keeping the client in bed is safer than ambulating to the bathroom. However, a foley catheter is an invasive procedure that could place the client at risk of infection. There is a better option. 3. Incorrect: The client has experienced syncope and is bradycardic. Keep the client in bed. This is a safety issue. 4. Incorrect: This client is newly admitted with a diagnosis of syncope. The exhibit shows the heart rate is bradycardic with PVCs, which are non-perfusing beats. In and out straight catheterization is an invasive procedure that could place the client at risk for infection. There is a better option.)

An elderly client with a history of CAD has just been admitted to the telemetry unit following a syncopal episode at home. The admitting nurse places EKG leads on the client and notes the following rhythm on the monitor. When the client indicates the need to void, the nurse knows that what would be the safest action? 1. Request prescription for a foley catheter. 2. Assist client with the use of a bedpan. 3. Provided incontinent pads to the client. 4. Have UAP ambulate client to the bathroom.

2. Assist client with the use of a bedpan. (2. Correct: The exhibit shows sinus rhythm with premature ventricular contractions (PVC's), and more specifically, bigeminy. The safest approach for a syncopal client with this rhythm is the use of a bedpan for bathroom needs. Even with assistance, this client would be at risk for falls when ambulating. 1. Incorrect: Because the client has experienced syncope and is having frequent PVCs, keeping the client in bed is safer than ambulating to the bathroom. However, a foley catheter is an invasive procedure that could place the client at risk of infection. There is a better option. 3. Incorrect: Using incontinent pads, either on the bed or personal pads for the client, is only appropriate if the client is unable to control urinary flow, or requests the use of same. It is embarrassing to ask clients who are continent to void onto a bed pad. 4. Incorrect: This client is newly admitted with a diagnosis of syncope. The exhibit shows frequent PVC's, which are non-perfusing beats. Even with assistance from the UAP, ambulating to the bathroom is not the safest action for this client.)

When explaining to caregivers how to reduce the risk of falls in their elderly parent, the nurse should educate about which measure? Select all that apply 1. Allow the parent to wear shoes that are most comfortable. 2. Assure there is adequate lighting with minimal glare. 3. Use sharply contrasting colors at edges of stairs. 4. Install grab bars beside the shower, tub, and toilet. 5. Encourage the parent to have an inside pet for comfort. 6. Rearrange the furniture for the parent to prevent stagnation.

2. Assure there is adequate lighting with minimal glare. 3. Use sharply contrasting colors at edges of stairs. 4. Install grab bars beside the shower, tub, and toilet. (2., 3., and 4. Correct: Adequate lighting with minimal glare is best to assure there is the amount of illumination needed for safe mobility. Marking the edges of stairs with sharply contrasted colors can help to reduce falls by alerting the elderly client of the change in the elevation of the walkway. The risk of falls in the bathroom can be diminished by installing grab bars to help stabilize the elderly client as they make position changes or transition from the tub, shower, or toilet. 1. Incorrect: Just because the shoes are the most comfortable does mean that they are safe to wear. They may have slick soles, be loose fitting, or have other unsafe issues. Improperly fitting shoes can create a hazard and increase the risk of falls. Unsafe footwear is one of the more common, treatable causes of falls in the elderly. 5. Incorrect: Pets often get in the path of individuals when walking and can create a tripping hazard. Having pets in the home is another one of the more common, treatable causes of falls in the elderly. 6. Incorrect: Elderly individuals manage better when the surroundings are familiar and are kept in the same arrangement. Changes in the environment can increase the risk of falls when objects are no longer where the older adult is accustomed to them being placed.)

The nurse would make which recommendations when conducting community health teaching about obesity to a group of adolescents? Select all that apply 1. Limit TV viewing and video game playing to 4 hours a day 2. At least 60 minutes of moderate-intensity activity daily 3. Exercise should be structured 4. A strict diet should be followed avoiding all junk food and drinking water only 5. Set a goal of at least 11,000 to 13,000 steps each day

2. At least 60 minutes of moderate-intensity activity daily 5. Set a goal of at least 11,000 to 13,000 steps each day (2. & 5. Correct: 60 minutes of moderate-intensity physical activity 7 days a week. Girls should take a least 13,000 steps daily and boys should take 11,000 steps daily. 1. Incorrect: TV viewing and video game playing should be 2 or fewer hours each day. 3. Incorrect: Exercise does not need to be structured. 4. Incorrect: If the diet is too restrictive, it is likely to fail.)

The nurse is caring for a hypertensive client who has been taking a loop diuretic while hospitalized. Upon discharge, the nurse must teach the client about the need for adequate electrolyte intake through foods and/or dietary supplements. Which foods should the nurse suggest to the client? 1. Cereals and breads 2. Avocados and apricots 3. Table salt and spinach 4. Blueberries and strawberries

2. Avocados and apricots (2. Correct: Avocados, apricots, milk, fruit juices, bananas and cantaloupe are good sources of potassium. Loop diuretics deplete the electrolyte potassium. 1. Incorrect: Cereals and breads are good sources of B vitamins. Since the client is losing potassium they need foods that are high in potassium. Cereals and breads are not high in potassium. 3. Incorrect: Table salt and spinach are good sources of sodium, but the hypertensive client usually should limit intake of sodium. The client is taking a potassium depleting diuretic and needs potassium rich foods. Spinach is high in potassium but the table salt makes this option incorrect. 4. Incorrect: Blueberries and strawberries both are relatively low in potassium. Clients on loop diuretics are losing potassium and need to consume foods high in potassium.)

What preoperative education should the nurse provide to a client scheduled for a transsphenoidal hypophysectomy? Select all that apply 1. There will be a large dressing covering the forehead. 2. Avoid drinking from a straw. 3. The head of the bed will be elevated 30 degrees. 4. After surgery, your urinary output will be monitored. 5. Use a sponge when doing oral care.

2. Avoid drinking from a straw. 3. The head of the bed will be elevated 30 degrees. 4. After surgery, your urinary output will be monitored. 5. Use a sponge when doing oral care. (2., 3., 4., & 5. Correct: Drinking from a straw can damage the surgical area, so the client should not use a straw until approved by the primary care provider. The client will return from surgery with the head of the bed elevated to about 30 degrees. This will allow for gravity to assist with draining of any cerebrospinal fluid. Surgery can damage the posterior lobe of the pituitary gland, which makes the client urinate frequently and feel thirsty. This is diabetes insipidus. If the client puts out too much urine, then fluid volume deficit or shock can occur. The incision is just above the gumline, so the client should not brush the front teeth. Oral care is best performed using a sponge until the incision heals. 1. Incorrect: Pituitary surgery can be removed through traditional microscopic or endoscopic technique. So, there is no dressing on the forehead.)

A client diagnosed with a 5 cm decubitus on the heel has a current serum albumin level of 3.0 g/dL (30 g/L). The nurse plans to provide dietary teaching focusing on which foods? Select all that apply 1. Green beans 2. Chicken 3. Cottage cheese 4. Shrimp 5. Almonds

2. Chicken 3. Cottage cheese 4. Shrimp 5. Almonds (2, 3., 4., & 5. Correct: These foods are high protein foods. A four-ounce serving of chicken, boiled or grilled, contains about 110 calories, and around 25 grams of whole, complete protein. A ½ cup of cottage cheese is 82 calories, and has 14 grams of protein. A three-ounce serving of shrimp contains roughly 18 grams of protein. A one-ounce serving of almonds contains about six grams of protein. 1. Incorrect: Green beans only contain 1.49 grams of protein. The client needs foods high in protein to promote tissue growth.)

A client has been admitted with a diagnosis of septic shock and has been successfully intubated. The nurse performs and documents a rapid assessment. Which information from the assessment requires the most immediate action by the nurse? Blood Pressure: 88/42 mmHg Heart Rate: 112 bpm Respiratory Rate: 32 breaths/min Temperature: 103°F (39.4°C) Oxygen Saturation: 94% Heart tones irregular, distant. Face flushed and warm. Extremities cool and mottled. Radial pulses faintly palpable. Pedal pulses non-palpable. Denies chest pain. Breath sounds audible bilaterally with adventitious sounds noted in left lung base. Grimaces with light abdominal palpation over pelvic bone. Urine amber and cloudy with red streaks. 100 mL urine output in foley catheter bag. Opens eyes and moves to command. Pupils equal, round, and react to light. 1. Lung assessment finding 2. Blood pressure reading 3. Elevated temperature 4. Urine description and output

2. Blood pressure reading (2. Correct: The low blood pressure indicates that systemic tissue perfusion is not adequate. The blood pressure needs to be improved rapidly. 1. Incorrect: The oxygen saturation is 94%, so the adventitious lung sounds do not need immediate intervention. 3. Incorrect: The second priority is to treat the infection that is likely the cause of the temperature elevation and hypotension. 4. Incorrect: This is the likely cause of the sepsis, but the priority is to improve the blood pressure. The second priority is to treat the infection.)

Which assessment finding identified in a client diagnosed with Guillain-Barre Syndrome would indicate that the nurse needs to notify the primary healthcare provider? 1. Vital lung capacity of 900 mL. 2. Breathlessness while talking. 3. Heart rate of 98 beats per minute. 4. Respiratory rate of 24 breaths per minute.

2. Breathlessness while talking. (2. Correct: Breathlessness while talking indicates respiratory fatigue. Preparation for intubation needs to be made. 1. Incorrect: If the vital lung capacity drops below 800 mL, mechanical ventilation is warranted. 3. Incorrect: Imminent signs of respiratory failure include a heart rate greater than 120 beats per minute or less than 70 beats per minute. 4. Incorrect: Imminent signs of respiratory failure include a respiratory rate greater than 30 breaths per minute.)

A client with a diagnosis of endocarditis and a new peripherally inserted cential catheter (PICC) line has been discharged home to receive daily intravenous antibiotics for six more weeks. The home health nurse is making an assessment visit today. What instruction by the nurse is most important initially? 1. Take antibiotics before dental procedures. 2. Brush and floss teeth at least twice daily. 3. Report any flu like symptoms immediately. 4. Include rest periods throughout the day.

2. Brush and floss teeth at least twice daily. (2. Correct: Poor dental hygiene is one of the chief causes of endocarditis in adults, leading to growth of vegetation on heart valves, emboli, strokes, or even death. Instructions on proper oral care is considered primary or preventative teaching and encourages the client to take an active role in personal health care. Decreasing mouth bacteria or disease will decrease the potential for a reoccurrence of endocarditis. 1. Incorrect: Although primary healthcare providers may order antibiotics prior to a dental visit, it depends on what procedure the dentist is going to perform. Invasive mouth procedures where bleeding is likely generally require pre-visit antibiotics. However, this is not the most important information by the nurse initially. 3. Incorrect: Flu like symptoms are an indication of a possible exacerbation or reoccurrence of endocarditis. The client would be instructed to report such signs as fever, chills, malaise, or night sweats immediately to the primary healthcare provider. While it is important for the client to understand what to report, preventative measures are more important at this time. 4. Incorrect: Infection within the heart is very serious and, despite aggressive treatment, may have lasting effects on the client's cardiovascular system. Decreasing the workload of the heart during treatment and recovery time would certainly assist with the healing process. However, the need for frequent rest periods throughout the day is determined by a variety of factors, such as the client's age and morbidity factors, general health, amount of damage to the heart, and response to antibiotics. Rest is not the most important instruction the nurse must present initially.)

What signs/symptoms would the nurse expect to find in a client admitted to the unit with a diagnosis of Cushing's disease? Select all that apply 1. Hyperpigmentation 2. Buffalo hump 3. Hirsutism 4. Acne 5. Moon face 6. Hypertension

2. Buffalo hump 3. Hirsutism 4. Acne 5. Moon face 6. Hypertension (2., 3., 4., 5., & 6. Correct: Cushing's disease results in an increase in glucocorticoids, mineralocorticoids, and sex hormones resulting in a buffalo hump, hirsutism, acne, moon face, and hypertension. 1. Incorrect: Hyperpigmentation occurs when the body has too little cortisol, such as with Addison's disease.)

The nurse assesses a client post thyroidectomy for complications by performing which assessment? 1. Accucheck 2. Chovostek's 3. Ballottement 4. Ice water colonic

2. Chovostek's (2. Correct: A positive Chovostek's and Trousseau's is indicative of tetany and low calcium. This can occur when a couple of parathyroids are accidently removed when the thyroid is removed. 1. Incorrect: Accucheck assesses for blood glucose levels, which is not the problem post thyroidectomy. 3. Incorrect: This assessment technique is used in examining the abdomen when ascites is present. It is done by palpating the abdomen to detect excessive amounts of fluid (ascites). 4. Incorrect: If you have never heard of it, no one else has either. The phrase implies using ice water to cleanse the colon and this would never be a good thing, especially for someone post thyroidectomy that would be intolerant to extremes in temperature.)

The nurse is providing care to a client who is post laparoscopic cholecystectomy. Which finding would be of concern? Select all that apply 1. Right upper quadrant abdominal discomfort 2. Clay colored stool 3. Light yellow urine 4. Pruritus 5. Icteric sclera

2. Clay colored stool 4. Pruritus 5. Icteric sclera (2., 4., & 5. Correct: Injury to nearby structures, such as the bile duct, liver and small intestine can occur after this surgery. Clay colored stools and jaundice of the sclera are caused by recurring stricture or stone of the common bile duct. Pruritus occurs when bile reaches the skin. 1. Incorrect: The gallbladder is in the right upper abdominal quadrant, so discomfort would not be uncommon in this area postoperatively. 3. Incorrect: Light yellow colored urine is normal.)

What signs/symptoms would the nurse expect to find in a client diagnosed with osteoarthritis (OA) in the knee? Select all that apply 1. Sjogren's syndrome 2. Clicking sound when knee bends 3. Fever 4. Pain that is worse after activity 5. Severe fatigue

2. Clicking sound when knee bends 4. Pain that is worse after activity (2., & 4. Correct: Loss of cartilage between bone joints produces the clinking or cracking sound heard when the joint bends. Pain is worse after an activity involving the affected joint or toward the end of the day. 1. Incorrect: Sjogren's (SHOW-grins) syndrome is a disorder of the immune system identified by its two most common symptoms — dry eyes and a dry mouth. It is not seen in osteoarthritis. 3. Incorrect: Fever is not associated with Osteoarthritis but is associated with rheumatoid arthritis (RA). 5. Incorrect: Severe fatigue is seen with rheumatoid rather than osteoarthritis.)

What assessment findings would the nurse expect when evaluating whether treatment has been effective for a client hospitalized with systolic heart failure? Select all that apply 1. 3+ pedal edema 2. CVP of 6 mm Hg 3. One day weight loss of 2 pounds (0.9 kg) 4. Purse-lip breathing 5. Pale nail beds 6. Urine output at 50 mL/hr

2. CVP of 6 mm Hg 3. One day weight loss of 2 pounds (0.9 kg) 6. Urine output at 50 mL/hr (2., 3. & 6. Correct: Normal CVP is 2-6 mm Hg. This CVP is within normal range so treatment is effective. Weight loss indicates that fluid is being removed and a urine output of 50mL/hour indicates that renal perfusion is adequate. All three assessents indicate improvement. 1. Incorrect: 3+ pedal edema would indicate that the client is not better. 4. Incorrect: Purse-lip breathing is seen when client is still short of breath. 5. Incorrect: Pale conjuctiva, nail beds, buccal mucosa are signs of impaired gas exchange.)

A client diagnosed with primary pulmonary hypertension is admitted to the hospital. What does the nurse expect the client to mention when reviewing the client's current treatment regimen? Select all that apply 1. Aminoglycosides 2. Calcium channel blockers 3. Digoxin 4. Diuretics 5. Oxygen 6. Vasodilators

2. Calcium channel blockers 3. Digoxin 4. Diuretics 5. Oxygen 6. Vasodilators (2., 3., 4., 5., & 6. Correct: Calcium channel blockers are given to help relax the muscles in the walls of the blood vessels. Digoxin can help the heart beat stronger and pump more blood. It can help control the heart rate if dysrhythmias are experienced. Diuretics are prescribed to eliminate excess fluid from the body. Oxygen therapy may be prescribed to treat pulmonary hypertension, especially if the client lives at a high altitude or has sleep apnea. Vasodilators are used to open narrowed blood vessels. 1. Incorrect: Aminoglycosides are antibiotics used to treat infections.)

A client rescued from a house fire is being treated for burns to both arms and suspected inhalation injury. What data collected by the nurse has the highest priority? 1. Estimation of total surface burn area 2. Characteristics of cough and sputum 3. Calculation of client weight and age 4. Extent of edema to arms

2. Characteristics of cough and sputum (2. Correct: A client rescued from a burning house is presumed to have inhaled super-heated air during that process. Though calculating fluid replacement is vital to the client's survival, the ABCs dictate the highest priority is airway. Noting any cough or sputum can help determine whether prophylactic intubation may be necessary. 1. Incorrect: The total amount of body surface burned is crucial information needed to determine fluid replacement using the Parkland Formula. However, though IV fluids are necessary, calculating the burn percentage is not the highest priority. 3. Incorrect: The client's age is not an immediate priority, although a complete health history will be essential to the final outcome. The client's weight will be used to calculate fluid replacement; however, there is a higher initial priority. 4. Incorrect:. A burn causes cellular damage that leads to edema. Depending on the location and extent of that edema, circulation could be greatly impaired. However, when monitoring a burned client, the ABCs place circulation third on the priority list.)

A nurse is reviewing serum laboratory data for four clients. Which client would require the most immediate assessment? A: Thyroid-stimulating hormone (TSH) 12 mU/L (12µ​U/mL) Normal Range: 2 -10 mU/L (2-10 µ​U/mL)​ B Free T4 (thyroxine) 5.0 ng/dL (0.39 pmol/L) Normal Range: 0.8 - 2.8 ng/dL (10-36 pmol/L) C Growth hormone 8 ng/mL (8 mcg/L) Normal Range: 0-6 ng/mL (0-6 mcg/L) D Glucose 150 mg/dL (8.3 mmol/L) Normal Range: 70 - 110 mg/dL (3.9-6.2 mmol/L) 1. Client A 2. Client B 3. Client C 4. Client D

2. Client B (2. Correct: An excess of thyroid hormone is the most life-threatening of the findings listed due to its effects on the cardiovascular system of hypertension and tachycardia. The client should be assessed for impending thyroid storm. 1. Incorrect: An elevated TSH level occurs in hypothyroidism. TSH is needed to ensure proper synthesis and secretion of the thyroid hormones which are essential for life. Not life-threatening. 3. Incorrect: An elevated growth hormone produces acromegaly with resulting bone and soft tissue deformities and enlarged viscera. But this is not life threatening. 4. Incorrect: Though the glucose level is elevated, a level of 150 mg/dL (8.3 mmol/L) does not require immediate assessment or intervention.)

A client with diabetes has a history of ignoring the primary healthcare provider's prescription for daily medication management of the illness. The client has been working toward a health promotion goal of increased adherence to prescribed medication regimen. Which outcome suggests that the client has met the health promotion goal? 1. Client has lost five pounds. 2. Client takes medication as prescribed. 3. Client has been hospitalized twice for complications of diabetes. 4. Client walks one mile per day.

2. Client takes medication as prescribed. (2. Correct: The outcome directly addresses medication adherence, the major focus of the health promotion plan. 1. Incorrect: This is a positive outcome; however, the focus is on medication adherence. 3. Incorrect: This outcome would indicate possible non-adherence to the medication regimen. 4. Incorrect: While this is a positive outcome for anyone's health, the focus is medication adherence.)

A client is being scheduled for a cat scan (CT) of the abdomen with contrast. When considering client safety, what should be the priority action for the nurse to implement? 1. Verify that informed consent has been provided. 2. Confirm with client the accuracy of allergies listed. 3. Force fluids following procedure. 4. Monitor output following procedure.

2. Confirm with client the accuracy of allergies listed. (2. Correct: When considering client safety, the nurse should confirm allergies with the client. Clients should be asked about allergies to iodine or shellfish. The radiocontrast agents in the dye contain iodine and have resulted in severe reactions and even death in a few cases. If the client is allergic to iodine, the healthcare provider should be notified before the CT is performed. The use of contrast dye for the procedure will typically be omitted to avoid the risk of a severe reaction. 1. Incorrect: It is very important that a client receive information regarding risks and benefits of a procedure before providing consent (informed consent), but assuring that the consent was provided is not the priority for client safety over the risk of a severe reaction to the dye. 3. Incorrect: Again, it is very important to implement increased fluid intake following procedures, such as this CT of the abdomen, to help flush the dye through the kidneys. However, the safety priority remains the potential for a severe reaction that should be avoided by asking about allergies to iodine. 4. Incorrect: Monitoring urine output is an important nursing action following the CT because sometimes dye can lead to kidney problems or can increase problems in clients with existing renal disease. But this is not a priority over assessing for allergies that could lead to severe reactions.)

A client reports to the nurse, "I just do not feel well. Something is wrong." The client's vital signs are BP: 130/88, HR: 102, RR: 28. What should the nurse do next? 1. Administer PRN anxiolytic. 2. Connect to oxygen saturation monitor. 3. Reassure the client that everything is okay. 4. Instruct on relaxation technique.

2. Connect to oxygen saturation monitor. (2. Correct: Look at HR and RR. This data is telling you that the client is restless and has tachycardia, think hypoxia FIRST when you see these 2 symptoms. 1. Incorrect: Anxiolytic medications are used to treat anxiety, however, in a client with hypoxia, this would decrease the respiratory rate so much that respiratory arrest could occur. Don't be a killer nurse. 3. Incorrect: This would be incorrect because everything is not okay. Remember, you must pick an answer that fixes the problem. This choice ignores it. 4. Incorrect: This will not fix the problem and is unsafe. The client is hypoxic, and this would make you a killer nurse.)

An elderly, confused client with dehydration is admitted to the medical unit. Which intervention would be appropriate for the RN to delegate to the LPN? 1. Encourage client contribution in care decision making. 2. Reinforce the teaching plan with the client's family. 3. Maintain fresh fluids at bedside. 4. Evaluate I&O for adequate fluid replacement.

2. Reinforce the teaching plan with the client's family. (2. Correct: The LPN can reinforce teaching. 1. Incorrect: The LPN can encourage the client to participate in care decision making. However, the client is confused and is not capable at this time to be involved in their plan of care. 3. Incorrect: This can best be accomplished by the unlicensed assistive personnel (UAP), it can be done by LPN but not best use of resources. 4. Incorrect: Evaluation is a role of the RN, not LPN. LPN can observe and data collect but not assess and evaluate on the NCLEX.)

What signs/symptoms would the nurse expect to find in a client diagnosed with pernicious anemia? Select all that apply 1. Pain 2. Smooth, red tongue 3. Burning feeling in feet 4. Lightheadedness 5. Dyspnea on exertion

2. Smooth, red tongue 3. Burning feeling in feet 4. Lightheadedness 5. Dyspnea on exertion (2., 3., 4., & 5. Pernicious anemia symptoms could include a smooth tongue that is red in color rather than a healthy pink. And neurological problems such as a burning feeling in the feet, slow reflexes, and disorientation. Light headedness, dyspnea on exertion, fatigue, and breathlessness are anemia symptoms that clients often report. 1. Incorrect. Pain is a symptom seen in sickle cell anemia.)

The nurse should teach the client with chronic pancreatitis how to monitor for which problem that can occur as a result of the disease? 1. Hypertension 2. Diabetes 3. Hypothyroidism 4. Graves disease

2. Diabetes (2. Correct: Insulin is produced in the pancreas. When the client has chronic pancreatitis, the pancreas becomes unable to produce insulin, thus resulting in diabetes. 1. Incorrect: No, not associated with chronic pancreatitis. 3. Incorrect: No, not associated with chronic pancreatitis. 4. Incorrect: No, not associated with chronic pancreatitis.)

A client is curious about visible appearance changes related to menopause. What menopausal changes, in general, would the nurse explain to the client? 1. Bone loss and fractures. 2. Loss of muscle mass. 3. Improved skin turgor and elasticity. 4. A reduction in waist size.

2. Loss of muscle mass. (2. Correct: Visible changes associated with menopause include loss of muscle mass, increased fat tissue leading to thicker waist, dryness of the skin and vagina, hot flashes, sleep abnormalities, and mood changes. 1. Incorrect: Bone loss is dependent on bone mass, weight-bearing exercise, and nutrition. Some bone loss may occur, but may not lead to fractures. 3. Incorrect: A decrease in turgor and elasticity may occur as we grow older. 4. Incorrect: There is increased fat tissue with an increase in waist size.)

The nurse is updating the client's plan of care 24 hours after admission. What data would indicate to the nurse that the client is improving? While in the emergency department, 68 year old client being treated for flu symptoms, became symptomatic with an episode of atrial tachycardia which was successfully treated with cardioversion. After stabilization, the client was admitted to the telemetry unit with a diagnosis of the flu, and a history of angina. Treatment included: Bedrest with bathroom privileges. Continuous cardiac monitoring remaining in NSR ½ Normal Saline at 75 mL/hour. 2 gm Low sodium diet. Peramivir 600 mg IVPB times one dose. ECG every 8 hours times three - NSR Select all that apply 1. Troponin T - 0.10 ng/mL 2. Coughing up moderate amount of clear to white sputum 3. Urinary output past 8 hours - 225 mL 4. BP - 100/64, Respirations - 18/min, Temperature - 99.2° F (37.3° C) 5. Current Telemetry ECG

2. Coughing up moderate amount of clear to white sputum 4. BP - 100/64, Respirations - 18/min, Temperature - 99.2° F (37.3° C) 5. Current Telemetry ECG (2., 4., & 5. Correct: Clients who have had the flu are going to cough and expectorate sputum for a while. The important thing is that the sputum is not green which would indicate a bacterial infection. These vital signs have improved since admit. A low-grade temperature may continue for several days. This ECG strip shows that the client is in a normal sinus rhythm, which is a good thing. 1. Incorrect: Troponin T values greater than or equal to 0.1 ng/mL are a prognostic sign in patients with ischemic heart disease and most other situations. 3. Incorrect: The average UOP here is 28 mL/hour and means the client is still dehydrated or the kidneys are not working properly. Remember, this client had Peramivir IVPB.)

The nurse observes an unlicensed assistive personnel (UAP) performing AM care for a client with a plaster leg cast applied 12 hours ago. Which action by the UAP should the nurse intervene? 1. Lifting the affected leg with the palms of the hands 2. Covering the affected leg with a blanket to avoid chills 3. Placing plastic over the entire cast prior to bathing 4. Elevating the casted leg on two pillows

2. Covering the affected leg with a blanket to avoid chills (2. Correct: Plaster cast take 24-48 hours to completely dry. During this time they release heat. The new cast should not be covered so that heat from the cast can evaporate. If the heat is not allowed to dissipate, drying will take longer. 1. Incorrect: Plaster cast take 24-48 hours to completely dry. During this time the cast should be handled carefully as to avoid indentations. Handling the cast carefully with the palms and not the fingertips will prevent indentations. Indentations in the cast could cause skin breakdown inside the cast. 3. Incorrect: Yes, will keep cast from getting wet. Plaster casts should never get wet. The plaster cast does not hold up well in water. Wet casts can also irritate the skin underneath. 4. Incorrect: Yes, may elevate the leg. Fractures are prone to swelling. Elevating fractures while casted is a common occurrence. Elevation prevents swelling.)

A client has been admitted to the ICU following an extensive anterior-wall myocardial infarction. What change in vital signs would alert the nurse the client may be experiencing cardiogenic shock? Select all that apply 1. Bounding pulse of 65. 2. Decreased urinary output. 3. Blood pressure of 80/40. 4. Cold and clammy skin. 5. Slowed respiratory rate.

2. Decreased urinary output. 3. Blood pressure of 80/40. 4. Cold and clammy skin. (2., 3., & 4. Correct: Cardiogenic shock is a condition in which inadequate blood flow leads to poor tissue perfusion throughout the body. This type of shock is usually the result of a diastolic dysfunction in the ventricles caused by a variety of heart issues or diseases. In shock, systolic blood pressure levels drop quickly below 100, further decreasing tissue perfusion as well as lowering level of consciousness. The resulting reduction in kidney perfusion can decrease urinary output to the point of oliguria. The body will compensate by shunting blood inward to vital organs, decreasing peripheral perfusion and resulting in cold, clammy extremities. If not treated immediately, the client can die. 1. Incorrect: In any type of shock, the pulse increases to pump existing volume throughout the body to perfuse vital organs. 5. Incorrect: The respiratory rate will increase with the heart rate since the body is not receiving enough oxygen secondary to poor perfusion.)

The nurse is evaluating a client for compliance to the prescribed diabetic program by checking recent lab results. Based on the lab data, what should the nurse conclude regarding the client? Fasting Blood Glucose: 90 mg/dL (4.995 mmol/L) Hemoglobin A1C: 6.5% 1. At risk for developing hypoglycemia. 2. Demonstrating good control of blood glucose. 3. At risk for developing Somogyi phenomenon. 4. Demonstrating signs of insulin resistance.

2. Demonstrating good control of blood glucose. (2. Correct: These are normal lab values indicating good control of blood glucose. 1. Incorrect: The client is not at risk for developing hypoglycemia. Fasting glucose is currently normal. 3. Incorrect: Somogyi pnenomenon occurs when there is a rebound hyperglycemia. The client is given too much insulin and their blood glucose level drops. The body attempts to compensate and releases hormones (epinephrine) that causes glycogenolysis. This leads to a rebound increase in the blood glucose levels. The insulin dose for this client should be decreased. Dawn's phenomena occurs when there is an increase in the blood glucose in the early morning hours due to release of growth hormone and cortisol. The client's insulin dose should be increased. 4. Incorrect: There are no signs of insulin resistance demonstrated from these normal values.)

The nurse is preparing to educate a group of clients on how to decrease the risk of developing recurrent renal calculi. What topics should the nurse include? Select all that apply 1. High-purine foods to consume 2. Discuss diuretic use to prevent urinary stasis 3. Straining urine with each void 4. Maintaining a daily water intake of at least 2 liters 5. Foods low in calcium

2. Discuss diuretic use to prevent urinary stasis 4. Maintaining a daily water intake of at least 2 liters 5. Foods low in calcium (2., 4., & 5. Correct: Diuretics are often used to prevent urinary stasis and further calculus formation. Thiazides decrease calcium excretion into the urine. Daily fluid intake should be 2-3 liters per day to ensure good renal function. Most stones are calcium stones, so decrease calcium in the diet to reduce the chance of calcium stones. 1. Incorrect: Clients with a history of renal calculi should be on a low-purine diet. When purines are digested, they produce a waste product called uric acid, which needs to be decreased in clients with renal calculi and with history of gout. 3. Incorrect: Straining is only necessary for the client with acute signs/symptoms of renal calculi.)

What preoperative information should the nurse provide to the client who is scheduled for an exercise stress test tomorrow morning? 1. Eat a light breakfast two hours before the test. 2. Dress in loose, comfortable clothing. 3. Take nitroglycerin dose 15 minutes prior to test. 4. Limit drinks with caffeine to 8 ounces (240 mL) within 12 hours.

2. Dress in loose, comfortable clothing. (2. Correct: The client should dress in loose, comfortable clothing the day of the test because the stress test consists of intense exercise. 1. Incorrect: Don't eat or drink anything except water for 4 hours before the test. The fullness from a meal makes it difficult to perform the stress test. 3. Incorrect: Clients are asked to hold beta-blockers, calcium channel blockers, and nitroglycerin medicines prior to a stress test. These medications either increase or slow down the heart rate, which can affect the test. 4. Incorrect: The client should avoid products containing caffeine for 24 hours prior to the stress test. Caffeine increases the heart rate and can affect the results of the test.)

The nurse is teaching a group of high school students about car accident prevention. Who would the nurse include as the highest risk for a motor vehicle crash (MVC)? 1. Males who have just turned 19 years of age. 2. Drivers who have recently acquired a driver's license. 3. A group of students that carpool to the senior prom. 4. Female students who drive to weekly football games.

2. Drivers who have recently acquired a driver's license. (2. Correct: According to the Centers for Disease Control and Prevention(CDC), crash risk is particularly high during the first year that teenagers are eligible to drive. Though teenagers who are 19 years old, carpooling to the senior prom, and driving to weekly football games are also at risk for an MVC, they are not the highest-risk teenage group. 1. Incorrect: The risk for all teens is higher than any other driving group, however, by the age of 19, the teen has generally been driving for several years and the statistical risk of having an accident drops. 3. Incorrect: Carpooling to the senior prom does not by itself increase a teen's risk for having a wreck. Driving while under the influence of alcohol will, so a designated driver is key. 4. Incorrect: Driving to a football game weekly does not by itself increase a teen's risk for having an accident. The female gender does not increase the risk of MVC.)

Immediately following a below-the-knee amputation (BKA), the nurse positions the client to prevent complications. What intervention related to position of the residual limb is a priority at this time? 1. Flat on the bed 2. Elevate foot of the bed 3. In a position of comfort 4. In a dependent position

2. Elevate foot of the bed (2. Correct: It is normal to experience post-operative swelling after a BKA. Immediately after surgery, the foot of the bed should be elevated to reduce swelling. An ACE compression bandage will be used to reduce swelling and prevent hemorrhage. The other positions would not be as appropriate since swelling is an issue after a below-the-knee amputation. 1. Incorrect: Flat on the bed will not relieve swelling. Post-operatively for a BKA, hemorrhage and swelling are the biggest concerns immediately following surgery. 3. Incorrect: Position of comfort may increase swelling. Immediately following a BKA, elevating the foot of the bed and the ACE compression wrap are used to present hemorrhage and swelling. Positioning for comfort is not appropriate. 4. Incorrect: Placing in a dependent position will increase swelling. Swelling post-operative is a normal occurrence and elevating the foot of the bed and the use of an ACE wrap will help prevent swelling.)

What should the nurse teach the client following a right knee arthroscopy? Select all that apply 1. Apply ice to right knee continuously for the first 24 hours. 2. Elevate the right knee when sitting. 3. Notify the primary healthcare provider of tingling in the right leg. 4. Gradually start an exercise program to prevent scarring. 5. Place a plastic bag over wound when showering.

2. Elevate the right knee when sitting. 3. Notify the primary healthcare provider of tingling in the right leg. 4. Gradually start an exercise program to prevent scarring. 5. Place a plastic bag over wound when showering. (2., 3., 4. & 5. Correct: Elevating the joint for several days will reduce swelling and pain. Tingling to the extremity could mean nerves have been damaged. Exercise is gradually started to strengthen muscles surrounding the joint and prevent scarring of surrounding soft tissues. The client needs to keep the site as clean and dry as possible. 1. Incorrect: Continuous ice can cause tissue damage.)

Which nursing interventions will help to prevent a contracture post-operatively in a client with a below the knee amputation? Select all that apply 1. Keep the residual limb elevated on a pillow at all times 2. Ensure the residual limb is positioned flat on the bed 3. Position the client prone several times a day 4. Keep head of bed elevated with knees up. 5. Apply anti-embolism stockings to the unaffected leg

2. Ensure the residual limb is positioned flat on the bed 3. Position the client prone several times a day (2. & 3. Correct: We want the limb flat on the bed to prevent a contracture, the prone position will also stretch out any flexion that has occurred as a defensive withdrawal to muscle spasms. 1. Incorrect: This will promote a contracture. Flexion contractures are avoided by preventing hip flexion and elevation of the limb on pillows. 4. Incorrect: This will promote rather than prevent a contracture. The client must avoid positions that promote hips being flexed. 5. Incorrect: Anti-embolism stockings have nothing to do with contractures.)

An elderly client who lives alone is being discharged home following a total hip replacement. The home care nurse is completing a safety assessment of the home environment prior to the client's arrival. Which conditions would require modifications to ensure client safety? Select all that apply 1. Wall-to-wall carpeting 2. Entrance throw rugs 3. Downstairs bathroom 4. Rail-free porch stairs 5. Step stool in kitchen

2. Entrance throw rugs 4. Rail-free porch stairs 5. Step stool in kitchen (2., 4., & 5. Correct: Throw rugs are loose fall hazards that should be removed or tacked down to prevent tripping, particularly for a client whose mobility is impaired by hip surgery. Both inside and outside stairs should have hand rails to provide stability when in use. The presence of a step stool in the kitchen indicates that some items in the cupboards are out of reach for the client. Rearranging frequently used items to within the client's reach would be much safer than using a step stool. 1. Incorrect: Wall to wall carpeting does not generally present safety issues since it is attached to the floor on all four sides and does not move even if the client were shuffling on the surface of the carpet or using a walker. 3. Incorrect: The existence of a downstairs bathroom is a positive feature that alleviates the need for the client to climb stairs frequently during the day. Stairs are challenging for elderly adults, plus this client also has limited mobility following a total hip replacement.)

The nurse has been working on a health plan promoting increased physical activity for a sedentary client. Which client outcome would indicate that the interventions were successful? 1. Walks 10 minutes per day periodically. 2. Establishes a routine of 30 minutes of brisk walking three days per week. 3. Reports there is not enough time for exercise. 4. Reports walking daily for about two weeks out of the last three months.

2. Establishes a routine of 30 minutes of brisk walking three days per week. (2. Correct: A successful outcome would indicate a change in behavior. A routine of walking three times per week indicates behavioral change. Therefore, the interventions were successful. 1. Incorrect: Periodic walking does not demonstrate a positive outcome for the plan. Perhaps barriers exist that keep the client from a routine. Further assessment would be needed. 3. Incorrect: The client's report indicates perceived barriers to action. The client needs help to identify small blocks of time. 4. Incorrect: The behavior changed for a brief period; however, the new behavior did not continue. Further evaluation should be performed.)

The client with ulcerative colitis calls the clinic and reports increasing abdominal pain and increased frequency of loose stools. The client asks the nurse to clarify foods that can be eaten with ulcerative colitis. What foods should the nurse suggest? Select all that apply 1. Dried beans 2. Fish 3. Apples 4. Yogurt 5. Scrambled eggs

2. Fish 5. Scrambled eggs (2., & 5. Correct: Fish and scrambled eggs are both high in protein and low in fiber. Foods high in fiber are irritating to the GI tract and should be avoided. A food diary is needed to determine triggers for flare-ups. 1. Incorrect: Fiber in the beans will increase motility. 3. Incorrect: Fiber in apple will increase motility. 4. Incorrect: Dairy products should be avoided in times of flare-ups as dairy is often a cause of flare ups.)

An elderly client is admitted to the floor with vomiting and diarrhea for three days. The client is receiving IV fluids at 200 mL/hr via pump. What would be the priority nursing action? 1. Obtaining Intake and Output 2. Frequent lung assessments 3. Vital signs every shift 4. Monitoring the IV site for infiltration

2. Frequent lung assessments (2. Correct: IV fluids at 200 mL/hr is a rapid infusion rate. The elderly adult is at risk for circulatory overload and should be closely monitored during rapid infusion rates. Lung assessments are important in detecting fluid overload. The client may experience shortness of breath and moist crackles on auscultation. 1. Incorrect: I and O are important, but less priority than lung assessment in the elderly client. 3. Incorrect: Vital signs should probably be more frequent than every shift on the elderly client with dehydration. 4. Incorrect: The IV site should be monitored for infiltration, but will not be priority over lung assessment in the elderly client.)

The nurse is admitting an elderly client reporting abdominal pain. During assessment, the client answers inappropriately or just smiles in response to questions. What should the nurse suspect is the most likely cause for this behavior? 1. Developmental delay 2. Hearing difficulty 3. Pain 4. Confusion

2. Hearing difficulty (2. Correct: Clients unable to hear properly are often not aware of the deficit, or may prefer not to acknowledge the situation. Inappropriate responses may indicate the client did not hear the question correctly. Smiling or nodding is occasionally an automatic reaction by a client unable to understand the conversation. 1. Incorrect: Inappropriate responses, or even just smiling, are not indicative of a developmental delay. Many other causes can account for such behaviors, particularly in elderly clients. There is not enough evidence to support a developmental delay. 3. Incorrect: Although the client may have abdominal discomfort, the behavior noted by the nurse does not suggest extreme pain. In fact, the client does occasionally smile, which is not generally noted with pain or discomfort. 4. Incorrect: It is never appropriate to assume that advanced age is automatically associated with confusion or dementia. A client's age should never be automatically equated with mental or physical decline without physical assessment and data to support that theory.)

After completing the initial morning assessment of a client, the nurse notes that a dose of intranasal desmopressin is to be administered. What action is most important for the nurse to take? 0815. Client reports drowsiness and a mild headache rated 3 on a scale of 0-10. Requests acetaminophen for pain. Denies photophobia. PERRLA. Skin warm/dry. Heart sounds regular with apical pulse of 104/min. No murmurs/gallops/clicks/rubs noted. Lungs clear bilaterally. 1+/4+ pitting edema noted at ankles. Pulses equal bilaterally, 3+/4+. 7-3: Intake - 1200 mL, Output - 1220 mL 3-11: Intake - 850 mL, Output - 450 mL 11-7: Intake - 640 mL, Output - 100 mL 0800 - BP: 110/68, P: 88, R: 18, Weight: 150 lbs (68 kg) 1200 - BP: 108/64, P: 84, R: 16 1600 - BP: 112/70, P: 78, R: 20 2000 - BP: 108/66, P: 82, R: 16 2400 - BP: 122/78, P: 88, R: 18 0400 - BP: 130/84, P: 96, R: 18 0800 - BP: 146/88, P: 104, R: 20, Weight: 155 lbs (70 kg) Drug Guide: Desmopressin Therapeutic Effects: Prevention of nocturnal enuresis. Maintenance of appropriate body water content in diabetes insipidus (DI). Control of bleeding in certain types of hemophilia or von Willebrand's disease. Intranasal: Onset 1 hour, Peak 4-5 hours, Duration 8-20 hours Use Cautiously in: Angina pectoris; Hypertension; Clients at risk for hyponatremia. Nursing Implications: Monitor urine and plasma osmolality and urine volume frequently. Assess for symptoms of dehydration. Weigh client daily. Assess for edema. Toxicity and overdose: Signs and symptoms of water intoxication include confusion, drowsiness, headache, weight gain, difficulty urinating, seizures, coma. Treatment of overdose: Decrease dose and, if severe, administer furosemide. Client teaching: Instruct on intranasal administration. Prime pump prior to use by pressing down 4 times. Take missed dose as soon as remembered, but do not double dose. Desired Outcome: Decreased frequency of nocturnal enuresis. Decrease in urine volume. Relief of polydipsia. Increased urine osmolality. Control of bleeding in hemophilia. 1. Measure urine osmolality. 2. Hold desmopressin dose. 3. Administer acetaminophen for headache. 4. Instruct client on intranasal administration of desmopressin.

2. Hold desmopressin dose. (2. Correct: This client is in fluid volume excess, so the desmopressin needs to be held. The nurse needs to discuss the dose with the primary healthcare provider. 1. Incorrect: Urine osmolality can be assessed but it is not the priority. There are lots of data findings indicating that the client is in a fluid volume excess. 3. Incorrect: There is nothing wrong with giving the client acetaminophen for reports of a headache, but it is not the most important action for the nurse to take. 4. Incorrect: Teaching is an important nursing task. However, it is not the most important task now. Teaching can be done later.)

The nurse is caring for a client diagnosed with heat exhaustion. Which finding by the nurse suggests a problem? 1. Temperature 101 degrees F (38.3 degrees C) 2. Hot, dry skin 3. Profuse sweating 4. Headache

2. Hot, dry skin (2. Correct: The body cools itself by sweating and allowing that sweat to evaporate. This requires enough fluid in the body to make sweat, air circulating across the skin, and low enough air humidity to allow that sweat to evaporate. 1. Incorrect: With heat stroke the body's temperature reaches more than 104 degrees F (40 degrees C). 3. Incorrect: Sweating is seen in heat exhaustion. Sweating stops with heat stroke. 4. Incorrect: Clients with heart exhaustion usually have flu like symptoms with headache, weakness, nausea and/or vomiting.)

What electrolyte imbalance should the nurse monitor for when caring for a client diagnosed with chronic alcoholism? Select all that apply 1. Hypochloremia 2. Hypokalemia 3. Hypophosphatemia 4. Hypomagnesemia 5. Hypocalcemia

2. Hypokalemia 3. Hypophosphatemia 4. Hypomagnesemia 5. Hypocalcemia (2., 3., 4., & 5. Correct: The number one way of getting rid of potassium is through the kidneys. What does alcohol make you do? Diuresis. Acute hypophosphatemia is seen in up to 50% of patients over the first 2-3 days after they are hospitalized for alcohol overuse. Hypophosphatemia is manifested as rhabdomyolysis (muscle breakdown) and weakness of the skeletal muscles. Magnesium deficiency occurs due to that increase in diuresis as well. Hypomagnesemia is often accompanied by hypocalcemia, or lowered calcium levels, which may be aggravated by a deficiency of vitamin D. 1. Incorrect: Hypochloremia is usually caused by excess use of loop diuretics, nasogastric suction, vomiting or diarrhea due to small bowel abnormalities, and loss of fluids through the skin occurring because of trauma such as burns.)

The nurse cares for a client who is scheduled for an upper GI series. The nurse teaches the client about the test. Which statement by the client indicates an understanding of the nurse's teaching? 1. I'll have to take a strong laxative the morning of the test. 2. I'll have to drink contrast while x-rays are taken. 3. I'll have a CT scan after I'm injected with a radiopaque contrast dye. 4. I'll have an instrument passed through my mouth to my stomach.

2. I'll have to drink contrast while x-rays are taken. (2. Correct: In an upper GI series (sometimes called a barium swallow test), the client swallows barium contrast while x-rays are taken. 1. Incorrect: Laxatives are taken the night before a colonoscopy to ensure stool is cleared from the colon. Waiting to take the laxative the morning of the test would be ineffective and uncomfortable for the client. 3. Incorrect: Radiopaque dye injected before a CT (computed tomography) scan is not part of a GI series. This would be a totally different diagnostic test from the upper GI. 4. Incorrect: In a gastroscopy (sometimes called a gastric endoscopy), a scope is passed through the mouth to the stomach to visualize the inner lining of the upper GI tract.)

A client has been admitted with a stroke on the right side of the brain. What clinical manifestations does the nurse expect to find when assessing this client? Select all that apply 1. Right sided hemiplegia 2. Impaired judgment 3. Depression 4. Impaired language comprehension 5. Impulsiveness 6. Impaired speech

2. Impaired judgment 5. Impulsiveness (2. & 5. Correct. The client with right sided brain damage will have left sided hemiplegia and will exhibit impulsive behavior and impaired judgment. 1. Incorrect. This is seen with left-brain damage. 3. Incorrect. This is seen with left-brain damage. 4. Incorrect. This is seen with left-brain damage. 6. Incorrect. This is seen with left-brain damage.)

The nurse is caring for a client with bladder cancer who is 2 days post an ileal conduit. Assessment of the urinary output verifies that the urine has flecks of mucus and the hourly output has gone from 200 mL at 8:00 am to 140 mL at 10:00 am. What is the nurse's priority action? 1. Check for leakage from the stoma 2. Increase fluids to 2000 mL/24 hours 3. Monitor the site for signs of infection 4. Perform a bladder scan

2. Increase fluids to 2000 mL/24 hours (2. Correct: Increase fluids. Why? Well we have a drop in output, but it's still well within normal range and the client is two days post-procedure (not immediate post-op, when we would be most concerned about bleeding). So increase fluids to see if the output picks back up. Also, high fluid intake is helpful to flush the ileal conduit. 1. Incorrect: This is an appropriate nursing action, but is not the priority and does not address the decreased urine output. 3. Incorrect: This is an appropriate nursing action but not the priority. It does not address the decreased urinary output. 4. Incorrect: We would scan the bladder if we were worried about urinary retention. It's a little premature for that, but we could scan the bladder after increasing fluids. Remember this is a priority question.)

Which signs/symptoms noted by the nurse would support a client history of chronic emphysema? Select all that apply 1. Atelectasis. 2. Increased anteroposterior (AP) diameter. 3. Breathlessness. 4. Use of accessory muscles with respiration. 5. Leans backwards to breathe. 6. Clubbing of fingernails

2. Increased anteroposterior (AP) diameter. 3. Breathlessness. 4. Use of accessory muscles with respiration. 6. Clubbing of fingernails (2., 3., 4., & 6. Correct: Emphysema is described as a permanent hyperinflation of lung beyond the bronchioles with destruction of alveolar walls. Airway resistance is increased, especially on expiration. Inspection reveals dyspnea on exertion, barrel chest (anteroposterior diameter is equal to transverse diameter), tachypnea, and use of accessory muscles with respiration. Clubbing of fingernails is due to chronically decreased oxygen levels. 1. Incorrect: Atelectasis is collapse of alveolar lung tissue, and findings reflect presence of a small, airless lung. This condition is caused by complete obstruction of a draining bronchus by a tumor, thick secretions, or an aspirated foreign body, or by compression of lung. 5. Incorrect: Client tends to lean forward (orthopnea) and uses accessory muscles of respiration to breathe.)

The nurse is caring for a client following a total thyroidectomy. What findings would alert the nurse to potential complications? Select all that apply 1. Neck dressing intact, clean and dry 2. Increased blood pressure and pulse 3. High-pitched, harsh respirations 4. Vocal quality weak and clear 5. Left-sided cheek twitching

2. Increased blood pressure and pulse 3. High-pitched, harsh respirations 5. Left-sided cheek twitching (2, 3 and 5. Correct: There are several potential complications following a thyroidectomy. One life-threatening problem is the potential for a thyroid storm in which a large bolus of thyroid hormone is dumped into the system, causing increased blood pressure and pulse which could lead to intracranial hemorrhage. High pitched, harsh respirations indicate increasing edema and the potential for obstructed airway. A third potential problem may occur if one or more parathyroid glands are removed, placing the client at risk for hypocalcemia, as evidenced by a positive Chvostek's sign. 1. Incorrect: The surgical dressing around the neck should definitely be intact and dry post-op. Any drainage that may develop would take a while to seep through the dressing. No concerns here. 4. Incorrect: The client has just had surgery on the thyroid, causing swelling that could affect vocal cords. A weak initial voice is not unusual as long as the vocal quality is clear.)

A client is transported to the emergency department following a 20-foot fall from a ski lift. The nurse records initial assessment findings on the chart. Based on that data, what actions should the nurse implement immediately? Blood pressure 90/40 Heart rate 125 Respiratory rate 30 and labored JVD 3 cm Sub-Q emphysema noted to right shoulder area. Select all that apply 1. Apply occlusive dressing to chest. 2. Initiate large gauge I.V. line. 3. Prepare for chest tube placement. 4. Administer high-flow oxygen. 5. Position client on right side.

2. Initiate large gauge I.V. line. 3. Prepare for chest tube placement. 4. Administer high-flow oxygen. (2, 3 and 4. Correct: Based on the assessment data recorded by the nurse, the client most likely has a tension pneumothorax secondary to blunt force trauma from the fall. Immediate actions must focus on preventing tracheal deviation and a fatal outcome. The need for intravenous fluids and medications in any trauma requires at least one large-bore I.V. line or more. This client will need immediate chest tube placement to relieve increasing intrathoracic pressure. While preparing the client for this procedure, high-flow oxygen should be administered via non-rebreather mask because of the client's respiratory distress. 1. Incorrect: There is no indication in the question of an open chest wound, or that a dressing is needed. The occlusive chest dressing will be placed over the insertion site of the chest tube after placement is completed. 5. Incorrect: This trauma client will be secured to a back board, most likely with a C-collar in place, until x-rays confirm there has not been a cervical spine injury. Placing the client on the right side is counterproductive and in fact could further impair respiratory efforts.)

A client is transported to the emergency department following a 20 foot fall from a ski lift. The nurse records initial assessment findings on the chart. Based on that data, what actions should the nurse implement immediately? BP 90/40; HR 125; RR 30 and labored; + jugular venous distention (JVD) with subcutaneous emphysema noted to right shoulder area. Select all that apply 1. Apply occlusive dressing to chest. 2. Initiate large gauge IV line. 3. Prepare for chest tube placement. 4. Administer high flow oxygen. 5. Position client on right side.

2. Initiate large gauge IV line. 3. Prepare for chest tube placement. 4. Administer high flow oxygen. (2., 3. & 4. Correct: Based on the assessment data recorded by the nurse, the client most likely has a tension pneumothorax secondary to blunt force trauma from the fall. Immediate actions must focus on preventing tracheal deviation and a fatal outcome. The need for intravenous fluids and medications in any trauma requires at least one large bore IV line or more. This client will need immediate chest tube placement to relieve increasing intrathoracic pressure. While preparing the client for this procedure, high-flow oxygen should be administered via nonrebreather mask because of the client's respiratory distress. 1. Incorrect: There is no indication in the question of an open chest wound, or that a dressing is needed. The occlusive chest dressing will be placed over the insertion site of the chest tube after placement is completed. 5. Incorrect: This trauma client will be secured to a back board, most likely with a cervical collar in place, until x-rays confirm there has not been a cervical spine injury. Placing the client on the right side is counterproductive and in fact could further impair respiratory efforts.)

Two hours after admission, a client reports palpitations, chest discomfort, and light-headedness. The nurse connects the client to a cardiac monitor and notes a weak, thread pulse, and a BP of 90/50. Which action should the nurse take? Select all that apply 1. Administer Lidocaine 50 mg intravenous push (IVP). 2. Initiate oxygen at 2 liters per nasal cannula. 3. Apply oxygen saturation monitor to client. 4. Prepare for immediate synchronized cardioversion. 5. Perform carotid massage. 6. Begin cardiopulmonary resuscitation.

2. Initiate oxygen at 2 liters per nasal cannula. 3. Apply oxygen saturation monitor to client. 4. Prepare for immediate synchronized cardioversion. (2., 3., & 4. Correct. This client has a rapid heart rate of 188/min. The actual rhythm is atrial tachycardia, but can also be identified as supraventricular tachycardia because the heartrate is greater than 150/min. This client is considered unstable so requires oxygen therapy, with O₂ saturation monitoring, and synchronized cardioversion. 1. Incorrect. Lidocaine is not indicated for an atrial or supraventricular dysrhythmia. 5. Incorrect. Carotid massage is not within the scope of practice of the nurse. Asystole could result. 6. Incorrect. This client has a pulse, so CPR is not needed now.)

What should a nurse teach a group of teenage boys who admit to using smokeless tobacco? Select all that apply 1. Smokeless tobacco increases risk for lung cancer. 2. Inspect mouth frequently for lesions. 3. White patches in mouth should be reported to healthcare provider. 4. Risk for stomach cancer can be decreased by not swallowing smokeless tobacco juice. 5. Report decreased saliva to primary healthcare provider. 6. Smoking cessation.

2. Inspect mouth frequently for lesions. 3. White patches in mouth should be reported to healthcare provider. 6. Smoking cessation. (2., 3., & 6. Correct: The mouth should be inspected frequently for painless lesions that do not heal. This may be a sign of oral cancer and should be reported to the primary heathcare provider. White patches (leukoplakia) is a sign of potential oral cancer as well. Nicotine is addictive and is found in smokeless tobacco. Clients using smokeless tobacco can benefit from smoking cessation information/classes. 1. Incorrect: Use of smokeless tobacco increases the risk developing of esophageal cancer, cancers of the mouth, throat, cheek, gums, lips, tongue, pancreatic cancer, stomach cancer, kidney cancer. 4. Incorrect: This is an incorrect statement. Some amount of tobacco juice will be swallowed and can lead to esophagus and stomach cancer. 5. Incorrect: Decreased saliva is not associated with oral cancer.)

A client's arterial blood gas report has arrived at the nurses' station. Based on the results what interventions are required by the nurse? pH - 7.47 PaCO₂ - 29 HCO₃⁻ -23 PO₂ 95%. Select all that apply 1. Start oxygen at 4 liters/min 2. Instruct on taking slow deep breaths 3. Re-breath into a paper bag 4. Calm the client 5. Administer anxiolytic

2. Instruct on taking slow deep breaths 3. Re-breath into a paper bag 4. Calm the client 5. Administer anxiolytic (2., 3., 4. & 5. Correct: This client is in respiratory alkalosis and it's acute because the kidneys have not kicked in. Fix the problem by slowing the respirations and rebreathing the CO₂. The hyperventilating client is either in a panic or hysterical so calm them and give an anxiolytic if necessary. 1. Incorrect: The client does not have an oxygen problem; they have a CO₂ problem. Their O₂ is normal.)

A client arrives to the emergency department after an altercation resulting in a knife wound to the abdomen. The nurse makes the following observation. Which intervention should the nurse perform? Picture Select all that apply 1. Place the client in trendelenburg position. 2. Instruct the client to lie quietly in a low Fowler's position. 3. Apply abdominal binder to gently place the intestines back in the abdominal cavity. 4. Cover exposed intestine with sterile dressings moistened with sterile saline. 5. Notify the surgeon at once.

2. Instruct the client to lie quietly in a low Fowler's position. 4. Cover exposed intestine with sterile dressings moistened with sterile saline. 5. Notify the surgeon at once. (2., 4. & 5. Correct: Low Fowler's position and staying calm and quiet help to minimize protrusion of body tissues. Cover exposed intestines with sterile dressings moistened with sterile saline solution. Have someone notify the surgeon at once and you stay with the client and stay calm. 1. Incorrect: The client should be placed in the low Fowler's position and instructed to lie quietly. These actions minimize protrusion of body tissues. 3. Incorrect: Never push eviscerated abdominal contents back, and do not apply pressure with an abdominal binder. This client needs to go back to surgery.)

The nurse, assessing the lung sounds of a client diagnosed with pneumonia, notes diminished lung sounds and dull percussion in the lower lungs bilaterally. What intervention is correct by the nurse? 1. Place the client in a left lateral recumbent position. 2. Instruct the client to perform incentive spirometry every hour. 3. Encourage the client to increase fluid intake of at least 50 mL/ hour. 4. Have the client use the bedpan to avoid overexertion and exacerbation of symptoms.

2. Instruct the client to perform incentive spirometry every hour. (2. Correct: Incentive spirometry promotes maximum lung expansion, mobilizes secretions, and encourages coughing. 1. Incorrect: The client should be placed in a semi-Fowlers position and told to change positions frequently. 3. Incorrect: The nurse should encourage hydration up to 2 to 3L/day for adequate hydration. 4. Incorrect: The nurse encourages rest but the client must continue to move to enhance secretion clearance and pulmonary ventilation.)

A client's absolute neutrophil count (ANC) is 750/mm³. Which measures should the nurse take to protect the client? Select all that apply 1. Prohibit the client from shaving. 2. Instruct the client to wear a mask when leaving the hospital room. 3. Remove fresh flowers and plants from the client's room. 4. Ask visitors to perform hand hygiene before entering the client's room. 5. Instruct client to avoid flossing of teeth.

2. Instruct the client to wear a mask when leaving the hospital room. 3. Remove fresh flowers and plants from the client's room. 4. Ask visitors to perform hand hygiene before entering the client's room. (2., 3. & 4. Correct: If a client's ANC is less than 1000/mm³, the client is at risk for infection. Instructing the client to wear a mask outside of the hospital room protects the client from infection. The soil in fresh flowers and plants can carry bacteria and fungi, which can cause infection. Performing hand hygiene is the best way to prevent the spread of infection. 1. Incorrect: Not allowing the client to shave would be an appropriate intervention for someone with a low platelet count. 5. Incorrect: Not allowing the client to floss the teeth would be an appropriate intervention for someone with a low platelet count. The client needs good oral care to prevent infections in the mouth.)

A client's absolute neutrophil count (ANC) is 750/mm³. Which measure should the nurse take to protect the client? Select all that apply 1. Prohibit the client from shaving. 2. Instruct the client to wear a mask when leaving the hospital room. 3. Remove fresh flowers and plants from the client's room. 4. Ask visitors to perform hand hygiene before entering the client's room. 5. Instruct client to avoid vigorous flossing of teeth.

2. Instruct the client to wear a mask when leaving the hospital room. 3. Remove fresh flowers and plants from the client's room. 4. Ask visitors to perform hand hygiene before entering the client's room. (2., 3. & 4. Correct: Normal ANC is 1500-8000/mm³. If a client's ANC is less than 1000/mm³, the client is at risk for infection. Instructing the client to wear a mask outside of the hospital room protects the client from infection. The soil in fresh flowers and plants can carry bacteria and fungi, which can cause infection. Performing hand hygiene is the best way to prevent the spread of infection. 1. Incorrect: Not allowing the client to shave or vigorous flossing of teeth would be an appropriate intervention for someone with a low platelet count. This client has a low white cell count. 5. Incorrect: Not allowing the client to shave or vigorous flossing of teeth would be an appropriate intervention for someone with a low platelet count.)

What interventions should the nurse include when planning care for a client post heart transplant? Select all that apply 1. Place on airborne precautions. 2. Instruct visitors to wash hands prior to entering the room. 3. Maintain strict aseptic technique. 4. Initiate pulmonary hygiene measures. 5. Provide for early ambulation.

2. Instruct visitors to wash hands prior to entering the room. 3. Maintain strict aseptic technique. 4. Initiate pulmonary hygiene measures. 5. Provide for early ambulation. (2., 3., 4., & 5. Correct: The transplant recipient is at high risk for infection due to the suppression of the body's normal defense mechanisms. All of these interventions decrease the incidence of the client developing an infection. The heart transplant client is prescribed medications to reduce the risk of organ rejection by inhibiting or suppressing the immune system. Handwashing is the main defense against infection. Pulmonary hygiene measures are are implemented to maintain open airways and prevent respiratory infections. The pulmonary measures can include oral hygiene, deep breathing exercises, mucus-controlling agents, and intermittent positive-pressure breathing. Pulmonary hygiene helps to decrease the development of pneumonia. Early ambulation helps increase general strength and lung expansion. Also ambulation increases circulation,peristalisis,and joint mobility, Emotionally ambulation improves self-esteem and feelings of independence. 1. Incorrect: The client needs to be protected from everyone else so a private room and protective isolation are needed. All persons entering the room must wash hands well and wear a mask and gloves. The client must wear a mask when leaving the room.)

A RN is observing an unlicensed nursing personnel (UAP) feed a client who is on aspiration precautions. Which action by the UAP would require the nurse to intervene? 1. Elevating the head of the bed to a 90 degree angle 2. Instructing the client to lean the head back slightly when swallowing. 3. Adding a thickening agent to liquids. 4. Feeding the client small amounts of food per bite.

2. Instructing the client to lean the head back slightly when swallowing. (2. Correct: This is an incorrect action, and needs intervention by the nurse. The chin should be flexed to prevent the risk of aspiration. A chin down or chin tuck maneuver is widely used in dysphagia treatment to prevent aspiration. 1. Incorrect: This is a correct action. The head of the bed should be elevated which aids in esophageal peristalsis and swallowing is aided by gravity. 3. Incorrect: This is a correct action. Thickened liquids are easier to swallow without aspirating. Drinking liquids thickened will help to prevent choking and stops fluid from entering the lungs. 4. Incorrect: This is a correct action by the UAP, so the nurse does not need to intervene. Smaller amounts of food can be chewed more thoroughly and swallowed with less risk for aspiration.)

Which assessment finding in a client 5 hours post open cholecystectomy would require the nurse to notify the surgeon? 1. Absent bowel sounds. 2. Jackson Pratt drain has 90 mL of blood. 3. Urinary output of 180 mL since return from surgery. 4. Client report of abdominal pain of 8/10.

2. Jackson Pratt drain has 90 mL of blood. (2. Correct. An open cholecystectomy will usually result in the placement of a drain. The drainage should be green (bile). Blood is a problem and needs immediate intervention. 1. Incorrect. It is not uncommon for bowel sounds to be absent after abdominal surgery. This client is only 5 hours postoperative. The client needs to remain NPO until bowel sounds return. 3. Incorrect. The urine output is greater than than 30 mL/hour which is an acceptable amount. There is not a baseline to compare to, so greater 30mL/hour is not abnormal. 4. Incorrect. Pain for this client is an expected finding 5 hours after surgery.)

What interventions should the nurse include in the care plan of a client admitted with Guillain-Barre syndrome? Select all that apply 1. Assess for descending paralysis. 2. Keep a sterile tracheostomy at the bedside. 3. Monitor for heart rate above 120/min. 4. Maintain in side-lying, supine position. 5. Have client perform active range of motion (ROM) every 2 hours while awake.

2. Keep a sterile tracheostomy at the bedside. 3. Monitor for heart rate above 120/min. (2. & 3. Correct: This client is at risk for respiratory paralysis as the disease progresses. An emergency tracheostomy may need to be performed so the nurse should watch out for imminent signs of respiratory failure. Signs include heart rate that is more than 120 bpm or lower than 70 bpm and respiratory rate of more than 30 bpm. The nurse should assess for signs of respiratory distress and prepare for intubation if needed. 1. Incorrect: Ascending paralysis should be assessed for with this disease. Paralysis begins in the lower extremities and moves upward. 4. Incorrect: The client should be assisted to a position with head of bed elevated for full chest excursion. 5. Incorrect: The nurse should perform passive range of motion exercises. Active exercise should be avoided during the acute phase as the client is easily fatigued and muscles are weak. Passive ROM stimulates circulation, improves muscle tone and increases joint mobilization.)

A nurse is educating several unlicensed assistive personnel (UAP) about a dietary prescription for clear liquids. Which selections by the UAP indicate to the nurse an understanding of a clear liquid diet? Select all that apply 1. Vanilla custard 2. Lemon jello 3. Tomato juice 4. Sprite 5. Banana popsicle

2. Lemon jello 4. Sprite 5. Banana popsicle (2., 4., & 5. Correct: These are considered clear liquids. You can see through them. The banana popsicle and lemon jello in a liquid state can be seen through. 1. Incorrect: This would be on a full liquid diet. A full liquid diet allows clear liquids along with thin hot cereals, strained cream soups, juices, milkshakes, custard, pudding and liquid nutritional supplements. 3. Incorrect: This would be on a full liquid diet. A full liquid diet allows clear liquids along with thin hot cereals, strained cream soups, juices, milkshakes, custard, pudding and liquid nutritional supplements.)

A nurse is participating in a cancer risk screening program. Which signs/symptoms would indicate to the nurse that a client needs further investigation? Select all that apply 1. Unexplained weight gain of 10 pounds 2. Leukoplakia 3. Prolonged hoarseness 4. Hematuria 5. Persistent abdominal bloating

2. Leukoplakia 3. Prolonged hoarseness 4. Hematuria 5. Persistent abdominal bloating (2., 3., 4., & 5. Correct: White patches inside the mouth or white spots on the tongue may be leukoplakia, which is a pre-cancerous area that is caused by frequent irritation. It is often caused by smoking or other tobacco use. People who smoke pipes or use oral or spit tobacco are at high risk for leukoplakia. If untreated, it can become mouth cancer. A cough that does not go away and prolonged hoarseness may be a sign of cancer. Hematuria may be a sign of bladder or kidney cancer and needs further investigation. Although women may experience bloating with changes in the menstrual cycle, constant bloating should be investigated to rule out ovarian cancer. 1. Incorrect: Unexplained loss of weight or loss of appetite may indicate some types of cancer. Weight gain is not typically associated with cancer.)

Following a motor vehicle accident, a client is brought to the emergency room with shallow, labored respirations. The client is intubated and placed on a ventilator. What is the nurse's priority action immediately after the intubation? 1. Suction to clear all secretions 2. Listen for bilateral breath sounds 3. Secure the endotracheal tube 4. Obtain x-ray to verify tube placement

2. Listen for bilateral breath sounds (2. Correct: All actions are important but assessment is the nurse's immediate priority. Clear and equal bilateral breath sounds along with equal chest wall movement would confirm that the endotracheal tube has been correctly inserted into trachea. 1.Incorrect: Although suctioning after intubation is an appropriate action, the nursing process requires assessment first. Clearing secretions will be more effective if the endotracheal tube is actually in the correct location in the lungs. 3.Incorrect: Prior to securing the endo tube to the client's mouth/face, it is more important to verify that tube is in the correct location within the lungs. If the tube is not placed properly, the client will not be adequately ventilated, and the tape would have to be removed for reinsertion. 4.Incorrect: Follow up chest x-rays are needed to verify that the endotracheal tube has been properly placed in the lungs. However, this action is not an immediate priority for the nurse. Assessment is the nursing priority.)

The nurse is reviewing the Centers for Disease Control and Prevention's (CDC's) guidelines for immunization recommendations with a group of parents whose children are preparing to attend college in the fall. Which immunization recommendations should the nurse include? Select all that apply 1. Rotavirus 2. Meningococcal 3. Herpes zoster 4. Seasonal influenza 5. Human papilloma virus

2. Meningococcal 4. Seasonal influenza 5. Human papilloma virus (2., 4., & 5. Correct: Meningococcal vaccine protects against bacterial meningitis and is recommended for students entering college. Influenza vaccine is recommended annually for protection against the viruses predicted to be most common for the season. Human papilloma virus vaccine is recommended for protection against the virus which causes cervical and anal cancers. 1. Incorrect: Rotavirus vaccine is recommended during infancy. Rotavirus is the most common cause of diarrheal disease among infants and children. 3. Incorrect: Herpes zoster vaccine is recommended for adults, over the age of 60 to reduce the risk of getting shingles.)

A client who must use crutches, is being taught by the nurse how to perform a three-point gait. What information should the nurse provide? 1. Move right crutch forward, then left foot. Next move left crutch forward, then right foot. 2. Move both crutches forward without bearing weight on the affected leg, then move the unaffected leg forward. 3. Move left crutch and right foot forward together, then move the right crutch and left foot forward together. 4. Move both crutches ahead together, then lift body weight by the arms and swing both legs to the crutches.

2. Move both crutches forward without bearing weight on the affected leg, then move the unaffected leg forward. (2. Correct: This method is correct for the three-point gait. Client has to bear weight on the unaffected foot and both crutches. The affected leg does not touch the ground. 1. Incorrect: This is the four-point alternate gait. This type of gait is used commonly when =both legs are weakened. 3. Incorrect: This is the two-point alternate gait. Two point requires at least partial weight bearing on each foot. 4. Incorrect: This is the swing-to gait. This gait is indicated for individuals with limited use of lower extremities and trunk instability.)

A client admitted for debridement of a leg wound has been diagnosed with vancomycin-resistant enterococci (VRE). What is the nurse's priority action? 1. Place with another client in contact isolation for methicillin-resistant staphylococcus aureus (MRSA). 2. Move the client to a private room with contact precautions. 3. Alert staff to use masks, goggles and gown to provide care. 4. Notify family members to gown and glove before entering room.

2. Move the client to a private room with contact precautions. (2. Correct: VRE normally lives in healthy intestines, but in an immunocompromised client, it can be found in a number of locations, including urinary tract, intestines, blood or wounds. A client diagnosed with VRE must be placed alone in contact isolation for the entire hospital stay, or until there are three negative cultures of that area, each one week apart. 1. Incorrect: The location of this client's VRE is not noted, and it cannot be assumed that it is in the leg wound. A client with VRE must be placed in a private room. This client definitely cannot be placed in the same room as a client with MRSA just because both require contact isolation. Cross-contamination may occur between the two clients. 3. Incorrect: VRE requires contact precautions. Anyone entering the room must first apply a gown and gloves. Before exiting the room, the gown and gloves are removed, discarded inside the room and hands washed prior to leaving. Goggles and mask are not required in this situation. 4. Incorrect: Anyone entering the room will need to follow the contact isolation protocols, including staff, visitors and family. A gown and gloves must be used when entering the room, then removed and discarded prior to leaving the room. However, this is not the nurse's priority.)

What signs/symptoms would the nurse expect to assess in a client diagnosed with acute pericarditis? Select all that apply 1. Petechiae on trunk 2. Muffled heart sounds 3. Pericardial friction rub 4. Pulsus paradoxus 5. Chest pain on deep inspiration

2. Muffled heart sounds 3. Pericardial friction rub 5. Chest pain on deep inspiration (2., 3., & 5. Correct: Muffled heart sounds are indicative of pericarditis. Fluid is between the heart and the chest wall; heart sounds are lowered and distant. A pericardial friction rub is a classic symptom of acute pericarditis. Chest pain is the most common symptoms of pericarditis, and is aggravated by deep inspiration, coughing, position change, and swallowing. 1. Incorrect: Petechiae on the trunk, conjunctiva, and mucous membranes are indicative of endocarditis. 4. Incorrect: Pulsus paradoxus is an exaggerated decrease of systolic blood pressure during inspiration exceeding 12 mmHg. It is the hallmark of cardiac tamponade.)

The nurse is teaching a group of adults how to check skin lesions for signs of melanoma. What should the nurse include? Select all that apply 1. Symmetrical shape 2. Multiple colors with a lesion 3. Odd looking lesion 4. Poorly defined border of lesion 5. Diameter of lesion 6 mm

2. Multiple colors with a lesion 3. Odd looking lesion 4. Poorly defined border of lesion 5. Diameter of lesion 6 mm (2., 3., 4., & 5. Correct: Multiple colors, varying colors from one area to another or uneven distribution of color could mean cancer. Some moles don't exactly fit the criteria for ABCDs of melanoma, but they can be cancerous nonetheless. If you see a mole that looks odd, or you see sudden changes in size, color or shape, get it checked. An irregular scalloped or poorly defined border of the growth could be a sign of melanoma. The diameter of a melanoma is usually bigger than 1/4 inch or 6 mm when diagnosed, but they can be smaller. 1. Incorrect: An asymmetrical shape can be an indicator of melanoma.)

A client is admitted with abdominal pain, distention, fever, dehydration, (+) Cullen's sign and a rigid boardlike abdomen. Which interventions would help control the client's pain in the acute period? Select all that apply 1. Small frequent feedings 2. NG tube to low suction 3. Side-lying position with head elevated 4. Hydromorphone by PCA pump 5. IV isotonic solutions

2. NG tube to low suction 3. Side-lying position with head elevated 4. Hydromorphone by PCA pump (2., 3., & 4. Correct: The pain is caused by inflammation of the pancreas and the autodigestive process. How do we fix the pain? Rest the pancreas and decrease the secretion of digestive enzymes or better known as...empty and dry. The NG tube will keep the stomach empty and dry. Side-lying with head elevated decreases tension on the abdomen and may ease pain. The hydromorphone is an analgesic and will help with the pain. 1. Incorrect: We want them NPO. If you selected this option, then you are telling the NCLEX Lady that you are going to constantly stimulate the release of digestive enzymes with frequent feedings and keep the pancreas inflamed and painful. 5. Incorrect: IV solutions will keep them hydrated and correct electrolyte disturbances but not relieve pain.)

A nurse draws arterial blood gases (ABGs) for a client diagnosed with Guillain-Barré syndrome while the client is receiving oxygen therapy at 2 liters per nasal cannula. How should the nurse interpret the client's ABG result? pH: 7.32, PaCO₂: 51 mmHg, HCO₃⁻: 28, PaO₂: 91 mmHg 1. Hypoxemia with respiratory acidosis 2. Normal oxygenation with respiratory acidosis 3. Partially compensated respiratory alkalosis 4. Fully compensated respiratory acidosis

2. Normal oxygenation with respiratory acidosis (2. Correct: The client has respiratory acidosis with normal oxygenation. The pH of 7.32 indicates a state of acidosis. An elevated PaCO₂ of 51 mm Hg indicates that carbon dioxide is being retained and that the source of the acidosis is respiratory in nature. A HCO₃⁻ level of 28 is elevated slightly, indicating that the body is beginning to compensate for the acidosis, but not enough to correct the body's pH. The oxygenation level, PaO₂, is within the normal reference range. 1. Incorrect: A normal oxygen level does not indicate hypoxemia (low oxygen levels in the blood). 3. Incorrect: The pH level of 7.32 indicates a state of acidosis, not a partially compensated respiratory alkalosis. 4. Incorrect: A pH of 7.32 indicates a state of acidosis, not normal acid-base balance. In order for it to be fully compensated respiratory acidosis, the pH would need to be normal.)

After reinforcing dietary teaching to a client diagnosed with Crohn's Disease, the nurse recognizes client understanding when the client selects which low-residue foods? Select all that apply 1. Broccoli 2. Oatmeal 3. Green peas 4. Spaghetti 5. Cantaloupe 6. Raisins

2. Oatmeal 4. Spaghetti 5. Cantaloupe (2, 4,& 5 Correct: A low residue diet is recommended for clients with inflammatory bowel diseases such as Crohn's Disease, diverticulitis or Ulcerative Colitis. This special diet is designed to decrease fiber in order to limit bowel peristalsis while still including nutritional elements for clients. Cooked oatmeal or pasta are both good choices as well as fruits with no skin and little pulp. Insoluble fiber—the kind in raw veggies, fruits, and nuts—draws water into the colon and can worsen diarrhea for those with IBD. But oatmeal has soluble fiber, which absorbs water and passes more slowly through your digestive tract. Cantaloupe is an excellent choice, since it is a great source of nutrients but has little pulp. 1. Incorrect: Broccoli is a very fibrous vegetable that causes excessive peristalsis, even when cooked. This will create excessive gas and increase discomfort for clients. 3. Incorrect: Green or yellow peas are rough, fibrous vegetables that will cause gas and cramping for most clients with inflammatory bowel diseases, even if cooked. This vegetable is discouraged along with beans, lentils, seeds, and nuts. 6. Incorrect: Raisins are high in fiber, as are other dried fruits such as prunes. Even cooked raisins tend to increase peristalsis which will lead to cramping and excessive bowel movements daily. Clients also need to avoid most fresh fruits with skin, pulp, or seeds.)

The telemetry unit nurse is assessing a newly admitted client following a fall at home. The client has been diagnosed with a left sided cerebrovascular accident (CVA), including aphasia, and a sprained wrist. What is the most effective method the nurse could use to assess the client's pain? 1. Monitor vital signs for elevations. 2. Observe client's non verbal behaviors. 3. Assess sleeping position client chooses. 4. Ask client to point to the pain rating scale.

2. Observe client's non verbal behaviors. (2. Correct: The client has had a left sided stroke which damages the left hemisphere of the brain. Although the question does not specify whether this is receptive or expressive aphasia, the client may have great difficulty identifying the location or amount of pain. Because of the client's difficulty in communicating at this time, the nurse must rely on non-verbal cues such as facial expressions, vocalizations (moaning, crying) or client attention to the injured portion of the body (massaging or holding the painful area). 1. Incorrect: Despite the fact that vital signs often become elevated in the presence of pain, this is not a reliable indicator, particularly since the vital signs could be impacted by the recent CVA. Additionally, tolerance to pain varies, and changing vitals would not provide the most accurate data about the severity or even the location of the client's pain. 3. Incorrect: This method is the least reliable approach in evaluating client discomfort. Waiting for the client to fall sleep delays effective treatment, assuming the client is able to rest at all while experiencing pain. Also, the nurse is assuming that the client would be able to position self in a manner indicating what area is most painful. The impact of a stroke in the left brain might prevent the client from accurately locating or identifying the exact painful area. 4. Incorrect: The client has had left hemisphere damage to the brain with resulting aphasia. There is no data provided in the question regarding the category of aphasia; therefore, the nurse would be aware the client may not be able to indicate the correct location or severity of pain, even if utilizing the smiling face picture scale. This client also may not be able to understand instructions on the use of the scale, or to self evaluate the actual level of pain.)

What assessments would be appropriate for the school nurse to perform related to school safety practices and emergency preparedness? Select all that apply 1. Teach about gun control laws. 2. Observe for gaps or changes in levels of sidewalks. 3. Identify which students have special healthcare needs. 4. Locate all entrances and exits to buildings. 5. Identify threats and hazards in the school and surrounding community. 6. Perform a check of all fire extinguishers.

2. Observe for gaps or changes in levels of sidewalks. 3. Identify which students have special healthcare needs. 4. Locate all entrances and exits to buildings. 5. Identify threats and hazards in the school and surrounding community. 6. Perform a check of all fire extinguishers. (2., 3., 4., 5., & 6. Correct: One of the first things that a school nurse should do is to assess where an accident might happen. Observing for gaps or changes in the level of sidewalks is an example of this assessment. The school nurse should assess for special healthcare needs in the event that the school enters a time of extended lockdown. Some students would require attention during the time of lockdown, such as diabetics who could not wait to receive insulin or have food available. All entrances to the schools must be identified to know where a potential entry for intruder might could occur. Some access points may need to be changed to reduce risk to students. Becoming familiar with all exits is crucial to planning timely and safe evacuation of students if needed. The school nurse can draw upon a wealth of information that exists regarding threats or events that have occurred in the past at the school or in the local community in order to plan for possible future events. Fire extinguishers should be checked on a regular schedule for assessment of access, date of expiration, and functionality. 1. Incorrect: Teaching about laws on gun control is not an assessment, but rather an intervention that can be done. Teaching is not the initial step of the nursing process. Assessment comes first.)

What teaching points should the nurse include when educating a client how to prevent a venous stasis ulcer? Select all that apply 1. Elevate legs above heart for 5 minutes, twice a day. 2. Perform leg exercises regularly. 3. Wear graduated compression stockings. 4. Treat itching with prescribed topical corticosteroids. 5. Minimize stationary standing.

2. Perform leg exercises regularly. 3. Wear graduated compression stockings. 4. Treat itching with prescribed topical corticosteroids. 5. Minimize stationary standing. (2., 3., 4., & 5. Correct: Regular leg exercises improve calf muscle function. Wearing graduated compression stockings will help prevent dilation of lower extremity veins, pain, and a heavy sensation in the legs that typically worsens as the day progresses. Itching can cause the client to scratch, which leads to skin breakdown. Topical corticosteroids can decrease itching and should be used as prescribed. Minimize stationary standing as much as possible to decrease pooling of blood in the lower extremities. 1. Incorrect: Elevate legs above heart for 30 minutes three times a day will minimize edema and reduce intraabdominal pressure.)

While preparing to administer intravenous of chemotherapy the nurse accidently pulls the tubing apart, spilling the solution onto the floor. After clamping the tubing, what is the nurse's immediate action? 1. Use disposable towels to clean up the liquid. 2. Obtain spill kit specific to this type of solution. 3. Complete an incident report for supervisor. 4. Call housekeeping to help clean up the floor.

2. Obtain spill kit specific to this type of solution. (2. Correct: Chemotherapy spill kits are pre-packaged supplies specific to the type of cytotoxic drugs used and are kept in close proximity to the location the chemo is administered. These kits vary slightly but all follow the basic guidelines. Individuals cleaning up the spill must be completely covered head to toe to prevent any contact with the drug. This includes inhalation. This option contains the word solution, which also appears in the question. 1. Incorrect: Disposable towels are not acceptable to clean up a chemotherapy spill. Although these towels are absorbent for kitchens and bathrooms, only special absorbent pads can be used to clean up cytotoxic drugs. 3. Incorrect: While it is true that the nurse will need to complete an incident report regarding the chemotherapy spill, it is certainly not the nurse's immediate action. Focus on staff and client safety first. 4. Incorrect: The responsibility for cleaning up cytotoxic drugs is for the nursing staff involved at the time. Special training and knowledge is required to handle this issue.)

The nurse is planning care for a client admitted with a diagnosis of new onset myasthenia gravis. Which nursing interventions should the nurse include in order to decrease the risk of aspiration? Select all that apply 1. Provide thin liquids such as water with meals. 2. Offer small bites of food. 3. Allow client to rest between each bite of food. 4. Offer small meals in the morning and larger meals in the evening. 5. Position client upright with head tilted slightly back when eating. 6. Provide meals 30 minutes before administration of cholinesterase inhibitor medication.

2. Offer small bites of food. 3. Allow client to rest between each bite of food. (2., & 3. Correct: Offer the client small bites and instruct to chew well, eat slowly, swallow after each bite, and swallow frequently. Allow the client to rest while chewing and in between bites to restore strength. 1. Incorrect: Provide highly viscous foods and thickened liquids that are easy to chew and swallow. Thin liquids are more likely to cause aspiration. 4. Incorrect: Offer large meals in the morning and small meals in the evening. The client is more fatigued as the day progresses, so a smaller meal is best in the evening. 5. Incorrect: Position the client upright with head slightly forward when eating and drinking, using compensatory maneuvers (chin tuck, head turn) as necessary. 6. Incorrect: Adjust the client's eating schedule to optimize medication efficacy. Typically, meals should be taken during periods of optimal strength (such as during the earlier part of the day, 30 minutes after administration of cholinesterase inhibitor medications, or after rest periods).)

A pregnant client has been receiving daily heparin injections for a history of deep vein thrombosis (DVTs) during pregnancy. Which laboratory test result should be immediately reported to the primary healthcare provider? 1. PT of 13 seconds 2. PTT of 22 seconds 3. INR of 1.0 4. Hemoglobin of 11 g/dL (6.8266 mmol/L)

2. PTT of 22 seconds (2. Correct: The test that monitors the efficacy of heparin is the PTT. The normal range for a PTT is 30-40 seconds, but desired outcome of heparin therapy is PTT of 1.5-2.5 times the control without signs of hemorrhage. This client's PTT is below therapeutic range so it is not preventing DVT formation. The dose of heparin will need to be increased. 1. Incorrect: PT monitors the efficacy of warfarin, which is contraindicated in pregnancy because it crosses the placenta which means the fetus would be receiving the medication. PT is measured in seconds. Most of the time, results are given as what is called INR (international normalized ratio). If a client is not taking blood thinning medicines, such as warfarin, the normal range for PT results is 11 to 13.5 seconds. Normal value ranges may vary slightly among different laboratories. Some labs use different measurements or test different samples. 3. Incorrect: INR monitors the efficacy of warfarin, which is contraindicated in pregnancy because it crosses the placenta which means the fetus would be receiving the medication. (Normal INR of 0.8 to 1.1.) If the client is taking warfarin to prevent blood clots, the primary healthcare provider will most likely choose to keep the INR between 2.0 and 3.0. 4. Incorrect: A hemoglobin of 11 g/dL (6.8266 mmol/L)​ is adequate in pregnancy. In pregnancy, there is an increase in plasma volume of the blood in order to help supply oxygen and nutrients to mother and baby. There can be a 20% increase in the total number of red blood cells, but the amount of plasma increases even more causing dilution of those red cells in the body. A hemoglobin level of pregnancy can naturally lower to 10.5 gm/dL (6.5163 mmol/L) representing a normal anemia of pregnancy.)

A nurse receives a client in the post anesthesia care unit following application of a long leg cast to the left leg due to a fractured tibia and fibula. Which interventions should the nurse initiate? Select all that apply 1. Elevate foot of bed 30 degrees. 2. Palpate bilateral pedal pulses. 3. Apply ice packs to fracture site. 4. Mark break through bleeding. 5. Assess client's ability to move toes

2. Palpate bilateral pedal pulses. 3. Apply ice packs to fracture site. 4. Mark break through bleeding. 5. Assess client's ability to move toes (2., 3., 4., & 5. Correct: The priority nursing assessment focuses on any intervention that maintains good circulation to the extremity and prevents complications that can impair mobility. This must include checking distal pulses in both legs to compare the strength of the pulse on both the affected and unaffected side. The nurse should also decrease swelling and risk of compartment syndrome by applying ice to fracture site, assess for bleeding, and check for tingling, coldness, numbness, and ability to move toes; in other words - neurovascular/sensation checks. 1. Incorrect: The affected leg should be elevated, but not both. The nurse should place the affected leg on a pillow and not raise the foot of the bed since this would raise both extremities.)

What interventions should the nurse initiate while caring for a client who has a cooling blanket in place? Select all that apply 1. Assess temperature every hour. 2. Perform comparison check with another thermometer periodically. 3. Assess client skin condition hourly. 4. Turn blanket off when temperature is at goal temperature. 5. Observe for signs of chilling.

2. Perform comparison check with another thermometer periodically. 3. Assess client skin condition hourly. 5. Observe for signs of chilling. (2., 3., & 5. Correct. Perform comparison check with another thermometer periodically to ensure there is no problem associated with equipment failure. For cooling treatments, extended periods of cooling can cause areas of decreased perfusion, skin burns, and tissue injury. Chilling can increase metabolism and body needs. 1. Incorrect: Check the client's temperature every 15 minutes. If the client is cooled too quickly, chilling, increased metabolism, and adverse reactions may occur. 4. Incorrect: The blanket will not immediately return to room temperature and will continue to cool the client even after it is turned off. Turning it off shortly before the goal temperature is achieved will prevent altering the client's core temperature beyond the desired outcome.)

A client returns to the unit post scleral buckling of the right eye. Which nursing interventions should the nurse include? Select all that apply 1. Approach client from the right side. 2. Place personal items within easy reach. 3. Maintain eye patch over right eye. 4. Administer antiemetic for reports of nausea. 5. Assist client to turn, cough, and deep breathe every 2 hours. 6. Place client prone for 1 hour.

2. Place personal items within easy reach. 3. Maintain eye patch over right eye. 4. Administer antiemetic for reports of nausea. (2., 3., & 4. Correct: Place all personal articles and the call light within easy reach. These measures prevent stretching and straining by the client. An eye patch or shield will prevent injury to the affected eye. We do not want the client to vomit, so administer an antiemetic for reports of nausea. Vomiting will increase intraocular pressure. 1. Incorrect: Approach the client on the unaffected side. This approach facilitates eye contact and communication. 5. Incorrect: The goal is to prevent anything that will increase intraocular pressure. That means coughing should be avoided. 6. Incorrect: Lying prone will increase intraocular pressure. After surgery for a detached retina, the client is positioned so that the detachment is dependent. For example, if the outer portion of the right retina is detached, the client is positioned on the right side. Positioning so that the detachment is inferior maintains pressure on that area of the retina, improving its contact with the choroid.)

A client has experienced a cerebrovascular accident (CVA) which resulted in left homonymous hemianopia. Based on this fact, what measures will the nurse include in the client's initial plan of care? Select all that apply 1. Approach the client from his left side. 2. Place the client's meal on the right side of the over bed table. 3. Request a consult for an ophthalmologist. 4. Stand directly in front of the client when addressing. 5. Have client look at the left side of the body.

2. Place the client's meal on the right side of the over bed table. 5. Have client look at the left side of the body. (2. & 5. Correct: Homonymous hemianopia is blindness in half of the visual field. The client has lost half of the visual field in the left side of both eyes. To avoid startling the client who has lost vision in half of their visual field and so the client can better view the food, the nurse should approach the client from the right side. Neglect of the left side can occur because the left side is out of the visual field. Encourage the client to intentionally look at the left side of the body to avoid neglect. 1. Incorrect: Approaching the client from the left side is inappropriate. The client cannot see in the left half of the visual field and could be startled. 3. Incorrect: An ophthalmologist cannot fix this problem. The problem is due to damage in the central nervous system, not in the eyes. 4. Incorrect: Standing in front of the client does not address the client's visual field deficit. This would give the client an altered view of the nurse since the left half of the visual field is affected.)

A client with a deep partial-thickness burn to the right forearm has returned from surgery with a skin graft to the burned area. Which graft site intervention would the nurse implement within the first 24 hours? 1. Monitor temperature every 12 hours. 2. Position arm to prevent pressure to the graft site. 3. Prepare to change the 1st dressing within 24 hours. 4. Perform passive range of motion exercises to the right arm.

2. Position arm to prevent pressure to the graft site. (2. Correct: The arm should be situated so there is no compression on the graft site. Applying pressure to the graft may cause the graft to move which may result in damage to the graft site. 1. Incorrect: The temperature should be monitored at least every 8 hours or less. An elevated temperature may indicate an infection under the graft or at another area. If the temperature is elevated, the source of the infection should be addressed. 3. Incorrect: The first dressing change is planned 2 to 5 days after surgery. The dressing is not changed in the first 24 hours to prevent any disturbance to the skin graft. 4. Incorrect: The burn is located on the forearm. Passive range of motion exercises are implemented to improve joint function. The client is not having difficulties with the wrist, elbow, or shoulder joints. The movement to the arm should be done cautiously to prevent any damage to the graft.)

After applying oxygen using bi-nasal prongs to a client who is having chest pain, the nurse should implement which intervention? 1. Have the client take slow deep breaths in through the mouth and out through the nose. 2. Post signs on the client's door and in the client's room indicating that oxygen is in use . 3. Apply Vaseline petroleum to both nares and 2 x 2 gauze around the oxygen tubing at the client's ears. 4. Encourage the client to hyperextend the neck, take a few deep breaths and cough.

2. Post signs on the client's door and in the client's room indicating that oxygen is in use . (2. Correct: This is an oxygen therapy safety precaution that the nurse should implement after applying oxygen. It is also the only correct and safe option in the question. 1. Incorrect: The bi-nasal prongs would mean that the oxygen is going in through the nose. Breathing deeply through the mouth and out through the nose would not increase oxygenation for a client having chest pain and would disrupt the flow of oxygen through the nose. 3. Incorrect: The nurse should avoid using petroleum products where oxygen is in use because they are flammable. 4. Incorrect: These client actions have nothing to do with oxygen administration and would cause more distress to the client with chest pain.)

What is the primary electrolyte that the nurse should be aware to monitor for in a client who is receiving an insulin infusion? 1. Sodium 2. Potassium 3. Calcium 4. Phosphorus

2. Potassium (2. Correct: Insulin causes movement of potassium into the cells, which can lead to a severe reduction in serum potassium if not regulated appropriately. A severe decrease in serum potassium could be fatal. 1. Incorrect: Although insulin has been shown to increase sodium reabsorption in the kidneys, the change is not as rapid and not as life threatening as the change in potassium can be. 3. Incorrect: A significant change in the calcium level is not anticipated with the insulin infusion. 4. Incorrect: A significant change in the phosphorous level is not anticipated with the insulin infusion.)

The nurse is preparing to administer a dose of potassium iodide 300 mg by mouth to a client diagnosed with hyperthyroidism. The nurse has not administered this medication before and is using a drug reference to review information about the medication. Which client and drug reference information supports the nurse's decision to hold the potassium iodide dose and notify the primary healthcare provider? Client Information: Medical diagnosis: Hyperthyroidism Current vital signs: BP 142/88, HR 102, R 20 Medical history: Hypertension Physical examination: Alert/oriented. PERRLA. Skin warm/dry. Lungs sounds clear bilaterally. Normal S1/S2 without murmurs, clicks, rubs. Lab test results: Glucose- 98 mg/dl (5.4 mmol/L), Sodium- 139 mEq/L (139 mmol/L), Potassium- 5.5 mEq/L (5.5 mmol/L), Creatinine - 0.9 mg/dL (79.5 µmol/L), Creatinine Clearance 110 mL/min Current medications: Losartan 50 mg one by mouth daily, Methimazole 10 mg by mouth daily Drug Reference: Medication: Potassium iodide Classification: Antithyroid agent Indications: Adjunct with other antithryoid drugs in preparation for thyroidectomy. Treatment in thyrotoxic crisis. Radiation protectant following radiation emergencies or administration of radioactive iodine. Contraindications/Precautions: Hypersensitivity; hyperkalemia; pulmonary edema; impaired renal function. Use cautiously in tuberculosis; bronchitis; cardiovascular disease. Adverse reactions/Side effects: Confusion, weakness, GI BLEEDING, diarrhea, nausea, vomiting, hyperkalemia, tingling, joint pain. Interactions: Use with lithium may cause increased hypothyroidism. Increases the antithyroid effects of methimazole and propylthiouracil. Increased hyperkalemia may result from combined use with potassium-sparing diuretics, Ace inhibitors, angiotensin II receptor antagonists or potassium supplements. Route/Dose: 300-500 mg three times a day by mouth Select all that apply 1. Creatinine - 0.9 mg/dL (79.5 µmol/L) 2. Potassium- 5.5 mEq/L (5.5 mmol/L) 3. Glucose- 98 mg/dl (5.4 mmol/L) 4. Taking losartan 50 mg one by mouth daily. 5. Currently taking methimazole 10 mg by mouth daily. 6. Creatinine Clearance 110 mL/min

2. Potassium- 5.5 mEq/L (5.5 mmol/L) 4. Taking losartan 50 mg one by mouth daily. 5. Currently taking methimazole 10 mg by mouth daily. (2., 4., & 5. Correct: The medication is potassium iodide, which can lead to hyperkalemia when administered, so it is contraindicated if the client already has hyperkalemia. This client's potassium level is 5.5 mEq/L (5.5 mmol/L), which would support the nurse holding the medication and contacting the primary healthcare provider. Additionally, the drug guides states that potassium iodide increases the antithyroid effect of methimazole and propylthiouracil. Increased hyperkalemia may result from combined use with potassium-sparing diuretics, Ace inhibitors, angiotensin II receptor antagonists or potassium supplements. This client is currently on both losartan, an ARB, and methimazole. 1. Incorrect: This is a normal creatinine level. Normal range is 0.8 - 1.4 mg/dL (70-124 µmol/L) in males and 0.56-1.0 mg/dL (50-88 µmol/L) in females. 3. Incorrect: Potassium iodide does not affect glucose and this is a normal glucose level. 6. Incorrect: The normal creatinine clearance is 75-125 mL/min. Therefore, 110 mL/min is within normal limits and would not require withholding the potassium iodide.)

The nurse is making an initial homecare visit to a client following a stroke. The client has right arm weakness and a limp in the right leg. While evaluating the client's ability to prepare food, the nurse is most concerned about what actions? Select all that apply 1. Uses skid-proof shoes when walking in kitchen. 2. Pours boiling water from pan into cup of tea. 3. Heats food in microwave instead of the oven. 4. Uses electric chopper to dice up vegetables. 5. Prepares and cooks large casserole in oven.

2. Pours boiling water from pan into cup of tea. 3. Heats food in microwave instead of the oven. 4. Uses electric chopper to dice up vegetables. 5. Prepares and cooks large casserole in oven. (2., and 5. Correct: The nurse is evaluating client safety at home following a stroke, observing for any actions that might be unsafe. With right arm weakness and instability of the right leg, pouring boiling water directly from a pan into a cup is dangerous. Pouring liquid from an open pan could easily spill or splash hot water on the client. Additionally, placing a large, filled casserole into a hot oven could throw the client off balance, again leading to burns. 1. Incorrect: The nurse would not be concerned about the client using skid-proof shoes, particularly in a kitchen where there could be food or liquids on the floor. This is safe action when cooking food. 3. Incorrect: Because of physical disabilities, it is safer for the client to use a microwave than the oven. The microwave has a small opening and no overwhelming heat when the door is opened. The smaller plates or bowls used are easier and safer for the client to handle. 4. Incorrect: An electric chopper is far safer than trying to manage a knife with a weakened hand. Placing vegetables into the chopper and closing the lid prevents the client from being exposed to an open knife blade.)

A client with a history of myasthenia gravis (MG) has been discharged from the hospital following a thymectomy. When teaching the client how to prevent complications, the home care nurse emphasizes what daily actions are most important? Select all that apply 1. Include daily weight lifting exercises. 2. Practice stress reduction techniques. 3. Complete chores early in the day. 4. Take medications on time and prior to meals. 5. Eat three large meals daily.

2. Practice stress reduction techniques. 3. Complete chores early in the day. 4. Take medications on time and prior to meals. (2., 3. & 4. Correct: Myasthenia gravis is a chronic autoimmune disorder characterized by progressive muscle weakening and chronic fatigue. Clients become weaker throughout the day, contributing to the potential for complications. Stress reduction techniques are important since stress can contribute to a myasthenic crisis, a severe respiratory emergency. Daily tasks, including ADL's, should be completed early in the day when the client has the most energy. Medications for MG, including neostigmine and pyridostigmine, must be taken on time and prior to meals. 1. Incorrect: Clients with myasthenia gravis are instructed to include gentle daily exercise combined with periods of rest throughout the day. Weight lifting would be too strenuous and would quickly tire this client, possibly leading to a myasthenia crisis. 5. Incorrect: Because of the difficulty in chewing or swallowing, multiple small meals throughout the day are safer and more beneficial to a client with myasthenia gravis. Medications are timed in relation to meals, so consistent but smaller meals would be more beneficial for the client.)

What preferred lab value would the nurse expect to see the primary healthcare provider prescribe for a client admitted with generalized malnutrition? 1. Albumin 2. Prealbumin 3. Iron 4. Calcium

2. Prealbumin (2. Correct: The preferred lab value to screen for generalized malnutrition is prealbumin. This assessment is preferred because it decreases more quickly when nutrition is not adequate. 1. Incorrect: Albumin is a major serum protein that is below normal in clients who have inadequate nutrition. However, it can take weeks to drop. 3. Incorrect: Low serum iron and anemia indicate an iron deficiency. Again, the prealbumin will decrease sooner than other lab values that assess nutrition level. 4. Incorrect: Older women may have low calcium levels which place them at risk for bone demineralization. But, prealbumin provides more data on generalized nutrition.)

A client with asthma has been admitted to the emergency room with sustained burns to the upper torso, face, and neck as a result of a steam injury when a pressure cooker exploded at home. Which intervention is the nurse's priority? 1. Initiate high flow oxygen. 2. Prepare for endotracheal intubation. 3. Administer 1000 mL of lactate ringers (LR). 4. Assess for head and neck injuries.

2. Prepare for endotracheal intubation. (2. Correct: Intubation must be accomplished quickly while a tube can still be inserted. The burn clients neck and facial area may become edematous due to capillary permeability. This can be done while assuming the client may have a head and neck injury due to the explosion. 1. Incorrect: The clients airway is the initial concern. The ventilation of the client is the next step. Respiratory distress is an increase potenial due to pre existing diagnosis of asthma. 3. Incorrect: Fluid resuscitation is needed, however, airway comes first. The client will need IV fluids to replace the transfer of plasma to interstitual tissue. 4. Incorrect: Inserting a endotracheal tube to maintain an airway is the nurse's priority intervention. The client should then be evaluated for any head and neck injuries.)

Which prescription by the emergency room primary healthcare provider for a client who fell from a ladder should the nurse question? 1. Record intake and output hourly. 2. Prepare the client for lumbar puncture. 3. Perform neurologic checks every 10 minutes. 4. Schedule a brain computed tomography (CT) scan.

2. Prepare the client for lumbar puncture. (2. Correct: The traumatic injury to the brain from the fall may result in increased intracranial pressure. The reduction of pressure in the lumbar spine during a lumbar puncture may result in the potential for herniation of the brain. A lumbar puncture should not be performed. 1. Incorrect: Brain damage can result in metabolic and hormonal dysfunctions. Brain injuries may result in disorders of sodium regulation and endocrine function. Strict intake and output are important to monitor for any fluid changes. 3. Incorrect: The client should be assessed frequently to continue to evaluate their neurological status. The Glasgow Coma Scale (GCS), corneal and gag reflexes, vital signs should be assessed for any variations. 4 Incorrect: The client experienced a blunt trauma to the head after falling off the ladder. A computed tomography of the brain is prescribed to evaluate hemorrhage and trauma to the brain.)

What should the nurse who is educating about the most common initial visual changes associated with glaucoma inform the client? 1. Central vision is lost. 2. Progressive tunnel vision occurs. 3. Sudden flashes of light in the eyes. 4. Eye floaters begin to be noticed.

2. Progressive tunnel vision occurs. (2. Correct: If glaucoma is not diagnosed and treated early, an individual starts to lose peripheral vision, or the area of vision outside the central field of sight. People who have glaucoma experience tunnel vision and cannot see objects to the side, near the head, or by their feet. Central vision can be lost later if the disease progresses. 1. Incorrect: Central vision loss is the classic visual disturbance for macular degeneration but peripheral vision is usually maintained. 3. Incorrect: Individuals experiencing retinal detachment may have sudden flashes of light in the affected eye, but this is not an initial visual change related to glaucoma. 4. Incorrect: Eye floaters are more common in eye disorders such as retinal detachment or may occur associated with the aging process.)

Following escharotomy of a circumferential burn to the arm, which assessment is the best indicator when evaluating the effectiveness of this procedure? 1. Decreased pain in the extremity 2. Prompt capillary refill < 2 seconds after blanching 3. Bleeding at the site of the incision 4. Ability of the client to wiggle his/her fingers

2. Prompt capillary refill < 2 seconds after blanching (2. Correct: The objective of creating an incision through the eschar is to relieve the pressure and restore circulation. If nail beds blanch and refill promptly, blood is flowing into the limb. 1. Incorrect: Decreased pain is not an indicator of circulation to a limb. It's a good thing, but is not a definitive evaluation of circulation. 3. Incorrect: Bleeding would indicate that circulation was improved in the incision area, but to assure improved circulation of the total arm, capillary refill is the best assessment. 4. Incorrect: Movement (motor) is a neurological check, and the right answer will involve a circulatory (vascular) check.)

A client diagnosed with advanced cirrhosis is admitted with dehydration and elevated ammonia levels. While discussing dietary issues, the client requests larger portions of meat with meals. Which response by the nurse provides the most accurate information to the client? 1. I will ask the dietician to add more meat with dinner. 2. Protein must be limited because of elevated ammonia levels. 3. You need to drink more fluids because of your dehydration. 4. We can ask for between meal snacks with more carbohydrates.

2. Protein must be limited because of elevated ammonia levels. (2. Correct: Normally, protein is broken down into ammonia, which the liver converts into urea, and the kidneys then easily excrete. However, in a diseased liver, this conversion is not possible, and ammonia continues to build up in the body, ultimately affecting the brain. The nurse would be aware that additional protein would be harmful for this client. 1. Incorrect: Increasing meat at mealtimes would be detrimental to the client's health. When protein is taken into the body, a healthy liver will convert this into urea that is then excreted by the kidneys. However, this client's impaired liver is not able to make that conversion; therefore, the ammonia levels would continue to increase. The nurse can discuss with the client other foods that might safely be added to meals. 3. Incorrect: While it is true this client is dehydrated, the issue is that the client wants to increase the amount of meat at mealtimes. This response does not address the client's request nor does it provide any teaching that would help the client once discharged. 4. Incorrect: Although this response indicates that the nurse is focusing on the client's issue with food, this reply does not address the request for more meat with meals. This would be the appropriate opportunity to educate the client on the need to limit daily protein in the diet.)

What interventions should the nurse include when teaching a client how to prevent and treat fungal infections of the feet? Select all that apply 1. Apply cornstarch to the feet after bathing. 2. Put terbinafine hydrochloride cream 1% on affected areas twice a day for two weeks. 3. Wear socks at all times until infection has cleared up. 4. Wash feet daily with soap and water. 5. Wear shower sandals when showering in public places. 6. Wear shoes that allow the feet to breathe.

2. Put terbinafine hydrochloride cream 1% on affected areas twice a day for two weeks. 4. Wash feet daily with soap and water. 5. Wear shower sandals when showering in public places. 6. Wear shoes that allow the feet to breathe. (2., 4. 5, & 6. Correct: Athlete's foot is treated with topical antifungal in most cases. Severe cases may require oral drugs. The feet must be washed daily with soap and water and dried thoroughly since the fungus thrives in moist environments.Steps to prevent athlete's foot include wearing shower sandals in public showering areas and wearing shoes that allow the feet to breathe. 1. Incorrect: Clients with fungal skin infections should avoid the use of cornstarch. The carbohydrates in cornstarch may provide nutrition to fungal infections and should be avoided. 3. Incorrect: Allow feet to have exposure to the air. The feet must be kept clean and dry since fungus thrives in moist environments. Keeping the feet covered all the time causes a dark, moist environment for the fungus to thrive.)

The nurse is caring for a client following gastric bypass surgery. The client reports dizziness, sweating and palpitations after eating meals. The nurse would recommend which actions to alleviate these symptoms? Select all that apply 1. Increase liquids with food. 2. Reduce intake of carbohydrates. 3. Eat small, frequent meals daily. 4. Sit semi-recumbent for meals. 5. Remain upright for one hour after eating.

2. Reduce intake of carbohydrates. 3. Eat small, frequent meals daily. 4. Sit semi-recumbent for meals. (2, 3, & 4. Correct: The symptoms described indicate the client is experiencing dumping syndrome, an adverse response following gastric or bariatric surgery. Clients may also experience tachycardia, nausea or cramping with the intake of food due to surgical restructuring of the gastrointestinal tract. Because this will be a lifetime issue, the nurse must teach the client to adjust eating habits and patterns. Reduction of carbohydrates will help decrease the problem since carbohydrates speed through the digestive track too quickly. Eating smaller, more frequent meals in a semi-recumbent position will further slow food through the digestive tract and eliminate most of the uncomfortable symptoms. 1. Incorrect: Increasing liquids while eating will speed food processing and increase the side effects. Clients are instructed to eliminate all fluids during meals. In some cases, clients may also need to eliminate fluids for one hour before and immediately after meals in order to control symptoms and slow the progress of food through the digestive tract.)

The client is undergoing progressive ambulation on the third day after a myocardial infarction. Which clinical manifestation would indicate to the nurse that the client should not be advanced to the next level? 1. Facial flushing 2. Reports shortness of breath 3. Heart rate increase of 10 beats/min. 4. Systolic blood pressure increase of 10 mm Hg

2. Reports shortness of breath (2. Correct: The onset of shortness of breath could be an indicator that the client should not advance to the next level. The client should be instructed to stop and rest if chest pain or shortness of breath occurs. While in a rehabilitation program, it is imperative to give the client very specific guidelines for physical activity so overexertion will not occur. 1. Incorrect: Facial flushing is not life-threatening. The client can advance to the next level. 3. Incorrect: An increase in heart rate of 10 beats a minute is an expected finding with physical activity. This would not prevent the client from advancing to the next level. 4. Incorrect: An increase in systolic BP is an expected finding with physical activity.)

A nurse is caring for a client who had an abdominal hernia repair 16 hours ago. What interpretation should the nurse make based on the results of the client's arterial blood gases (ABGs)? pH - 7.32 PaO₂ - 93% PaCO₂ - 48 HCO₃⁻ - 24 1. Metabolic acidosis 2. Respiratory acidosis 3. Metabolic alkalosis 4. Respiratory alkalosis

2. Respiratory acidosis (2. Correct: The pH is 7.32 (normal 7.35-7.45) which means acidosis. The paCO₂ of 48 (normal 35-45) indicates a respiratory problem. The arterial blood gas results indicate that the client is in respiratory acidosis. 1. Incorrect: This is a respiratory problem. The bicarb is within normal limits, eliminating a metabolic problem. 3. Incorrect: The pH is low which indicates acidosis. The bicarb is within normal limits, eliminating a metabolic problem. 4. Incorrect: The pH is low, which indicates acidosis.)

The medical surgical nurse is admitting a client diagnosed with deep vein thrombosis (DVT) of the right leg. The client suddenly begins to report shortness of breath. Which additional early signs/symptoms indicative of a complication would the nurse need to report to the primary healthcare provider immediately? 1. Tachycardia with tachypnea. 2. Restlessness and dizziness. 3. Pain in the lower right leg. 4. A positive Homan's sign.

2. Restlessness and dizziness. (2. Correct: The worst complication of a DVT is the potential for a pulmonary embolism, resulting when part of the blood clot breaks free and travels to the lungs. This life-threatening complication presents with symptoms of hypoxia, including restlessness, agitation, or dizziness. The client may also develop chest pain, depending on the size of the clot. 1. Incorrect: While these symptoms may require further assessment, the question does not provide any parameters for vital signs. Individually, tachycardia and tachypnea could be attributed to pain, anxiety, or even hospitalization. There is not enough information provided to necessitate an immediate call to the Primary healthcare provider. 3. Incorrect: Pain in the affected extremity is not an unexpected finding with this diagnosis, although the nurse would need to further assess and evaluate the level and location of the pain in relation to the blood clot. This symptom is not surprising and would not require immediately alerting the primary healthcare provider. 4. Incorrect: The Homan's sign was a method formerly used to assess for the presence of a DVT and was performed by dorsiflexing the foot of the affected leg in an effort to elicit pain. However, this technique has proven to be unreliable and is no longer part of the assessment process.)

What signs or symptoms should the nurse assess for when monitoring a client who has a brain injury? Select all that apply 1. Increased pulse 2. Rhinorrhea 3. BP 150/60 4. Papilledema 5. Projectile vomiting

2. Rhinorrhea 3. BP 150/60 4. Papilledema 5. Projectile vomiting (2., 3., 4. & 5. Correct: Rhinorrhea is an important symptom that is associated with leakage of cerebrospinal fluid (CSF) The pulse pressure of 150/60 is 90 {greater than 40 is a sign of increased intracranial pressure (ICP)}. Signs of increased intracranial pressure also include papilledema, elevated systolic pressure, widened pulse pressure, decreased pulse, and slow respirations. Projectile vomiting is classically associated with increased ICP. 1. Incorrect: The major focus of a client with a brain injury is increased ICP, which is associated with bradycardia.)

The nurse has just received a client from the special procedures lab for a liver biopsy. What is the position of choice for this client post procedure? 1. Fowler's 2. Right side 3. Left side 4. Prone

2. Right side (2. Correct: How do you stop bleeding from a puncture site? With pressure, right? Yes. So where is the liver? In the right upper abdomen under the rib cage. So position the client on the right side so that pressure is applied to the liver's puncture site. Then apply pressure with a sand bag or rolled up towel. This will help to stop bleeding. 1. Incorrect: This will not help control the bleeding. Pressure needs to be applied to the liver, so we want the liver coming forward toward the abdominal wall and pressure to be applied with a sand bag or rolled up towel. 3. Incorrect: The liver is on the right, not the left. Without the liver next to the abdominal wall, pressure cannot be exerted on the liver's puncture site. 4. Incorrect: We don't turn client onto abdomen. You will not be able to assess for bleeding with the client in this position.)

A client has been trying to implement a low fat diet for prevention of heart disease and enhancement of weight loss. He further reports that his wife shows her love by preparing rich foods and pastries. Which action should the nurse make? 1. Suggest that the client prepare all meals at home. 2. Schedule a meeting with husband and wife to discuss diet and health. 3. Suggest that the client limit intake to one serving of each food at meals. 4. Ask the client to give his wife a cookbook with low fat recipes.

2. Schedule a meeting with husband and wife to discuss diet and health. (2. Correct: The meeting with the wife and husband together may help to gain the support of the wife. She may not realize that meal preparation is actually serving as a barrier to successful change. Also, the importance of the opinions and behaviors of the wife are important to the client as he tries to engage in long-term behavioral change. 1. Incorrect: This intervention may actually increase barriers to change because the wife's feeling and support are necessary to maintain long-term change. 3. Incorrect: While this practice may reduce the intake of fat, the issue of spousal support should be addressed. 4. Incorrect: Open discussion with the wife about the need for low-fat meals is essential.)

The nurse is preparing a seminar for a group of clients diagnosed with irritable bowel syndrome. Which point should the nurse include? Select all that apply 1. Teach about a low fiber diet. 2. Schedule meals at regular times. 3. Fluid should be taken with meals. 4. Become active in yoga classes. 5. Keep a food diary for 2 weeks.

2. Schedule meals at regular times. 4. Become active in yoga classes. 5. Keep a food diary for 2 weeks. (2., 4. & 5. Correct: Eating at regular intervals and chewing foods slowly and thoroughly will help to manage symptoms. Additional strategies include maintaining good dietary habits with avoidance of food triggers. Stress management via relaxation techniques, yoga, or exercise are recommended. Identify irritating foods by keeping a food diary for 1-2 weeks. 1. Incorrect: This client needs a high soluble fiber diet to help control diarrhea and constipation. Dietary fiber and bulk help the client by establishing regular bowel elimination patterns with soft, bulky stools. 3. Incorrect: Although adequate fluid intake is necessary, fluid should not be taken with meals because this results in abdominal distention.)

The nurse is caring for a client with chronic pyelonephritis. Which lab value noted by the nurse indicates a problem? 1. Estimated glomerular filtration rate - 90 mL/min/1.73 m2 2. Serum creatinine - 2.1 mg/dL (186 micro mol/dL) 3. Blood urea nitrogen - 19 mg/dl (6.78 mmol/L) 4. Urine culture isolates Escherichia coli

2. Serum creatinine - 2.1 mg/dL (186 micro mol/dL) (2. Correct: Other than the glomerular filtration rate, the creatinine level is considered the best indicator of renal function, since it is not affected by diet or fluid intake. This creatine level is high and should alert the nurse to a progressive renal problem. 1. Incorrect: The estimated glomerular filtration rate (eGFR) is the best test to measure the client's level of kidney function. This eGFR is normal and would not indicate a problem. 3. Incorrect: Blood Urea Nitrogen (BUN) is not as sensitive an indicator of renal function as creatinine because factors other than renal failure can cause an increase in urea levels. And since this value is within normal limits, it does not indicate a problem. 4. Incorrect: E. coli is the common culprit of UTIs including pyelonephritis. This would not indicate a problem, but rather validate the diagnosis.)

A client is admitted to the emergency department with digoxin toxicity. Nursing assessment reveals cool skin, a slow, weak pulse, and a BP of 86/44. What initial action should the nurse take based on the assessment and cardiac rhythm strip? Exhibit 1. Administer sodium nitroprusside 0.3 mcg/kg/min IV. 2. Set up for transcutaneous pacing. 3. Have client perform vagal maneuver. 4. Draw blood for potassium level.

2. Set up for transcutaneous pacing. (2. Correct: This client is exhibiting symptomatic bradycardia, specifically 3rd degree heart block. Transcutaneous pacing is the treatment of choice. 1. Incorrect: Sodium nitroprusside is indicated for treatment of severe hypertension. 3. Incorrect: Vagal maneuvers are done for rapid, rather than slowed heart rates. The vagal response causes the heart rate to drop which could throw this client into asystole. 4. Incorrect: The number one concern here is that the client is unstable and needs pacing to increase the heart rate. Determining the potassium level can be done later.)

A client is to be discharged following a left modified-radical mastectomy. When reviewing ADL's to be completed at home, the nurse anticipates the client will experience the most difficulty doing what tasks? Select all that apply 1. Cooking a meal. 2. Shampooing hair. 3. Doing the laundry. 4. Vacuuming carpets. 5. Changing bed linens.

2. Shampooing hair. 3. Doing the laundry. 5. Changing bed linens. (2, 3, & 5. Correct: The modified-radical mastectomy is a surgical approach to cancer in which the breast tissue, nipple, and axillary lymph nodes are removed but the chest muscles remain intact. Following surgery, individuals usually experience pain and stiffness when resuming normal daily activities, particularly tasks which require stretching the arm above the head. Shampooing or drying hair would be challenging, as would moving loads of heavy laundry between washer and dryer. Also difficult is changing bed sheets because it involves lifting and stretching across the bed. 1. Incorrect: The process of cooking food can be modified in such a way the client would not need to extend the surgical arm above the head or in a painful position. 4. Incorrect: Vacuuming carpet does not require lifting or reaching if the client uses an upright sweeper. This task should not present difficult challenges and can be completed with the non-surgical arm.)

A community health nurse is presenting a seminar to a group of senior citizens on ways to reduce the risks of peripheral artery disease (PAD). What topics should the nurse include? Select all that apply 1. Anti-embolic stockings 2. Smoking cessation 3. Moderate exercise 4. Application of heat 5. Low cholesterol diet 6. Decrease blood pressure

2. Smoking cessation 3. Moderate exercise 5. Low cholesterol diet 6. Decrease blood pressure (2., 3., 5. & 6. Correct: Senior clients are at increased risk for peripheral artery disease for a variety of reasons, though many erroneously believe that this process is an unavoidable part of the aging process. Educating clients on preventative activities will help reduce incidence of atherosclerosis and improved mobility along with quality of life. Smoking is a major risk factor in developing PAD by contributing to arterial constriction. Clients can increase collateral circulation with a moderate exercise program of at least 30 minutes three times a week. A low cholesterol, heart healthy diet with more fruits and vegetables helps reduce cholesterol while decreasing blood pressure, both important goals towards controlling PAD. 1. Incorrect: Increasing arterial blood flow is important in the prevention or management of peripheral artery disease; however, anti-embolic stockings are designed to improve venous return in clients with decreased mobility. The use of these stockings would actually hinder arterial flow in lower extremities. 4. Incorrect: Clients with PAD often complain of cold extremities secondary to decreased arterial blood flow. But the application of heat such as use of a heating pad is unsafe and is always contraindicated in the elderly with PAD. Inability to sense temperature extremes may result in serious burns to lower extremities. Additionally, clients with PAD do not heal as well from injuries or wounds.)

After a thoracotomy, which interventions will the nurse initiate to reduce the risk of acute respiratory distress? Select all that apply 1. Allow 4 hours of rest between deep breathing and coughing exercises. 2. Splint the incision during deep breathing and coughing exercises. 3. Have the client drink a glass of water before coughing. 4. Perform percussion and vibration every 2 hours. 5. Promote incentive spirometer use several times per hour while awake.

2. Splint the incision during deep breathing and coughing exercises. 5. Promote incentive spirometer use several times per hour while awake. (2., & 5. Correct: Splinting helps with the ability to control pain and produce an effective cough. Incentive spirometry encourages deep inspiratory efforts, which are more effective in re-expanding alveoli than forceful expiratory efforts. 1. Incorrect: They need to cough more often than every 4 hours. It is the best when this is done every 2 hours. 3. Incorrect: It takes longer than a few minutes to liquefy secretions and, if the stomach is full, vomiting may occur which would put the client at risk for aspiration. 4. Incorrect: After the surgery, we do not want to percuss and vibrate the incision. Besides being extremely painful, this could potentially disrupt the suture line.)

A client, who is receiving an IV vesicant agent, reports pain at the intravenous site. What is the priority nursing action? 1. Apply a cold compress to the IV site 2. Stop the infusion 3. Check the IV for a blood return 4. Notify the primary healthcare provider

2. Stop the infusion (2. Correct: Stop the infusion to stop the vesicant from getting into the tissue and causing more extravasation. 1. Incorrect: This is a right response, but it's not what I would do first. You have to stop the infusion first. Why do we use a cold compress and not a warm compress? We don't want the vesicant to spread out through vasodilation (warm compress), we want to keep it contained, so cold compress to vasoconstrict. 3. Incorrect: You may do this but the priority with pain and swelling is to stop the infusion before more damage is done. 4. Incorrect: The healthcare provider may be notified, but first the infusion must be stopped to prevent further extravasation.)

An elderly client living in a long-term care facility fell 8 hours ago causing a laceration on the occipital area of the skull and steri-strips were applied for closure. Which signs/symptoms would indicate to the nurse that the client should be transferred to the emergency department? Select all that apply 1. Purposeful movement. 2. Sudden emotional outbursts. 3. Client report of blurred vision. 4. Pupils equal, react to light, and accommodation. 5. Bright red blood oozing from the wound. 6. Headache unrelieved by acetaminophen.

2. Sudden emotional outbursts. 3. Client report of blurred vision. 6. Headache unrelieved by acetaminophen. (2., 3., & 6. Correct: Signs/symptoms of increased ICP include: excessive sleepiness, inattention, difficulty concentrating, impaired memory, faulty judgment, depression, irritability, emotional outbursts, disturbed sleep, diminished libido, difficulty switching between two tasks, and slowed thinking. Abnormalities in vision and extraocular movements occur in the early stages of increased ICP. A headache that is unrelieved by acetaminophen would warrant further investigation. 1. Incorrect: This is a normal response and does not warrant further investigation. 4. Incorrect: This is a normal response and does not warrant further investigation. 5. Incorrect: The scalp is very vascular and oozing would be expected. Apply pressure to stop bleeding.)

A nurse is participating in a community health fair for middle aged individuals. Which points should the nurse stress for decreasing the risk of stroke? Select all that apply 1. Reduce dietary intake of unsaturated fat. 2. Swim or walk most days of the week. 3. Treat obstructive sleep apnea, if present. 4. Drink five or more 8 ounce glasses of water daily. 5. Decrease smoking to less than ½ pack a day.

2. Swim or walk most days of the week. 3. Treat obstructive sleep apnea, if present. 4. Drink five or more 8 ounce glasses of water daily. (2., 3., & 4. Correct: Aerobic or cardio exercise reduces risk of stroke in many ways. Exercise can lower blood pressure, increase HDL, lower cholesterol, and improve overall health of blood vessels and heart. Exercise helps in weight loss, controlling diabetes and reduces stress. Obstructive sleep apnea can decrease oxygen to the brain and place you at increased risk of stroke. Water helps to thin blood, which in turn makes it less likely to form clots. Drink water throughout the day, rather than all at once. 1. Incorrect: When choosing fats, pick unsaturated fat over saturated or trans fat. Saturated fat raises total blood cholesterol levels and low density lipoprotein. 5. Incorrect: Tobacco use is a major preventable risk factor for stroke and heart disease. Even if a person has smoked for years, the risk of stroke can be reduced by quitting all smoking.)

What should the nurse include when planning discharge teaching for a client post scleral buckling of the right eye? Select all that apply 1. Redness, tenderness and swelling should be gone within 2 days. 2. Teach to report seeing flashes of light immediately. 3. Place eye drops onto the cornea of the affected eye. 4. Wear eye shield during naps, and at night. 5. Have client demonstrate the correct technique for instilling eye drops.

2. Teach to report seeing flashes of light immediately. 4. Wear eye shield during naps, and at night. 5. Have client demonstrate the correct technique for instilling eye drops. (2., 4., & 5. Correct: The client should be taught the signs/symptoms of retinal detachment, such as seeing flashes of light, floating spots or blurred, "sooty" vision, or a veil-like curtain obscuring parts of the visual field. The client should wear either an eye shield or glasses during the day, during naps, and at night. The client will need to instill eye drops into the affected eye appropriately to avoid injury and infection. 1. Incorrect: The pain level should decrease within a few days, but the client will continue to have redness, tenderness, and swelling for a few weeks after surgery. 3. Incorrect: The client should tilt the head backward and slightly to the side, so the solution runs away from the tear duct and other eye to prevent contamination while depressing the lower lid with the finger of one hand. Tell the client to look up when the solution is dropped on the averted lower lid. Do not the place drop directly on the cornea.)

A primary healthcare provider documents the following Glasgow Coma Scale score in a client's medical record: "GCS 9 = E2 V4 M3 at 0720". What conclusions should a nurse draw from this documentation? Select all that apply 1. A severe head injury has been sustained. 2. The client opens the eyes in response to a pain stimulus. 3. The client's speech is incomprehensible. 4. Abnormal flexion is observed in the client. 5. The GCS assessment was performed at 7:20 am.

2. The client opens the eyes in response to a pain stimulus. 4. Abnormal flexion is observed in the client. 5. The GCS assessment was performed at 7:20 am. (2., 4., & 5. Correct: A score of "E2" indicates that the client opens the eyes only in response to a pain stimulus. A score of "M3" indicates that the client is exhibiting abnormal spastic flexion of the body (decorticate posture) and "0720" indicates that the assessment was conducted at 7:20 am. 1. Incorrect: "GCS 9" indicates that the client has sustained a moderate head injury. 3. Incorrect: "V4" indicates that the client is able to speak, but is confused and/or disoriented.)

What is indicated when caring for a client admitted with meningitis? 1. The client should be placed in a negative pressure room and health care providers should wear a N95 protective mask when in contact with the client. 2. The client's room door may remain open and health care providers should wear a facemask within 3 to 6 feet of the client. 3. The client should be placed in a private room and no face mask is needed. 4. The only precaution needed is hand hygiene.

2. The client's room door may remain open and health care providers should wear a facemask within 3 to 6 feet of the client. (1. Incorrect: Meningococcal infections are not spread by small airborne organisms. Meningococcal infection is spread by large particle droplets. 2. Correct: Meningococcal infections are transmitted by large particle droplets. Meningitis can be spread by respiratory and throat secretions or lengthy contact. 3. Incorrect: Meningococcal infections are not spread by contact with a person's skin or contaminated items. Meningococcal infection is spread by large particle droplets. 4. Incorrect: Meningococcal is not spread by contact with a person's skin or contaminated items, but by large particle droplets. Hand hygiene alone is not sufficient to prevent the spread of meningococcal infections.)

After discontinuing a peripherally inserted central line (PICC), it is most important for the nurse to record which information? 1. How the client tolerated the procedure. 2. The length and intactness of the central line catheter. 3. The amount of fluid left in the IV solution container. 4. That a dressing was applied to the insertion site.

2. The length and intactness of the central line catheter. (2. Correct: This is the most important information that needs to be documented. This information would be important in determining if a potential safety issue/complication could occur as a result of the PICC line being removed or a portion of the line breaking off before removal. 1. Incorrect: This is not the most important information that needs to be documented. There are no client safety issues with charting the client's tolerance of the procedure. 3. Incorrect: This would be charted so the intake and output could be calculated. This is not the most important data that needs to be documented related to the removal of the PICC line. 4. Incorrect: This would need to be documented because a dressing is applied to the insertion site after removal. However, this is not the most important data that would need to be documented after this procedure.

The nurse is irrigating an acid chemical burn on a client's arm. Which would indicate to the nurse that irrigation can be stopped? 1. Client's pain rating has decreased from 6 to 2 on a 0 to 10 pain scale. 2. The pH value of the runoff solution is 7.0. 3. Client reports a burning sensation in the affected arm. 4. Capillary refill is less than 2 seconds in the affected arm.

2. The pH value of the runoff solution is 7.0. (2. Correct: A pH of 7 is nonacidic, so the solution's pH indicates that the acid chemical has been removed. 1. Incorrect: Pain could indicate acid is still present. 3. Incorrect: A burning sensation may indicate acid is still present. 4. Incorrect: Capillary refill is not an indication that all acid has been removed.)

An unresponsive client begins to vomit. What intervention by the nurse would have the highest priority? 1. Suction the client's mouth. 2. Turn the client onto their side. 3. Apply oxygen per face mask. 4. Insert an oropharyngeal airway.

2. Turn the client onto their side. (2. Correct: To prevent aspiration the first thing to do is turn the client onto their side. Leaving the client in the supine position will allow vomitus to get into the lungs when the client breathes. 1. Incorrect: Suctioning can be done after you turn the client onto the side. They could aspirate while you get the suction equipment put together to suction. 3. Incorrect: Applying oxygen will not prevent aspiration. Turn onto side. 4. Incorrect: The first thing to do is to turn to the side to protect the airway. Don't waste time. The oropharyngeal airway will not prevent aspiration. It is used to maintain an open airway.)

What should a nurse include when preparing to educate a female client on how to prevent recurrent cystitis? Select all that apply 1. Drink at least eight, 4 ounce glasses of water per day. 2. Urinate as soon as the urge occurs. 3. Avoid irritating perineum with harsh soap. 4. Empty your bladder post coitus. 5. Avoid use of a diaphragm.

2. Urinate as soon as the urge occurs. 3. Avoid irritating perineum with harsh soap. 4. Empty your bladder post coitus. 5. Avoid use of a diaphragm. (2., 3., 4. & 5. Correct: The client should void as soon as the urge occurs and completely empty the bladder. The client should not use excessive soap or harsh soap to minimize irritation of the urethra. Women should void immediately after sexual intercourse. This helps prevent bacteria from moving into the urethra. Pressure on the urethra may cause irritation and urinary tract infections in women who use diaphragms. 1. Incorrect: A client with normal renal function and who does not have heart or kidney disease needs to drink 2200 to 2700 mL of fluid daily. Increasing fluid intake helps flush out solutes or particles that collect in the urinary system.)

Following nasal surgery, the nurse suspects a client has developed diabetes insipidus. The nurse knows what laboratory results provide evidence of diabetes insipidus? Select all that apply 1. White blood cells of 9,500 mm³ (9.5 x 10⁹/L) 2. Urine specific gravity of 1.004 3. Serum sodium level of 149 mEq/L (149 mmol/L) 4. Hemoglobin of 20 g/dL (200 g/L) 5. Glucose of 100 mg/dL (5.6 mmol/L)

2. Urine specific gravity of 1.004 3. Serum sodium level of 149 mEq/L (149 mmol/L) 4. Hemoglobin of 20 g/dL (200 g/L) (2, 3 and 4. Correct: Diabetes insipidus results when the body is deficient in anti-diuretic hormone (ADH), resulting in a fluid volume deficit and shock. Blood becomes concentrated and urine dilute because of extreme loss of water. Specific gravity is very low, as evidenced by a lab result of 1.004. However, serum levels of sodium (149 mEq/L (149 mmol/L)) and hemoglobin (20 g/dL (200 g/L)) are high due to concentration. 1. Incorrect: Normal white blood cell count is 5,000 to 10,000 mm³ (5-10 x 10⁹/L). This WBC result is normal and does not require action by the nurse. 5. Incorrect: Diabetes insipidus is not related to the disease diabetes mellitus. The blood glucose level in this question is normal.)

A community health nurse prepares a presentation about decreasing the risk of the spread of influenza in the community. Which information should the nurse include in the presentation? 1. The flu is spread via the influenza vaccine. 2. Use a shirtsleeve when coughing or sneezing. 3. Tissues are the most effective means to decrease the spread of the influenza. 4. Antibiotics are effective in treating influenza.

2. Use a shirtsleeve when coughing or sneezing. (2. Correct: A shirtsleeve should be used as a barrier when coughing or sneezing. This prevents germs being spread via the hands. 1. Incorrect: The flu vaccine contains a dead virus that is not capable of causing the flu. Clients may experience flu-like symptoms from the flu vaccine, but they won't contract the virus. 3. Incorrect: Tissues are effective in decreasing the spread of the flu if disposed of in the trash after use. Hand washing is also very important in decreasing the spread of germs. 4. Incorrect: Antibiotics are not effective in treating the flu. The flu is treated with antipyretics, fluids, and rest. Antibiotics are used for infections, not viruses.)

Which meal option should the client diagnosed with gout select? 1. Tuna salad on bed of lettuce, apple slices, coffee 2. Vegetable soup, whole wheat toast, skim milk 3. Roast beef with gravy sandwich, baked chips, diet coke 4. Spinach salad with chick peas and asparagus, apple, tea

2. Vegetable soup, whole wheat toast, skim milk (2. Correct: Gout is pain and inflammation that occurs when too much uric acid crystallizes and deposits in the joints. This is a good choose as it is low in purine and fat. Purines are broken down into uric acid. A diet rich in purines can raise uric acid levels. Meat and seafood increase the risk of gout. Dairy products may lower risk for gout. 1. Incorrect: The client should not eat tuna, which is high in purine. 3. Incorrect: Gravy is a high purine food and should be avoided. Also avoid artificial sweeteners. 4. Incorrect: Although spinach, and asparagus can be consumed in moderation, they still contain purines, so it is not as good of a choice as the vegetable soup, toast and skim milk.)

The nurse is cleaning and dressing a foot ulcer of a diabetic client. Which actions are appropriate? Select all that apply 1. Uses a clean basin and washcloth to clean the ulcer. 2. Wears sterile gloves to clean the ulcer. 3. Cleans ulcer with normal saline. 4. Warms saline bottle in microwave for 1 minute. 5. Cleans ulcer in a full circle, beginning in the center and working toward the outside.

2. Wears sterile gloves to clean the ulcer. 3. Cleans ulcer with normal saline. 5. Cleans ulcer in a full circle, beginning in the center and working toward the outside. (2., 3., & 5. Correct: The nurse needs to wear sterile gloves when cleaning the wound. Normal saline solution is the preferred cleansing agent because, as an isotonic solution, it doesn't interfere with the normal healing process. Gently clean the wound in a full or half circle, beginning in the center and working toward the outside. 1. Incorrect: Sterile supplies should be used with this procedure because the client is at risk for infection and gangrene. Gauze and salve should be used instead of a wash cloth. 4. Incorrect: Before you start, make sure the cleansing solution is at room temperature. Do not heat in the microwave. It could scald the client!)

The nurse is caring for a client diagnosed with Addison's disease. Which finding would indicate to the nurse that a client has received excessive mineralocorticoid replacement? Select all that apply 1. Oily skin 2. Weight gain of 4 pounds in one week 3. Loss of muscle mass in extremities 4. Blood glucose of 58 mg/dL 5. Serum potassium of 3.2 mEq

2. Weight gain of 4 pounds in one week 5. Serum potassium of 3.2 mEq (2., & 5. Correct: Remember that aldosterone is a mineralocorticoid, which causes the client to retain sodium and water. Retaining sodium and water will cause the client's weight to increase. Also remember, any sudden gain in weight is due to water retention. Too much aldosterone makes you retain too much sodium and water and lose potassium. Normal potassium is 3.5-5.0 mEq/L, so a lowering of potassium could indicate high levels of aldosterone. 1. Incorrect: Oily skin would be seen with an increase in sex hormones such as testosterone and estrogen. Oily skin is not common with mineralocorticoids like aldosterone. 3. Incorrect: Too many glucocorticoids will cause the breakdown of protein and fat but muscular weakness and increased fatigue is seen with too little mineralocorticoids. 4. Incorrect: Too many glucocorticoids will inhibit insulin, causing the serum blood glucose level to go up. Normal blood glucose is 70-110.)

A female client receiving chemotherapy for breast cancer reports vomiting, stomatitis, and a 10 pound weight loss over the past month. The primary healthcare provider orders an antiemetic and daily mouthwashes. When the home care nurse evaluates the client one week later, what change described by the client would best indicate improvement? 1. Eating three meals daily. 2. Weight gain of two pounds. 3. No further mouth pain. 4. Improved skin turgor.

2. Weight gain of two pounds. (2. Correct: Chemotherapy typically causes gastrointestinal disturbances severe enough to interfere with a client's ability to eat or absorb nutrients. A ten pound weight loss over one month is significant but expected because of the reported vomiting and stomatitis. A weight gain of two pounds in a week would be the best specific indicator of improvement. 1. Incorrect: The ability to eat three meals daily does not mean that the client is actually absorbing those nutrients successfully. This option suggests that the antiemetic is working well, but there is not enough evidence to demonstrate significant client improvement. 3. Incorrect: The client's denial of any further mouth pain signifies that the mouthwashes have decreased mouth inflammation and stomatitis. While this is a positive change in the client's condition, it is not the best evidence noted by the nurse. 4. Incorrect: Skin turgor specifies the hydration status of a client. Since this client had previously been vomiting, improved skin turgor would indicate the antiemetic is working well and the client is able to retain fluids. While this is a positive change, it is not the most significant indicator of client improvement.)

What information would the nurse include when participating in community health training about sexually transmitted infections? 1. Clients are screened for chlamydial infection and/or gonorrhea only if the client is experiencing cervical discharge, dyspareunia, and dysuria. 2. Women with chlamydial infection or gonorrhea are likely to be asymptomatic. 3. In many instances, chlamydia infection and/or gonorrhea will go away without intervention. 4. It is only necessary for females to be treated for chlamydial infections and/or gonorrhea due to the potential damage to a female's reproductive system.

2. Women with chlamydial infection or gonorrhea are likely to be asymptomatic. (2. Correct: Chlamydial infections of the cervix and gonorrhea often produce no symptoms. 1. Incorrect: All sexually active women aged 25 and younger should be screened annually​. 3. Incorrect: If left untreated, chlamydial infections and gonorrhea can result in serious complications​. 4. Incorrect: Partners do need to be treated​.)

A 35 year old client, concerned about weight, asks a clinic nurse, "What is my BMI?" The client weighs 135 pounds and is 5 feet 2 inches tall. Determine the client's BMI to the nearest tenth? Ans:______

24.7 (Standard formula: BMI = (703 × lb) ÷ in² BMI = (703 × 135) ÷ (62)² BMI = (94,905) ÷ (3,844) BMI ≈ 24.7 Metric formula: BMI = kg ÷ m² BMI = (135 ÷ 2.2) ÷ (((5 × 12) + 2) × 2.54 ÷ 100)² BMI = (135 ÷ 2.2) ÷ ((62 × 2.54) ÷ 100)² BMI = (135 ÷ 2.2) ÷ (157.48 ÷ 100)² BMI = (135 ÷ 2.2) ÷ 1.5748² BMI ≈ 24.7 Ans: 24.7)

A client diagnosed with human immunodeficiency virus (HIV) is to be sent home today. The nurse has initiated discharge instructions on the proper handling of blood and body fluid at home. The nurse knows the teaching is successful when the client makes what statement? 1. "As long as it's my home, I can use normal cleaning methods." 2. "I must scrub with hot, soapy water and allow it to air dry." 3. "I should clean area with a 10% mixture of bleach and water." 4. "I must sterilize with isopropyl alcohol and rinse with ammonia."

3. "I should clean area with a 10% mixture of bleach and water." (3. Correct: The proper method to clean spills of blood or body fluids at home is to use a 10% solution of household bleach, which means 9 parts of water to 1 part bleach. It is recommended to leave the bleach solution on the contaminated area for 10 to 20 minutes, and then rinse with hot water. Any towels or cloths used to clean the area should be double bagged and discarded. 1. Incorrect: This demonstrates a false sense of security. Even in a home environment, visitors and family could become contaminated with the HIV virus. Microscopic amounts of blood or body fluids could contaminate others, and therefore proper cleaning methods must be followed even at home. 2. Incorrect: Hot, soapy water will not kill the HIV virus on hard surfaces, regardless of the type of soap or the temperature of the water. Additionally, air drying will not decrease the virulence of the virus. 4. Incorrect: Isopropyl alcohol, or rubbing alcohol, does not inactivate the HIV virus, even if rinsed with ammonia. Also, mixing household chemical cleaners, such as bleach with ammonia, can create dangerous fumes that are toxic to humans.)

A nurse has been educating a client newly diagnosed with diabetes, about proper foot care. The nurse knows teaching will need to be reinforced again when the client makes what statement? Select all that apply 1. "I should cut my toenails with nail clippers." 2. "Drying both feet thoroughly is important." 3. "I should never use nail polish on my toes." 4. "Weekly foot inspection must include the soles of the feet." 5. "I need larger shoes that don't pinch my toes."

3. "I should never use nail polish on my toes." 4. "Weekly foot inspection must include the soles of the feet." 5. "I need larger shoes that don't pinch my toes." (3, 4 and 5 Correct: The nurse is evaluating the client for an understanding of proper diabetic foot care; therefore, an incorrect statement would require further instruction. There is no reason a client with diabetes could not use nail polish on toenails. Inspection of both feet, including the soles of the feet, must be done daily and not weekly. Most importantly, properly fitted shoes are crucial to prevent complications that might result in a blister or eventually an amputation. 1. Incorrect: Diabetic clients are advised to use small clippers rather than scissors which could result in an injury from cutting too deep. It is easier to cut toenails straight across, as prescribed, with clippers. This comment indicates the client understood the teaching. 2. Incorrect: It is crucial to thoroughly dry feet following a shower or if feet get wet, since moist skin can breakdown easily. Although diabetics have very dry skin, no lotion should be applied between toes. Absorbent, white cotton socks are best for diabetic clients, and may need to be changed more than once daily. This response indicates the client understands the information from the nurse.)

The nurse is teaching a client diagnosed with salmonellosis about how to decrease the transmission to others. Which statement by the nurse would require follow up? 1. "I will wash my hands after feeding pets." 2. "I will use a meat thermometer to cook food to safe temperature." 3. "I will clean my hands with water before handling food." 4. "I will use disposable dishes until infection free."

3. "I will clean my hands with water before handling food." (3. Correct. Salmonellosis is caused by the bacteria salmonella. Hands should be washed with soap and warm water. Only washing with water is not correct and requires the nurse to do further teaching with the client. 1. Incorrect: This statement indicates the patient understands teaching. The client should wash hands after contact with animals, their food or treats, and their living environment. 2. Incorrect: A meat thermometer should be used to ensure foods are cooked properly. Undercooked meat and unpasteurized milk is a source for the organism. 4. Incorrect: Disposable dishes help prevent the spread of infection. It prevents the organism from being transferred on dirty dishes.)

Which statement by a client diagnosed with infectious mononucleosis indicates to the nurse that education has been successful? 1. "I should let my primary healthcare provider know if I start having pain in the side of my stomach" 2. "I can return to my normal activities in 5 days." 3. "I will not let others drink from my glass." 4. "My immediate family needs to get vaccinated against mononucleosis."

3. "I will not let others drink from my glass." (3. Correct: Infectious mononucleosis, caused by the Epstein-Barr virus is transmitted by saliva and intimate physical contact like kissing, sharing of utensils, and eating/drinking after others. 1. Incorrect: The client should observe for left upper quadrant abdominal pain radiating to the left scapula as this is an indicator of splenic rupture, a complication of infectious mononucleosis. 2. Incorrect: This is too soon. Most people get better in 2 to 4 weeks; however, some people may feel fatigued for several more weeks. Occasionally, the symptoms of infectious mononucleosis can last for 6 months or longer. 4. Incorrect: There is no vaccine to protect against infectious mononucleosis. The best way of the Epstein-Barr virus is to eliminate contact with oral secretions.)

A nurse is caring for a client who is diagnosed with diabetic ketoacidosis (DKA). Which primary healthcare provider prescription is appropriate during the first 24 hours of treatment for this client? 1. 0.45% saline solution (NaCl) at 50 mL/hr 2. 3% saline solution (NaCl) at 125 mL/hr 3. 0.9% saline solution (NaCl) at 1,000 mL/hr times 2 4. Dextrose 5% in lactated Ringer's solution at 150 mL/hr

3. 0.9% saline solution (NaCl) at 1,000 mL/hr times 2 (3. Correct: 0.9% saline solution should be infused at a rate of at 1,000 mL/hr times 2, up to a total of 10 L in the first 24 hours of treatment. This client will be in a fluid volume deficit or shock and fluid replacement is essential. 1. Incorrect. 0.45% saline would not be administered at 50 mL/hr because the first goal of fluid therapy in DKA is to restore volume in a severely volume depleted client, thus we need normal saline at a faster rate to replace fluid volume. 2. Incorrect. 3% saline solution would be contraindicated in this client because it is a hypertonic solution that would worsen the client's dehydration. 4. Incorrect. Dextrose 5% in lactated Ringer's solution at 150 mL/hr is inappropriate because the blood sugar is too high for infusion of a dextrose containing solution.)

The nurse is caring for a client admitted with heart failure associated with an acute MI. At which time point did the nurse begin to intervene incorrectly? Perscriptions: IV D5W at 20 mL/hr, Nitroglycerin 25 mg in 250 mL D5W IV administered per protocol. Nitroglycerin Protocol: 1. Dosing a. Nitroglycerin may be mixed in D5W or NS. b. If titrating nitroglycerin for pulmonary edema or heart failure, titrate until systolic BP is 120 mmHg or below. 2. Client monitoring a. If pain resolves completely, maintain infusion at current rate of administration. b. If pain continues, increase the infusion rate by 5 mcg/min every 5 minutes until pain resolves or systolic BP falls below 100 mmHg. c. Maximum dose is 200 mcg/min. d. If systolic BP falls below 90 mmHg during titration, decrease the infusion rate by 10 mcg/min and give a 250 ml NS bolus IV. If BP remains below 90 mmHg, discontinue the infusion. Nurses Notes: 1110: IV D5W started with 20-gauge catheter to left hand at 20 ml/hr via infusion pump. Client reports chest pain at 8/10. BP - 180/102, HR - 108, RR - 30. 1115: Nitroglycerin 25 mg added to glass bottle of D5W 250 mL and connected to lowest left hand IV site with primary tubing and attached to infusion pump at 3 mL/hr. Client reports chest pain at 9/10. BP - 182/100, HR - 110, RR - 28. 1120: Client reports chest pain at 8/10. BP - 168/90, HR - 108, RR - 26. Nitroglycerin infusion increased to 6 mL/hr via infusion pump. 1125: Client reports chest pain at 7/10. BP - 154/94, HR - 100, RR - 24. Nitroglycerin infusion increased to 12 mL/hr via infusion pump. 1130: Client reports chest pain at 2/10. BP - 130/80, HR - 86, RR - 24. Nitroglycerin infusion increased to 15 mL/hr via infusion pump. Flow Rates: 5 mcg/min = 3 mL/hr 10 mcg/min = 6 mL/hr 15 mcg/min = 9 mL/hr 20 mcg/min = 12 mL/hr 25 mcg/min = 15 mL/hr 30 mcg/min = 18 mL/hr 1. 1115 2. 1120 3. 1125 4. 1130

3. 1125 (3. Correct: At 1125, the nurse failed to follow protocol for nitroglycerin infusion. The nurse increased the IV rate by 6 mL/hr (going from 10-20 mcg/min). 1. Incorrect: The nurse mixed the nitroglycerin appropriately and connected the tubing at the correct IV site. The infusion rate was started at 3 mL/hr which delivered the appropriate starting dose at 5 mcg/min. 2. Incorrect: At 1120 the client is still hurting and the BP is above 120 systolic, so the nitroglycerin infusion can be increased by 5 mcg/min which would increase the rate to 6 mL/hr. 4. Incorrect: At 1130 the client is still hurting and the BP is above 120 systolic, so the nitroglycerin infusion can be increased by 5 mcg/min which would increase the rate to 15 mL/hr.)

The nurse is preparing to discharge four clients from the unit. Which client is most likely to warrant a referral to other agencies or community outreach programs? 1. 45 year-old client who had nasal surgery. 2. 50 year-old client postop mastectomy. 3. 72 year-old client with diabetes and obesity. 4. 80 year-old client with a diagnosis of delirium caused by dehydration.

3. 72 year-old client with diabetes and obesity. (3. Correct: The elderly client with diabetes and obesity is likely to need referrals at the time of discharge, whether to physical therapy, home health, weight loss program or other agencies. 1. Incorrect: There is no need to think that this client who is 45 years old will need assistance after nasal surgery. 2. Incorrect: This 50 year-old client postop mastectomy is mobile and does not need referral to other agencies or community outreach programs. 4. Incorrect: Delirium is an acute illness that resolves once the physical stressor is eliminated. In this case the delirium was caused by dehydration that has been corrected if the client is ready for discharge. There is no indication that the client is in need of post-hospital care.)

The nursing supervisor is reviewing several instances in which restraints have been used. The nurse is aware the only acceptable use of restraints is what? 1. An elderly male had a chest restraint applied after crawling over bed rails several times. 2. An Alzheimer client's room door is closed to prevent wandering during shift change. 3. A confused client with a closed head injury had hand mitts applied after pulling out IV 4. A dementia client with sundowners is placed in Geri-chair with lap belt at nurse's station.

3. A confused client with a closed head injury had hand mitts applied after pulling out IV (3. Correct: Restraints are considered a last resort when caring for a client, whether soft cloth or chemical restraints. The most acceptable use is to prevent a client from harming self or others. In this instance, a confused client has previously pulled out a prescribed IV. Therefore, the use of hand mitts is the most appropriate, least-restrictive method to prevent the client from further self-harm. 1. Incorrect: There are several problems here. The client had side rails up, which are considered a form of restraint and in many facilities are no longer permitted. By applying a chest restraint, the client has been restrained twice. Just because a client is elderly does not mean restraints are needed. This restraint is not acceptable. The nurse should provide regular toileting periods and determine why this client is climbing out of bed. 2. Incorrect: Closing a client into a room is overly restrictive and unsafe. This Alzheimer's client needs to be observed and closing the room door prevents visual access. Additionally, closing the door may violate fire safety codes in certain facilities. At shift change, when staff is occupied with report, special arrangements should be made so that the client can be observed and not restrained. 4. Incorrect: Depending on the facility, placing a client upright at night, using a Geri-chair and a lap belt is overly restrictive. A client with dementia is challenging, particularly in the presence of sundowner syndrome. However, keeping a client upright all night, belted into a chair for the purpose of observation, is neither safe nor healthy for the client.)

Which client diagnosis would a prescription for an intravenous infusion of 1000 mL normal saline with 20 mEq (20 mmol) potassium chloride be appropriate? Select all that apply 1. Major burn injury 2. Kidney disease 3. Abdominal cramping with diarrhea 4. Diabetic Ketoacidosis (DKA) 5. Hypokalemia

3. Abdominal cramping with diarrhea 4. Diabetic Ketoacidosis (DKA) 5. Hypokalemia (3., 4. & 5. Correct: Clients with abdominal cramping with diarrhea, diabetic ketoacidosis, and hypokalemia are safe to receive normal saline with potassium chloride. A primary electrolyte found in the lower GI tract is potassium. Therefore, diarrhea can result in excessive losses of potassium and associated hypokalemia can occur. When insulin is given to the client in DKA, it causes a transport of both glucose and potassium out of the blood and into the cell, resulting in hypokalemia. Finally, a client who has hypokalemia from other causes would need potassium replacement as well. 1. Incorrect: Tissue destruction from a major burn will cause release of potassium from the cell and into the blood. Thus, hyperkalemia occurs. An IV infusion with potassium will make the problem worse. 2. Incorrect: With kidney disease and the resulting diminished renal function, the client is at risk for sodium and potassium retention.)

The nurse is providing care to a client who had an endoscopic retrograde cholangiopancreatogram (ERCP) two hours ago. Which finding would indicate a possible complication? 1. Occasional cough 2. Sore throat reported 3. Abdominal pain rated 8/10 4. Drowsy

3. Abdominal pain rated 8/10 (3. Correct: The client should not have severe pain after the ERCP. Severe pain along with a distended abdomen is suspicious and should be of concern to the nurse. 1. Incorrect: A continuous cough would be indicative of a problem, such as perforation of the esophagus. An occasional cough is not of concern. 2. Incorrect: It is not abnormal for the client to experience a sore throat for 1-2 days post procedure as the instrument is passed through the mouth and esophagus to view the bile and pancreatic ducts. 4. Incorrect: It is normal for the client to be drowsy for several hours after the procedure as conscious sedation is used during the ERCP procedure.)

A client is admitted with an acute episode of diverticulitis. What symptom would the nurse promptly report to the primary healthcare provider? 1. Midabdominal pain radiating to the shoulder 2. Nausea and vomiting periodically for several hours 3. Abdominal rigidity with pain in the left lower quadrant 4. Elimination pattern of constipation alternating with diarrhea

3. Abdominal rigidity with pain in the left lower quadrant (3. Correct: Pain in the lower left quadrant with abdominal rigidity indicates the client is experiencing a perforated diverticuli and is a medical emergency. Abdominal rigidity indicates either perforation or internal bleeding. Both of these symptoms are considered an "acute abdomen" and are emergencies. 1. Incorrect: Midabdominal pain radiating to the shoulder is a common s/s for a client with cholecystitis but is not a medical emergency. 2. Incorrect: Nausea and vomiting periodically for several hours is often seen with diverticulitis but is not a medical emergency. 4. Incorrect: Elimination pattern of constipation alternating with diarrhea indicates a partial bowel obstruction and may require further investigation, but this is not a medical emergency.)

A small community has experienced a mudslide that hit a restaurant causing mass casualties. What would the nurse do first? 1. Assess the immediate area for electrical wires on the ground. 2. Attend to victim injuries as they are encountered. 3. Activate the community emergency response team. 4. Triage and tag victims according to injury.

3. Activate the community emergency response team. (3. Correct: With mass casualties, community response teams are needed. 1. Incorrect: This would be the second step so that further injuries are not encountered. 2. Incorrect: Triage must occur before treatment of anyone so that an accurate assessment of level of injuries can be made. With mass casualties, a color tag system is usually implemented. 4. Incorrect: This would be the third step.)

Which clients would the nurse monitor for the development of hypovolemic shock? Select all that apply 1. Having an allergic reaction form multiple wasp stings 2. Post-operative cervical spinal cord surgery 3. Addisonian crisis 4. Partial thickness burns over 50% total body surface area (TBSA) 5. Type 2 diabetic with hyperglycemic hyperosmolar nonketotic coma (HHNK)

3. Addisonian crisis 4. Partial thickness burns over 50% total body surface area (TBSA) 5. Type 2 diabetic with hyperglycemic hyperosmolar nonketotic coma (HHNK) (3., 4. & 5. Correct: A client in Addisonian crisis loses sodium and water and can have hypovolemic shock. A client with 50% burns is shifting fluid to the tissues because of the tissue damage of the burns, increasing permeability. A client with type 2 diabetes and an infection can develop HHNK. This massive polyuria can cause shock. With polyuria, think shock first. 1. Incorrect: The nurse would monitor for anaphylactic shock with this client. 2. Incorrect: The nurse would monitor for neurogenic shock with this client.)

A client has returned to the unit following an upper gastrointestinal series (Upper GI). What is the nurse's priority action? 1. Keep client NPO until the gag reflex returns. 2. Perform an immediate cleansing enema. 3. Administer 30 mLs milk of magnesia orally. 4. Monitor vital signs every ten minutes until stable.

3. Administer 30 mLs milk of magnesia orally. (3. Correct: An Upper GI involves the ingestion of a barium based contrast under fluoroscopy to view the esophagus, stomach, and small intestine. Following such a procedure, it is vital for the client to pass all the barium before a blockage occurs. The client is encouraged to drink large amounts of fluid and is administered an over the counter laxative, such as milk of magnesia, to remove barium. 1. Incorrect: The client's gag reflex was not inactivated. The reflex must remain intact in order for the client to drink the barium based contrast during the test. 2. Incorrect: The barium would not yet have reached the colon following the Upper GI and therefore a cleansing enema would not be effective. If the client had received a lower GI, an enema may have been ordered. 4. Incorrect: The client is fully awake and conscious during the entire procedure. No medications were administered that would alter the vital signs; therefore, every 10 minute vital signs are not necessary.)

A nurse is caring for a client on the second day after a thoracotomy. The client reports incisional pain. The nurse assesses the client and evaluates the vital signs. Based on the data documented in the chart, what action should the nurse take first? Client reports incisional pain as 8/10. Wound is clean and dry, without redness, edema, or drainage. Shallow respirations noted at 24/min. Adventitious lung sounds noted in bilateral bases. Oral temperature 100º F/37.8ºC Heart rate 92/min and regular BP 130/80 Respirations 24/min 1. Have client cough and deep breathe. 2. Administer acetaminophen for fever 3. Administer the prescribed analgesic 4. Assist the client to ambulate.

3. Administer the prescribed analgesic (3. Correct: The client described in this question is post thoracotomy. With ANY post-op client, the number one concern, especially as a brand new nurse, is preventing pneumonia. A thoracotomy is very painful and the client is unlikely to breathe deep unless the pain is relieved. Temperature of 100º F/37.8ºC, HR 92, respirations 24, bilateral crackles (indicating atelectasis) all reveal this client is heading for pneumonia. 1. Incorrect: Coughing and deep breathing exercises are exactly what the client needs, but the client will not cough and deep breathe if it hurts. Give pain medication first. 2. Incorrect: Acetaminophen is not potent enough to relieve pain. The goal is to "fix the problem". The problem is that the client is not properly deep breathing due to pain. 4. Incorrect: Assisting the client to ambulate is a good idea, but the nurse has to fix the problem, and the problem is that the client is not deep breathing.)

When planning post procedure care for a client who is having a barium enema, what must the nurse include? 1. Cardiac monitoring for potential arrhythmias 2. Monitoring urinary output 3. Administration of a laxative or enema after the procedure 4. Reordering the client's diet

3. Administration of a laxative or enema after the procedure (3. Correct: The client must expel the barium post procedure. If the barium is not eliminated, it can harden in the colon and cause an obstruction. 1. Incorrect: It is not standard practice to place clients on a cardiac monitor after a barium enema. 2. Incorrect: Monitoring urine output has nothing to do with this procedure and does not answer the specific question related to this diagnostic procedure. 4. Incorrect: Reordering the client's diet is important but is not as life-threatening as a bowel obstruction.)

A client is admitted to the emergency department after sustaining burns to the chest and legs during a house fire. Which assessment should the nurse perform immediately? 1. Respiratory 2. Cardiac 3. Airway 4. Neurological

3. Airway (3. Correct: A fire in an enclosed area causes concern for carbon monoxide poisoning. In addition to the burns to the chest, there is the added potential for airway damage. 1. Incorrect: Important to assess respiratory status but not before airway. 2. Incorrect: Important to assess cardiac, but not #1. 4. Incorrect: This assessment would be done, but not #1.)

A client is admitted for management of chronic obstructive pulmonary disease (COPD). What finding would be of concern to the nurse? 1. Pursed lip breathing 2. Productive cough with thick white sputum 3. Ankles with 2+ pitting edema 4. Barrel chest

3. Ankles with 2+ pitting edema (3. Correct: Swelling in the legs or feet is a serious symptom and should be of concern to the nurse. To make up for the damage to the lungs, the heart must pump harder to get enough oxygen to the rest of the body. Further investigation is needed. 1. Incorrect: Pursed lip breathing helps keep the alveoli open to allow for better oxygen and carbon dioxide exchange. Clients with COPD are taught how to breathe through pursed lips. 2. Incorrect: A chronic, persistent cough is often the first symptom to develop, and may be present every day as the disease progresses. The cough may or may not be productive. If it is productive, white means free of infection, so that is a good thing. 4. Incorrect: As COPD progresses, the antero-posterior diameter of the chest increases. This increase is called a barrel chest and is not of concern to the nurse.)

A child with a radial fracture reports itching below the casted area. What is the appropriate nursing action to relieve itching? 1. Allow the child to use a Q-tip to scratch the area. 2. Assess the fingers and areas above the cast to identify areas of irritation. 3. Apply an ice pack for 10-15 minutes. 4. Raise the cast above the level of the heart to reduce itching sensation.

3. Apply an ice pack for 10-15 minutes. (3. Correct: This will change the sensation. Normally the answer is use a cool blow dryer, but they wanted to see if you would be flexible with what you know. Use an ice pack that will not get the cast wet. 1. Incorrect: A Q-tip is soft, trying to make you feel like this is okay to put into a cast. But objects should never be placed under the cast. 2. Incorrect: How does assessing the fingers decrease itching? It doesn't. 4. Incorrect: Raising the cast above the level of the heart will help reduce swelling under the cast in the first 24 to 72 hours after a cast is applied, but it does not help with itching.)

A client with a history of eczema has been admitted with cellulitis of the left forearm. Which admission order should the nurse question immediately? 1. Start IV of normal saline at 100 mL per hour. 2. Keep left arm elevated on pillow at all times. 3. Apply ice packs to affected area every shift. 4. Ibuprophen 800 mg po every 6 hours prn pain.

3. Apply ice packs to affected area every shift. (3. Correct: Cellulitis is a bacterial skin infection resulting in warm, redden and edematous tissue, sometimes accompanied by fever and chills. Swelling in the affected area impedes blood flow and increases pain. In order to decrease the edema, warm, moist compresses are used to stimulate circulation and speed reabsorption of the fluid within the tissue. This order should be questioned immediately. 1. Incorrect: An infection serious enough to require hospitalization indicates this client is either septic or may need intravenous antibiotics. Fluids are a primary treatment for sepsis along with bedrest and antibiotics. A basic solution of normal saline at 100/mL per hour would be an appropriate order for this client. 2. Incorrect: The swelling characteristic in cellulitis in painful and diminishes circulation to the area. Elevation on one or two pillows at all times helps to improve blood flow so that healing can take place. In some facilities, clients are provided with a wedge shaped cushion that provides better support of the affected area. This order is appropriate. 4. Incorrect: Ibuprophen provides relief from both pain and inflammation associated with cellulitis. A dose of 800 milligrams by mouth every 6 hours as needed for pain would be appropriate for this client. This is not an order the nurse should question.)

A client who has had a stroke presents with lethargy, facial droop, and slurred speech. The client has a history of gastroesophageal reflux disease (GERD). From this history, what does the nurse recognize as an increased risk for this client? 1. Diminished colonic motility 2. Esophageal hemorrhage 3. Aspiration pneumonia 4. Stress ulcers

3. Aspiration pneumonia (3. Correct: Anyone who has had a stroke is at risk for aspiration, especially with a history of reflux disease. It is important to remember that the stomach is full of acid. When aspiration of this acid occurs, it causes irritation to the lung tissue. The client can develop a severe pneumonitis. That's what could kill the client, so this answer takes priority. 1. Incorrect: Diminished colonic motility may become a problem, but aspiration pneumonia is more acute. Remember airway, breathing, and circulation will take priority. 2. Incorrect: Esophageal hemorrhage is seen with esophageal varices, not reflux disease. 4. Incorrect: GERD is not associated with increased risk for stress ulcers, but GERD can also lead to strictures and/or precancerous lesions called Barrett's esophagus.)

The nurse is performing sterile wound care for partial thickness burns on a client's lower right leg. Prior to initiating this procedure, what action should the nurse complete first? 1. Position client upright with right leg elevated. 2. Obtain wound culture before cleaning wound. 3. Assess current pain level and medicate. 4. Encourage client to verbalize concerns.

3. Assess current pain level and medicate. (3. Correct: Wound care on burns is a painful process, particularly with partial thickness burns (formerly referred to as second degree) because nerve endings are intact and exposed. Pre-medicating is a priority action, since pain medication can take up to 30 minutes to activate within the body. Clients are more cooperative and heal faster when pain is well controlled. 1. Incorrect: Proper visualization during wound care is vital, as is client comfort during the procedure. However, completion of this process does not require the client to be in an upright position. In fact, that may be counter productive at this time. Additionally, whether the right leg needs elevated depends on the size or location of the burn on the right leg, and that information has not been provided in the question. 2. Incorrect: While it is true that any wound culture must be obtained prior to cleaning the affected area, this action is not presently the nurse's first priority. Consider the nursing process and choose another option. 4. Incorrect: Therapeutic communication is an on going process during any client interaction, particularly when the nurse needs to explain an upcoming procedure. Allowing the client to express fears, verbalize concerns or ask questions enhances cooperation. Although this exchange of information is occurring throughout this period of time, the nurse has another priority action that should be completed first.)

The nurse sees the following rhythm on the cardiac monitor for a client recovering from a myocardial infarction. What would be the nurse's first action upon entering the client's room? Exhibit 1. Attempt defibrillation 2. Begin CPR 3. Assess for carotid pulse 4. Administer lidocaine

3. Assess for carotid pulse (3. Correct: Although the rhythm strip looks like ventricular fibrillation, you must first check the client. Assess for consciousness, airway, breathing, circulation first. 1. Incorrect: Assess the client first. Do not rely on the strip alone. It may be artifact. If there is no pulse, then you defibrillate. 2. Incorrect: Assess the client first. Defibrillate, then CPR. 4. Incorrect: While CPR is in progress after defibrillation, start IV, if one is not available, then give lidocaine.)

Following a lumbar puncture, the client reports a headache on a pain scale of 8 out of 10. What priority action should the nurse perform? 1. Instruct the client to drink at least 8 ounces of water. 2. Close room blinds to darken the environment. 3. Assist the client into a supine position in bed. 4. Notify primary healthcare provider of client's complaints.

3. Assist the client into a supine position in bed. (3. Correct: The most frequent cause of headache following a lumbar puncture is loss of, or leaking, of cerebrospinal fluid from the puncture site. Positioning a client is an important nursing responsibility, particularly in this situation since the supine position could help to stop any leaking. Following this, the nurse will pursue additional actions as ordered by the primary healthcare provider, which may include increasing fluids or even a blood patch. 1. Incorrect: Although increasing fluids may help clients under specific circumstances, it is not the priority action in this situation. Additionally, the primary healthcare provider may order IV fluids rather than PO fluids. 2. Incorrect: A darkened room can be beneficial for clients with severe migraine headaches, but would not be useful to this client. Headaches following a lumbar puncture are caused by the loss of cerebrospinal fluid and would not respond to a quiet, dark environment. 4. Incorrect: Although the primary healthcare provider should indeed be notified of this situation, the nurse's priority action should first focus on stabilizing the client by addressing the cause of this problem and positioning the client.)

The nurse is caring for a burn client 48 hours after the burn occurred. What would be the nurse's priority assessment? 1. Measure the abdominal girth. 2. Administer pain medication. 3. Auscultate the lungs every 2 hours. 4. Inspect the burn for infection.

3. Auscultate the lungs every 2 hours. (3 Correct: After 48 hours, the fluid in the interstitial spaces will begin to shift back into the vascular space and can lead to fluid volume excess. Excess fluid can back up into the lungs, so auscultation of the lungs takes priority. Remember: Airway, breathing, then circulation. 1. Incorrect: No indication of need to measure abdominal girth. Fluid is now shifting out of the tissue and abdominal cavity back to the vascular space. Worry about fluid volume excess now. 2. Incorrect: Not priority over pulmonary function. Pain is a priority from the client's perspective, but remember pain never killed anyone. However, fluid in the lungs will! Lung assessment takes priority. 4. Incorrect: Not priority over pulmonary function. Again, we want to inspect for infection, but it is not the priority over assessing the lungs.)

The nurse is caring for an oncology client with a WBC-5.5 x 10³/mm³, Hgb-12g/dL, PLT-90 x 10³/mm³. Which measure should be instituted? 1. Protective isolation 2. Oxygen therapy 3. Bleeding precautions 4. Strict intake and output

3. Bleeding precautions (3. Correct: Yes. That is the only value that is not a normal level, and it is way too low, so this client is at risk for bleeding. Bleeding precautions are the appropriate intervention. A normal platelet count ranges from 150,000 to 450,000 platelets per microliter of blood. Having more than 450,000 platelets is a condition called thrombocytosis; having less than 150,000 is known as thrombocytopenia. 1. Incorrect: The WBC is okay. An average normal range is between 4,500 and 10,000 white blood cells per microliter (mcL). Leukopenia is the medical term used to describe a low WBC count. 2. Incorrect: There is no indication of hypoxia in stem, and the Hgb is normal, so the client is not anemic. A low hemoglobin count is generally defined as less than 13.5 grams of hemoglobin per deciliter (135 grams per liter) of blood for men and less than 12 grams per deciliter (120 grams per liter) for women. 4. Incorrect: There is no indication for I & O measurement. I&O should be done with clients who have a fluid volume, cardiac, or renal problem.)

A client has been admitted with advanced cirrhosis. The nurse's assessment reveals an abdominal girth increase of 5 inches (12.7 cm) and a weight increase of 6 lbs. (2.72 kg) since yesterday's measurements. Based on this data, what would be the nurse's priority assessment? 1. Stool for occult blood 2. Ammonia blood level 3. Blood pressure 4. Level of consciousness

3. Blood pressure (3. Correct: In ascites, the client is in FVD and we worry about shock. If my blood pressure drops, I will have decreased perfusion of my vital organs. Poor perfusion leads to organ damage and failure. 1. Incorrect: We are worried about bleeding because the liver is sick, but this is not the first priority in this case. Third spacing (ascites) has increased placing the clietn at risk for FVD and shock. 2. Incorrect: Ammonia level would indicate a worsening liver condition, but this is not a priority here. 4. Incorrect: LOC is a good indicator of perfusion, but it may be affected by other factors, such as the ammonia level, as well. The BP will tell us the most about shock, and that is what I am worried about here.)

The nurse is preparing to administer scheduled medications for a client. Which medication would require clarification prior to administration? Diagnosis: Heart failure Current vital signs: BP 110/64, HR 70, R 18 Allergies: Sulfonamides Medical history: Hypertension Lab results: Glucose- 98 mg/dl (5.4 mmol/L), Sodium- 142 mEq/L (142 mmol/L), Potassium- 3.8 mEq/L (3.8 mmol/L), Digoxin level - 0.8 ng/mL (1.02 nmol/L) Diet: 2 gm Sodium Scheduled procedures: Echocardiogram Chest x-ray Scheduled medications to administer: Digoxin 0.125 IV push every morning Sacubitril/valsartan 24/26 mg by mouth twice a day Bumetanide 0.5 mg by mouth twice a day Potassium chloride 20 mEq by mouth three times a day 1. Digoxin 2. Sacubitril/valsartan 3. Bumetanide 4. Potassium chloride

3. Bumetanide (3. Correct: Bumetanide is a loop diuretic. What is worrisome about giving this medication is the fact that the client is allergic to sulfonamides. It is contraindicated because there is a cross-sensitivity with thiazides and sulfonamides. 1. Incorrect: Digoxin is a cardiac glycoside. There is nothing in the chart or other medications that prevent this medication from being administered. 2. Incorrect: Sacubitril/valsartan is a combination medication used to reduce the risk of cardiovascular death and hospitalization for heart failure. The client should not take this medication within 36 hours before or after taking any ACE inhibitor or other ARB medication. Watch for hypotension, hyperkalemia, and impaired renal function. There are no indications of adverse effects in this question. 4. Incorrect: Administering potassium chloride is acceptable since this client is on a loop diuretic which depletes potassium and digoxin. You do need to monitor for hyperkalemia as well since the client is on sacubitril/valsartan. The serum potassium level is normal in this client.)

Following surgery, a client has an indwelling urinary catheter attached to a collection bag. The nurse empties the collection bag at 0900. At the change of shift at 1500, the collection bag contains 100 mL of urine. The system has no obstructions to urinary flow. What would be the nurse's most appropriate initial response? 1. Elevate the head of the client's bed. 2. Start giving the client 8 ounces of oral fluid per hour. 3. Check circulation and take the vital signs of the client. 4. Continue monitoring, because this is an expected finding.

3. Check circulation and take the vital signs of the client. (3. Correct: A urine output (U/O) of 100 mL over a 6 hour period is dangerously low. This client could be experiencing hypovolemic shock. In clients who are "shocky", the kidneys stop making urine to try to hold on to what little volume the body has left. The nurse is checking the vital signs for low BP and increased HR, indicators of hypovolemic shock. Also, when the urine output is this low, the client is at risk for renal failure. 1. Incorrect: Elevating the head of the client's bed is a good choice when the client is having difficulty breathing, but not here. Raising the HOB will cause the BP to drop lower. Clients in shock should be supine. 2. Incorrect: Normally, pushing fluids is a good choice if the urine output were low. 100 mL over six hours requires more aggressive treatment to combat shock. 4. Incorrect: This is not an expected finding. Urine output less than 240 mL in an eight hour time frame should alert the nurse to a serious problem such as shock.)

A client being prepared for surgery is to be given a pre-operative medication. What is the nurse's priority action when administering the medication? 1. Verify client has signed all consent forms. 2. Escort the client to the bathroom to void. 3. Check that identification band is in place. 4. Raise side rails and put call bell in place.

3. Check that identification band is in place. (3. Correct: All the actions mentioned are important, but the priority is client identification. Regardless of whether the nurse is administering medication, preparing the client to leave for surgery or for testing, the I.D. band must be in place during the entire hospitalization. If the band falls off or is removed for any reason, the client must be re-identified and banded before proceeding with any orders. 1. Incorrect: While it is important for a client to sign pre-operative forms, this is not the nurse's responsibility. The primary healthcare provider or surgeon must witness and verify the client has signed all consent forms prior to surgery. 2. Incorrect: Having the client void prior to transport to the O.R. can easily be accomplished by any staff member, including a UAP or LPN. This action would be extremely important if the nurse was going to administer a narcotic or sedative. But the scenario does not indicate the type of pre-op med to be administered. 4. Incorrect: Again, raising side rails is generally completed after administration of a pre-operative medication, especially narcotics or sedatives. The call bell should always be within the client's reach, and although side rails are an important safety factor, it is not the first priority.)

The nurse is caring for a client with a closed head injury. Three days after admission, urinary output for 8 hours was 1800 mL. In response to this data, what would be the appropriate nursing action? 1. Hydrate the client with 500 mL of IV fluid in the next hour. 2. Monitor BUN and creatinine. 3. Check urine specific gravity. 4. Recognize this as a side effect of dexamethasone.

3. Check urine specific gravity. (3. Correct: For any client with a head injury and abnormally high urinary output, the nurse knows the client is at risk for ADH (anti-diuretic hormone) problems. The pituitary gland is located in the brain. ADH is produced in the pituitary gland. In head injured clients, ADH can get messed up. If the client does not have enough ADH large volumes of water will be lost in the urine. The name of this disease is diabetes insipidus (DI). Large volume losses place the client at risk for shock. The nurse knows to further investigate the problem by checking a urine specific gravity. For clients in DI, the urine specific gravity will be very, very low because they are losing so much water. When you see the letters DI, think of the "D" for diuresis and think SHOCK first. 1. Incorrect: Administration of 500 mL of fluid over one hour is possible if the client were in shock. The stem of the question, however, does not indicate this client is in shock. 2. Incorrect: Monitoring BUN and creatinine does not help identify diabetes insipidus. 4. Incorrect: Decadron can cause fluid retention, not increased urinary output.)

The ICU nurse is caring for a client with massive head injuries. The nurse notices that the client's respirations have a rhythmic increase and decrease of rate and depth and include brief periods of apnea. How would the nurse document this respiratory pattern? 1. Apneusis 2. Ataxic 3. Cheyne-Stokes 4. Cluster

3. Cheyne-Stokes (3. Correct: The respiratory pattern described is Cheyne-Stokes. A client with massive head injuries is at risk for this breathing pattern due to an injury with the cerebal hemispheres. 1. Incorrect: Apneusis is characterized by a sustained inspiratory effort. It does not typically have a period of apnea. 2. Incorrect: Ataxic respirations have an irregular, random pattern of deep and shallow respirations with irregular apneic periods. The irregularity of it differentiates ataxic respirations from Cheyne-Stokes respirations. 4. Incorrect: Cluster breathing is characterized by a closely grouped series of gasps followed by a period of apnea. There is no rhythmic increase and decrease as in Cheyne-Stokes respirations.)

The nurse is assigned to bathe a client diagnosed with dementia. Which nursing intervention should the nurse implement? 1. Increase the volume of the television. 2. Finish the bath as soon as possible. 3. Clean the face and hair at the end of the bath. 4. Delegate another nurse to distract the client.

3. Clean the face and hair at the end of the bath. (3. Correct: The client with dementia begins to be separated from reality. Gradually the client will decrease their ability to perform activities of daily living. These steps in the bathing process should be performed last because bathing the face and washing their hair may upset the client. 1. Incorrect: Because the client is having difficulty processing sensory input, the loud volume on the television will increase the anxiety of the client. The television should be turned off during the bath. 2. Incorrect: Finishing the bath as soon as possible does not address the client's inability to process the bath procedure. The client's bath should not be rushed. The nurse should proceed with the bath in a calm and controlled manner that will reduce the client's anxiety. 4. Incorrect: Options that transfer nursing responsibility to other members of the healthcare team usually are incorrect.)

What information should the nurse include when providing community teaching on burn prevention strategies? Select all that apply 1. Have chimney professionally inspected every 5 years. 2. Microwave a baby bottle rather than heating on the stove. 3. Clean the lint trap on the clothes dryer after each use. 4. Keep anything that can burn at least 3 feet (0.91 meters) away from space heaters. 5. Hold a hot beverage or hold a child, not both at the same time. 6. Home hot water heater should be set at a maximum of 120°F (48.8°C).

3. Clean the lint trap on the clothes dryer after each use. 4. Keep anything that can burn at least 3 feet (0.91 meters) away from space heaters. 5. Hold a hot beverage or hold a child, not both at the same time. 6. Home hot water heater should be set at a maximum of 120°F (48.8°C). (3., 4., 5., & 6. Correct: Lint that accumulates in the lint trap of a dryer can cause a fire, so the lint trap should be cleaned after each use. Space heaters need space at least three feet (0.91 meters) away from anything that can burn. A hot beverage can easily spill on a child by accident when trying to handle both the beverage and child at the same time. Home hot water heater should be set at a maximum of 120°F (48.8°C), especially when small children, the elderly, or diabetics are in the home. 1. Incorrect: A chimney should be professionally inspected every year prior to use. It should also be cleaned if necessary. 2. Incorrect. Never microwave a baby bottle. The formula inside the bottle can become scalding hot in the center.)

The nurse is caring for a client following a transurethral resection of the prostate (TURP). The client has a 3 way irrigation catheter in place. Which observation would indicate the need to slow the irrigation? 1. Clots in urine 2. Bladder pressure 3. Clear urine 4. Bladder spasms

3. Clear urine (3. Correct: The irrigation is regulated so that the urine is free of clots and slightly pink tinged. When it becomes clear after surgery, the fluid is going too fast and not clearing any blood clots effectively. 1. Incorrect: The irrigation should be increased if you see clots in order to keep the catheter patent. 2. Incorrect: Bladder pressure may mean that the indwelling urinary catheter is obstructed. Either increase flow or manually irrigate catheter to ensure patency and no retention of fluid in the bladder. 4. Incorrect: Bladder spasms occur with clots so you do not want to slow the irrigation if this happens. This would indicate the need for increasing the irrigation fluid rate.)

The nurse has just received shift report from the off-going nurse. Which client is the priority and should be seen first by the nurse? 1. Client with a chest tube who has bubbling in the suction control chamber of the closed drainage unit (CDU). 2. Client with emphysema with moderate expiratory wheezing. 3. Client post op with a pulse of 120 bpm who the off-going nurse reports as "anxious". 4. Client with pneumonia who reports pain in the chest and a bad cough.

3. Client post op with a pulse of 120 bpm who the off-going nurse reports as "anxious". (3. Correct: Post op, mild tachycardia and anxious...well that sounds like hypoxia to me. If I have to choose just one client to go see, then I better go see the one that might be experiencing respiratory distress and hypoxia! 1. Incorrect: Is this good bubbling or bad bubbling? Good, right? We expect bubbling in the suction chamber. 2. Incorrect: Wheezing, especially upon exhalation, is commonly seen in clients with emphysema. It does indicate that air is being forced through narrow passages and air trapping is a manifestation of emphysema. However, the client who is anxious and has tachycardia is exhibiting early signs of hypoxia and takes priority. 4. Incorrect: Well, when a client has pneumonia, pain in the chest and a bad cough are expected findings, so this is not the priority over the client with signs of hypoxia.)

The community health nurse plans to educate a client diagnosed with tuberculosis (TB) how to avoid spreading the disease to others. What should the nurse include when educating this client? 1. Wear a N95 respirator when around family at home. 2. Have adult family members get the TB vaccine. 3. Complete TB medication regimen. 4. Live at a sanatorium until cured of TB.

3. Complete TB medication regimen. (3. Correct: The best way to prevent the spread of TB is by completing the medication regimen. TB bacteria die very slowly, and so the drugs have to be taken for quite a few months. Even when a client starts to feel better they can still have bacteria alive in their body. So the person needs to keep taking the TB treatment until all the bacteria are dead. All the drugs must be taken for the entire period of TB treatment. If the entire treatment is not taken then the bacteria may not all be killed. They may then become resistant to the TB drugs which then don't work. 1. Incorrect: This is not required. It is expensive and must be fitted to each family member. Cloth or disposable masks can be used. Also, the client should cover mouth when coughing or sneezing, as TB is spread through the air. 2. Incorrect: The vaccine, BCG, works against a severe type of TB that affects young children but it is not effective against the type of bacteria found in adults. That severe type of TB is called military TB, is rare in the US, so the vaccine isn't used on young children here. 4. Incorrect: This used to be common treatment protocol, but no longer.)

A client who is four days post-op cholecystectomy complains of severe abdominal pain. During the initial assessment the client states, "I have had two almost black stools today." Which nursing action is most important? 1. Start an IV with D5W at 125 mL/hr 2. Insert a nasogastric tube 3. Contact the primary healthcare provider 4. Obtain a stool specimen

3. Contact the primary healthcare provider (3. Correct: What's going on inside? They are hemorrhaging. Assume the worst. The primary healthcare provider is the only one who can stop the bleeding. 1. Incorrect: There's nothing wrong with starting an IV, but isn't the client bleeding while you do this? 2. Incorrect: How does that help the bleeding stop? It doesn't. 4. Incorrect: You are going to wait on a stool specimen and Hemoccult? Don't delay care, notify the primary healthcare provider first.

Which signs and symptoms will the nurse include when teaching a client about indicators of recurrent nephrotic syndrome? Select all that apply 1. Dysuria 2. Hematuria 3. Foamy urine 4. Periorbital edema 5. Weight loss

3. Foamy urine 4. Periorbital edema (3. & 4. Correct: Foamy urine, which may be caused by excess protein in the urine, is seen with nephrotic syndrome. Swelling (edema), particularly around the eyes (periorbital) and in the ankles and feet, is a symptom. 1. Incorrect: Dysuria would be a symptom of disorders such as kidney stone or UTI, rather than nephrotic syndrome. 2. Incorrect: Proteinuria rather than hematuria is seen. 5. Incorrect: Weight gain is seen with renal disorders due to poor renal function and increased fluid volume.)

What should the nurse teach a client about testicular self examination? 1. This exam should be performed bi-annually. 2. The exam should be performed during a cold shower. 3. Gently roll each testicle with slight pressure between the fingers. 4. The epididymis should feel like a hard, knotty rope.

3. Gently roll each testicle with slight pressure between the fingers. (3. Correct: Examine one testicle at a time. Use both hands to gently roll each testicle, with slight pressure, between the fingers to feel for lumps, swelling, soreness or a harder consistency. 1. Incorrect: All men 15 years and older need to perform this examination monthly. 2. Incorrect: The exam should be performed during or right after a warm shower or bath when the the scrotum is less thick. 4. Incorrect: The epididymis should feel soft, rope like, and slightly tender to pressure. It is located at the top of the back part of each testicle. It is not a lump.)

The nurse is preparing to administer a dose of sacubitril/valsartan 24/26 mg by mouth. The nurse has not administered this medication before and is using a drug reference to review information about the medication. Which client and drug reference information supports the nurse's decision to withhold the sacubitril/valsartan? Medical diagnosis: Heart failure Current vital signs: BP 110/64, HR 70, R 28 Medical history: Hypertension Physical examination: Alert/oriented. PERRLA. Skin warm/dry. PMI 6 intercostal space 1 cm left of midclavicular line. Bilateral crackles noted to posterior lung fields. Lab test results: Glucose- 98 mg/dl (5.4 mmol/L) Sodium- 142 mEq/L (142 mmol/L) Potassium- 4.8 mEq/L (4.8 mmol/L) Digoxin level - 0.8 ng/mL (1.02 nmol/L) Hct - 43% Current medications: Digoxin 0.125 IV push every morning Trandolapril 2 mg by mouth daily Potassium chloride 20 mEq by mouth three times a day Furosemide 20 mg by mouth daily Drug Reference: Medication: Sacubitril/valsartan Classification: Neprilysin inhibitor + angiotensin II receptor blocker Indications: Heart failure Contraindications/Precautions: -Hypersensitivity to sacubitrilor valsartan -History of angioedema related to previous ACE inhibitor or ARB therapy -Concomitant use or use within 36 hours of ACE inhibitors -Concomitant use of aliskiren in patients with diabetes Adverse reactions/Side effects: Hypotension, Hyperkalemia, Dizziness, Decreased hgb/hct, Angioedema Interactions: ACE inhibitors increase risk of angioedema. Aliskiren increases risk of hyperkalemia. Route/Dosage: Clients not currently taking an ACE inhibitor or angiotensin II receptor antagonist: Initially, sacubitril 24 mg/valsartan 26 mg twice daily. Maintenance dosage: Double dosage of sacubitril/valsartan every 2-4 weeks, as tolerated, to a target maintenance dosage of sacubitril 97 mg/valsartan 103 mg twice daily. Select all that apply 1. Bilateral crackles noted to posterior lung fields. 2. Potassium- 4.8 mEq/L (4.8 mmol/L). 3. Currently taking trandolapril 2 mg by mouth daily. 4. Concomitant use or use within 36 hours of ACE inhibitors. 5. ACE inhibitors increase risk of angioedema. 6. Decreased Hematocrit.

3. Currently taking trandolapril 2 mg by mouth daily. 4. Concomitant use or use within 36 hours of ACE inhibitors. 5. ACE inhibitors increase risk of angioedema. (3., 4., & 5. Correct: This client is currently taking trandolapril, which is an ACE inhibitor. The drug reference guide specifically said, "concomitant use (of sacubitril/valsartan) or use within 36 hours of ACE inhibitors" is contraindicated because giving with ACE inhibitors can increase the risk of angioedema. 1. Incorrect: Bilateral crackles noted to posterior lung fields. 2. Incorrect: Potassium- 4.8 mEq/L (4.8 mmol/L). This is within normal limits and would not require withholding the sacubitrin/valsartan. 6. Incorrect: This client's Hematocrit is 43%, which is normal. Normal values: Adult males: 42-52% (0.42-0.52). Adult women: 37-47% (0.37-0.47). Therefore, this would not influence the administration of the sacubitril/valsartan.)

A client calls the nurses' station to report chest pain. The nurse looks at the client's cardiac rhythm strip, then hurries into the client's room to find the client unresponsive and without a pulse. What initial action should the nurse take? Exhibit 1. Administer Epinephrine 1mg IV push. 2. Begin cardiopulmonary resuscitation (CPR) for 2 minutes. 3. Defibrillate at 120 joules. 4. Insert supraglottic airway device.

3. Defibrillate at 120 joules. (3. Correct: This client had a witnessed arrest with pulseless ventricular tachycardia. The first thing that the nurse should do is to defibrillate the client at 120-200 joules. Subsequent dosing is increased in a stepwise fashion as needed. 1. Incorrect: Epinephrine, a vasopressor is used to produce vasoconstriction and to raise the BP. It is used for the treatment of Ventricular fibrillation and pulseless ventricular tachycardia for its vasoconstrictive effects. It is not the first action however. If defibrillations do not work, then CPR is done for two minutes prior to administration of epinephrine. 2. Incorrect: Defibrillation should be done immediately unless it is not readily available. Then CPR is performed until the defibrillator arrives. 4. Incorrect: Consider an advanced airway after defibrillations fail to revert the client to Normal Sinus Rhythm.)

Which client would the nurse monitor for the development of hypovolemic shock? Select all that apply 1. Admitted with acute myocardial infarction (MI) 2. Post-operative hip replacement with spinal anesthesia 3. Diagnosed with Addisonian crisis 4. A 10 year old with 40% Total body surface area (BSA) burns 5. Admitted with severe vomiting and diarrhea

3. Diagnosed with Addisonian crisis 4. A 10 year old with 40% Total body surface area (BSA) burns 5. Admitted with severe vomiting and diarrhea (3., 4. & 5. Correct: With addisonian crisis, the client is losing too much sodium and water from the vascular space which can lead to hypovolemic shock. A 10 year old child with 40% burns is considered to have a major burn. With major burns, the client loses fluid from the vascular space because of increased permeability. This sends fluid to the tissues and leads to hypovolemic shock.​ The circulating blood in your body may drop with the loss of too many other body fluids. Severe vomiting and diarrhea may cause hypovolemic shock due to this loss of body fluids. 1. Incorrect: I would worry about cardiogenic shock with this client. 2. Incorrect: I would worry about neurogenic shock with this client.)

During a health fair, a client asks the nurse about the methods used to detect prostate cancer. What should the nurse tell the client about the detection process? 1. Abdominal x-rays to detect the presence of lesions and masses. 2. A serum calcium test to detect elevated levels, which may indicate bone metastasis. 3. Digital rectal exam (DRE) and prostate-specific antigen (PSA) test to evaluate the prostate. 4. A magnetic resonance image (MRI) study to detect tumors and other abnormal growths.

3. Digital rectal exam (DRE) and prostate-specific antigen (PSA) test to evaluate the prostate. (3. Correct: Prostate cancer is the second most common type of cancer and the second leading cause of cancer death in men. Early detection improves outcome. DRE and PSA should be offered annually beginning at age 50 to men who have a life expectancy of at least 10 years and at age 45 in high-risk groups. The DRE estimates the size, symmetry, and consistency of the prostate gland while the PSA measures for elevated levels consistent with prostatic pathology, although not necessarily cancer. Declining PSA levels are useful in determining efficacy of treatment for prostate cancer. 1. Incorrect: Radiologic studies are not screening tools for this disease. 2. Incorrect: Hypercalcemia may indicate cancerous bone involvement, but it's not a screening tool. 4. Incorrect: MRI is a diagnostic tool, not a screening tool.)

A client diagnosed with celiac disease has been prescribed a gluten-free diet. Which meal, if chosen by the client, would indicate to the nurse that the client understands this diet? 1. Cream based chicken soup 2. Breaded baked chicken with peas 3. Grilled catfish with mixed vegetables 4. Marinated talapia with green beans

3. Grilled catfish with mixed vegetables (3. Correct: Fresh meats, fish and poultry (not breaded, batter-coated or marinated), fruits and vegetables are allowed on a gluten-free diet. 1. Incorrect: This may contain hidden gluten as wheat may be used as a thickener. 2. Incorrect: Fresh meats, fish, and poultry are allowed, but not if breaded, batter-coated, or marinated. 4. Incorrect: Fresh meats, fish, and poultry are allowed, but not if breaded, batter-coated, or marinated.)

The nurse is monitoring the healing of a full-thickness wound to a client's right thigh. The wound has a small amount of blood during the wet to dry dressing change. What action should the nurse initiate next? 1. Notify the primary healthcare provider. 2. Obtain wound culture. 3. Document the findings. 4. Remove dressing and leave open to air.

3. Document the findings. (3. Correct: Look at the clues: full thickness wound, small amount of blood, wet to dry dressing. With a full thickness wound there is destruction of the epidermis, dermis, and subcutaneous tissues going down to the bone. ​So you would expect to see a small amount of blood or drainage wouldn't you? Yes. This is expected. Simply document this normal finding. 1. Incorrect: Is there really anything to worry about in this situation? No, so you do not need to notify healthcare provider. Now, with most questions on NCLEX there is something to worry about but just not with this one. 2. Incorrect: No, bleeding is not a sign of infection which is what you would be worried about if you got a wound culture. 4. Incorrect: Probably not, just a sign of blood flow in healing wound. Wet to dry dressing helps to debride the wound. So if you remove the dressing will debridement occur? No.)

The nurse is discharging a client post right radial percutaneous transluminal coronarey angioplasty (PTCA) with stent insertion. Which instructions should the nurse give the client to reduce the risk of complications? Select all that apply 1. Do not use the wrist to lift more than 5 pounds (2.27 kg) for 24 hours. 2. Stop taking aspirin in one week. 3. Drink at least 8 glasses of water a day. 4. Wear loose fitting sleeves. 5. Do not shower or soak in a tub for one week. 6. Take short walks around your house.

3. Drink at least 8 glasses of water a day. 4. Wear loose fitting sleeves. 6. Take short walks around your house. (3., 4., & 6. Correct: Drink eight to ten glasses of water to flush the contrast material from the client's system. The client should wear loose sleeves. We do not want any constriction to the surgical site. In general, the client will need to take it easy for the first two days after getting home. The client can expect to feel tired and weak the day after the procedure, but it is important to take walks around the house. This will help prevent blood clots. 1. Incorrect: The client should limit the use of the wrist. It is important to allow the artery to heal. So, no straining of the wrist. Do not use the wrist used in the procedure to lift more than 2 pounds (0.9 kg) for 24 hours. 2. Incorrect: Clients are maintained on aspirin indefinitely after percutaneous coronary intervention to prevent future thrombotic events. 5. Incorrect: The client can shower after the pressure dressing is removed (usually the day after surgery). The client should keep the area clean and dry when not showering.)

What signs and symptoms does the nurse expect a client diagnosed with bacterial pneumonia to exhibit? Select all that apply 1. Asymmetrical chest expansion 2. Night sweats 3. Dyspnea 4. Tachypnea 5. Pleuritic chest discomfort 6. Increased tactile fremitus

3. Dyspnea 4. Tachypnea 5. Pleuritic chest discomfort 6. Increased tactile fremitus (3., 4., 5., & 6. Correct: The most common presenting signs and symptoms of pneumonia are cough, fever, chills, dyspnea, tachypnea, and pleuritic chest pain. If consolidation is present, increased tactile fremitus (vibration of the chest wall produced by vocalization) may be noted. 1. Incorrect: Asymmetrical chest expansion occurs if the client has a collapsed lung from a pneumothorax or hemothorax. 2. Incorrect: Night sweats is a common symptom of tuberculosis, not pneumonia.)

The nurse is discussing frostbite prevention with a group of teenagers who participate in cold weather activities. What information should the nurse provide? Select all that apply 1. Limit alcohol intake when out in cold weather. 2. Dress in several layers of tight fitting clothing. 3. Eat well-balanced meals. 4. Synthetic clothes absorb moisture and become wet quickly. 5. Wear a wool headband over the ears. 6. Wear several pairs of socks.

3. Eat well-balanced meals. 5. Wear a wool headband over the ears. (3., & 5. Correct: Eat well-balanced meals and stay hydrated. Doing this even before going out in the cold will help the person stay warm. People should not diet or restrict food or fluid intake when participating in winter outdoor activities. Malnutrition and dehydration contribute to cold related illnesses and injury. Heavy woolen or windproof materials make the best headwear for cold protection. 1. Incorrect: Don't drink alcohol if planning to be outdoors in cold weather. Alcoholic beverages cause the body to lose heat faster. 2. Incorrect: Dress in several layers of loose, warm clothing. Air trapped between the layers of clothing acts as insulation against the cold. 4. Incorrect: Synthetic clothes should be worn because it moves moisture away from the body and dries quicker. Cotton clothing, especially as an undergarment, holds moisture and becomes wet quickly and contributes to the development of hypothermia and frostbite. 6. Incorrect: Wearing several pairs of socks can decrease circulation and lead to frostbite. Instead, wear socks and sock liners that fit well, wick moisture and provide insulation.)

The nurse is caring for a client with acute renal failure. The morning assessment findings indicate the client has become confused and irritable. Which finding is most likely responsible for the change in behavior? 1. Hyperkalemia 2. Hypernatremia 3. Elevated blood urea nitrogen (BUN) 4. Limited fluid intake

3. Elevated blood urea nitrogen (BUN) (3. Correct: A client with acute renal failure will have an increased (BUN). Significant elevation in BUN may result in nausea, vomiting, lethargy, fatigue, impaired thought processes, and headache. 1. Incorrect: Hyperkalemia can result from acute renal failure. Symptoms of hyperkalemia do not include confusion and irritability. Hyperkalemia may cause muscle weakness, muscle twitching, and flaccid paralysis. 2. Incorrect: Clients with renal failure retain fluid and are at risk for dilutional hyponatremia. Increased or decreased sodium levels can cause confusion, but this client is not at risk for hypernatremia. 4. Incorrect: Clients in acute renal failure should have limited fluid intake. This will not lead to confusion.)

What should the nurse emphasize when teaching clients how to decrease the risk of chronic obstructive pulmonary disease? Select all that apply 1. Avoid exposure to individuals with respiratory infections. 2. Increase intake of Vitamin C. 3. Eliminate exposure to second hand smoke. 4. Avoid prolonged exposure to occupational dusts and chemicals. 5. Get a yearly influenza and pneumococcal vaccination.

3. Eliminate exposure to second hand smoke. 4. Avoid prolonged exposure to occupational dusts and chemicals. (3., & 4. Correct: The most important environmental risk factor for COPD is cigarette smoking. Second hand smoking also contributes to COPD. Risk factors for COPD include prolonged and intense exposure to occupational dust and chemicals as well as indoor and outdoor air pollution. 1. Incorrect: Exposure to individuals with respiratory infections does not increase risk of chronic obstructive pulmonary disease. Respiratory infections may cause an acute exacerbation in a client with existing COPD. 2. Incorrect: Increasing intake of vitamin C does not decrease risk of obstructive pulmonary disease.)

The nurse just received an arterial blood gas (ABG) report that shows a borderline high PCO₂ on a client who had chest surgery. What should be the priority nursing intervention? 1. Tell the client to breathe faster. 2. Medicate for pain and ambulate. 3. Have client use the incentive spirometer. 4. Prepare to administer bicarbonate to buffer.

3. Have client use the incentive spirometer. (3. Correct: This client had chest surgery and the pCO₂ is high. What are you worried about? Hypoventilation. Yes, the client is probably hurting due to the incision and does not want to take deep breaths. In order to get rid of the excess CO₂ the client needs to turn, cough, and deep breath. Incentive spirometry can be provided to assist the client with this effort. 1. Incorrect: Breathing faster will only work for a few minutes. The problem is the client needs to breathe deeper to get more oxygen to the tissue and more CO₂ out of the lungs. Hyperventilating will lead to respiratory alkalosis. 2. Incorrect: No more sedation! The client is not breathing enough. Walking would be okay. This client needs to take deep breaths. 4. Incorrect: No, we want the client to blow off the CO₂. Bircarb will make the pH happy for a short period of time but will not correct the problem. The problem is shallow respirations, so fix the problem.)

During day shift, staff notifies the nurse that an elderly client seems slightly confused and has become incontinent. Upon assessing the client, the nurse notes an increased pulse with blood pressure lower than normal. What action by the nurse takes priority? 1. Call primary healthcare provider stat. 2. Notify family that client is confused. 3. Have staff collect a urine specimen. 4. Apply oxygen at 2/L via nasal cannula.

3. Have staff collect a urine specimen. (3. Correct: In the elderly, symptoms of urinary tract infections (UTIs) may vary from standard manifestations usually seen in younger client. An older client may initially show neurologic signs such as confusion or falls in addition to frequency, incontinence or lower abdominal pain. Those clients with recurring UTIs may even have a standing prescription written for a urinalysis anytime confusion is noted. 1. Incorrect: Although the primary healthcare provider will need to be notified about the changes occurring with the client, the nurse should focus on addressing current client needs, including assessing and stabilizing the client if necessary. 2. Incorrect: The nurse will definitely notify the family when a diagnosis of urinary tract infection is confirmed; however, there is no need to report the confusion to the family until all data has been collected and the primary healthcare provider has determined the outcome. 4. Incorrect: There is no indication of any respiratory issues requiring the nurse to provide oxygen for the client.)

A client arrives at the emergency department (ED) in obvious emotional distress, reporting perioral numbness and tingling of the fingers and toes. The nurse notes a respiratory rate is 56/min. What should be the initial intervention performed by the nurse? 1. Send the client for a CT of the head. 2. Place on 100% O₂ per non-rebreathing face mask. 3. Have the client breathe into a paper bag. 4. Administer diazepam 2 mg IV push.

3. Have the client breathe into a paper bag. (3. Correct: Recognize the respiratory rate is too fast. This client is hyperventilating and blowing off too much CO₂ which has resulted in symptoms of respiratory alkalosis, perioral numbness, and tingling of the fingers and toes. The nurse should try to help calm the client and encourage the client to slow the rate of breathing. This will help hold onto CO₂. By breathing into a paper bag, the client will re-breathe CO₂ therefore increasing the CO₂ level. 1. Incorrect: The client is not demonstrating signs of a stroke. A CT is not warranted based on the information provided. 2. Incorrect: Administration of O₂ is not warranted at this time. The client is blowing off too much CO₂ and needs to re-breathe CO₂ using a paper bag. Increasing O₂ will not fix the problem of emotional distress. 4. Incorrect: Diazepam has sedative effects. Although hysterical clients may have to be sedated to decrease the respiratory rate, the less invasive means of using the paper bag should be attempted first.)

What information should be included when a nurse is teaching a group of college students about the transmission of hepatitis B and human immunodeficiency virus (HIV)? 1. HIV is transmitted via toilet seats whereas hepatitis B is not. 2. HIV is transmitted by sexual contact whereas hepatitis B is not. 3. Hepatitis B is more readily transmitted via needle sticks than HIV. 4. Neither virus is transmitted via body fluids.

3. Hepatitis B is more readily transmitted via needle sticks than HIV. (3. Correct: Hepatitis B virus (HBV) and HIV can be transmitted in similar ways, but hepatitis B is more infectious. Studies show hepatitis B is more readily transmitted via needle sticks than HIV. More than 1 million people currently have HIV in the United States. Hepatitis B is 50-100 times more infectious then HIV. 1. Incorrect: Neither virus is transmitted via toilet seats. Both are spread by contact with infected body fluids such as blood, semen and vaginal fluid, or from a mother to her baby during pregnancy or delivery. 2. Incorrect: Both hepatitis B and HIV are transmitted via body fluids through sexual contact. Therefore, condoms should be used during sexual contact. Using a latex condom reduces the chances of hepatitis B and HIV being passed on during sex. Syringes and other injecting drug equipment should never be shared. 4. Incorrect: Both hepatitis B and HIV are transmitted via body fluids through sexual contact. Standard precautions should be implemented for both HIV and hepatitis B. The CDC recommends HBV vaccination for people who are at risk for or living with HIV, including men who have sex with men (MSM); people who inject drugs; household contacts and sex partners of people who have HBV; people with multiple sex partners; anyone with a sexually transmitted infection; people with diabetes; and health care and public safety workers who may be exposed to blood on the job.)

What is the priority electrolyte imbalance for the nurse to monitor when caring for a client post op thyroidectomy? 1. Hypercalcemia 2. Hyperkalemia 3. Hypocalcemia 4. Hypomagnesemia

3. Hypocalcemia (3. Correct: Why is hypocalcemia the correct answer? The complication from removal of the thyroid is possible injury or removal of parathyroid glands. This produces a disturbance in calcium levels. The calcium levels fall resulting in hypocalcemia. 1. Incorrect: No, the calcium is not elevated. With possible removal of the parathyroid glands during thyroidectomy the calcium will decrease. 2. Incorrect: Potassium is not the priority electrolyte to monitor post thyroidectomy. 4. Incorrect: No, magnesium is not the priority electrolyte to monitor post thyroidectomy.)

The nurse evaluates an electrocardiogram (EKG) and notices a U-wave. The nurse suspects that this occurrence is caused by which electrolyte imbalance? 1. Hypermagnesemia 2. Hypocalcemia 3. Hypokalemia 4. Hyponatremia

3. Hypokalemia (3. Correct: The most serious complications of hypokalemia are cardiac changes. Hypokalemia impairs myocardial conduction and prolongs ventricular repolarization. This can be seen by a prominent U-wave (a positive deflection following the T-wave on the EKG). The U-wave is not totally unique to hypokalemia, but its presence is a signal for the clinician to check the serum potassium level. 1. Incorrect: Remember hypermagnesemia results in the client having a sedated appearance, decreased deep tendon reflexes, decreased level of consciousness, decreased respiratory rate, and ultimately cardiac arrest. 2. Incorrect: In hypocalcemia, this client is not sedated and will have an increased nerve excitability, tetany, appearance of Trousseau's, and Chvostek's sign. Cardiac manifestations include Vtach. 4. Incorrect: Hyponatremia results in neurological symptoms: confusion, irritability, and ultimately coma.

Which comment made by a client scheduled for a lumbar laminectomy and discectomy indicates to the nurse that the client needs further teaching? 1. After the incision is healed, I can go for daily walks. 2. By the time I am discharged, my back and leg pain will be better. 3. I can turn by myself after surgery, but I will need help to get out of bed. 4. The staff will frequently check my feet and legs for feeling and movement.

3. I can turn by myself after surgery, but I will need help to get out of bed. (3. Correct: The client must log roll with assistance. The spine must be kept in proper alignment to allow the area time to rest and heal. The nurse should reinforce this information with the client. 1. Incorrect: After the incision heals, it is acceptable practice to go for daily walks so this is an accurate understanding of what the client can do after the surgery. 2. Incorrect: Successful laminectomies and discectomies will relieve back and leg pain so this is accurate, also. 4. Incorrect: The nurse knows it is very important to perform neurovascular checks after ANY orthopedic surgery. Any changes from their baseline should be reported to the primary healthcare provider immediately. This indicates successful teaching.)

A client who underwent a laparoscopic cholecystectomy is being discharged from an outpatient surgical center. Which statement by the client shows the nurse that discharge teaching has been effective? 1. I will need to eat a low fat diet since I no longer have a gallbladder. 2. I can expect drainage from the incisions for a few days. 3. I may have some mild pain from the procedure. 4. I should plan to limit my activities and not return to work for several weeks.

3. I may have some mild pain from the procedure. (3. Correct: After a laparoscopic procedure the client can expect to have some mild pain. Severe pain, however, would indicate a problem. 1. Incorrect: The client can resume their usual diet. The liver will produce enough bile to digest fats. The gallbladder stores bile. Without the gallbladder, the bile just drains from the liver. 2. Incorrect: The client should not have drainage from the incisions. There are 2-3 small incisions on the abdomen that do not normally have drainage. 4. Incorrect: The client can return to normal activities in 2 to 3 days. This is not considered a major surgical procedure with a large abdominal incision. Recover time is much shorter, allowing the client to return to normal activities sooner.)

A nurse is providing discharge teaching to a client who has had a cystectomy and formation of an ileal conduit. What client statement indicates that teaching was successful? 1. I should restrict my fluid intake to decrease the need to empty the drainage bag. 2. I will change my appliance daily to prevent skin excoriation from the leakage of urine. 3. I will change my drainage bag whenever it is leaking, giving special attention to my skin around the bag. 4. I will restrict going to events outside the home because leakage is common and embarrassing.

3. I will change my drainage bag whenever it is leaking, giving special attention to my skin around the bag. (3. Correct: The goal is to prevent skin irritation by changing the bag regularly and using proper equipment to prevent leakage. The client with an ileal conduit (urinary diversion) must be educated appropriately to ensure that self-care abilities are complete and safe. As long as the bag is not leaking and no skin breakdown is evident, changing the appliance bag only has to be done as needed, not daily. 1. Incorrect: Clients with ileal conduits are encouraged to increase fluid intake, as opposed to restricting it, to maintain adequate urine flow for prevention of urinary tract infections (UTI). 2. Incorrect: Changing the apparatus daily is too often and will cause trauma and skin breakdown. 4. Incorrect: Clients with ileal conduits are encouraged to maintain an active and normal lifestyle. People with ileal conduits have a generally low incidence of complications and high client satisfaction level due to the ease of care and minimal lifestyle changes.)

Which goal is the most important for the nurse to address for a client admitted to the cardiac rehabilitation unit? 1. Reduction of anxiety 2. Referral to community resources 3. Identification of lifestyle changes 4. Verbalization of energy-conservation techniques

3. Identification of lifestyle changes (3. Correct: On admission, the best starting point is to survey what is good and what needs to be changed. 1. Incorrect: No, people need some anxiety to change. 2. Incorrect: Not yet.This may be done, but it is not the most important thing right now. 4. Incorrect: For cardiac rehab we want to exercise, not conserve, at this point. Conserving energy is for times of hypoxia or angina.)

A client had an open cholecystectomy several days ago. What finding by the nurse should be reported to the primary healthcare provider immediately? 1. Respiratory rate of 30 2. Blood pressure reading of 104/50 3. Incisional pain with foul, green drainage 4. Urinary output of 75 mL straw colored urine

3. Incisional pain with foul, green drainage (3. Correct: The client is having incisional pain, which by itself could be expected following an open cholecystectomy. However, there should never be any foul, green drainage from an incision, as this indicates a post-operative infection. The nurse should report this immediately to the primary healthcare provider. 1. Incorrect: Although this respiratory rate seems slightly on the elevated side, this client has had recent surgery and is now having some complications. Combined with the pain, this rapid respiratory rate would be expected. 2. Incorrect: There is no baseline data provided regarding this blood pressure data. Without a reference to a client's previous blood pressure, it is impossible to form any opinion about this reading. We worry about a systolic BP of 90. 4. Incorrect: The information provided in the question does not give any parameters by which to evaluate the urine. Straw colored urine is a normal finding; however, there is no indication regarding the length of time it took to accumulate 75 mL of urine. Therefore, no decision can be formed about this finding.)

A client receiving palliative care is reporting constipation. What intervention should the palliative care nurse provide first? 1. Increase foods high in fiber. 2. Administer an enema 3. Increase fluid intake 4. Administer docusate sodium

3. Increase fluid intake (3. Correct: Increase fluid intake is correct. Dehydration is one of the most common causes of constipation. Fluids keep your stool soft and easy to pass. 1. Incorrect: Fiber should be increased. This is true but water is the first intervention that should be implemented. Fiber will increase bulk and help with passage of stool but fluids should be first. 2. Incorrect: Administering an enema would not be the first thing to try for constipation. Least invasive first. Avoid medicines as long as possible. 4. Incorrect: Docusate sodium is colace and a stool softener, although appropriate avoid medicines as long as possible.)

The client has pustules on the arm from intravenous drug abuse. The microbiology laboratory informs the nurse that the client's cultures are growing methicillin-resistant Staphylococcus aureus (MRSA). Which action would the nurse take? Select all that apply 1. Implement droplet precautions immediately. 2. Inform the client to wear a mask when ambulating in the hall. 3. Instruct the client on the importance of hand hygiene. 4. Cover the pustules to prevent drainage. 5. Allow pustules to drain freely.

3. Instruct the client on the importance of hand hygiene. 4. Cover the pustules to prevent drainage. (3. & 4. Correct: It is important that the nurse implement these interventions in order to prevent the spread of infection. If the client refuses to follow instructions, then isolation precautions are warranted. 1. Incorrect: Contact isolation should be instituted. 2. Incorrect: The client is placed on contact precautions not droplet precautions. The client would not wear a mask. 5. Incorrect: The pustules should be covered with a dressing. Opening the pustules will increase the chance of spreading the infection.)

A client has been admitted to the emergency department after repeated food binging and purging by vomiting and laxative abuse. The client reports leg pains and weakness. ECG reveals a depressed ST segment and flattened T wave. Based on this data, what does the nurse anticipate that this client will need to receive first? 1. Oral fluids 2. Kayexalate enemas 3. Intravenous potassium (KCl) 4. An antidiarrheal medication

3. Intravenous potassium (KCl) (3. Correct: Look at the clues in the stem: vomiting, laxative abuse, symptoms of hypokalemia including weakness, muscle cramps, and arrhythmias. Due to repeated laxative abuse and vomiting, the client has lost potassium. Normal potassium is 3.5-5.0 mEq/L. IV potassium is required for a severely low potassium. 1. Incorrect: Oral fluids are needed, but with symptoms this severe, IV resuscitation is needed with potassium. The client is exhibiting symptoms of severe hypokalemia. The potassium is prescribed to correct this imbalance. 2. Incorrect: Kayexalate is given for high potassium. This client's potassium is low. The therapeutic effect of kayexalate is to reduce the serum potassium level. 4. Incorrect: We are worried about low potassium here. This won't solve the problem. An antidiarrheal medication may be prescribed, but the client is exhibiting symptoms of hypokalemia. The client should be administered the IV potassium first to correct the low potassium level.)

What clinical manifestation does the nurse expect to see in a client suspected of having hypercalcemia? Select all that apply 1. Tachycardia 2. Positive Chvostek 3. Lethargy 4. Tachypnea 5. Decreased deep tendon reflexes

3. Lethargy 5. Decreased deep tendon reflexes (3., & 5. Correct: Hypercalcemia is a condition in which the calcium level in blood is above normal. Too much calcium in blood can weaken bones, create kidney stones, and interfere with heart and brain function. Hypercalcemia is usually a result of overactive parathyroid glands. Other causes include cancer, some medications, and taking too much of calcium and vitamin D supplements. Signs and symptoms of hypercalcemia range from nonexistent to severe. Lethargy and decreased deep tendon reflexes are two manifestations of hypercalcemia. 1. Incorrect: Bradycardia rather than tachycardia is seen with hypercalcemia. Remember - muscles are sedated. 2. Incorrect: A Negative Chvostek will be seen with hypercalcemia. It will be positive in hypocalcemia. 4. Incorrect: Hypercalcemia will result in a decreased, rather than increased respiratory rate.)

A client diagnosed with hypothyroidism has received dietary education from the nurse. Which snack selection chosen by the client would indicate that education has been successful? 1. Cup of almonds 2. Cheese and crackers 3. Popcorn 4. Sweet potato fries

3. Popcorn (3. Correct: Hypothyroidism clients tend to have constipation due to decreased motility of the GI tract and need increased fiber and fluid intake. Popcorn is high in fiber. 1. Incorrect: People with hypothyroidism have a slow metabolism and do not need high protein but a well balanced diet. Almonds are high in protein. 2. Incorrect: Cheese and crackers are high in sodium. This client is at risk for CAD, so sodium should be limited. 4. Incorrect: This client does not need high potassium, which fried sweet potatoes have. The high potassium dietary approaches to stop hypertension (DASH) diet is only for healthy clients with hypertension.)

A nurse is planning to teach a group of adult males in their 40's about health care promotion recommendations. Which recommendations should the nurse include? Select all that apply 1. Do bi-annual skin self-exam to check for new moles or changes in moles. 2. Comprehensive eye exam every 5 years starting at age 45. 3. Limit alcohol intake to no more than two drinks per day. 4. Yearly physical exam from a health care provider. 5. Get at least 30 minutes of moderate physical exercise on most days of the week.

3. Limit alcohol intake to no more than two drinks per day. 5. Get at least 30 minutes of moderate physical exercise on most days of the week. (3, & 5. Correct: If a client must drink alcohol, they should do so only in moderation. For men, that means up to two drinks a day for men age 65 or younger and one drink a day for men over age 65. The risk of various types of cancer, such as liver cancer, appears to increase with the amount of alcohol ingested and the length of time that one has regularly been drinking. Too much alcohol can also raise blood pressure. Physical exercise can go a long way toward managing stress and controlling weight. Controlling stress and obesity can decrease the risk of many health risks such as heart disease, diabetes, and stroke. 1. Incorrect: Do monthly skin self-exam to check for new moles or changes in moles. 2. Incorrect: Comprehensive eye exam every 2 years is recommended. Changes in vision is a relatively common problem for people in their 40s. They may find that glasses are needed for the first time in their life. They may need glasses to see at a distance or for reading. 4. Incorrect: Physical exam every 2-3 years when no health issues exist including height, weight, and BMI. Routine blood tests, urinalysis and mental health screening is conducted at this time.)

Which physical assessment finding noted in a client would be of concern to the nurse? 1. Generalized tympany with abdominal percussion 2. Nonpalpable spleen 3. Liver palpated 3 cm below right costal margin 4. Negative rebound tenderness

3. Liver palpated 3 cm below right costal margin (3. Correct: If the client has chronic obstructive pulmonary disease, large lungs, or a low diaphragm, the liver may be normal and palpable at 0.4 to 0.8 inches (1-2 cm) below the right costal margin. Palpating the liver 3 cm below the right costal margin is abnormal and would be of concern to the nurse. 1. Incorrect: The purpose of percussing the abdomen is to estimate the size of the liver and determine the presence of fluid, distention, and masses. Air produces a higher pitched, hollow sound termed tympany. Tympany is the predominant percussion sound of the abdomen. 2. Incorrect: The spleen is normally not palpable. 4. Incorrect: The nurse should check any problem areas on the abdomen for rebound tenderness. If present, rebound tenderness indicates peritoneal inflammation.)

What turning method should the nurse use to turn a client who has a spinal injury? 1. Lateral transfer 2. Slide sheet procedure 3. Logrolling 4. Mechanical lift transfer

3. Logrolling (3. Correct: Logrolling is used for the client who has a spinal injury. This technique keeps the client's body in straight alignment at all times. 1. Incorrect: Lateral transfer uses a spinal board to move the client from one bed to another. 2. Incorrect: Slide sheets enable clients to be slid up a surface or over to their side, that is, up the bed or rolled over in the bed. The difference is that all of the body may not be kept in perfect alignment as with logrolling. 4. Incorrect: A mechanical lift is used to move client from a bed to chair or chair to bed.)

A gunshot victim is brought by ambulance to the emergency room with an open pneumothorax. A bio-occlusive dressing is applied to the chest. The nurse then notes increased dyspnea and sub-q emphysema in the client. What is the nurse's priority action? 1. Prepare client for insertion of chest tube. 2. Apply a non-rebreather with 100% oxygen. 3. Loosen one side of the bio-occlusive dressing. 4. Obtain a tracheostomy kit and call the surgeon.

3. Loosen one side of the bio-occlusive dressing. (3. Correct: An open pneumothorax, also referred to as a sucking chest would, allows outside air to rush into the chest cavity. Because outside air has greater pressure than intrathoracic air, the pressure builds up quickly creating a mediastinal shift that collapses all structures in the thoracic cavity. This is referred to as a tension pneumothorax and can be caused by securing all four sides of the bio-occlusive dressing. The nurse should check to see that the dressing is loose on one side. If it is not, one side of the dressing must be released to allow the air to escape from the chest. 1. Incorrect: Although this client will ultimately need a chest tube to remove the air that entered the chest cavity, this is not the nurse's priority action. 2. Incorrect: A dyspneic client may certainly need supplemental oxygen; however, there are not enough parameters provided to determine whether the client is truly hypoxic. 4. Incorrect: An emergency tracheostomy is not the initial treatment for dyspnea or sub-q emphysema. Such an invasive procedure would be used only in a life-threatening situation.)

A client diagnosed with a hemorrhagic stroke is being transferred to the medical unit from the intensive care unit. Which nursing intervention should the nurse initially implement? 1. Administer an osmotic diuretic. 2. Complete a neurological assessment. 3. Maintain the head of the bed at 30 degrees. 4. Instruct the client to take a stool softener daily.

3. Maintain the head of the bed at 30 degrees. (3. Correct: Hemorrhagic strokes are the result of ruptured vessel bleeding in the cranial cavity. This action will result in increased intracranial pressure (ICP). ICP can cause a decrease in the brain's metabolism and hypoxia of the brain tissue. The head of the bed should be elevated to decrease the increased intracranial pressure which can reduce damage to the brain. The intervention of raising the head of the bed to 25 -30 degrees is directly related to a decrease in ICP. 1. Incorrect: An osmotic diuretic is administered to increase the osmotic effect on the kidneys which will decrease ICP. An osmotic diuretic is initiated during the acute care protocol for a stroke. 2. Incorrect: A neurological assessment would be done upon admission to the medical unit. But maintaining the head of the bed at 30 degrees is the initial action. 4. Incorrect: The readiness of the client to learn should be evaluated prior to initiating teaching. Due the client's immediate transfer from ICU, this is not the apparent time to begin to the initiate client teaching. Ways to avoid straining during a bowel movement instruction is not the priority nursing intervention.)

A client is returned to the surgical unit following gastric/esophageal repair of a hiatal hernia, with an IV, NG tube to suction, and an abdominal incision. To prevent disruption of the esophageal suture line, what is most important for the nurse to do? 1. Assess the wounds for drainage. 2. Give ice chips sparingly. 3. Maintain the patency of the NG tube. 4. Monitor for the return of peristalsis.

3. Maintain the patency of the NG tube. (3. Correct: Maintain the patency of the NG tube. On any post-op client, the nurse is responsible for preventing disruption of the suture line. (Disrupture of any suture line, since disruption could be life-threatening.) The nurse is responsible for keeping the NGT patent to prevent accumulations of gastric secretions and blood in the stomach. Accumulation of fluid in the stomach can cause pressure on the suture line and places the client at risk for disruption of the suture line and hemorrhage. The nurse knows never to allow pressure or stretching on suture lines. 1. Incorrect: Assessing the wound for drainage is important, but when there is something more life-threatening, that is the priority answer. Disrupting the sutures is more life-threatening. 2. Incorrect: This person is absolutely NPO. Giving ice chips is contraindicated as it could disrupt the suture lines. 4. Incorrect: It is important to monitor for return of peristalsis, but this is not life-threatening.)

What is the most effective method of stroke prevention that the nurse should teach to the public? 1. Administering platelet inhibitors to prevent clot formation. 2. Undergoing transluminal angioplasty to open a stenosed artery and improve blood flow. 3. Maintaining normal weight, exercising, and controlling comorbid conditions. 4. Administering tissue plasminogen activator (tPA).

3. Maintaining normal weight, exercising, and controlling comorbid conditions. (3. Correct: Although administering platelet inhibitors, tPA, and undergoing transluminal angioplasty may improve cerebral blood flow, the goals of stroke prevention include health promotion for the healthy individual and education and management of modifiable risk factors to prevent a stroke. Health promotion focuses on a healthy diet, weight control, regular excercising, no smoking, limited alcohol consumption and routine health assessments. 1. Incorrect: This is not the most effective method to prevent a stroke. Anti platelet drugs are usually the chosen treatment to prevent stroke in clients who have had a TIA. 2. Incorrect: This is not the most effective method to prevent a stroke. Transluminal angioplasty is an invasive procedure to improve blood flow. 4. Incorrect: This is not the most effective method to prevent a stroke. tPA is administered IV to reestablish blood flow through a blocked artery in a client with acute onset of ischemic stroke symptoms.)

A new nurse asks the charge nurse for assistance in interpreting arterial blood gases (ABGs) for a client. What acid/base imbalance should the charge nurse tell the new nurse these ABGs indicate in the client? pH - 7.5 PaO₂ - 94% PaCO₂ - 58 HCO₃⁻ - 35 Select all that apply 1. Metabolic acidosis 2. Respiratory alkalosis 3. Metabolic alkalosis 4. Respiratory acidosis 5. Uncompensated 6. Partially compensated 7. Fully compensated

3. Metabolic alkalosis 6. Partially compensated (3., & 6. Correct: Partially compensated metabolic alkalosis is indicated by these ABGs. The pH is 7.5 (normal 7.35-7.45) which is high, which means alkalosis. The PaCO₂ is 58 (normal 35-45) which is high. Greater than 45 is acidosis from too much CO₂. The HCO₃⁻ is 35 (normal 22-26) which is high. A high bicarb level equals alkalosis. The HCO₃⁻ matches the pH as both indicate alkalosis. The initial problem was a kidney problem or metabolic alkalosis. The lungs are trying to compensate by holding on to more acid. So the correct answer is partially compensated metabolic alkalosis. 1. Incorrect: A pH of greater than 7.45 indicates alkalosis rather than acidosis. So this option is incorrect. 2. Incorrect: The PaCO₂ would be low rather than high if the problem was respiratory alkalosis. 4. Incorrect: A pH of greater than 7.45 indicates alkalosis rather than acidosis. So this option is incorrect. 5. Incorrect: If the ABGs indicated that compensation had not begun (uncompensated) then the CO₂ would be normal. Since it is high, the lungs are attempting to compensate for the metabolic alkalosis retaining more acid (Decrease breathing to hold on to acid). 7. Incorrect: Full compensation does not occur until the pH is normal. The pH is still abnormal here.)

A client has been admitted to the med-surg floor with lower abdominal pain and bloating, fever, chills, and vomiting. Following a Cat scan, a diagnosis of diverticulitis is made. What action by the nurse is most appropriate after the initial assessment? 1. Obtain a stool specimen for ova and parasites. 2. Prepare client for emergency exploratory surgery. 3. Notify dietary the client will need a clear liquid diet. 4. Give client a heating pad to ease abdominal pain.

3. Notify dietary the client will need a clear liquid diet. (3. Correct: When diverticulum become severely inflamed or infected, an individual may need treatment in a hospital setting with intravenous antibiotics, antispasmodics and rest. In order to allow the inflammation in the gastrointestinal tract to heal, it is most important for the client to be placed on clear liquids for several days. 1. Incorrect: Diverticulitis is inflammation within the colon caused by food particles becoming lodged in the diverticulum (out-pouching). This is generally diagnosed with a CBC, Cat scan and possibly a colonoscopy. A stool specimen is ordered when the presence of ova (eggs) and parasites are suspected in the bowel, which are not present in diverticulitis. 2. Incorrect: Clients who experience an exacerbation of diverticulitis are treated medically with dietary modifications, liquids, bed rest, antibiotics and even antispasmodics. Surgery is rarely needed unless a diverticulum ruptures, causing an abscess, peritonitis or severe bleeding. 4. Incorrect: Although warmth may help to ease some mild abdominal discomfort, the use of a heating pad is never acceptable in a hospital setting. This is a safety issue for clients, with the potential for burns or electric shock. Additionally, warmth should not be utilized for lower abdominal discomfort unless specifically ordered by the primary healthcare provider.)

A middle-aged client has a strong positive family history of type 2 diabetes mellitus. What should the nurse teach the client regarding the best method to prevent or delay the development of this disease? 1. Test serum glucose values monthly. 2. Avoid starches and sugars in the diet. 3. Obtain a normal body weight and exercise regularly. 4. Maintain a normal serum lipid panel.

3. Obtain a normal body weight and exercise regularly. (3. Correct: Genetics and body weight are the most important factors in the development of type 2 diabetes mellitus. The client cannot alter his genetics. Therefore, a normal body weight is imperative. Regular exercise reduces insulin resistance and permits increased glucose uptake by cells. This serves to lower insulin levels and reduce hepatic production of glucose. 1. Incorrect: Monthly glucose monitoring is not sufficient. It will tell you when the client becomes a diabetic but will not prevent it from happening. 2. Incorrect: Starch and sugar intake should be decreased, not avoided. 4. Maintaining a normal serum lipid panel may not be achievable in some clients, but it is always the goal. Medication may be needed.)

A client reports dizziness and weakness while walking down the hall. The nurse notes the client's cardiac rhythm displayed on the telemetry monitor. What actions should the nurse take? Select all that apply 1. Have client ambulate back to bed. 2. Initiate 100% oxygen per nonrebreather mask. 3. Obtain client's blood pressure. 4. Prepare for cardioversion. 5. Auscultate lung sounds. 6. Administer nitroglycerin 1 tab SL.

3. Obtain client's blood pressure. 4. Prepare for cardioversion. 5. Auscultate lung sounds. (3. & 5. Correct: The client is dizzy and weak. This client is at risk for falling, so think safety and get the client back in bed. Use a wheelchair to accomplish this. Then obtain the client's BP. It may be low indicating poor tissue perfusion to the vital organs. One cause of premature ventricular contractions (PVCs) includes heart failure, so assess the lungs for adventitious sounds. 1. Incorrect: This client is dizzy and weak. Having the client ambulate back to the bed is a safety risk. The client could fall. 2. Incorrect: Oxygen may abate the PVCs; however, it should be initiated at 2 liters/NC rather than at 100%. Start with the least amount of oxygen that could relieve symptoms. 4. Incorrect: Cardioversion is not indicated with an underlying rhythm that is normal (NSR) with PVCs. Oxygen may decrease the PVCs. If not, medication can be administered to decrease the rate of the PVCs. 6. Incorrect: Nitroglycerin would be given if the client is experiencing chest pain or is suspected of having an MI. Get the client back in bed and provide the client with oxygen at 2 L/NC first.)

While suctioning a client's endotracheal (ET) tube, the nurse notes that the client's heart rate has gone from 78 to 44. The nurse stops suctioning the ET tube. What is the nurse's best action? 1. Deflate the ET tube cuff. 2. Have the client cough several times in a row. 3. Oxygenate the client with 100% oxygen. 4. Notify the primary healthcare provider.

3. Oxygenate the client with 100% oxygen. (3. Correct: The drop in pulse rate indicates acute hypoxia, which can be caused by suctioning. The nurse should stop suctioning and oxygenate with 100% oxygen. 1. Incorrect: There is no indication to deflate the ET cuff. Routine ET cuff deflation is not recommended. 2. Incorrect: This is a vagal maneuver that can be done to increase parasympathetic tone and decrease the conduction of the electrical impulses to the heart, usually done for treatment of supraventricular tachycardias. 4. Incorrect: The primary healthcare provider does not have to be called unless the client does not respond to oxygen.)

What signs/symptoms would the nurse expect to assess in a client diagnosed with Guillain-Barre' Syndrome? Select all that apply 1. Opisthotonos 2. Seizures 3. Paresthesia 4. Hemiplegia 5. Hypotonia 6. Muscle aches

3. Paresthesia 5. Hypotonia 6. Muscle aches (3., 5., & 6. Correct: Guillain-Barre' Syndrome is an acute, rapidly progressing, and potentially fatal form of polyneuritis. It is characterized by ascending, symmetric paralysis affecting the cranial and peripheral nerves. Signs and symptoms include paresthesia, hypotonia, areflexia, muscle aches, cramps, orthostatic hypotension, hypertension, bradycardia, facial flushing, facial weakness, dysphagia, and respiratory distress. 1. Incorrect: Opisthotonos is extreme arching of the back and retraction of the neck. This is seen with tetanus, not with Guillain-Barre' Syndrome. 2. Incorrect: Seizures can be associated with many neuromuscular problems but are not typical with Guillain-Barre' syndrome. Look for seizures with such problems as increasing ICP, infections of the brain, high fever, epilepsy. 4. Incorrect: Hemiplegia, paralysis on one side of the body, is not seen. There is a symmetric paralysis starting in the lower extremities and ascending through the body. In other words, weakness begins in the feet and progresses upward. The client gets better in reverse order.)

A client is admitted to the hospital reporting chills, fatigue and left lower leg pain for nearly a week. During initial assessment, the nurse notes wide-spread swelling and redness of left ankle in addition to a fever of 103.5° F (39.72° C). Which admission order should the nurse implement first? 1. Perform sterile wound care to lower leg. 2. Start I.V. for administration of antibiotics. 3. Place client on bedrest with left leg elevated. 4. Draw blood for serial cultures and lab work.

3. Place client on bedrest with left leg elevated. (3. Correct: Osteomyelitis is a serious inflammation of bone tissue caused when bacteria or fungus has entered the body through an open wound, an infected prosthetic, or even animal bite. Symptoms include fever, chills, nausea, and fatigue with decreased mobility in the affected extremity. The client can quickly become septic as the illness spreads through the system. Bedrest along with massive doses of antibiotics are necessary to prevent the spread of the infection, resulting in possible bone death or even amputation. 1. Incorrect: Since the nurse is in the process of admitting this client, wound care is not a priority action. There are more urgent orders to be implemented in order to stabilize the client. 2. Incorrect: Intravenous antibiotics are generally prescribed for up to six weeks, and the client may need a PICC line to continue antibiotic therapy in the home setting. While starting an I.V. line for antibiotic administration is important, this is not the most crucial first action. 4. Incorrect: Lab tests can provide valuable diagnostic information about clients with osteomyelitis. The Healthcare provider would most likely order a complete blood count (CBC) and sediment rate, expecting elevations in both. Blood cultures would also confirm whether the infection has become systemic. However, a venipuncture can wait until a more important action has been completed.)

A client who had a triple lumen catheter placed in the right subclavian vein 30 minutes ago reports chest discomfort and shortness of breath. The assessment reveals BP 92/58, HR 104, Resp 28, and unequal breath sounds over lung fields. What problem should the nurse suspect this client is exhibiting? 1. Myocardial infarction 2. Atelectasis 3. Pneumothorax 4. Pneumonia

3. Pneumothorax (3. Correct: Pneumothorax is the number one potential complication of all central venous access devices. Signs and symptoms include chest pain, dyspnea, shoulder or neck pain, irritability, palpitations, light-headedness, hypotension, cyanosis, and unequal breath sounds. 1. Incorrect: The hints point to a pneumothorax rather than an MI. The triple lumen catheter and unequal breath sounds are the biggest hints. 2. Incorrect: Atelectasis is the collapse of alveoli and is caused by a blockage of the air passages or by pressure on the outside of the lung. Examples of causes of atelectasis are mucus that plugs the airways, anesthesia, pleural effusion, prolonged bedrest with few position changes, and shallow breathing. 4. Incorrect: Pneumonia is an infection that causes inflammation of the air sacs in one or both lungs. The air sacs may fill with fluid or pus (purulent material), causing a cough with phlegm or pus, fever, chills, and difficulty breathing.)

The nurse is caring for a female client who is at risk for renal failure. The nurse has completed the initial assessment of the most recent lab results so that any concerns can be reported to the primary healthcare provider. Which assessment finding warrants further action? 1. Hemoglobin of 12 g/dl (120 g/L) 2. Hematocrit of 38% (0.38) 3. Potassium levels of 5.2mEq/L (5.2 mmol/L) 4. BUN of 15 mg/dl. (5.35 mmol/L)

3. Potassium levels of 5.2mEq/L (5.2 mmol/L) (3. Correct: Potassium is excreted primarily through the kidneys. When the kidneys are not working, potassium is being held. The normal value is 3.5 - 5.0 mEq/L (3.5 - 5.0 mmol/L). This potassium level is high and warrants further action. 1. Incorrect: This hemoglobin level is within normal limits for the female client. Normal hemoglobin values are 11.7-15.5g/dL (117-155g/L). 2. Incorrect: This hematocrit value is within the normal range for a female client. Normal hematocrit values are 35-45% (0.35-0.47). 4. Incorrect: This BUN level is within normal limits. Normal BUN levels are 10-20mg/dL (3.6-7.1mmol/L). Just because this client is in renal failure would not indicate a need to select this option.)

A nurse is caring for a client admitted to the hospital for a total hip replacement. In preparing the post-operative plan of care for this client, the nurse recognizes which goal as the highest priority? 1. Prevent complications of shock. 2. Prevent dislocation of prosthesis. 3. Prevent respiratory complications. 4. Prevent skin breakdown.

3. Prevent respiratory complications. (3. Correct: The postoperative client with a total hip replacement is at risk for thromboembolism and fat emboli which can travel to the lungs and cause respiratory distress. Without proper turning, coughing, and deep breathing, pneumonia and atelectasis may occur. So preventing respiratory complications is high on the priority list. Remember the ABCs - airway, breathing, then circulation. Preventing respiratory complications is the highest priority because of the possibility of sudden death from the complications of deep vein thrombosis and pulmonary embolism. 1. Incorrect: This client is at risk for hemorrhage and/or hematoma formation related to surgical trauma to blood vessels (the hip is a very vascular area) and use of anticoagulants or antiplatelet agents before and after surgery. So the nurse will need to monitor for shock caused by loss of volume. The nurse should monitor drains, wound dressings, and intake and output. But remember, Airway and Breathing take priority. 2. Incorrect: Dislocation of the prosthesis is another complication to worry about. It will cause pain and possible deformity and is very important, but airway is the priority. Dislocation of the hip prosthesis is related to weakness of the hip muscles, improper positioning or movement of the operative extremity, and/or noncompliance with weight-bearing limitations. 4. Incorrect: The client is at risk for skin breakdown if not turned and repositioned properly or ambulated as soon as prescribed. However, Airway is still the priority for this client.)

A clinic nurse is educating a client diagnosed with Bell's Palsy. What is the most important educational point the nurse must emphasize to the client? 1. Physical therapy will be needed to maintain muscle tone of the face. 2. Massage the face several times daily using a gentle upward motion. 3. Proper methods of closing eyelids and eye patching. 4. Acupuncture may provide great improvement in symptoms.

3. Proper methods of closing eyelids and eye patching. (3. Correct: Even though all are educational points that need to be provided to the client, this is the most important point. Keratitis, or the inflammation of the cornea, is one of the most dangerous complications for a client with Bell's palsy. As a precautionary measure, the nurse must ensure that the cornea is protected even if the eyelids will not close. 1. Incorrect: Physical therapy will be needed, however care to prevent eye injury takes priority. 2. Incorrect: This can be done once the client's facial sensitivity to touch decreases and the client can tolerate touching the face. Preventing eye injury takes priority. 4. Incorrect: Acupuncture may provide a potential small improvement in function. The priority however, is protection of the eye.)

The nurse is planning a teaching session with the family members of a client diagnosed with moderate Alzheimer's disease. Which topic is most important for the nurse to discuss? 1. Encouraging dependence on family members 2. Performing passive range of motion 3. Providing a safe environment 4. Monitoring vital signs every 8 hours

3. Providing a safe environment (3. Correct: A safe home environment is a priority. When you see a safety answer always consider it. This client has a memory deficit and may wander away, leave food on the stove cooking or burn themselves with hot water. Safety is a priority. 1. Incorrect: It is important to promote independence in self-care activities to promote dignity and autonomy. The client cannot make decisions alone but the family can give the client choices to pick from., Never promote dependency. 2. Incorrect: Active range of motion and regular exercise are encouraged, but this is not the most important topic. The stem does not mention that the client is mobility impaired. Walking is usually intact until late stages of Alzheimer's. 4. Incorrect: Focus is on cognitive and behavioral symptoms. V/S would be monitored as needed. This client is at home with family and nothing indicates the need to take the client's vital signs three times a day.)

A client admitted to a long-term care facility is legally blind and partially deaf. How would the nurse best provide for the client's safety in the event of an emergency? Select all that apply 1. Have roommate lead client out of the room to safety area. 2. Assign a specific UAP every shift to escort client to safety. 3. Research established protocols utilized by emergency groups. 4. Discuss best communication methods with client and family. 5. Plan for the supervisor to be responsible for evacuating the client.

3. Research established protocols utilized by emergency groups. 4. Discuss best communication methods with client and family. (3 & 4. Correct: When faced with a new or challenging situation involving client safety, the nurse manager should employ the Nursing Process to assess needs and collect contributing data. Asking for input from emergency preparedness groups, such as the Red Cross or FEMA, could provide ideas about assisting individuals with sensory deficits. Secondly, the nurse should discuss the situation with both client and family to determine appropriate methods of communicating with client, particularly in an emergency situation. 1. Incorrect: The responsibility for client safety should never be placed on a roommate or even family members. Staff should be accountable for client safety at all times. 2. Incorrect: Assigning a specific UAP each shift to locate and escort client to a safe area would be confusing. Changing protocols every shift creates a hazardous situation for staff and clients. 5. Incorrect: The facility supervisor is responsible for all aspects of an emergency, including activation of alarms, coordinating evacuation of staff and clients, and initiating facility emergency protocols such as closing fire doors or turning off oxygen valves. It would not be safe for the supervisor to also be responsible for a single individual.)

A client reporting right thigh pain is admitted to a local hospital with a diagnosis of deep vein thrombosis (DVT). During the admission assessment, the client develops new signs/symptoms. The nurse would be most concerned about what sign/symptom? 1. Swelling along vein of leg 2. Right foot begins to tingle 3. Restlessness 4. Warmth over affected area

3. Restlessness (3. Correct: A change in client's behavior or level of consciousness indicates possible decreased oxygenation to the brain. When there is a known DVT, the nurse would be concerned about a potential stroke from a clot that has broken off from the main thrombus. 1. Incorrect: The client has just been diagnosed with a right thigh DVT; therefore, symptoms are still evolving. It is not unusual for edema to continue to increase, though the nurse should observe carefully for additional issues. 2. Incorrect: Tingling of the foot on the affected side is expected since the affected thigh edema and the clot are compressing the circulation and nerves that extend into the foot. The nurse must monitor the situation carefully to prevent further complications. 4. Incorrect: Pain and warmth are typical early signs of a DVT. Additionally, the tissue becomes red and inflamed from the internal edema. The nurse is aware some of the client's symptoms are still evolving, so this sign is not the most alarming at this time.)

A hospitalized client diagnosed with rheumatoid arthritis is receiving IV methylprednisolone every six hours. What is the best method for the nurse to provide client safety? 1. Place "fall precautions" sign above client's bed. 2. Change the intravenous site for steroids daily. 3. Restrict any visitors with visible illnesses. 4. Put client on full contact precautions.

3. Restrict any visitors with visible illnesses. (3. Correct: Rheumatoid arthritis is an autoimmune disease that affects not only body joints but also organs of the body. Receiving methylprednisolone as treatment further suppresses the immune system, making the client even more at risk of infection. Restricting visitors with colds, respiratory problems and other infectious processes is the best method to protect the client. 1. Incorrect: The question states the diagnosis is rheumatoid arthritis, but there is no indication the client is unsteady or needs to be on "Fall Precautions". Although the client is fatigued and has brittle bones, there is no evidence the client needs assistance ambulating. A sign is not necessary. 2. Incorrect: Most facilities have policies to change an IV site at specific intervals, usually every three days. Changing the site daily exposes the client to an increased chance of infection from the invasive procedure. Steroids do not irritate veins and do not require frequent site changes. 4. Incorrect: There is no rationale for contact precautions since the client's disease process is not contagious. The main concern is to protect the client from other individuals.)

What is the nurse's first priority when treating a client with a chemical burn? 1. Attach client to a cardiac monitor. 2. Apply a sterile bandage. 3. Rinse the area with copious amounts of water. 4. Remove the client's clothing.

3. Rinse the area with copious amounts of water. (3. Correct: The first action in treating a chemical burn is to rinse the affected area with large amounts of cool water. 1. Incorrect: This is necessary with electrical burns. 2. Incorrect: This may come later, not first priority. 4. Incorrect: This can be accomplished while you are rinsing them with water.)

The nurse is caring for a client immediately following a bilateral salpingo-oophorectomy. Which position would be best for this client? 1. Fowler's 2. Modified Sims 3. Side-lying 4. Supine

3. Side-lying (3. Correct: We want to position for comfort with the knees flexed and on the side for airway. 1. Incorrect: Avoided to prevent pooling and edema in pelvis. 2. Incorrect: Partial lying on stomach is going to be painful. 4. Incorrect: Stretching out straight puts pressure on the abdomen and should be avoided.)

The nurse is caring for a client who is unresponsive during a postictal state. Which position is correct for this client? 1. Orthopneic 2. Dorsal recumbent 3. Sims' 4. Reverse trendelenburg

3. Sims' (3. Correct: Sim's is a semi prone position where the client is halfway between lateral and prone positions. Often used for enemas or other examinations of the perianal area. Sim's is used for unconscious client's because it facilitates drainage from the mouth and prevents aspiration. 1. Incorrect: Orthopneic position places the client in a sitting position with arms resting on an overbed table. It allows maximum expansion of the chest. This would not be a safe position for an unresponsive client. 2. Incorrect: Dorsal recumbent is a back lying position where the shoulders are slightly elevated on pillows. it is used after surgeries and anesthetics. 4. Incorrect: Reverse trendelenburg is where the body the body is completely straight but the head is elevated and the feet are down. This position helps with gastroesophageal reflux disease, snoring, and with some surgeries.)

A client being discharged home following hip surgery is prescribed to use a walker. While observing the client walk across the room, the nurse is most concerned when the client does what? 1. Applies shoes securely before ambulating with walker. 2. Checks walker to be certain the legs are securely locked. 3. Slides walker slowly forward when walking across the room. 4. Places walker to right of the chair after sitting down in chair.

3. Slides walker slowly forward when walking across the room. (3. Correct: The nurse is observing the client ambulate with a walker prior to discharge, to determine whether the client is using the assistive device safely. The nurse becomes concerned upon noting the client sliding the walker during ambulating. The correct use of a walker involves the client lifting and placing the walker approximately one-foot length ahead, then stepping into the non-moving walker. It is important for the walker to remain stationary when the client takes a step forward. 1. Incorrect: This action by the client is appropriate. Proper, gripping footwear should be worn by the client at all times when ambulating. This prevents the possibility of slipping and falling. There is no cause for concern with this action. 2. Incorrect: Another smart move is to verify the cross bars are securely locked before ambulating. When a walker is folded for storage, the locks are unlatched. When the walker is open, the locks must click into place to verify the device is safe for ambulating. No concerns here. 4. Incorrect: When a client sits down, the walker can be placed to either side of the chair. The most important factor is for the client to use the walker to safely maneuver into the chair rather than placing the walker aside before sitting down. Placing the walker next to the chair after being seated is appropriate.)

A nurse is caring for a client who had a total hip replacement 2 days ago. What assessment finding would be a priority concern for the nurse? 1. Small amount of red drainage on the surgical dressing. 2. Continues to report pain in hip when being repositioned. 3. Temperature of 101.8°F (38.7°C). 4. Slight swelling in the leg on the affected side.

3. Temperature of 101.8°F (38.7°C). (3. Correct: A low grade fever is normal following hip surgery but a temperature of 101.8ºF (38.7ºC) two days postoperatively is higher than the expected slight increase and should be a priority concern. The development of an infection is one of the major complications for clients following hip surgery. Therefore, fever that persists above 101ºF that is accompanied by chills, diaphoresis, or increasing drainage and odor from the incision should be reported. 1. Incorrect: A small amount of red, bloody drainage on the dressing is expected as part of the normal healing process. 2. Incorrect: Some pain during repositioning after hip surgery is normal and can generally be managed with analgesics. 4. Incorrect: Swelling in the operative leg is a normal part of the postoperative process after hip surgery. Normal swelling is lessened in the morning but tends to re-accumulate throughout the day. This can be minimized by elevating the client's legs or having the client lie down for approximately 45 minutes during the day.)

The nurse caring for a client who had a transurethral resection of the prostate (TURP) would increase the flow of the continuous bladder irrigation for which assessment data? 1. The drainage is continuous but slow. 2. The drainage is cloudy and dark yellow. 3. The drainage is bright red. 4. No drainage of urine or irrigation solution is noted.

3. The drainage is bright red. (3. Correct: Indicates blood and increasing the flow helps flush the catheter. 1. Incorrect: Continuous irrigation causes continuous drainage. 2. Incorrect: The color is noted and color from pink to amber is expected. 4. Incorrect: Indicates a possible obstruction.)

The nurse is caring for an elderly client who is approaching death and expressing intense despair and anxiety. Based on Erikson's theory, the nurse recognizes that this client's despair and anxiety would most likely be based on what? 1. An inappropriate desire for youthfulness and staying young. 2. The decision to never marry. 3. The lack of a sense of wholeness, purpose, and a life well lived. 4. The fear of experiencing a painful death.

3. The lack of a sense of wholeness, purpose, and a life well lived. (3. Correct: Older adults who view their lives as purposeful and full have an increased ability to view death as a meaningful part of life. 1. Incorrect: As people age they lose physical function and don't look as youthful as they once did. But the stage the elder adult faces is despair and anxiety regarding the life lived. 2. Incorrect: Individual choice that may or may not lead to satisfaction. 4. Incorrect: Conversely, older adults who view their lives as meaningless and full of lost opportunities view their approaching death with despair and conflict.)

A nurse is caring for a client who has chest pain. Which statement made by the client leads the nurse to suspect angina instead of a myocardial infarction(MI)? 1. I became dizzy when I stood up. 2. I was nauseated and began vomiting. 3. The pain started in my chest and stopped after I sat down. 4. The pain was not relieved after taking 3 nitroglycerine tablets.

3. The pain started in my chest and stopped after I sat down. (3. Correct: Chest pain brought on by exercise and stopped with rest is the hallmark of angina. If it were an MI, the pain would continue even with rest or position changes. 1. Incorrect: This indicates orthostatic hypotension which is not definitive for angina or MI. 2. Incorrect: Vomiting is a symptom of an MI not angina and is a bad sign related to the acute pain from the MI. This type of pain stimulates the vagus nerve, which causes the heart rate, BP and cardiac output to decrease and this is never good with a heart client. 4. Incorrect: This is the picture of MI, not angina.)

Which assessment by the nurse indicates a tension pneumothorax? 1. Sudden hypertension and bradycardia 2. Productive cough with yellow mucus 3. Tracheal deviation and dyspnea 4. Sudden development of profuse hemoptysis and weakness

3. Tracheal deviation and dyspnea (3. Correct: Yes, as pleural pressure on the affected side increases mediastinal displacement occurs with resultant respiratory and cardiovascular compromise. Symptoms of tension pneumothorax include dyspnea, chest pain radiating to the shoulder, tracheal deviation, decreased or absent breath sounds on the affected side, neck vein distention and cyanosis. 1. Incorrect: Hypoxia causes tachycardia rather than bradycardia. The client would more likely to be hypotensive due to decreased cardiac output. 2. Incorrect: Yellow mucus indicates infection, such as from pneumonia. This does not indicate a tension pneumothorax. 4. Incorrect: Profuse hemoptysis and weakness may indicate a serious condition such as a ruptured vessel, but it is not an indication of a mediastinum shift.)

A client reports excruciating paroxysmal facial pain occurring after feeling a cool breeze and drinking cold beverages. Based on this client's reports, what disorder does the nurse suspect? 1. Bell's palsy 2. Submucous cleft palate 3. Trigeminal neuralgia 4. Temporomandibular joint disorder (TMD)

3. Trigeminal neuralgia (3. Correct: Trigeminal neuralgia is an ongoing pain condition that affects the trigeminal nerve in the face. People who have this condition say the pain feels like an intense electric shock. The pain is triggered by things such as brushing teeth, washing the face, shaving, or putting on makeup. Even a light breeze against the face might trigger the onset of pain. 1. Incorrect: Bell's palsy is a condition in which the muscles on one side of the face become weak or paralyzed. It affects only one side of the face at a time, causing it to droop or become stiff on that side. It is caused by trauma to the facial nerve. 2. Incorrect: A submucous cleft palate (SMCP) results from a lack of normal fusion of the muscles within the soft palate as the fetus is developing in utero. An SMCP can include a very wide or split (bifid) uvula, translucency of the tissue along the middle of the soft palate, and a notch in the back of the hard palate. 4. Incorrect: The temporomandibular joint is a hinge that connects the jaw to the temporal bones of the skull. It allows the jaw to move up and down and side to side, so a person can talk, chew, and yawn. Pain or tenderness in the face, jaw joint area, neck and shoulders, and in or around the ear when chewing, speaking, or opening the mouth wide are signs/symptoms of TMD.)

The nurse is caring for a client diagnosed with heart failure who has developed pulmonary edema. Which findings best indicate that the client's medications are having a therapeutic effect? Blood pressure (BP) of 190/122, apical heart rate of 134, respiratory rate (RR) of 34 and urine output of 28 mL per hour. Perscriptions: Digoxin 0.125 mg IV now Furosemide 40mg IV now Select all that apply 1. Respiratory rate of 34/min 2. Blood pressure 90/50 3. Urine output of 100 mL over the last hour 4. Apical heart rate of 88/min 5. Blood pressure of 160/90

3. Urine output of 100 mL over the last hour 4. Apical heart rate of 88/min 5. Blood pressure of 160/90 (3., 4. & 5. Correct: With pulmonary edema the client will have a rapid, irregular heart beat. The heart rate has decreased to within normal range. The blood pressure has decreased, and the urine output has increased. 1. Incorrect. The respiratory rate is still too high, so the pulmonary edema has not resolved. 2. Incorrect. The blood pressure is too low. We worry when the blood pressure gets to 90 systolic, or 30mmHg off the baseline.)

A client admitted to the hospital following a fall has a history of Alzheimer's disease with apraxia. The nurse knows the client will need priority assistance with what activity? 1. Ambulating to the bathroom. 2. Understanding instructions. 3. Using utensils at mealtime. 4. Identifying objects in room.

3. Using utensils at mealtime. (3. Correct: Apraxia is a motor disorder of voluntary movements in which the individual can no longer execute purposeful activity, even though there is adequate mobility, strength, and coordination. This loss of ability to carry out previously learned movements could occur secondary to brain injury or a disease process such as Alzheimer's disease. The client has the ability to pick up utensils but is unable to use them correctly, which may affect the client's nutritional status. 1. Incorrect: Apraxia does not affect the ability to ambulate to the bathroom, although the client may not be able to follow cleanliness procedures once in the bathroom. However, there is another activity is of more concern. 2. Incorrect: The ability to understand is not affected by apraxia, which is a disorder in which the client loses the ability to perform purposeful movement. The client is still able to comprehend instructions at this point. There is another situation in which the client will need assistance. 4. Incorrect: The client is still able to identify objects in the environment; however, the diagnosis of apraxia indicates the client cannot use previously known objects correctly. Because of this situation, there is another area in which assisting the client is of more importance.)

A client tells a clinic nurse of plans to travel to Europe by plane. What tips should the nurse provide the client regarding prevention of clot formation? Select all that apply 1. Do not cross legs longer than 15 minutes at a time. 2. Get up and move around the plane every 4 hours. 3. Wear compression stockings while traveling. 4. Frequently move legs while sitting. 5. Avoid coffee while traveling.

3. Wear compression stockings while traveling. 4. Frequently move legs while sitting. 5. Avoid coffee while traveling. (3., 4. & 5. Correct: Compression stockings put gentle pressure on the leg muscles. Studies in healthy people have shown that wearing compression stockings minimizes the risk for developing DVTs after long flights. It is important for passengers to keep moving their legs to help the blood flow, even when waiting in the airport terminal. Alcohol and coffee contribute to dehydration, which can lead to thickened blood and increased risk for DVT. 1. Incorrect: Do not cross legs at all. 2. Incorrect: The client should get up and move around at least every 2 hours. When walking, the muscles of the legs squeeze the veins and move blood to the heart.)

What signs/symptoms would the nurse expect to assess in an elderly client diagnosed with acute decompensated heart failure (ADHF)? Select all that apply 1. Thick, white sputum 2. Crackles that clear with coughing 3. Wheezing 4. Orthopnea 5. Apical pulse 88/min 6. S3 gallop

3. Wheezing 4. Orthopnea 6. S3 gallop (3., 4., & 6. Correct: These S/S are commonly seen with ADHF because the heart is unable to keep up with the excess fluid in the vascular system. Wheezes indicate a narrowing of the bronchial lumen caused by engorged pulmonary vessels. Orthopnea is difficulty breathing when lying flat. The client will want to sit up rather than lie down. In older adults and the elderly with heart disease, an S3 often means heart failure. An S3 gallop is an early diastolic filling sound indicating an increase in left ventricular pressure. 1. Incorrect: The client with ADHF will have a frothy sputum with a pinkish color. 2. Incorrect: Crackles do not clear with coughing. 5. Incorrect: This is a normal pulse rate. Clients with ADHF will have tachycardia to keep blood moving in a forward direction.)

The nurse manager is making rounds in a long-term care facility and discovers an unfamiliar client standing in the hallway in a puddle of liquid. What is the nurse manager's priority action? 1. Ask client to state name and room number. 2. Find dry clothes and clean client completely. 3. Wipe up puddle of liquid and call housekeeping. 4. Contact unit charge nurse to identify the client.

3. Wipe up puddle of liquid and call housekeeping. (3. Correct: The issue in this scenario is safety. Whether the nurse manager knows this client is not important at the moment. Liquid on the floor poses a safety hazard for clients, visitors, and staff. The priority action is to remove this risk immediately before an injury occurs. 1. Incorrect: Even though the nurse does not recognize this particular client, the priority concern is not focused on identifying this individual. 2. Incorrect: While the client will certainly need clean, dry clothes, the most immediate concern is a safety issue. The nurse can summon a UAP to help the client after addressing the safety issue. 4. Incorrect: The identity of the client is not the initial priority at this time. Consider the whole picture when thinking about safety.)

A nurse educator has completed an educational program on interpreting arterial blood gases (ABGs). The educator recognizes that education was successful when a nurse selects which set of ABGs as compensated respiratory alkalosis? 1. pH - 7.4, PaCO₂ - 40, HCO₃⁻ - 24 2. pH - 7.48, PaCO₂ - 29, HCO₃⁻ - 22 3. pH - 7.44, PaCO₂ - 30, HCO₃⁻ - 18 4. pH - 7.46, PaCO₂ - 32, HCO₃⁻ - 20

3. pH - 7.44, PaCO₂ - 30, HCO₃⁻ - 18 (3. Correct: This set of ABGs indicate compensated respiratory alkalosis. The pH is normal. Both the PaCO₂ and Bicarb are abnormal, but the PaCO₂ is low. The bicarb is low to get rid of base. Compensation has occurred. 1. Incorrect: These are normal ABGs. 2. Incorrect: This set of ABGs indicates respiratory alkalosis. Compensation has not occurred as the pH is still abnormal and the bicarb is still normal. 4. Incorrect: This set of ABGs indicates partially compensated respiratory alkalosis. The pH is still abnormal. Both the PaCO₂ and HCO₃⁻ are abnormal. The pH matches the CO₂ as both are alkalotic. The reduction in bicarb shows attempt to compensate.)

Which initial arterial blood gas (ABG) results would the nurse expect on a client who has overdosed on aspirin (ASA)? 1. pH 7.54, PaCO₂ 41, PaO₂ 63, SaO₂ 91, HCO₃⁻ 36 2. pH 7.24, PaCO₂ 37, PaO₂ 83, SaO₂ 95, HCO₃⁻ 18 3. pH 7.49, PaCO₂ 30, PaO₂ 88, SaO₂ 92, HCO₃⁻ 25 4. pH 7.12, PaCO₂ 28, PaO₂ 72, SaO₂ 93, HCO₃⁻ 10

3. pH 7.49, PaCO₂ 30, PaO₂ 88, SaO₂ 92, HCO₃⁻ 25 (3. Correct: This ABG result indicates respiratory alkalosis. Aspirin stimulates the respiratory center and causes an increase in respiratory rate and depth. This causes respiratory alkalosis by blowing off CO₂ and causing the pH to increase. Losing CO₂ (acid) makes the client more alkalotic, which is reflected with an increased pH, decreased PaCO₂ and normal HCO​₃⁻. 1. Incorrect: This ABG result indicates metabolic alkalosis. The pH is high, PaCO​₂ is normal and HCO​₃⁻ is high. Normal pH is 7.35-7.45, normal PaCO​₂ is 35-45, normal HCO​₃⁻ is 22-26. 2. Incorrect: The client with an initial aspirin overdose will have a respiratory alkalosis. 4. Incorrect: This ABG also indicates metabolic acidosis. The problem in the stem would result in a respiratory problem.)

Dietary teaching has been initiated for a client newly diagnosed with acute diverticulitis. The nurse knows that further instruction is necessary when the client makes what statement? 1. "I must include a lot of fluid in my daily routine." 2. "I need to take my antibiotics at the same time daily." 3. "Rest and mild exercise are important for my recovery." 4. "Decreasing fiber in my diet can help prevent recurrences."

4. "Decreasing fiber in my diet can help prevent recurrences." (4. Correct: Diverticula are small, bulging pouches that can form in the lining of the digestive system, most often the lower portion of the colon. Diverticulitis is an inflammation in those pouches, which fill with retained material causing infection and inflammation. To diminish the chances of an exacerbation, the client is taught to increase fiber in the diet. This client's comment indicates the need for further instruction. 1. Incorrect: A client with diverticulitis needs large quantities of fluid daily to prevent dehydration and to avoid possible bowel blockage. Sluggish bowels also increase the potential for bacterial growth and a recurrent infection. This statement is accurate and does not require additional teaching. 2. Incorrect: Most clients hospitalized with diverticulitis will continue to take oral antibiotics for a period of time after discharge. Consistency with medication is always important to maintain adequate blood levels of the antibiotic. The client has made an accurate statement so no further teaching needed. 3. Incorrect: Once the client is discharged home, rest will be important to recovery. However, a mild form of daily exercise, such as short walks, can help with recovery by boosting the body's own immune system. It is obvious the client comprehends the nurse's teaching and will not require further instruction here.)

A client newly diagnosed with Celiac disease is being instructed on a gluten-free diet. What statement by the client would indicate to the nurse that further teaching is needed? 1. "I will still have occasional abdominal discomfort." 2. "I may need to take iron or vitamin supplements." 3. "I can have eggs but no wheat toast for breakfast." 4. "I should avoid fresh apples and strawberries."

4. "I should avoid fresh apples and strawberries." (4. Correct:The nurse is evaluating client statements for any lack of understanding and the need to provide further instruction. With Celiac disease, intestinal villi become inflamed whenever gluten is introduced to the gut through food intake. However, fresh fruits and vegetables do not contain gluten; therefore, fresh apples and strawberries would definitely be acceptable foods for this client. This statement by the client is inaccurate, indicating the need for further explanation by the nurse. 1. Incorrect:The client correctly acknowledges that some episodes of abdominal discomfort may still occur, since it is nearly impossible to totally eliminate gluten. Despite buying "gluten-free" products, occasionally small amounts of gluten may contaminate foods and causing symptoms to resurface. Eating in a restaurant may also be a challenge for those with Celiac disease. Because the client recognizes these possible symptoms, teaching was successful. However, the question asks for evidence the client needs further instruction. 2. Incorrect: This is an accurate statement by the client about Celiac disease. Because inflammation of the intestinal villi may lead to poor absorption of nutrients or anemia, clients may indeed need to take supplements for extended periods of time. This response does not indicate any problems with the client's comprehension of teaching. 3. Incorrect: It is important for a Celiac client to eat as healthy and diverse a diet as possible, since malnutrition occurs secondary to poor nutrient absorption in the bowel. Protein is a vital component in the diet, including such choices as eggs, dairy and beans. Those foods creating the worst symptoms include grains like wheat, rye, and barley as well as the "malt barley" used as a thickening agent in certain products. The client has precisely stated that a breakfast including eggs but minus the wheat toast would be appropriate, evidence that teaching was successful.)

A client is to be discharged following treatment for hepatitis A. The nurse knows teaching was successful when the client makes what statement? 1. "I should never eat fresh salad in a restaurant." 2. "I must wait two years before traveling abroad." 3. "I will need blood work once a month for a year." 4. "I will be able to donate blood when I am well."

4. "I will be able to donate blood when I am well." (4. Correct: Hepatitis A is a virus acquired from food or water contaminated with fecal material, causing inflammation in liver cells. Though some antibodies will remain in the blood permanently, an individual can donate blood once fully recovered from the illness. 1. Incorrect: Hepatitis A can be spread in several ways, including ingestion of food or drink that is contaminated with fecal material. However, there is no need to avoid fresh salad or vegetables from a reputable restaurant where food is appropriately and safely prepared. 2. Incorrect: Hepatitis A generally resolves within a few weeks, sometimes without any treatment. Even if the illness was acquired while traveling, there is no need to avoid overseas travel once the client has recovered. 3. Incorrect: Hepatitis A shows symptoms in 2 to 4 weeks, and the client is usually recovered within several weeks or a month. Though the primary healthcare provider may check blood work for the presence of specific antibodies, the client does not need monthly bloodwork for a year.)

The edrophonium (Tensilon) test has been prescribed for a client. Which statement by the client would indicate to the nurse that the client understands this test? 1. "This medication will be given to me as an IM injection immediately after my muscles are tired." 2. "This test will determine if I have multiple sclerosis." 3. "The test is positive if my muscles do not get stronger after injection with this medication." 4. "I will be asked to perform a repetitive movement to test my muscles."

4. "I will be asked to perform a repetitive movement to test my muscles." (4. Correct: If the primary healthcare provider suspects myasthenia gravis (MG), the client will be asked to perform a repetitive movement to test a group of muscles. 1. Incorrect: The medication is given IVP after the muscle group has become fatigued. 2. Incorrect: The edrophonium (Tensilon) test is used to diagnose myasthenia gravis. 3. Incorrect: A person tests positive for MG if their muscles get stronger after being injected with edrophonium (Tensilon).)

A renal transplant client has received discharge education. Which statement by the client indicates that further teaching is necessary? 1. "I will need to notify my primary healthcare provider if I develop a fever." 2. "I need to check my BP daily and report an increased BP." 3. "I will tell my primary healthcare provider if I become easily fatigued." 4. "I will be on steroids for 3 months, then I will not have to take them."

4. "I will be on steroids for 3 months, then I will not have to take them." (4. Correct: This statement indicates a need for further teaching. Doses of immunosupressive agents are often adjusted, but the client will be required to take some form of immunosuppressive therapy for the entire time that the client has the transplanted kidney. 1. Incorrect: Yes, fever is a sign of post transplant rejection. Additional signs and symptoms include oliguria, edema, increasing blood pressure, weight gain and swelling or tenderness over the transplanted kidney, flu-like symptoms, shortness of breath and/or fatigue. 2. Incorrect: Yes, hypertension is a sign of post transplant rejection. Additional signs and symptoms include oliguria, edema, increasing blood pressure, weight gain and swelling or tenderness over the transplanted kidney, flu-like symptoms, shortness of breath and/or fatigue. 3. Incorrect: Yes, fatigue is a sign of post transplant rejection. Additional signs and symptoms include oliguria, edema, increasing blood pressure, weight gain and swelling or tenderness over the transplanted kidney, flu-like symptoms, shortness of breath.)

The nurse instructs a client diagnosed with chronic obstructive pulmonary disease (COPD) about positions to use during times of dyspnea. Which statement by the client indicates that further teaching is needed? 1. "Lying on my side, propped up on three pillows is a good resting position." 2. "While sitting up, I will rest my elbows on my knees." 3. "If I become short of breath while walking, I will lean slightly forward and place my hands against the wall." 4. "I will open my lips and exhale quickly to decrease air trapping."

4. "I will open my lips and exhale quickly to decrease air trapping." (4. Correct: This statement made by the client indicates a lack of understanding. Clients with COPD should use slow pursed-lip breathing to increase the flow of expired air and delay the collapse of the small airways. This forced exhalation reduces air trapping. So, further teaching is necessary for this client. 1. Incorrect: High side lying will help improve gas exchange and decrease the work of breathing. 2. Incorrect: Sitting in the tripod position and sitting up while leaning over a table both allows for maximal chest expansion, which helps the client during periods of dyspnea. 3 Incorrect: Leaning against a wall allows the accessory muscles to work to facilitate breathing, thus relieving episodes of dyspnea.)

A client has been diagnosed with genital herpes. Which comment indicates understanding of the disease and prevention of the spread of the disease? 1. "I can be treated and then no one else is at risk." 2. "Using condoms will keep my sex partner from acquiring the disease." 3. "If I have no sores, I am not contagious to anyone." 4. "My sex partner should be tested because we have not always used condoms."

4. "My sex partner should be tested because we have not always used condoms." (4. Correct: The sex partner may become infected even if using a condom. The condom does not always cover all lesions. Condoms do, however, reduce the likelihood of getting/transmitting the disease. 1. Incorrect: Sex partners can acquire the disease even if no open sores are present. Treatment manages outbreaks but does not cure the disease. 2. Incorrect: Condoms decrease the risk. Abstinence is the only guaranteed way to not expose your partner. 3. Incorrect: Sex partners may get the disease even if no open sores are present; therefore, they should be tested for the disease.)

A client is to be discharged following cataract removal with lens implantation. What statement by the client indicates to the nurse that teaching has been successful? 1. "I must keep both eyes covered till my check-up." 2. "I should only have pain for about two days." 3. "I will no longer have to wear reading glasses." 4. "My vision will be blurry for a couple weeks."

4. "My vision will be blurry for a couple weeks." (4. Correct: Following cataract removal, a new lens is sutured in place, which slightly alters the corneal curve. Newer surgical approaches involve the use of a "suture-less glue" but that method is less common. Although the client's vision will eventually improve and stabilize, minor blurring may exist during the 6 to 12 week healing period. After that time, any remaining visual issues can be corrected with glasses. 1. Incorrect: Only the operative eye is protected by an eye patch during the healing process. The primary healthcare provider will remove that covering at the first post-operative checkup. Covering both eyes would pose a greater safety risk and decrease the client's self care abilities. 2. Incorrect: Pain following cataract surgery is the sign of a serious complication and should be reported to the surgeon immediately. Clients may experience a small amount of serous drainage or scratchy sensation, but should not have pain. 3. Incorrect: Implantation of a new lens causes a mild astigmatism that will be permanent. The client may still need to use corrective lenses, even if just for reading.)

An elderly client is to be ambulated for the first time following a hip replacement. The client refuses to get out of bed, indicating an extreme fear of falling. What statement by the nurse is most therapeutic? 1. "Don't be afraid because I will not let you fall." 2. "Your doctor says you must walk twice today." 3. "I'll get another nurse to help so you won't fall." 4. "What worries you most about getting out of bed?"

4. "What worries you most about getting out of bed?" (4. Correct: The nurse needs to focus on the client's psychological as well as physical needs. An open-ended question or statement encourages the client to elaborate and share concerns that the nurse needs to address. It would be inappropriate to force the client to participate in an activity that causes extreme fear and distress. 1. Incorrect: The nurse is dismissing the client's right to experience a specific emotion, rather than actively seeking the reason behind those feelings. The nurse is not utilizing appropriate communication techniques. 2. Incorrect: This tactless response focuses on the orders provided by the primary healthcare provider, rather than the client's expressed concerns. Such a comment by the nurse is non-therapeutic because it ignores the client's psychological needs. 3. Incorrect: Although the nurse offers a solution to the client, there is no chance for the client to verbalize feelings and concerns. It is more important to present the client with the therapeutic opportunity to discuss fears.)

A client has received 850 mL of an isotonic solution intravenously in less than 60 minutes. Which central venous pressure (CVP) reading noted by the nurse indicates a problem related to the amount of intravenous fluids infused? 1. 1 mm of Hg 2. 3 mm of Hg 3. 6 mm of Hg 4. 10 mm of Hg

4. 10 mm of Hg (4. Correct: Normal CVP is 2-6 mmHg. This client has received an isotonic solution amount of time. Remember that isotonic fluids stay "where I put them". The vascular space will increase in volume. More volume, more pressure. 1. Incorrect: This CVP reading indicates fluid volume deficit. There is no indication in the stem that the client is losing fluid. 2. Incorrect: This is a normal CVP reading. Normal CVP is 2 to 6 mm of Hg. 3. Incorrect: This is a normal CVP reading. Normal CVP is 2 to 6 mm of Hg.)

The head nurse on a busy surgical unit is evaluating several fresh post-operative clients. Which observation should the nurse report immediately to the primary healthcare provider? 1. A post transurethral resection client with cherry colored urine 2. A post mastectomy client drains 40 mL of bloody drainage within 3 hours of the surgery 3. A post ileostomy client with a beefy red stoma and mucus drainage 4. A post thyroidectomy client reporting tingling in toes and fingers

4. A post thyroidectomy client reporting tingling in toes and fingers (4. Correct: One potential risk during a thyroidectomy is the accidental removal of some or all of the parathyroid glands. The client would develop signs and symptoms of hypocalcemia from decreased blood levels of calcium. As muscles become rigid and twitch, the resulting tetany would cause the client to experience a tingling sensation in toes and fingers. The nurse needs to notify the primary healthcare provider so that a calcium level can be drawn and the client given supplemental calcium. 1. Incorrect: Following a transurethral resection of the prostate (TURP), it is normal for urine to be cherry red in color. Continuous bladder irrigation will keep clots from developing over several days and the deep red color of urine is an expected finding following this procedure. 2. Incorrect: Mastectomy clients return from surgery with one or more drains placed under skin flaps in the breast tissue. These drains are part of a collection system that allows serous drainage to be removed from the surgical site, thus enhancing the healing process. Because this client has just returned from surgery, 40 mL over 3 hours is not an excessive amount. This is an expected finding that does not need to be reported. 3. Incorrect: The sign of a healthy stoma immediately post-op is a beefy red appearance and slightly elevated above the level of the abdomen. It is expected that the stoma will have a mucoid discharge for a day or so until normal stool begins to form again. This client displays normal post-operative findings with nothing unexpected.)

A client is admitted for management of ulcerative colitis. What sign/symptom would be of immediate concern to the nurse? 1. Tenesmus 2. Hyperactive bowel sounds 3. Ten bloody diarrhea stools in 8 hours 4. Abdominal guarding

4. Abdominal guarding (4. Correct: Guarding is a completely involuntary response of the muscles. In other words, you have no control over it. It's a sign that your body is trying to protect itself from pain. It can be a symptom of a very serious and even life-threatening medical condition. 1. Incorrect: Tenesmus is the urge to move your bowels even if you've just emptied your colon. This is a common symptom of an ulcerative colitis flair and would not be of immediate concern to the nurse. 2. Incorrect: Hyperactive bowel sounds can mean there is an increase in intestinal activity. This may happen with diarrhea or after eating. This client has ulcerative colitis so hyperactive bowel sounds during a flare is expected. 3. Incorrect: The colon is ulcerated and unable to absorb water, so 10-20 bloody diarrhea stools are the most common symptom of ulcerative colitis and would not be of immediate concern to the nurse.)

A client with end-stage kidney disease (ESKD) is admitted to the hospital with shortness of breath and has a serum potassium level of 5.5 mEq/L (5.5 mmol/L). What would be the priority nursing action? 1. Report the client has uremic fetor to the charge nurse. 2. Assign the client to the room closest to the nurse's station. 3. Provide the client with assistance with dressing and bathing. 4. Administer sodium polystyrene sulfonate 15 grams/60mL by mouth.

4. Administer sodium polystyrene sulfonate 15 grams/60mL by mouth. (4. Correct: Hyperkalemia is the most immediate life-threatening issue at this time and must be addressed. 1. Incorrect: Uremic fetor is a gastrointestinal sign and symptom of end-stage kidney disease that should be reported but it is not the first priority. 2. Incorrect: Disorientation is a sign and symptom of end-stage kidney disease but the stem does not indicate the client is disoriented at this time so a room closest to the nurse's station is not the priority. 3. Incorrect: The client may need assistance with bathing and dressing but this is not the priority at this time.)

The emergency room nurse is assessing a client reporting severe abdominal pain for several hours prior to arrival at the hospital. Assessment findings include slight mottling of the lower extremities and a pulsating mass near the umbilicus. Which actions should the nurse implement immediately? Select all that apply 1. Position client on the left side. 2. Apply warm blankets to legs. 3. Administer I.M. pain medication. 4. Alert the operating room staff. 5. Notify the primary healthcare provider. 6. Palpate mass to determine size.

4. Alert the operating room staff. 5. Notify the primary healthcare provider. (4. & 5. Correct: The client's symptoms indicate the presence of an aortic abdominal aneurysm that may be dissecting (rupturing) at this time. This is a life-threatening emergency and the client will need urgent surgery to survive. The nurse should immediately notify the healthcare provider and alert the operating room staff of impending surgery. 1. Incorrect: These are the classic symptoms of a dissecting abdominal aneurysm, a life-threatening situation requiring immediate surgery. Positioning the client on either side is contraindicated as that action may cause further internal bleeding, complete rupture of the aneurysm, or death. 2. Incorrect: Mottling of lower extremities accompanied by severe abdominal pain suggests a dissecting abdominal aneurysm. The discoloration of lower extremities indicates compromised circulation secondary to interrupted blood flow because of the aneurysm. This client would not benefit from warm blankets but rather needs immediate surgery to survive. 3. Incorrect: Pain medications in general are not administered until an exact diagnosis is confirmed, since relieving pain would mask those signs or symptoms needed to verify the problem. While the client may be given medications at some point, this is not the life-saving action the nurse must take immediately. 6. Incorrect: The client's symptoms are suggestive of a dissecting abdominal aneurysm, a life-threatening emergency requiring immediate surgical intervention. It is never acceptable for the nurse to palpate an abdominal mass, particularly a pulsating mass, since this would likely cause complete rupture of the blood vessel and immediate death.)

The charge nurse is making assignments for the evening shift. Which client would be an appropriate assignment for a new LPN/VN graduate? 1. A middle aged adult admitted with syncope. 2. An adolescent with skin grafts to right hand. 3. A young adult receiving IV chemotherapy. 4. An elderly adult diagnosed with diverticulitis.

4. An elderly adult diagnosed with diverticulitis. (4. Correct: Diverticulitis is a bowel disorder of undetermined origin, characterized by pain to the lower abdomen, along with bloating, fever and diarrhea. Treatment may include hospitalization, antibiotics, liquid diet and bedrest. Because there is usually no bleeding involved, this would be an appropriate assignment for a new LPN/VN graduate. 1. Incorrect: A new admission requires assessments that must be completed by a registered nurse. Additionally, syncope could be an indication of a serious cardiac issue; therefore, this would not be a client suitable for the LPN/VN. 2. Incorrect: Although sterile wound care is within the scope of practice for an LPN/VN, skin grafts require special assessment techniques during dressing changes, in order to determine quality of tissue perfusion and potential for rejection. This client should be assigned to a more experienced nurse. 3. Incorrect: This client will need extensive assessments of multiple body systems while receiving I.V. chemotherapy, requiring an experienced registered nurse with knowledge of both chemo drugs and different types of cancer. This client would not be appropriate for a new LPN/VN.)

The postanesthesia care unit has received several postoperative clients. While encouraging the clients to cough and deep breathe, the nurse realizes that coughing poses the greatest risk to which client? 1. A female with an abdominal hysterectomy 2. A male who had a right upper lobectomy 3. An adolescent with an open appendectomy 4. An elderly client who had cataract removal

4. An elderly client who had cataract removal (4. Correct: Cataract removal involves replacing the eye's lens with an artificial lens that is permanently sutured into place. Coughing would increase intraocular pressure in this client and risk dislodging the lens and eye sutures. The nurse needs to monitor and prevent additional potentially harmful actions such as sneezing, vomiting, bending over, or straining. 1. Incorrect: Coughing is a potential complication following any surgery in which there are external sutures. However, a priority post op action is to encourage the client to cough and deep breathe to prevent possible atelectasis; therefore, the nurse must instruct this client to splint the lower abdomen prior to taking deep breaths and coughing. This client is not at the highest risk. 2. Incorrect: Any type of thoracic or cardiac surgery places the client at high risk for pneumonia or atelectasis. Deep breathing is particularly important for this client. The nurse will definitely need to encourage this client to cough and deep breathe while assisting with splinting the surgical incision. This client would be at a greater risk by not coughing. 3. Incorrect: An open appendectomy indicates this was not done laparoscopically, but rather that this client will have abdominal sutures. Despite this fact, the client must be encouraged to cough and deep breathe to prevent complications. The nurse will instruct the client to splint the lower abdomen to protect the suture line while coughing and deep breathing.)

The nurse is caring for a client who has been intubated and placed on a ventilator. The nurse hears the ventilator alarm and enters the client's room to find the high pressure alarm sounding. The client is very agitated with a respiratory rate of 40/min, arterial BP 98/48, oxygen saturation 82%, cardiac monitor showing sinus tachycardia at 138 bpm. What action should the nurse take first? 1. Turn off alarm, then check ventilator settings. 2. Increase FiO₂ settings to 100%. 3. Hyperventilate client. 4. Auscultate lung sounds.

4. Auscultate lung sounds. (4. Correct: When an alarm sounds, the first action by the nurse is to assess the client. In this situation, assessment of lung sounds, chest movement, and respiratory effort should indicate which respiratory complication the client may be experiencing. Depending on the assessment findings, the other actions may be necessary. 1. Incorrect: Depending on the assessment findings, the other actions may be necessary. Check the ventilator after checking the client. 2. Incorrect: Depending on the assessment findings, the other actions may be necessary. 3. Incorrect: Depending on the assessment findings, the other actions may be necessary.)

The nurse is caring for a client who has been intubated and placed on a ventilator. The nurse hears the ventilator alarm and enters the client's room to find the high pressure alarm sounding. The client is very agitated with a respiratory rate of 40; arterial line BP 98/44; oxygen saturation 82%; cardiac monitor sinus tachycardia at 138. What action should the nurse take first? 1. Turn off alarm, then check ventilator settings. 2. Increase FiO₂ setting to 100%. 3. Hyperventilate client, then suction ET tube. 4. Auscultate lung sounds.

4. Auscultate lung sounds. (4. Correct: When an alarm sounds, the first action by the nurse should be to assess the client. In this situation, assessment of lung sounds, chest movement, and respiratory effort should indicate which respiratory complication the client may be experiencing. Depending on the assessment findings, the other actions may be necessary. Excessive positive pressure can result in lung complications, including a pneumothorax. This could quickly progress to a tension pneumothorax. Therefore, the nurse should consider any sudden changes in oxygen saturations and signs of respiratory distress as life threatening. Immediate assessment of the client is warranted with actions taken based on the findings. 1. Incorrect: Depending on the assessment findings, this action may be necessary. Check the ventilator after checking the client. The ventilator is checked following the client assessment to assure that it is working properly and that the settings are appropriate. 2. Incorrect: Depending on the assessment findings, this action may be necessary. The goal of mechanical ventilation is optimal gas exchange by maintaining oxygen delivery and alveolar ventilation. The lowered oxygen saturation could be the result of the underlying illness, but since there was an abrupt change, mechanical factors should be considered as well. The nurse would need to institute other measures to promote gas exchange in addition to consideration of increasing the fraction of inspired oxygen. 3. Incorrect: Depending on the assessment findings, this action may be necessary. Ventilation use increases the production of secretions, regardless of the initial reason for ventilating support. The client must be assessed first for the presence of secretions by auscultating the lungs bilaterally. If excessive secretions are present, suctioning should be performed with caution to prevent damage to the airway mucosa.)

A client, who is connected to a cardiac monitor, is found unresponsive, apneic, and pulseless. What action should the nurse initiate first? 1. Administer Atropine 1mg IVP. 2. Prepare for transcutaneous pacing. 3. Defibrillate at 200 joules. 4. Begin cardiopulmonary resuscitation (CPR).

4. Begin cardiopulmonary resuscitation (CPR). (4. Correct: The key here is that the client is apneic and pulseless. In other words, the client is dead. There is no cardiac output. Start CPR. 1. Incorrect: Atropine can be given, but the first action is to begin CPR. 2. Incorrect: Transcutaneous pacing can be done, but this client is dead. Perform CPR until pacing can be established. 3. Incorrect: Defibrillating this ventricular rhythm will wipe out the rhythm all together which could lead to asystole.)

The nurse is teaching a community education class on alternative therapies. Which alternative therapy that uses substances found in nature should the nurse include? 1. Energy therapies. 2. Mind-body interventions. 3. Body-based methods. 4. Biologically-based therapies.

4. Biologically-based therapies. (4. Correct: Biologically-based therapies use substances found in nature such as herbs, foods, and vitamins. 1. Incorrect: Energy therapies use energy fields. Substances found in nature are biologically-based therapies. 2. Incorrect: Mind-body interventions use the mind to help affect the function of the body. Substances found in nature are biologically-based therapies. 3. Incorrect: Body-based methods use movement of the body. Substances found in nature are biologically-based therapies.)

An Orthodox Jewish client receives the following lunch tray. What is the nurse's priority action? Lunch Menu: Spaghetti and meatballs in sauce Tossed salad with vinegrette dressing Hot rolls with butter Dessert: Fruit cocktail or cookie Milk - coffee available 1. Nothing, since this is a healthy and acceptable lunch. 2. Ask the client to eat the lunch so food is not wasted. 3. Remove the tossed salad so the client can eat the other foods provided. 4. Call dietary to immediately make a new tray for the client.

4. Call dietary to immediately make a new tray for the client. (4. Correct: One Jewish religious belief contends that dairy and meat cannot be served or eaten at the same meal. The tray includes meatballs in the spaghetti and milk served with the meal. Nothing on this tray could be consumed by the client, and an entirely new tray must be prepared immediately. 1. Incorrect: The "healthiness" of the lunch is not the issue. The Jewish faith contends that milk and meat can neither be eaten nor served at the same meal. Because both items are on the same tray, this meal would not be acceptable for this client. 2. Incorrect: Because milk and meat have been placed on the same tray, the client cannot eat anything from this contaminated meal. Expecting the client to eat anything from this tray is not acceptable. 3. Incorrect: The issue is not the tossed salad. The problem is that dairy (milk) has been served along with a meat product at the same meal. This is not acceptable for those of the Jewish faith, and the entire tray must be removed from the client's room.)

An 18 year old football player is admitted to the ortho unit after a femur fracture. He is scheduled for a rod to be placed in the morning, but suddenly develops severe shortness of breath, a petechial rash on his chest, and unstable vital signs. What should the nurse do first? 1. Decrease rate of IV fluids. 2. Neurovascular checks of affected leg. 3. Elevate the head of the bed. 4. Call the active response team.

4. Call the active response team. (4. Correct: The client is exhibiting symptoms of a fat embolism, particularly with the petechial rash on his chest and severe shortness of breath. Due to his age, high risk behaviors with contact sports, and the large long bone fracture, he is the classic example of a client that may experience a fat embolus. This constitutes a medical emergency and activation of the response team. 1. Incorrect: This does not affect breathing here and will do nothing to resolve the fat embolism. 2. Incorrect: Neurovascular checks of the leg will not help the client's breathing and are not the first priority for the nurse. 3. Incorrect: The nurse may elevate HOB to assist with breathing unless client is hypotensive. Either way, this is not the best first answer.)

The nurse is caring for a post op client who is drowsy but arousable. The client will take a few deep breaths when instructed but drifts to sleep when left alone. The O₂ saturation while sleeping drops to 82% on 3 liters of nasal oxygen. The client received a dose of oxycodone/acetaminophen 2 tabs one hour ago. What is the nurse's best action at this time? 1. Keep the O₂ sat machine at the bedside and set the alarm to beep loudly when O₂ sat drops below 93%. 2. Give bath to arouse client and then report that oxycodone/acetaminophen 2 tabs is too much for next dose. 3. Let the client sleep until he has rested, then discuss abuse potential of narcotics. 4. Call the primary healthcare provider and report client assessment findings.

4. Call the primary healthcare provider and report client assessment findings. (4. Correct: Yes, this client has unstable respirations and is in respiratory distress. The client needs naloxone, the antidote for narcotic overdose. Since that is not an option, you need to call the primary healthcare provider to get a prescription for the antidote. 1. Incorrect: That will work the first time, but the client is too sedated to remain awake and take deep breaths. The client will continue to have respiratory distress until naloxone can be given. 2. Incorrect: No, that won't fix the problem of too much medication. We need to fix the problem now. 3. Incorrect: No, client is too sedated. Naloxone is needed, so the nurse needs to notify the primary healthcare provider.)

While preparing an information sheet for a client diagnosed with a vancomycin-resistant enterococcus (VRE) urinary tract infection (UTI), the home health nurse should include which instructions? 1. Wash hands with hot water and soap when hands are soiled. 2. Gloves are not needed in the home since contamination with VRE has already occurred. 3. Wash hands before using the bathroom and after preparing food. 4. Clean the bathroom and kitchen with warm water and bleach.

4. Clean the bathroom and kitchen with warm water and bleach. (4. Correct: The bathroom and kitchen should be cleaned with warm water and bleach to decrease contamination. The client should wash hands after using the bathroom and before preparing food. 1. Incorrect: Instructing the client and family to wash with hot water can cause drying and cracking of the skin. Hands should be washed with all contacts. Washing hands is the single most important thing to do to prevent infection. 2. Incorrect: Gloves are needed with VRE to prevent spread of infection. Gloves are especially needed if contact with blood or other infectious materials is anticipated. 3. Incorrect: Hands should be washed after using the bathroom and prior to handling or preparing food.)

Which signs and symptoms would concern the nurse if assessed in a client post radical neck surgery? Select all that apply 1. Bradypnea 2. Flaccid muscle tone 3. Flushed and warm skin 4. Positive Trousseau's sign 5. Leg cramps 6. Decreased deep tendon reflexes

4. Positive Trousseau's sign 5. Leg cramps (4., & 5. Correct: A positive Trousseau's sign indicates that muscles are rigid and tight due to a low calcium level. Some of the parathyroids could have been removed resulting in hypocalcemia. Hypocalcemia will cause muscle twitching and painful muscle cramps. 1. Incorrect: The respiratory rate will decrease with hypermagnesemia and hypercalcemia. 2. Incorrect: Weak, flaccid muscle tone is seen with hypercalcemia. 3. Incorrect: Flushed and warm skin would be seen with hypermagnesemia due to vasodilation. 6. Incorrect: Decreased deep tendon reflexes would occur with hypermagnesemia or hypercalcemia.)

A client is admitted for treatment of fluid volume excess. The nurse reviews the admitting lab work and the primary healthcare provider's prescriptions. Which prescription would be of concern to the nurse? Sodium: 138 mEq/L (138 mmol/L) Potassium: 5.4 mEq/L (5.4 mmol/L) Calcium: 9.0 mg/dL (2.25 mmol/L) Glucose: 108 mg/dL (6 mmol/L) Orders: Bedrest 2 gram Na diet Spironolactone 25 mg by mouth once per day Potassium Chloride (KCl) 20 mEq by mouth twice a day 1. Bedrest 2. 2 gram Na diet 3. Spironolactone 4. Potassium Chloride (KCl)

4. Potassium Chloride (KCl) (4. Correct: The client has been prescribed spironolactone, a potassium sparing diuretic, so KCl supplement is not necessary. 1. Incorrect: Bed rest induces diuresis, which is good for this client. 2. Incorrect: This client needs to be on a low sodium diet to reduce fluid retention. 3. Incorrect: Spironolactone is a potassium sparing diuretic which can be prescribed for this client.)

The nurse receives the morning lab results of four clients during the change of shift report. Which client should the nurse assess first? 1. Vomiting and diarrhea with a potassium 3.3 mEq/L (3.3 mmol/L). 2. One day post-operative hip replacement with a Hct 30% (0.30) / Hgb 10 g/dL (100 g/L). 3. Pneumonia with a White Blood Cell (WBC) count of 12,000/mm³ (12 × 10⁹/L). 4. Diabetes with a Fasting Blood Sugar (FBS) of 40 mg/dL (2.2 mmol/L).

4. Diabetes with a Fasting Blood Sugar (FBS) of 40 mg/dL (2.2 mmol/L). (4. Correct: This is a critical value. The client who has a potential life-threatening problem should be assessed first. 1. Incorrect: The potassium level is low but not critical. Below 2.5mEq/L (2.5 mmol/L) is critically low potassium. 2. Incorrect: The hematocrit and hemoglobin are low but not critical. It is expected for the hemoglobin and hematocrit to drop after surgery. 3. Incorrect: The WBC count is high but not critical. A critically high value would be greater than 30,000/mm³ (12 × 10⁹/L).

A client presenting at the clinic has a history of systemic lupus erythematosus (SLE). Which finding would indicate to the nurse that the client is having a flare-up of the disease? 1. Alopecia 2. Arthritis of hands 3. Weight gain 4. Fever

4. Fever (4. Correct: Fever is the classic sign of a flare, or exacerbation of SLE. 1. Incorrect: Lupus can cause the hair on your scalp to gradually thin out, although a few people lose clumps of hair. 2. Incorrect: Most SLE clients will develop arthritis with their illness. Arthritis from SLE commonly involves swelling, pain, stiffness, and even deformity of the small joints of the hands, wrists, and feet. Sometimes, the arthritis of SLE can mimic that of rheumatoid arthritis, another autoimmune disease. 3. Incorrect: Weight gain is not a sign of exacerbation but is a side effect of long-term corticosteroid use.)

A community health nurse is assessing a migrant farmer who raises chickens. The nurse notes the client has developed a cough, fever, dyspnea, and hemoptysis. What infection should the nurse suspect? 1. Lyme disease 2. Toxoplasmosis 3. Tuberculosis 4. Histoplasmosis

4. Histoplasmosis (4. Correct: Histoplasmosis is a fungal infection transmitted through ingestion of soil contaminated by bird manure. 1. Incorrect: The classic symptom of Lyme disease is usually an expanding target-shaped or "bull's-eye" rash which starts at the site of the tick bite. Fever, headache, muscle aches, and joint pain may also occur. 2. Incorrect: Toxoplasmosis occurs from contact with cat feces. Symptoms may be influenza-like: swollen lymph nodes, headaches, fever, and fatigue, or muscle aches and pains. 3. Incorrect: TB is often suspected; however, the primary difference is exposure to bird feces.)

A client who has a long leg cast is reporting unrelieved pain. What should the nurse do first? 1. Apply a cool compress. 2. Elevate and reposition the leg. 3. Assess for breakthrough bleeding on the cast. 4. Monitor extremity for paresthesia.

4. Monitor extremity for paresthesia. (4. Correct: The worry here is compartment syndrome. Do a complete neurovascular assessment. Remember the 5 Ps: pain, pallor, pulse, paresthesia, and paralysis. 1. Incorrect: A cool compress may be helpful to decrease swelling but monitoring for compartment syndrome takes priority. 2. Incorrect: The nurse should elevate the extremity to decrease swelling, but it is not the priority over assessing neurovascular status. 3. Incorrect: The nurse should assess for breakthrough bleeding on the cast but this is not a priority over neurovascular assessment.)

Standard orders on the nurse's unit include an intravenous infusion of 0.45 NS 1000 mL with 20 mEq (20 mmol/L) potassium chloride to run at 100 mL per hour. This IV solution would be appropriate for which client diagnosis? Select all that apply 1. Addisonian crisis 2. Hypertension 3. Chronic renal failure 4. Cushing's disease 5. Hypokalemia

4. Cushing's disease 5. Hypokalemia (4. & 5. Correct: Clients with cramping, Cushing's disease, and hypokalemia are safe to receive normal saline with potassium chloride. 1. Incorrect: Clients with Addison's disease can have hyperkalemia if they experience an Addisonian crisis due to lack of aldosterone. When aldosterone is not secreted, sodium and water is released and potassium levels elevate in response to the hyponatremia. 2. Incorrect: Clients with hypertension need a hypotonic solution which will not increase the client's blood pressure. However, the client may or may not have hypokalemia. 3. Incorrect: Clients in chronic renal failure are retaining fluid and potassium. They do not need more potassium.)

A 68-year-old client with a history of angina presents to the emergency department (ED) reporting flu like symptoms progressively worsening over the past 24 hours.What action is most important for the nurse to initiate? "I have the flu. I have been vomiting every couple of hours, running a fever and my chest hurts." Pulse-132 beats/minute Respirations-26 breaths/minute Blood pressure-94/60 mmHg Temperature-101.3° F (38.5°C) orally Capillary refill - 4 seconds 1. Administer acetamenophen. 2. Initiate IV of Normal Saline at 250 mL/hour. 3. Notify radiology and lab of diagnostic test prescriptions. 4. Discuss IV prescription with primary healthcare provider.

4. Discuss IV prescription with primary healthcare provider. (4. Correct: This client needs fluid because of dehydration, but did you notice that this client is elderly and has a history of cardiac problems? I hope so, because giving this client NS rapidly could throw our heart client into pulmonary edema, which would be a bad thing! Talk to the primary healthcare provider. 1. Incorrect: Acetaminophen needs to be administered but it is not the most important thing for the nurse to do. Clarification regarding the IV fluid prescription is necessary here to prevent a possible complication. 2. Incorrect: If this client receives an isotonic IV solution at this rapid rate, the client will be at increased risk of developing FVE and pulmonary edema. 3. Incorrect: Again, the radiology and lab departments can be notified of the test prescriptions to be completed. However, the nurse can assign this task to the unit secretary.)

What assessment finding would alert the nurse that a client's open pneumothorax has progressed to a tension pneumothorax? Select all that apply 1. Subcutaneous emphysema 2. Shortness of breath 3. Tachypnea 4. Distended neck veins 5. Hypotension

4. Distended neck veins 5. Hypotension (4., & 5. Correct: As pressure builds inside the chest cavity, a tension pneumothorax can develop. Symptoms that are commonly seen with a tension pneumothorax include distended neck veins, distant heart sounds, and hypotension. Now, why does the tension pneumothorax client experience distended neck veins? Because the pressure is building up in the chest causing pressure on the right side of the heart. This is a bad thing! There is decreased venous return and this can lead to cardiovascular collapse. Because of the compromised or obstructive blood flow, cardiac output decreases and hypotension develops. 1. Incorrect: You would expect to see subq emphysema with an open pneumothorax and can still be seen with a tension pneumothorax. 2. Incorrect: SOB is expected with open pneumothorax and tension pneumothorax. 3. Incorrect: Tachypnea would be seen with both.)

Which risk factor should the nurse include when planning to educate a group of women about breast cancer? 1. Menopause before the age of 50 2. Drinking one glass of wine daily 3. Multiparity 4. Early menarche

4. Early menarche (4. Correct: Early menarche before age 12 is a known risk factor for breast cancer. The increased risk of breast cancer linked to a younger age at first period is likely due, at least in part, to the amount of estrogen a woman is exposed to in her life. A higher lifetime exposure to estrogen is linked to an increase in breast cancer risk. The earlier a woman starts having periods, the longer her breast tissue is exposed to estrogens released during the menstrual cycle and the greater her lifetime exposure to estrogen. 1. Incorrect: Studies show women who go through menopause after age 50 have increased risk of breast cancer. The risk for breast cancer increases as time period between menarche and menopause increases. 2. Incorrect: Small increase in risk with moderate alcohol consumption, not one glass of wine daily. Drinking low to moderate amounts of alcohol, however, may lower the risks of heart disease, high blood pressure and death. But, drinking more than one drink per day (for women) and more than two drinks per day (for men) has no health benefits and many serious health risks, including breast cancer. Alcohol can change the way a woman's body metabolizes estrogen (how estrogen works in the body). This can cause blood estrogen levels to rise. Estrogen levels are higher in women who drink alcohol than in non-drinkers. These higher estrogen levels may in turn, increase the risk of breast cancer. 3. Incorrect: Nulliparity (no pregnancies) is a known risk factor for breast cancer. Factors that increase the number of menstrual cycles also increase the risk of breast cancer, probably due to increased endogenous estrogen exposure.)

The client needs assistance to apply anti-embolism stockings each day in the long-term care facility. Today, as the nurse enters the room to apply the stockings, she finds that the client has been walking about the unit for 30 minutes. What should the nurse do first to lessen the risk of swelling of the lower extremities? 1. Ask the client to lie down and place the stockings on the legs. 2. Ask the client to sit on the bedside and place the stockings on the legs. 3. Tell the client that the nurse will return later to assist with the application. 4. Elevate the extremities in bed for 30 minutes before application.

4. Elevate the extremities in bed for 30 minutes before application. (4. Correct: The client should have extremities elevated to encourage venous return and reduce the risk of swelling before the stockings are applied. 1. Incorrect: To place the stockings on immediately will cause further venous stasis and swelling. 2. Incorrect: The extremities should be elevated for a period of time before application. 3. Incorrect: This instruction alone does not give the client adequate information about the need to keep the lower extremities elevated before applying the stockings.)

The nurse is providing care to a client who has returned to the long-term facility following cataract surgery. Which finding would indicate a possible complication? 1. Slightly swollen eyelid 2. Slight discomfort of the eye 3. "Bloodshot" appearance of the eye 4. Extreme pain in the eye

4. Extreme pain in the eye (4. Correct. The postoperative cataract client usually experiences little to no pain, and it can be managed with mild analgesics. If the pain is severe, there may be an increase in intraocular pressure, hemorrhage, or infection, and the surgeon should be notified. 1. Incorrect. Slight swelling of the eyelid is considered a normal finding following cataract surgery. 2. Incorrect. The postoperative cataract client usually experiences little to no discomfort following surgery. This is a normal finding. 3. Incorrect. Slight redness is an expected finding. Pay attention to the word "slight". Increased redness is cause for concern. Compare it to the non-operative eye.)

Following a hemorrhagic stroke, a client had a craniotomy with insertion of a ventriculostomy. Upon arrival in the ICU, the nurse's initial readings indicate an increase in intracranial pressure (ICP). What is the nurse's priority action? 1. Position client on the right side. 2. Call the primary healthcare provider. 3. Lower the head of the bed immediately. 4. Hyperventilate client with a bag valve mask.

4. Hyperventilate client with a bag valve mask. (4. Correct: A ventriculostomy is a temporary drain placed in the brain to remove excess cerebral spinal fluid in order to decrease intracranial pressure. Because the client's ICP readings are increasing, the nurse's initial action is to try to reduce that pressure by hyperventilating the client with a bag valve mask, also called an Ambu bag or manual resuscitator. This lowers cerebral CO₂ levels, causing vasoconstriction which temporarily decreasing blood flow and reducing pressure within the brain. 1. Incorrect: Clients experiencing increased intracranial pressure must be positioned in a neutral position, head midline and slightly elevated, generally with sandbags or immobilizers on either side of the skull to maintain that position. This allows for optimal drainage of cerebral spinal fluid (CSF) from the ventriculostomy. 2. Incorrect: The primary healthcare provider or surgeon will indeed need to be notified. However, the nurse's initial action is always focused on stabilizing the client if possible. In this case, the nurse can intervene prior to calling the primary healthcare provider. 3. Incorrect: Lowering the head of the bed is contraindicated following brain surgery since it increases blood flow to the brain, thus increasing intracranial pressure. The ideal position is head slightly elevated at 30 to 45 degrees with head immobilized in the midline position to improve drainage of CSF.)

What electrolyte imbalance should the nurse monitor for in a client diagnosed with hyperosmolar hyperglycemic state (HHS)? 1. Hypocalcemia 2. Hypermagnesemia 3. Hyperkalemia 4. Hyponatremia

4. Hyponatremia (4. Correct: Hyperglycemia can cause dilutional hyponatremia, so Normal Saline is administered to replace both fluid and sodium lost through increased urinary output. 1. Incorrect: Calcium is not affected in the client who is in HHS. 2. Incorrect: HHS does not specifically cause hypermagnesemia. We do know that magnesium is lost through the kidneys, so hypomagnesemia is possible with uncontrolled diabetes. 3. Incorrect: Serum potassium levels are usually normal when the client arrives with HHS. The potassium will drop as the large volume of NS is administered with IV insulin. Then we worry about hypokalemia.)

A client admitted with a diagnosis of end stage kidney disease (ESKD) has been prescribed a diet containing no more than 1 gram of phosphate per day. Which food item, if found on the client's meal tray should be removed by the nurse? 1. Skinless chicken breast 2. Green beans 3. Asparagus 4. Ice cream

4. Ice cream (4. Correct: Ice cream, a milk product is high in phosphate. 1. Incorrect: Fresh or frozen red meats without breading, marinades or sauce are better choices for a kidney diet. On average, fresh meat contains 65 mg of phosphorus per ounce and 7 grams of protein per ounce. A 3 ounce serving would have 150 mg of phosphorus. 2. Incorrect: Green peas, green beans and wax beans are low in phosphate, whereas black-eyed peas, lima beans, kidney beans, pinto beans, and lentils are high in phosphate. 3. Incorrect: A serving of asparagus has approximately 20 mg of phosphorus.)

The nurse checks the results of a urinalysis performed on a client with dehydration. Which results should the nurse expect to find? 1. Increased white blood cells 2. Presence of protein 3. Presence of ketones 4. Increased specific gravity

4. Increased specific gravity (4. Correct: Specific gravity is an indicator of hydration status and urine osmolality. In a dehydrated client, specific gravity is increased, indicating highly concentrated urine. 1. Incorrect: White blood cells should not be found in the urine unless an infection is present. Dehydration does not cause white blood cells in the urine. 2. Incorrect: Protein should not be found. Presence of protein indicates renal disease. In order to have proteinuria there must be damage to the glomeruli 3. Incorrect: Ketones should not be present. They are found in clients with poorly controlled diabetes or hyperglycemia, because ketones are a by-product of fat breakdown. Fats are broken down and used for energy when glucose cannot be transported into the cells because of lack of insulin.)

The nurse is caring for a client who sustained a head injury with possible seizure activity. The primary healthcare provider prescribes an EEG. What client teaching should the nurse provide to the client prior to this test? Select all that apply 1. Instruct client to be NPO after midnight. 2. Tell client not to wash their hair the night before the test. 3. Assure client that they may take sleeping pill the night prior to the EEG. 4. Instruct client not to drink caffeinated beverages the morning of the test. 5. Take routine medications the morning of the EEG with a sip of water.

4. Instruct client not to drink caffeinated beverages the morning of the test. 5. Take routine medications the morning of the EEG with a sip of water. (4., & 5. Correct: Caffeine is a stimulant and will speed up the electricity in the brain. We don't want to do anything that is going to alter a client's electrical activity in the brain before an EEG. The clients are encouraged to take their routine medications unless directed otherwise. 1. Incorrect: Fasting may cause hypoglycemia and could alter the EEG pattern. 2. Incorrect: The client should wash hair the night before the test. 3. Incorrect: No sedatives or hypnotics are given before the test. This could cause abnormal results on the EEG. In addition, the client may be directed to sleep less the night before the EEG if sleeping is desired during the EEG.)

Which nursing intervention should the nurse include when caring for a client with Alzheimer's disease being admitted to a long term care facility? 1. Offer multiple environmental stimuli at the same time to provide distraction. 2. Encourage the client to participate in activities such as board games. 3. Restrain the client in a chair to prevent falls when sundowning occurs. 4. Involve the client in supervised walking as a routine.

4. Involve the client in supervised walking as a routine. (4. Correct: A regular routine and physical activity help client's with Alzheimer's disease maintain abilities for a longer period of time. Physical activities promote strength, agility and balance. The client's walking should be supervised for client safety issues. 1. Incorrect: Environmental stimuli should be limited with clients with Alzheimer's Disease. The client can become agitated and/or more disoriented with an increase in environmental stimuli. 2. Incorrect: Board games would not be appropriate due to the client's cognitive and memory impairment. Board games require complex cognitive actions. 3. Incorrect: Restraints should be avoided because they increase agitation. The client may become agitated by the restriction of he restraints. Also the client may perceive the restraints as a threat.)

A client with a history of syncope and transient arrhythmias has been ordered a Holter monitor for 48 hours. The nurse knows that teaching has been effective when the client makes what statement? 1. No follow up care will be needed after the monitor is removed. 2. It is okay to shower or bath while wearing this equipment. 3. I have to take it easy and not exercise for the next two days. 4. It's important to write down all my activities during this time.

4. It's important to write down all my activities during this time. (4. Correct:The purpose of the Holter monitor is to detect cardiac irregularities over an extended period of time, in this case 48 hours. Although the monitor will record heart rate and rhythm for two days, it is vital for the client to keep a log or diary during that time, indicating the precise time and type of every activity. Additionally, this log needs to indicate any chest pain or palpitations the client experiences during that time, to assist the primary healthcare provider in diagnosing cardiac dysfunctions. 1. Incorrect: A Holter monitor is a mobile diagnostic test utilized by the cardiologist to help determine a cause for this client's syncopal episodes or arrhythmias. Once the client has the monitor and electrodes removed, the primary healthcare provider will analyze the data before meeting with the client to discuss the findings. Regardless of any suggested treatment options, the cardiologist needs a follow up visit with the client. 2. Incorrect: Showering or tub bathing is not permitted while wearing the Holter monitor as this may interfere with the functioning of the equipment. Only a careful sponge bath is permitted. Clients are also instructed to avoid heavy machinery, electric razors, microwave ovens and even hair dryers since can also affect accuracy and performance of the monitor. 3. Incorrect: The purpose of wearing Holter monitor for 24-48 hours is to diagnose cardiac arrhythmias during ADL's or exercise. The client cannot remove the monitor at any time during that period since that would cause inaccurate readings, or even the loss of valuable data. The client is instructed to complete all routine daily activities during that time, including work or exercise, to help identify actions that contribute to the symptoms or cardiac irregularities.)

A client with a history of angina has returned to the unit following a cardiac catherization. What nursing action has the highest priority? 1. Obtain vital signs every thirty minutes. 2. Assess pedal pulses every ten minutes. 3. Place the call bell within client's reach. 4. Keep affected extremity immobilized for 6 hours.

4. Keep affected extremity immobilized for 6 hours. (4. Correct: The greatest risk following a cardiac catherization is the potential for hemorrhage, most often from the insertion site. Therefore, the affected extremity must remain straight and immobilized for 4-6 hours after the procedure. 1. Incorrect: The frequency of vital signs is determined by facility protocol, but generally vital signs are obtained every ten minutes for the first half hour, then every fifteen minutes for another half hour. While vital signs provide valuable information to compare to baseline, another action is more important. 2. Incorrect: It is vital to assess pedal pulses in order to verify circulation following a catherization. The frequency is based on facility protocol. However, this action is not the highest priority. 3. Incorrect: Because the client is on bed rest, it is crucial for the client to be able to summon staff when needed. Despite the importance of this action, there is an even more important action.)

The nurse is reviewing morning laboratory results on four clients. Which lab finding should the nurse report to the primary healthcare provider immediately? Client diagnosed with deep vein thrombus who is receiving a heparin infusion: aPTT 85 sec Client diagnosed with possible appendicitis: WBC 18,000 per mm³ Client diagnosed with rheumatoid arthritis: Sed rate 100 mm/hr Client diagnosed with congestive heart failure receiving furosemide: K⁺ 2.9 mEq/L 1. aPTT 2. WBC 3. Sed rate 4. K⁺

4. K⁺ (4. Correct: Notice that all laboratory results are abnormal, based on the disease process of each client. However, the potassium level for the cardiac client is way below normal, most likely secondary to the furosemide. Levels that low can result in premature ventricular contractions (PVCs) or other arrhythmias, placing the client at risk for sudden onset of CHF. 1. Incorrect: The client is on the heparin infusion for a diagnosed deep vein thrombus (DVT). While normal aPTT levels should be between 20-36 seconds, the therapeutic levels of heparin are usually 2 ½ to 3 times normal to keep the blood thin. This result is expected and not alarming at this point. 2. Incorrect: Appendicitis is a serious infection that is treated with either antibiotics or surgery to remove the organ. While normal WBC values are 5,000 to 10,000, this elevated result is not unexpected for an infection. 3. Incorrect: A Sed rate or sedimentation rate, reveals inflammatory activity in the body and can be used to diagnose or monitor the status of an inflammatory disease process. The blood cells affected (erythrocytes) will settle to the bottom of a blood tube and that speed indicates the severity of the inflammatory process in the body. This is not an unexpected result in clients with rheumatoid arthritis.)

Which intervention should the nurse recommend to the adult child who is caring for an elderly parent diagnosed with Alzheimer's disease (AD)? 1. Give parent a small dog for company and comfort. 2. Reset the water heater to 125 degrees Fahrenheit (51.67 degree Celsius) to prevent burns. 3. Place mirrors in multiple locations so parent sees images of self. 4. Make floors and walls different colors.

4. Make floors and walls different colors. (4. Correct: People with Alzheimer's disease (AD) get more confused over time. They also may not see, smell, touch, hear and/or taste things they once did. By creating a contrast in color between the floors and walls makes it easier for the person with AD to see. 1. Incorrect: Be careful with small pets. The person with AD may not see the pet and trip over it. This is a safety issue. A fall could cause a major injury to the client. 2. Incorrect: The water heater should be set below 120 degrees Fahrenheit (48.8 degrees Celsius) to prevent burns. 3. Incorrect: Limit the size and number of mirrors in the home. Mirror images may confuse the person with AD. They may not recognize self and may see the person as a stranger.)

The nurse is providing care to a client who is 12 hours post abdominal aorta aneurysm (AAA) repair. Which assessment finding by the nurse would indicate a possible complication? 1. Urinary output 360 mL since surgery 2. Temperature 100.2 degrees F (37.9 degrees C) 3. 2+/4+ pedal pulses 4. Mean arterial pressure 50

4. Mean arterial pressure 50 (4. Correct. A normal range for mean arterial blood pressure is 70 to 110. A minimum of 60 is required to supply enough blood to nourish the coronary arteries, brain and kidneys. What is the complication you are worried about? Shock! 1. Incorrect. Normal UOP 30 mL x 12 hours = 360 mL 2. Incorrect. A slight increase in temperature postoperatively is not uncommon. 3. Incorrect. 2+/4+ is normal.)

Which statement by a client would indicate to the nurse that education about gastroesophageal reflux disease (GERD) has been successful? 1. It would be better for me to eat 3 small meals a day. 2. I need to avoid eating foods high in purine. 3. When going to sleep, I should lie on my side. 4. My last daily meal should not be within 2 hours of bedtime.

4. My last daily meal should not be within 2 hours of bedtime. (4. Correct: To avoid reflux the client should not eat within 2 hours of bedtime. Late night meals may increase discomfort and should be avoided. 1. Incorrect: The client should eat at least 6 smaller meals per day to help decrease reflux. Small, frequent meals help prevent over distention of the stomach. 2. Incorrect: The client with GERD should avoid high fat foods and increase high protein foods in an effort to lose weight. Eliminate foods high in purine with disorders such as gout. 3. Incorrect: The client should sleep with the HOB elevated on six inch blocks and several pillows under the upper body. Gravity fosters esophageal emptying.)

Two days after a client has a chest tube inserted, the nurse notes constant bubbling in the water seal chamber. What action should the nurse take? 1. Do nothing since this is normal. 2. Decrease the amount of suction. 3. Replace CDU unit with another one. 4. Notify primary healthcare provider.

4. Notify primary healthcare provider. (4. Correct: The water seal chamber is the middle of the three chambers and helps to create the one-way flow of drainage and air from the client to the CDU. The water seal chamber should bubble only intermittently when the client coughs, sneezes or breathes, creating a fluctuation of the water known as "tidaling". Constant bubbling in that chamber indicates an air leak somewhere in the system. Because the nurse cannot fix this independently, the primary healthcare provider must make that determination. 1. Incorrect: The water seal chamber helps create the one-way flow of drainage and air from the pleural space to the CDU. Constant bubbling in that chamber is not normal. 2. Incorrect: Constant bubbling in the water seal chamber is not controlled by the amount of suction. Decreasing suction would not alter the type of bubbling in the middle chamber. 3. Incorrect: Though the nurse may discover damage to the CDU unit itself, simply replacing the unit with a new one may not correct the problem in the water seal chamber.)

A client with chronic arterial occlusive disease has a bypass graft of the left femoral artery. Postoperatively, the client develops left leg pain and coolness in the left foot. What is the priority action by the nurse? 1. Elevate the leg. 2. Check distal pulses. 3. Increase the IV rate. 4. Notify the primary healthcare provider.

4. Notify the primary healthcare provider. (4. Correct: In this case, there is nothing on the list the nurse can do to fix the problem. The primary healthcare provider must be notified immediately. Anticipate that the client will be returning to surgery because these are symptoms of an arterial problem that needs to be addressed immediately. 1. Incorrect: Arterial circulation is improved by lowering the extremity. Remember to raise venous problems; lower arterial problems. These signs/symptoms indicate an acute, emergent change in the client's condition. In this case, the nurse is "worried" the client will lose the extremity. There is nothing the nurse can do to fix the problem, so calling the primary healthcare provider is the best answer. 2. Incorrect: Assessing the pulses is delaying treatment and does not fix the problem. In this question you have only 1 option, so you must go with what is best for the client. 3. Incorrect: Increasing the IV rate does nothing to fix the problem, and you have only 1 chance in this question to show the NCLEX lady that you are a safe nurse.)

A client with diabetes is hospitalized for debridement of a non-healing foot ulcer. Following the procedure, the nurse notes that the client has become confused and combative. The family expresses concern with the behavioral changes and requests that the client be restrained in bed. What is the nurse's priority action? 1. Notify the primary healthcare provider. 2. Apply a vest restraint as requested by family. 3. Move client to a room near the nurse's desk. 4. Obtain a finger-stick blood glucose level.

4. Obtain a finger-stick blood glucose level. (4. Correct: The client's behavior has negatively changed following the ulcer debridement procedure. The nurse's priority is to determine the cause of the client's confusion. The nurse is correct to investigate other possible causes for the behavior changes, including an abnormal glucose level in this diabetic client. 1. Incorrect: The nurse will indeed have to contact the primary healthcare provider about the client's change in behavior. However, the first priority would be to assess the client and collect data prior to placing that phone call. 2. Incorrect: The nurse understands that restraints cannot be applied by family request. Additionally, applying a restraint can often increase negative behavior while ignoring the actual cause. 3. Incorrect: Although assigning confused clients to a room near the nurses' station is an accepted practice, this does not determine the cause for the changing behavior and is not a priority at this time. The nursing priority is to assess the client for possible factors causing the behavior changes.)

A client with a terminal illness, asks the nurse about palliative care. What would be the nurse's best response? 1. Palliative care is a holistic way of finding a cure for a serious illness. 2. Palliative care begins when the client has 3 months or less to live. 3. Palliative care will require you to change to a palliative care healthcare provider. 4. Palliative care prevents and treats symptoms and side effects of disease and treatments.

4. Palliative care prevents and treats symptoms and side effects of disease and treatments. (4. Correct: This is a correct statement. The goal of palliative care is to help the client living with a chronic, life threatening illness. It focuses on the client's symptoms and the relief of these symptoms. Palliative care helps the client obtain their best quality of life throughout the course of their illness. 1. Incorrect: Palliative care is not aimed at cure. It is provided to clients who have chronic, life threatening illnesses. 2. Incorrect: Palliative care can begin at diagnosis. Hospice care is usually offered when the person has 6-12 months or less to live. 3. Incorrect: The client does not need to give up his or her primary healthcare provider. This is not a requirement of palliative care.)

A client has arrived at the emergency room reporting tingling to both lower legs over the past 24 hours. The only significant health history is a cold for the past week. During the nursing assessment, the client indicates that both thighs are feeling numb. What priority action should the nurse initiate immediately? 1. Assess bilateral pedal pulses. 2. Initiate a Code Blue. 3. Roll client onto left side. 4. Prepare for intubation.

4. Prepare for intubation. (4. Correct: The symptoms reported by this client indicate the onset of Guillian-Barre syndrome, an acute inflammatory disease that may occur following a respiratory illness and is characterized by progressive, ascending paralysis. The extent of paralysis varies by client, but airway is always the greatest concern. Because the client's lack of sensation is progressing, the nurse should anticipate the health care provider may need to immediately intubate to protect the airway. 1. Incorrect: Assessment is a primary essential part of the nursing process and provides clues to a client's status. This client has indicated a numbness to lower extremities for the last 24 hours, which is now extending up to the thighs. Determining the presence of pedal pulses is not the priority issue at this time. Another area presents a greater concern. 2. Incorrect: The code blue alarm is only initiated when a client has no heartbeat or spontaneous breathing. This client is still breathing and, therefore, a code blue is not necessary at this time. However, there is a potential problem that urgently needs to be addressed by the nurse. 3. Incorrect: This action serves no purpose for Guillian-Barre. Positioning is not beneficial for this client.)

The nurse is completing a focused assessment on a client post coronary artery bypass surgery (CABG). What finding warrants immediate attention by the nurse? 1. Central venous pressure (CVP) 6 mmHg 2. Mediastinal chest tube drainage of 70 mL in 1 hour 3. Incisional pain rated 9/10 4. Pulsus paradoxus

4. Pulsus paradoxus (4. Correct: Pulsus paradoxus is an indicator of cardiac tamponade. This is a severe complication after open heart surgery and is a medical emergency. The primary healthcare provider will need to be notified. 1. Incorrect: Normal CVP is 0-8 mm Hg. There is nothing to worry about here. 2. Incorrect: Worry about hourly drainage over 150 mL in 1 hour. 3. Incorrect: Post operative pain is expected. The nurse should administer prescribed pain medication.)

The nurse is teaching a group of clients who have osteoarthritis how to protect joints. What should the nurse include? Select all that apply 1. Use small joints and muscles. 2. Turn doorknobs clockwise. 3. Sit in a chair that has a low, straight back. 4. Push off with the palms of hands when getting out of bed. 5. Use hairbrush with extended handle.

4. Push off with the palms of hands when getting out of bed. 5. Use hairbrush with extended handle. (4. & 5. Correct: Pushing off with the palms of the hands is using a larger joint and muscles. Using the fingers will cause more joint injury. Use long handled devices such as a hairbrush with an extended handle to decrease stress on joints (in this case the wrist). 1. Incorrect: Larger joints and muscles can take more stress and weight than smaller ones. Using small joints again and again puts more stress on them and may lead to deformity. Try to spread the strain and weight over several joints. This helps you use each part of your body to its best advantage. 2. Incorrect: Do not turn a doorknob clockwise. Turn it counterclockwise to avoid twisting the arm and promoting ulnar deviation. 3. Incorrect: Sit in a chair that has a high, straight back. This will provide more support for the back.)

The nurse enters a client's room and finds the client masturbating. Which action by the nurse would be most appropriate for the nurse to take? 1. Ask the client to stop 2. Remain in the room until client has finished. 3. Document the activity in the client's chart. 4. Quietly leave the room

4. Quietly leave the room (4. Correct: Leaving the client's room, allows the client to have privacy. The client has the right to express self sexually in private. 1. Incorrect: The client has a right to express sexuality through masturbation, which is a normal way of finding sexual release. 2. Incorrect: Ignoring the behavior and continuing presence in the room will embarrass the client. 3. Incorrect: The nurse can chart the client's sexual activity in the chart. However, when the nurse enters the client's room and finds the client masturbating, the nurse first needs to leave the client's room quietly.)

A post-operative client becomes anxious and reports acute onset of chest pain when taking a deep breath and shortness of breath. Initial vital signs obtained by the nurse reveals tachycardia, hemoptysis, and a pulse oximeter reading of 90%. What intervention should the nurse initiate first? 1. Administer oxygen. 2. Obtain a blood pressure reading. 3. Connect to cardiac monitor. 4. Raise head of bed to 90 degrees.

4. Raise head of bed to 90 degrees. (4. Correct: This is your priority. This position will facilitate maximum lung expansion. It will also decrease venous return to the right side of the heart so that pressure decreases in the pulmonary vascular system. 1. Incorrect: Oxygen is needed, but the first thing the nurse should do is raise the head of the bed, so the client can breathe easier. Then get the oxygen set up. 2. Incorrect: Obtaining a blood pressure reading at this point is delaying treatment. The problem is a breathing problem. Do something to fix the breathing problem first. Then, you can continue your assessment by checking circulation status. 3. Incorrect: Connecting the client to a cardiac monitor is an appropriate intervention, but facilitating breathing takes priority and should be done first.)

What foods should the nurse inform the client to avoid for three days prior to a guaiac test? Select all that apply 1. Chicken 2. Carrots 3. Apple 4. Raw broccoli 5. Steak 6. Turnip greens

4. Raw broccoli 5. Steak 6. Turnip greens (4., 5., & 6. Correct: Foods that affect this test include raw broccoli, red meats such as steak, turnip greens, cantaloupe, radish, and horseradish. All of these could cause a false positive reading for the guaiac test. 1. Incorrect: Red meats such as steak should be avoided, but chicken is okay. 2. Incorrect: Carrots are not prohibited and will not affect the results of the test. 3. Incorrect: The client can eat apples with no effect on the test results.)

A client asks the nurse, "What causes hypermagnesemia?" The nurse should explain to the client that hypermagnesemia can occur secondary to what health problem? 1. Peripheral vascular disease 2. Dehydration 3. Liver failure 4. Renal insufficiency

4. Renal insufficiency (4. Correct: Magnesium is excreted primarily through the kidneys. When the client experiences renal insufficiency, magnesium is held. The incidence of hypermagnesemia is rare in comparison with hypomagnesemia, and it occurs secondary to renal insufficiency. 1. Incorrect: Peripheral vascular disease does not lead to hypermagnesemia 2. Incorrect: Dehydration leads to the electrolyte imbalance of hypernatremia, it does not cause hypermagnesemia. A client who has become dehydrated due to excessive urination may experience hypomagnesemia. 3. Incorrect: Liver failure does not lead to hypermagnesemia. Magnesium is regulated by GI absorption and renal excretion.)

The nurse is preparing to teach a client about post percutaneous transluminal coronary angioplasty (PTCA) care. Which teaching points should the nurse include? Select all that apply 1. Restricting oral fluids until the gag reflex has returned. 2. Encouraging early ambulation and deep breathing exercises. 3. Discontinuing medicines following percutaneous intervention. 4. Reporting any chest discomfort following percutaneous intervention. 5. Avoid lifting more than 10 pounds until approved by healthcare provider.

4. Reporting any chest discomfort following percutaneous intervention. 5. Avoid lifting more than 10 pounds until approved by healthcare provider. (4., & 5. Correct: The number one thing you are "worried" about post PTCA is re-occlusion or re-infarction, so report chest discomfort at once. Lifting more than 10 pounds can make the client bleed and would be contraindicated until cleared by the primary healthcare provider. 1. Incorrect: Fluids need to be increased to flush the dye used during the procedure from the kidneys. Oral fluids do not have to be restricted because the client does not have to be intubated for the procedure. 2. Incorrect: To ensure a stable clot is formed at the femoral access site, the client must remain on bed rest for a minimum of 4 hours. The client is at risk for hemorrhaging at the insertion site. DO NOT ambulate until it is certain that the clot is stable. 3. Incorrect: Medications are generally continued as before the procedure. Certain medications, like anticoagulants, may be held prior to the procedure, but typically all pre-procedure medications are resumed after PTCA.)

How should the nurse interpret the arterial blood gas (ABG) results of a client admitted with dehydration? pH - 7.49 PaCO₂ - 29 HCO₃⁻ - 24 Select all that apply 1. Metabolic acidosis 2. Respiratory acidosis 3. Metabolic alkalosis 4. Respiratory alkalosis 5. Uncompensated 6. Partially compensated 7. Fully compensated

4. Respiratory alkalosis 5. Uncompensated (4., & 5. Correct: The blood gases confirm respiratory alkalosis. Why? The pH is 7.49 (normal 7.35-7.45). This pH indicates alkalosis since it is high. Which other chemical says alkalosis? The PaCO₂ of 29 (normal 35-45) is low which indicates alkalosis. The HCO₃⁻ is normal. This means that the client is in uncompensated respiratory alkalosis. 1. Incorrect: The blood gases confirm respiratory alkalosis. The Bicarb is normal, so the problem is not metabolic. 2. Incorrect: The blood gases confirm respiratory alkalosis. The PaCO₂ of 29 (normal 35-45) is low which indicates alkalosis. For this client to bew in respiratory acidosis, the PaCO₂ would be greater than 45. 3. Incorrect: The blood gases confirm respiratory alkalosis. The Bicarb is normal, so the problem is not metabolic. 6. Incorrect: Compensation has not begun because the bicarb is normal. To compensate the bicarb would need to decrease to bring the pH down to normal. 7. Incorrect: Fully compensated would occur if the pH is normal with abnormal CO₂ and bicarb.)

A client, who arrives at the emergency department, reports flashes of light. What problem does the nurse suspect? 1. Cataract 2. Open angle glaucoma 3. Macular degeneration 4. Retinal detachment

4. Retinal detachment (4. Correct: Seeing flashes of light is one indication of retinal detachment. The client may also report floating spots or blurred, "sooty" vision. If detachment progresses rapidly, the client may report a veil-like curtain obscuring parts of the visual field. Early on, straight-ahead vision may be intact but, as detachment evolves, there is a loss of central and peripheral vision. 1. Incorrect: A cataract is a clouding that occurs over the eye lens. The client states it is like looking through a fogged-up window. 2. Incorrect: Glaucoma is a group of eye conditions that damage the optic nerve, often caused by an abnormally high pressure in the eye. Open-angle glaucoma results in patchy blind spots in the peripheral or central vision, frequently in both eyes. Tunnel vision occurs in the late stages. 3. Incorrect: Macular degeneration is the leading cause of irreversible vision loss in people over the age of 60. It occurs when the small central portion of the retina deteriorates. Although macular degeneration is almost never a totally blinding condition, it can be a source of significant visual disability. The first sign a person will notice is a gradual or sudden change in the quality of vision or that straight lines appear distorted. Other symptoms include dark, blurry areas or whiteout that appears in the center of vision. In rare cases, a change in color perception can occur.)

In what position should the nurse place a client diagnosed with gastric reflux? 1. Orthopneic 2. Semi-Fowler's 3. Sims' 4. Reverse Trendelenburg

4. Reverse Trendelenburg (4. Correct: The entire bed is tilted with the foot of the bed lower than the head of the bed. This position promotes gastric emptying and prevents esophageal reflux. 1. Incorrect: Orthopneic position has the client sit in the bed or at the bedside. A pillow is placed on the over-bed table, which is placed across the client's lap. The client rests arms on the over-bed table. This position allows for chest expansion and is especially beneficial to clients with COPD. 2. Incorrect: The head of the bed is elevated 30 degrees. This position is useful for clients who have cardiac, respiratory, and neurological problems and is often optimal for clients who have a nasogastric tube in place. 3. Incorrect: Sims' or semi-prone position has the client on the side halfway between lateral and prone positions. Weight is on the anterior ileum, humerus, and clavicle. The lower arm is behind the client while the upper arm is in front. Both legs are flexed but the upper leg is flexed at a greater angle than the lower leg at the hip as well as at the knee. This is a comfortable sleeping position for many clients, and it promotes oral drainage.)

A client is brought into the emergency department (ED) with nausea, vomiting and diarrhea after eating chicken at a picnic. The nurse suspects that this client has most likely contracted which infection? 1. Shigella 2. Escherichia coli 3. Clostridium Difficile 4. Salmonella

4. Salmonella (4. Correct: Salmonella is a gram negative bacillus found in animal sources such as chicken products, eggs, turkey, and some beef. Nausea, vomiting, and diarrhea after ingesting infected chicken would be the classic signs/symptoms. 1. Incorrect: Shigella infection is a gram negative organism that invades the lumen of the intestine and causes severe runny, bloody diarrhea which can be transmitted through the fecal/oral route. Improper hygiene is most likely cause. 2. Incorrect: Escherichia coli is the most common aerobic organism colonizing the large bowel. It is often linked to ingestion of undercooked contaminated beef and vegetables that have been contaminated by animal waste water. Signs and symptoms of E. coli include bloody diarrhea, severe cramps, nausea and vomiting, and renal failure. 3. Incorrect: Clostridium Difficile is a spore-forming bacterium usually preceded by antibiotics, which disrupt normal intestinal flora and allow the C. Difficile spores to proliferate within the intestine. C. difficile signs and symptoms can range from mild diarrhea to severe colitis.)

A client awaiting discharge for a broken left tibia is to be sent to physical therapy for crutches and crutch walking. The client reports having brought a pair of crutches borrowed from a family member. What is the most appropriate action for the nurse to take now? 1. Cancel physical therapy and allow client to leave. 2. Ask client to stand with crutches to check the size. 3. Tell client insurance will not permit use of old crutches. 4. Send client with crutches to physical therapy for evaluation.

4. Send client with crutches to physical therapy for evaluation. (4. Correct: The physical therapy department is best qualified to assist a client in adjusting to the use of crutches prior to discharge. Because the client wants to use older crutches, it is even more important for a physical therapist to determine whether it is safe for the client to do so. Physical therapy can evaluate the condition of the old crutches, the client's ability to manage that equipment and to walk safely with those crutches. 1. Incorrect: It is permissible for a client to use previously owned medical equipment. However, the stability of that equipment and the client's ability to use the equipment safely must be evaluated by physical therapy. Cancelling physical therapy would also violate the physician's orders and place the client at risk for injury upon discharge. 2. Incorrect: While the nurse may be able to adjust the old crutches to the client's height, crutch safety and walking should be evaluated by physical therapy to be certain the previous equipment is appropriate. 3. Incorrect: Insurance does not designate whether assistive medical devices can be reused by clients or if a new device must be purchased. It is cost effective to reuse durable medical equipment if it is appropriately suited to the client's current needs.)

Following hip replacement surgery, an elderly client is being transferred to a long term care facility for therapy. What priority action by the nurse best promotes continuity of care for the client? 1. Explain future care requirements to the family. 2. Call facility's nurse manager to give oral report. 3. Discuss client's needs with healthcare provider. 4. Send written summary of client needs to facility.

4. Send written summary of client needs to facility. (4. Correct: Written documentation is the most complete legal record for continuity of client care. In this format, the same specific information is then available to all staff having direct care contact with the client. 1. Incorrect: While the family will definitely need to be informed of the client's current and future therapeutic needs, such a discussion would have taken place prior to being discharged to long term care. Another action takes priority. 2. Incorrect: An oral report is vital prior to the client's arrival at a new facility so that an appropriate room and needed equipment can be available for the client's arrival. Though such an action is important, there is a better method to promote continuity of care. 3. Incorrect: Talking with the primary healthcare provider must be done at the time orders for transfer have been written to clarify specifics, which would then be relayed to the long term care facility. This is not the nurse's current priority.)

Which finding should take priority when the nurse is assessing the skin of a client diagnosed with diabetes? 1. Vitiligo of the chest. 2. Scleroderma to scapula and posterior neck region. 3. Redness of face and upper chest. 4. Small abrasion on great toe.

4. Small abrasion on great toe. (4. Correct: Skin breakdown on the foot is priority. Healing is likely to be impaired and the abrasion can be an entry point for microorganisms. There maybe other risk factors observed in the assessment; however, this finding should receive priority. 1. Incorrect: Vitiligo is a skin problem commonly associated with type I diabetes. The melanin containing cells are destroyed, resulting in patches of discolored skin. Vitiligo poses no harm to the client. 2. Incorrect: Scleroderma affects people with type 2 diabetes causing thickening of the skin to the upper back and neck. Scleroderma poses no harm to the client. 3. Incorrect: Redness should be noted and reasons found; however, this is not the priority finding. Redness/flushing can be due to many conditions but poses no obvious harm to the client.)

The nurse is making an initial home visit to a client newly diagnosed with diverticulitis. The client had been on a liquid diet but is now to begin solid foods appropriate for the disease process. The nurse knows dietary teaching has been successful when the client selects which meal? 1. Hamburger on sesame roll, macaroni and cheese, tossed salad 2. Lamb chop with brown rice, cooked broccoli, baked potato 3. Pork with sauerkraut, baked beans, and coconut cake 4. Spaghetti with meatballs, fruit cocktail, garlic bread

4. Spaghetti with meatballs, fruit cocktail, garlic bread (4. Correct: Diverticulitis is an inflammation within the small, out pouching which can develop in the colon. A low residue/low fiber diet limits the amount of food waste passing through the large intestine, allowing the intestinal tract to rest and heal. Cooked pasta, along with ground, well cooked meatballs, is tolerated well. Canned fruits like fruit cocktail are far better than fresh fruit to control diarrhea and cramping. Garlic bread is also acceptable. 1. Incorrect: A hamburger is well ground, cooked meat, which is acceptable for this client, but not when served on the sesame roll. Seeds and nuts tend to lodge in the diverticula, leading to pain or infection. Macaroni and cheese is a great menu item for the client. However, fresh vegetables, though healthy, do not breakdown easily, resulting in large amounts of undigested material passing through, or getting stuck, in the large intestine. 2. Incorrect: Lamb is an acceptable as part of a low residue diet, but clients are instructed to eat white rice rather than the whole grain brown rice. Vegetables which create gas even when cooked, such as broccoli, could lead to a serious exacerbation of diverticulitis. Well cooked potatoes are permitted, but not if prepared with the skin, such as the baked potato. 3. Incorrect: Several parts of this menu selection present a major problem for the client; in fact, only the pork is acceptable. Sauerkraut is prepared cabbage, and even when cooked, causes digestive and gas issues. Baked beans should be avoided for the same digestive reasons, and coconut is in the category with nuts or seeds.)

A client has been unable to eat due to protracted vomiting. Which alterations in the arterial blood gases would the nurse expect to find? 1. pH: 7.40, PaCO₂: 44, HCO₃⁻: 23 2. pH: 7.33, PaCO₂: 35, HCO₃⁻: 18 3. pH: 7.35, PaCO₂: 48, HCO₃⁻: 29 4. pH: 7.46, PaCO₂: 35, HCO₃⁻: 28

4. pH: 7.46, PaCO₂: 35, HCO₃⁻: 28 (4. Correct: The stomach as a lot of acid in it. So, if the client is vomiting a lot, then the client is losing acid. This will make the client alkalotic inside. Is this going to be a lung problem? No. So we are looking for ABGs that indicate that this client is in metabolic alkalosis. A pH of 7.46 is higher than the normal pH value of 7.45, which indicates alkalosis. The PaCO₂ is 35, which is on the low end of normal (34-45). The HCO₃⁻ is 28, which is higher than the normal HCO₃⁻ of 26, which indicates alkalosis. So the Bicarb (Kidney chemical) matches the pH. Metabolic alkalosis. 1. Incorrect: These are normal ABGs. pH: 7.40 (normal), PaCO₂: 44 (normal), HCO₃⁻: 23 (normal). 2. Incorrect: This is metabolic acidosis. pH: 7.33 (acid), PaCO₂: 35 (normal), HCO₃⁻: 18 (acid). 3. Incorrect: This is fully compensated respiratory acidosis. pH: 7.35 (acid), PaCO₂: 48 (acid), HCO₃⁻: 29 (alkaline).)

A client has been on the nursing unit for two hours following a retropubic prostatectomy for the treatment of prostate cancer. The client is receiving a continuous bladder irrigation of normal saline infusing at 1000 mL/hr. The client's urine output for the past two hours is 410 mL. What is the nurse's first action? 1. Inspect the catheter tubing for obstruction. 2. Irrigate the catheter with a large piston syringe. 3. Notify the primary healthcare provider. 4. Stop the irrigation flow.

4. Stop the irrigation flow. (4. Correct: The catheter output should be at least the volume of irrigation input plus the client's actual urine. A severe decrease in output indicates obstruction in the drainage system. The first action is to stop the irrigation flow to prevent further bladder distention. Bladder distention is one of the main causes of hemorrhage in the fresh post op period. 1. Incorrect: The next action is to check the external system for kinks or obstruction to assess if this is the cause of the decreased urine output. Obstruction of the catheter tubing can also cause bladder distention. 2. Incorrect: After the external system is checked for kinks or obstruction, and the client's urine output doesn't change, then the catheter is irrigated with 30 to 50 mL of normal saline using a large piston syringe. However, irrigating a new post-op client is not the safest or first action for the nurse. 3. Incorrect: Of the options listed here, this is the last intervention. If the obstruction is not resolved after irrigating the system, the primary healthcare provider must be notified.)

An elderly client comes to the clinic for a check-up. The client's daughter tells the nurse that her father's dementia symptoms become increasingly more difficult to handle in the evening. How would the nurse document this symptom? 1. Confabulation 2. Apraxia 3. Pseudodementia 4. Sundowning

4. Sundowning (4. Correct: Sundowning is a phenomenon where symptoms seem to worsen in the late afternoon and evening. Communication becomes more difficult, with increasing loss of language skills. Institutional care is usually required at this stage. 1. Incorrect: Confabulation is the term used for creating imaginary events to fill in memory gaps. This is sometimes associated with dementia, but more often with disorders like Korsakoff's syndrome, traumatic brain injuries or tumors. 2. Incorrect: Apraxia is the term used for the inability to carry out motor activities despite intact motor function. 3. Incorrect: Pseudodementia is depression. Depression is the most common mental illness in the elderly, but it is often misdiagnosed and treated inadequately. Cognitive symptoms of depression may mimic dementia.)

A client has just developed an abdominal wound evisceration post bowel resection. In what position should the nurse place the client? 1. Sims' position. 2. Dorsal recumbent. 3. Right side lying in the fetal position. 4. Supine, head of bed at 15 degrees with knees and hips bent.

4. Supine, head of bed at 15 degrees with knees and hips bent. (4. Correct: This position will decrease pressure on the suture line and abdomen. 1. Incorrect: Sims' is a semi-prone position where the client assumes a posture halfway between the lateral and prone positions. If you place the client in this position the bowel contents can protrude out of the wound even more. 2. Incorrect: In this position the client's head and shoulders are slightly elevated on a small pillow. This does not ease the tension as much as supine with HOB elevated 15 degrees and knees and hips flexed. 3. Incorrect: Turning the client on their side will allow the abdominal contents to protrude out of the wound even more.)

What should the summer camp nurse include when teaching a group of adolescents about West Nile Virus? 1. Antiviral medications are used to treat West Nile Virus. 2. Using insect repellent containing diethyltoluamide (DEET) will kill the virus when a mosquito makes skin contact. 3. Nothing can be done to prevent West Nile Virus. 4. Symptoms of West Nile Virus include headache, fever, and fatigue

4. Symptoms of West Nile Virus include headache, fever, and fatigue (4. Correct: The West Nile Virus begins with flu-like symptoms such as headache, fatigue, and fever. These symptoms, however, may continue for several months. 1. Incorrect: There is no medication to treat West Nile Virus infection. 2. Incorrect: Insect repellent repels the mosquito but has no effect on the virus the mosquito is carrying 3. Incorrect: There are prevention methods that can be initiated to attempt to prevent West Nile Virus, such as using insect repellant with DEET as instructed; dress in clothing that covers arms and legs; cover crib, stroller, and baby carrier with mosquito netting; use screens on windows and doors; repair holes in screens to keep mosquitoes outside; use air conditioning when available; sleep under mosquito net if sleeping outdoors; and check inside and outside home for standing water (where mosquitoes lay eggs).)

A client with Bell's palsy is having difficulty eating. Which action by the nurse will be most helpful? 1. Teach the client to perform active facial exercises several times a day. 2. Provide a liquid diet high in protein and calories that will be easily swallowed. 3. Provide oral hygiene after eating. 4. Teach the client to chew food on the unaffected side of the mouth.

4. Teach the client to chew food on the unaffected side of the mouth. (4. Correct: Maintenance of good nutrition is most important. Teaching the client to chew on the unaffected side will help the client avoid food trapping. This will decrease the risk of aspiration which prioritizes higher than the other options. 1. Incorrect: Performances of facial exercises is important in recovery from Bell's palsy and will help over a long period of time. This intervention is not the highest priority. 2. Incorrect: Liquids are too difficult for the client to manage, as lip closure and chewing are impaired. A purely liquid diet increases the risk for aspiration. 3. Incorrect: Providing oral hygiene is important to prevent dental caries; however, this is not more important than preventing aspiration.)

A client is admitted with irritable bowel syndrome (IBS) and shingles. The nurse is discussing the client assignments with the charge nurse. Which staff member should not be assigned to this client? 1. The nurse with a history of roseola. 2. The unlincesed assitive personnel (UAP) with no history of roseola. 3. The UAP with a history of chicken pox. 4. The LPN/VN with no history of chicken pox.

4. The LPN/VN with no history of chicken pox. (4. Correct: A nurse who has not had chicken pox could contract it and should not be assigned a client with shingles. Those who have not developed antibodies to the varicella zoster virus are susceptible to chicken pox. Chicken pox and shingles are both from the varicella virus. 1. Incorrect: Roseola is a rose colored rash and would not have any effect on the assignment. It is a generally mild infection that usually affects children by age 2, and rarely adults. It is caused by 2 strains of herpes virus, rather than the varicella virus. 2. Incorrect: Roseola is a rose colored rash and would not have any effect on the assignment. It is a generally mild infection that usually affects children by age 2, and occasionally adults. It is caused by 2 strains of herpes virus, rather than the varicella virus. There is no relationship between roseola and shingles. 3. Incorrect: Shingles is caused by a reactivation of the varicella-zoster virus (which causes chicken pox). Those who have not developed antibodies to the varcella-zoster virus are susceptible to chicken pox. Therefore, the UAP who had chicken pox could be assigned this client.)

Four clients are admitted to the medical-surgical unit. The nurse is aware that what client will need standard precautions only? 1. The client with chicken pox. 2. The client with rubeola. 3. The client with impetigo. 4. The client with pancreatitis.

4. The client with pancreatitis. (4. Correct: Standard precautions are observed with all clients admitted to the hospital, without the need for additional safeguards. The client with pancreatitis is not contagious and does not present any unique concerns other than the need for gloves and hand washing. 1. Incorrect: Chicken pox, also known as varicella zoster, requires airborne precautions. The virus can be spread through contact with the droplets, either touching or inhaling the droplet, while providing care for this client. 2. Incorrect: Measles, also called rubeola, is spread through droplet contact with the contaminated individual, including inhalation of the droplets. Airborne precautions are necessary when caring for a client diagnosed with rubeola. 3. Incorrect: Impetigo is a severe skin infection characterized by itchy, red, fluid-filled blisters caused by either staphylococcus or streptococcus bacteria. This skin infection is highly contagious, and requires contact precautions to protect staff and visitors.

A nurse is conducting an initial admission history on a client who is reporting bone pain secondary to cancer with metastasis to the bone. What does the nurse determine is the most important information to gather during this initial screening? 1. The physical assessment of the client 2. The hemoglobin and hematocrit levels 3. The amount of pain medication the client is receiving 4. The client's description of the pain

4. The client's description of the pain (4. Correct: The most important information to gather during the initial screening is the client's perception and description of the pain. Pain is subjective, based on the client's perception. This is also the primary complaint of the client upon admission. 1. Incorrect: The question is asking about the client's pain. The physical assessment is important but does not address the client's perception of their own pain. 2. Incorrect: RBCs are produced in the bone marrow. The H&H might be affected but will not be the cause of the pain and assessed later with admission lab and diagnostics. 3. Incorrect: The amount of pain medication is important, but is not the most important information to gather from a client who is reporting pain, particularly with cancer and metastatic bone pain.)

A client returns to the unit after having extracorporeal lithotripsy. Which would be the best indicator that the treatment has been effective? 1. The client is relieved of the pain. 2. The urine is free of red blood cells. 3. The urinary output has increased since return to the unit. 4. There is sediment in the urinary catheter drainage bag.

4. There is sediment in the urinary catheter drainage bag. (4. Correct: This answer provides visible proof that the renal calculi has been broken up by the shock waves. 1. Incorrect: Pain can occur because of spasm of smooth muscle when the stone is moving. 2. Incorrect: There will be blood in the urine for several days after treatment. 3. Incorrect: Blocked urine flow from stone fragments may cause decreased urine output.)

What would be the best way for the nurse to evaluate the effectiveness of fluid resuscitation during the emergent phase of burn management? 1. Weight increases by 2 pounds in 24 hours 2. Urinary output is greater than fluid intake 3. Blood pressure is 90/60 mmHg 4. Urine output greater than 35mL/hour

4. Urine output greater than 35mL/hour (4. Correct: Urine output of 30 to 50 mL/hour indicates adequate fluid replacement. 1. Incorrect: May indicate fluid retention. 2. Incorrect: Does not indicate fluid balance. 3. Incorrect: Blood pressure alone does not indicate adequate fluid balance.)

A client with cervical cancer received an internal cervical radiation implant. What should be the initial nursing action if the radiation implant becomes dislodged and is found lying in the bed? 1. Call the client's primary healthcare provider. 2. Pick up the implant immediately with gloved hands and place it in double biohazard bags. 3. Notify the radiology department. 4. Utilize long-handled forceps to pick up the implant and dispose of it in a lead container.

4. Utilize long-handled forceps to pick up the implant and dispose of it in a lead container. (4. Correct: If a client is receiving a radiation implant, a lead container and long-handled forceps should be placed in the client's room and kept for the duration of the therapy. If the implant becomes dislodged, the nurse should pick up the implant with long-handled forceps and place it in the lead container. 1. Incorrect: The placement of the implant into the lead container should be done initially. The primary healthcare provider may be notified but this is not the initial nursing action needed. 2. The implant should be picked up with long forceps for distance and reduction of contact. In addition, a biohazard bag is not sufficient for proper disposal of the radiation implant. 3. The initial action is to use long-handled forceps and dispose of the implant in a lead container. Calling the radiology department is delaying care and exposing individuals to the implant.)

What signs/symptoms would the nurse expect to find in a client admitted to the unit with a diagnosis of Addison's disease? Select all that apply 1. Moon face 2. Buffalo hump 3. Hirsutism 4. Acne 5. Hyperpigmentation 6. Hypotension

5. Hyperpigmentation 6. Hypotension (5., & 6. Correct: Addison's disease results in a decrease in glucocorticoids, mineralocorticoids, and sex hormones resulting in a darkening in skin color and hypotension. 1. Incorrect: Moon face occurs when the body has too much cortisol, such as with Cushing's. 2. Incorrect: Buffalo hump occurs with Cushing's. 3. Incorrect: An increase in facial hair occurs with Cushing's. 4. Incorrect: Acne occurs with Cushing's.)

The nurse, caring for a client post motor vehicle accident who sustained multiple crushing injuries, suspects that the client may be developing disseminated intravascular coagulation (DIC). Which assessment findings by the nurse suggest that the client is developing this complication? Select all that apply 1. Chest pain 2. Frothy sputum 3. Intermittent claudication 4. Subcutaneous emphysema 5. Petechiae 6. Blood oozing from chest tube insertion site

5. Petechiae 6. Blood oozing from chest tube insertion site (5., & 6. Correct: Petechiae are red dots on the surface of the skin seen because of minute hemorrhages within the dermal or submucosal layers of the skin. Oozing blood from invasive catheter sites is one sign of DIC. The client can have minor oozing of blood to bleeding from every orifice and into the tissues. 1. Incorrect: Chest pain may be a symptom of MI, or pulmonary embolus. It is not typical of DIC. 2. Incorrect: Think pulmonary edema and pulmonary embolus with frothy sputum. 3. Incorrect: Intermittent claudication is severe leg pain associated with decreased oxygenation to the leg muscles. 4. Incorrect: Subcutaneous emphysema is air under the skin, typically seen with chest tubes or tracheostomy insertion.)

An adult client has partial and full thickness burns over the anterior trunk and anterior and posterior aspects of both legs. Utilizing the rule of nines, what percentage of the body surface area is burned? Round your answer to the nearest whole number. Ans:______

54 (The anterior trunk counts for 18% of the body; entire right leg counts 18%; entire left leg counts 18%. Body surface on this client is 54%.)

The nurse initiates sterile wound care on a client's newly debrided foot ulcer. After removing the dressing and beginning a betadine cleanse, the client mentions an allergy to iodine not previously reported. Place the nursing actions in order of priority. Observe client for signs or symptoms of reaction. Ask client about the type of "allergic response". Cover wound with temporary sterile dressing. Notify primary healthcare provider of the allergy. Remove betadine solution from wound with normal saline.

Ask client about the type of "allergic response". Remove betadine solution from wound with normal saline. Cover wound with temporary sterile dressing. Observe client for signs or symptoms of reaction. Notify primary healthcare provider of the allergy. (First: Because many individuals confuse the term "allergy" with expected side effects, the nurse first needs to quickly determine the type of response this client may have previously experienced. Second: Whether or not the client is able to give a clear description of an allergic response, such as hives, swelling or reddened skin, the nurse must assume the worst and then immediately remove the betadine solution from the open wound/skin before any reaction occurs. Third: The previously unreported allergy may require a change in the plan of care; however, at this moment the nurse has a fresh wound exposed to air. The nurse should cover the wound with a dry sterile dressing. Fourth: Then observe the client for at least 10 minutes to determine the need for any emergency intervention in case of anaphylaxis. Fifth: Once it is determined the client is stable, the healthcare provider should be contacted regarding the new information, client's status and whether new wound care orders are needed.)

A nurse has been assigned to care for five clients. In what order should the nurse assess these clients after shift report? Place in priority order from highest to lowest priority. Client diagnosed with an arterial ulcer to the right leg who reports pain of 8/10. Client whose BP is reported by the UAP to be 200/102 at present. Client hospitalized to rule out abdominal aortic aneurysm who is reporting deep, aching pain in the flank area. Client diagnosed with peripheral vascular disease requesting information on smoking cessation. Client with Buerger's disease reporting numbness, tingling and cold in toes.

Client hospitalized to rule out abdominal aortic aneurysm who is reporting deep, aching pain in the flank area. Client whose BP is reported by the UAP to be 200/102 at present. Client diagnosed with an arterial ulcer to the right leg who reports pain of 8/10. Client with Buerger's disease reporting numbness, tingling and cold in toes. Client diagnosed with peripheral vascular disease requesting information on smoking cessation. (The first client the nurse should assess is the client reporting deep, aching flank pain with suspected abdominal aortic aneurysm. This is a sign of ruptured aortic aneurysm. This client is at greatest risk of death if the nurse does not do something. Clients who experience a ruptured aortic aneurysm may develop shock rapidly because the aorta is a large vessel and blood loss occurs quickly. Death can occur within minutes to hours. The second client who needs to be seen by the nurse is the one whose BP is high. This is the second most life threatening problem. The client is experiencing a hypertensive crisis which is a systolic pressure greater than or equal to 180 mmHg or diastolic greater than or equal to 120 mmHg. Uncontrolled blood pressure at this level can lead to progressive or impending end-organ dysfunction, including a possible cerebral vascular accident (CVA). This is an emergency situation and should be treated immediately. Third, the nurse should see the client experiencing leg pain due to arterial ulceration. The nurse needs to perform vascular checks to note if there is adequate arterial circulation. Although the symptoms listed are characteristic of arterial ulcers, the client may be experiencing further arterial occlusion. The goal for this client would be rapid surgical or medical approaches to help improve circulation to the areas. If circulation cannot be restored, amputation may be required. The fourth client to assess is the one diagnosed with Buerger's disease. These signs/symptoms are typical of this disorder. The disease is characterized by inflammation in the arteries that results in a vaso-occlusion type phenomena. The claudication, with symptoms described here, can quickly progress to a critical degree of limb ischemia. As it progresses, revascularization may not be possible, and amputation may be the only viable option. This is seen almost exclusively in heavy smokers or those who use other forms of tobacco. Medications are not generally helpful, so stopping tobacco use is basically the only way to stop the progression of this disease. The last client to assess is the client requesting information on smoking cessation. This is the most stable client. Although smoking is considered hazardous to the overall health status and can worsen this client's condition, cessation is not something that must be done as an emergency measure in this case. The other clients have more serious or life threatening issues than this client.)

In what order should the nurse address these client events that occur at the same time? Place in order of highest to lowest priority. Client who is on bedrest due to a deep vein thrombus attempting to get out of bed. The water seal chamber is empty in a client's closed chest drainage unit. Client's tracheostomy needs to be suctioned. UAP reports a heart rate of 40/min in a client. Client reporting urinary frequency and dysuria.

Client's tracheostomy needs to be suctioned. The water seal chamber is empty in a client's closed chest drainage unit. UAP reports a heart rate of 40/min in a client. Client who is on bedrest due to a deep vein thrombus attempting to get out of bed. Client reporting urinary frequency and dysuria. (The client with the highest need is the client who has a tracheostomy that needs to be suctioned. This client has an airway problem. Maintaining a patent airway is vital to life and is always the first priority. The next client to be seen is the client whose water seal chamber is empty which prevents the CDU from being a closed system. This can create a breathing problem. The purpose of the water seal chamber is to allow air to escape from the pleural space and yet prevent air from re-entering the pleural space. It is a one-way system. The water should be at the prescribed level (2 cm) to maintain this one-way water seal. If air is allowed to re-enter the pleural space, the lung can collapse again (pneumothorax). Once the other client's airway is suctioned, this would be the next priority. The third client to be seen is the client with a heart rate of 40/min which may be affecting cardiac output. This is a circulatory problem. Circulation follows airway and breathing in priority setting. The fourth client to be seen is the client on bedrest for a DVT. If the client gets up and ambulates, the clot can break lose and form an embolus. Although this could potentially be dangerous, it does not take priority over airway or circulatory issues that exist. The fifth client would be the one reporting frequency and dysuria. This client does not have a life-threatening problem. Therefore, this would be the lowest in priority from the events presented.)

A newly hired nurse has been instructed by the preceptor nurse on burn dressing techniques. The nurse knows teaching has been effective when the new nurse performs wound care in what order? Wash hands and apply sterile gloves. Remove the old dressing and discard. Clean burn and place sterile dressing. Set up sterile field and open packages. Wash hands and apply clean gloves. Medicate client with pain medication.

Medicate client with pain medication. Wash hands and apply clean gloves. Set up sterile field and open packages. Remove the old dressing and discard. Wash hands and apply sterile gloves. Clean burn and place sterile dressing. (The client must first be medicated for this painful procedure, and at least 30 minutes in advance so the drug has time to work. The nurse must then wash hands thoroughly and apply the clean (non-sterile) gloves. Depending on the type and extent of burns, the nurse may also need a gown to prevent contaminating the client. Although not mentioned here, the nurse would most certainly explain the procedure to the client, which could actually be accomplished while the nurse sets up the sterile field, opens sterile packages and pours the ordered cleansing fluids. Once properly set up, the nurse will gently remove the old dressing and discard along with the non-sterile gloves, per facility protocols. After washing hands a second time, the nurse will apply the sterile gloves to complete care. The burned area is cleaned, prescribed antibiotic cream is applied, and a new sterile dressing placed over the burn.)

The nurse walks into a client's room and discovers the radioactive uterine implant lying on the bed. In what order should the nurse properly dispose of the implant? Pick up implant with tongs Put on gloves Place implant in lead lined container Call radiation department to take the implant out of the room

Put on gloves Pick up implant with tongs Place implant in lead lined container Call radiation department to take the implant out of the room (The first thing the nurse should do is put on gloves. Second, pick up the implant with tongs. Third, place the implant in a lead lined container. Fourth, call the radiology department to take the implant out of the room.)

The nurse is demonstrating ostomy care to a client with a new stoma in the sigmoid area of the colon. The nurse knows teaching is successful when the client completes care in what order? Press flange into place and attach bag. Apply skin protectant and allow drying. Remove ostomy bag and old flange. Wash stoma with warm soapy water. Place stoma adhesive onto new flange. Cut center of new flange to fit stoma.

Remove ostomy bag and old flange. Wash stoma with warm soapy water. Apply skin protectant and allow drying. Cut center of new flange to fit stoma. Place stoma adhesive onto new flange. Press flange into place and attach bag. (Changing an ostomy appliance can be challenging for an individual without assistance from another individual. Successful completion of the procedure involves not only a willingness to learn, but also the client's physical capabilities to reach, view, and accomplish the task based on the type or location of stoma. The basic principles of stoma care are similar to any dressing change. The client must remove and dispose of the old, inspect and clean the area, then prepare and apply the new appliance. Though there are multiple steps, many clients can perform self-ostomy care with practice and minimal assistance. The ostomy flange is designed to remain in place for 3 to 5 days, while the bag can be changed, or cleaned, daily. There are many types of appliances, including some which are sealed and are simply thrown away at the end of each day. The steps have been simplified here, but are basic: First, remove both the ostomy bag and old flange. Second, thoroughly wash the entire abdomen with warm soapy water, being sure to wash stoma gently. (The client must be instructed to carefully inspect the skin for any signs of excoriation which should be reported to the primary healthcare provider). Third, a skin protectant is applied and allowed to dry. Fourth, the new flange is sized/cut to fit the stoma. Fifth, a thin "bead" of stoma paste is placed around the new flange opening. Lastly, Press flange into place and attach the bag. The bag is snapped over the center rim of the flange, and "burped" to allow any excess air to escape. This will create a strong seal. It is also vital to apply the clamp onto the bottom of the new bag to prevent leakage of stool.)

A nurse is reviewing the lab values for a group of clients in a psychiatric emergency department. Rank each lab result from greatest to least concern to the nurse. The client diagnosed with bipolar disorder who has a lithium level of 1.3 mEq/L The client admitted with a blood alcohol level of 0.08% (80 mg/dL) The client diagnosed with schizoaffective disorder who has a potassium level of 7.0 mEq/L (7 mmol/L The client diagnosed with schizophrenia who is taking clozapine and has a WBC count of < 3000 mm³ (3 × 10⁹/L)

The client diagnosed with schizoaffective disorder who has a potassium level of 7.0 mEq/L (7 mmol/L The client diagnosed with schizophrenia who is taking clozapine and has a WBC count of < 3000 mm³ (3 × 10⁹/L) The client diagnosed with bipolar disorder who has a lithium level of 1.3 mEq/L The client admitted with a blood alcohol level of 0.08% (80 mg/dL) (First, the client who has a potassium level of 7.0 mEq/L (7 mmol/L) should be of greatest concern to the nurse. The normal reference range for serum potassium is 3.5 mEq/L to 5.0 mEq/L (3.5-5 mmol/L) with a critical value of › 6.5mEq/L (6.5 mmol/L). This client is at high risk for cardiac death. Second,the client who has a WBC count of less than 3,000 mm³ (3 × 10⁹/L) should then be of concern to the nurse. A serious adverse effect of clozapine is agranulocytosis and, if the total white cell and/or neutrophil counts indicate agranulocytosis, clozapine should be immediately discontinued. Third, a client who has a lithium level of 1.3 mEq/L should be documented by the nurse. Therapeutic lithium levels range from 0.8 to 1.2 mEq/L and, since this client's serum lithium level is high, the client is at risk for lithium toxicity. Serum lithium levels greater than 2.0 mEq/L is toxic. Fourth, the client who has a blood alcohol level of 0.08% (80 mg/dL) is intoxicated, but this is not at a dangerous level. A blood alcohol level of 0.30% (300 mg/dL) or greater may cause severe central nervous system depression, unconsciousness, and possible death. Normal is 0-50 mg/dL or 0-0.05%.)

A nurse is in the mall when a shopper suddenly becomes non-responsive. Taking an available automatic external defibrillator (AED) from the wall, the nurse would immediately initiate interventions in what order? Uncover the client's chest. Press the shock button. Turn on the AED machine. Tell everyone to stand clear. Place pads on client's torso. Await analysis of rhythm.

Turn on the AED machine. Uncover the client's chest. Place pads on client's torso. Await analysis of rhythm. Tell everyone to stand clear. Press the shock button. (The procedure for use of an AED is consistent, whether in a medical setting or a public environment. First, turn on the AED and follow the visual and/or audio prompts. Second, the client is then placed supine and the skin of the chest is exposed (do not expose more than necessary). Next, the electrode pads are then placed to mid-chest and left lateral chest, as is pictured on the AED machine. Then, once the pads are attached, the nurse would wait for the machine to analyze the client's rhythm. If a shock is advised by the machine, the nurse will then shout "clear", being certain that no one is touching the client in any manner. The final action is to press the shock button when the AED advises to do so.)

The nurse is teaching a group of clients in cardiac rehabilitation how blood flows through the heart. In what order should the nurse present this information? List the order in which blood flows through the heart, starting from deoxygenated blood in the body. Right Atrium Vena Cava Aorta Left Ventricle Lungs Right Ventricle Left Atrium

Vena Cava Right Atrium Right Ventricle Lungs Left Atrium Left Ventricle Aorta (Deoxygenated blood comes from the body to the heart via the superior and inferior vena cava. From there blood enters the right atrium, then travels to the right ventricle. The right ventricle pumps the blood to the lungs via the pulmonary artery where the blood becomes oxygenated. From the lungs, oxygenated blood goes to the left atrium via the pulmonary vein, then to the left ventricle. The left ventricle pumps the blood out through the aorta to the body.)

The nurse is caring for a client receiving peritoneal dialysis. Place the steps for peritoneal dialysis in the correct order. Access Tenckhoff catheter Begin dwell time Assess effluent Warm dialysate Complete exchange

Warm dialysate Access Tenckhoff catheter Begin dwell time Complete exchange Assess effluent (First, warm the dialysate. Would you put cold or even cool dialysate in your peritoneal cavity? NO, it would feel uncomfortable but more importantly, it would vasoconstrict the vessels of the peritoneal membrane. Would that affect the success of the dialysis? Yes, you want dilated blood vessels to promote osmosis and diffusion. Second, access the Tenckhoff catheter, assess it for patency and look at the site for infection. Just like you would do with any IV access. Third, begin the dwell time. Fourth, complete exchange by removing effluent by gravity drainage. Fifth, assess effluent. What would I assess the effluent for? Color, clarity, amount...just like urine? What am I worried about? Infection. How would my effluent look if I had an infection? Cloudy. If I have an infection in my peritoneal cavity, I need immediate antibiotic therapy to prevent peritonitis and damage to the membrane. The primary healthcare provider is going to want to culture the effluent and start a broad spectrum antibiotic.)

The nurse is performing peritoneal dialysis on a client diagnosed with renal injury. In what order should the nurse perform this procedure? Drain fluid for 30 minutes. Turn client from side to side. Provide 30 minute dwell time. Warm dialysate. Infuse dialysate through peritoneal catheter.

Warm dialysate. Infuse dialysate through peritoneal catheter. Provide 30 minute dwell time. Drain fluid for 30 minutes. Turn client from side to side. (First, the dialysate should be warmed to body temperature by allowing it to sit out for a short period of time. Second, the dialysate is infused through the peritoneal catheter into the peritoneal cavity. Third, the prescribed dwell time should be provided. In this case the dwell time is 30 minutes. Fourth, allow the dialysate to drain for 20-30 minutes. Fifth, turn the client from side to side if all the drainage does not come out of the peritoneum.)

In what order will the nurse provide instructions to a client on using a cane? With cane on stronger side of body, support body weight with both legs. Advance weaker leg forward toward the cane. Move cane forward 6-10 inches (15 - 25 cm). Advance stronger leg forward toward cane.

With cane on stronger side of body, support body weight with both legs. Move cane forward 6-10 inches (15 - 25 cm). Advance weaker leg forward toward the cane. Advance stronger leg forward toward cane. (First, with cane on stronger side of body, support body weight with both legs. This will support the even distribution of weight away from the weaker side to promote a normal gait. Second, move cane forward 6-10 inches (15-25 cm). Moving the cane the approximate distance of a normal gait helps with stability. Third, advance weaker leg forward toward the cane. This allows the weight to be supported by the cane and the stronger leg. Fourth, advance stronger leg forward toward the cane. This allows the weight to be supported by the cane and weaker leg.)


Related study sets

BCS Cultural Geography Chapter 5

View Set

Accounting Chapter 14 - Statement of Cash Flows

View Set

Renal system NCLEX Type Questions

View Set

Macular Degeneration NCLEX questions

View Set

AP European History Semester 2 Final

View Set

TTT Exam 1 Chapter 2: Collecting Subjective Data: The Interview and Health History - ML8

View Set

The powers of state and local government

View Set